Download as docx, pdf, or txt
Download as docx, pdf, or txt
You are on page 1of 426

Table of Content

Part 1Costs: Concepts and Objectives

1. Management, the Controller, and Cost Accounting.................................................................. 1


2. Cost Concepts and the Cost accounting Information System.................................................. 5
3. Cost Behavior Analysis..................................................................................................................... 16

Part 2Cost Accumulation

4. Cost Systems and Cost Accumulation.......................................................................................... 32


5. Job Order Costing............................................................................................................................ 46
6. Process Costing................................................................................................................................. 61
7. The Cost of Quality and Accounting for Production Losses..................................................... 82
8. Costing By-Products and Joint Products..................................................................................... 101

Part 3Planning and Control of Costs

9. Materials: Controlling, Costing, and Planning.......................................................................... 116


10. Just-in-Time and Backflushing..................................................................................................... 134
11. Labor: Controlling and Accounting for Costs............................................................................ 146
12. Factory Overhead: Planned, Actual, and Applied..................................................................... 160
13. Factory Overhead: Departmentalization.................................................................................... 172
14. Activity Accounting: Activity-Based Costing
and Activity-Based Management.......................................................................................... 197

Part 4Budgeting and Standard Costs

15. Budgeting: Profits, Sales, Costs, and Expenses.......................................................................... 207


16. Budgeting: Capital Expenditures, Research and
Development Expenditures, and Cash: PERT/Cost........................................................... 222
17. Responsibility Accounting and Reporting.................................................................................. 237
18. Standard Costing: Setting Standards and Analyzing Variances............................................. 251
19. Standard Costing: Incorporating Standards
into the Accounting Records................................................................................................. 268

1
Part 5Analysis of Costs and Profits

20. Direct Costing and Cost-Volume-Profit Analysis..................................................................... 291


21. Differential Cost Analysis.............................................................................................................. 313
22. Planning for Capital Expenditures............................................................................................... 331
23. Economic Evaluation of Capital Expenditures........................................................................... 344
24. Decision-Making under Uncertainty.......................................................................................... 358
25. Profit Performance Measurements and
Intracompany Transfer Pricing.............................................................................................. 373

2
Chapter 1

MANAGEMENT, THE CONTROLLER,


AND COST ACCOUNTING

MULTIPLE CHOICE

B 1. An organizational concept that groups business functions around resources,


processes, and human interrelations is the:
A. resources function
B. functional-teamwork concept
C. processes function
D. line-staff concept
E. matching concept

E 2. The measurement of performance and the control of costs is aided the most
by:
A. organizational charts
B. continuous supervision
C. preparation for the future
D. planning
E. budgets and standards

3
A 3. All of the following are abbreviations for systems or processes that represent
changes in manufacturing technology, except:
A. CMA
B. JIT
C. CIM
D. CAD
E. FMS

D 4. The process of providing individuals with the authority to carry out their
assigned responsibilities is referred to as:
A. control circuit
B. objective setting
C. accountability
D. delegation
E. line-staff organization

A 5. The department that uses pertinent cost data to determine products that are
most profitable and sales policies is:
A. Marketing
B. Manufacturing
C. Treasury
D. Legal
E. Cost

4
Management, the Controller, and Cost Accounting 5

B 6. Examples of nonroutine planning include all of the following, except:


A. responses to the appearance of new competition
B. estimating the collection of receivables during the next month for the
purpose of making investment/borrowing decisions
C. responses to a proposed government regulation of the industry
D. responses to a significant change in consumer tastes
E. none of the above

B 7. The coordinated development of a company's organization with the cost and


budgetary system will lead to an approach to accounting and reporting called:
A. functional-teamwork system
B. responsibility accounting
C. line-staff organization
D. controllable segmentation
E. superior-subordinate relationship

E 8. The organizational group that advises or performs technical functions of an


enterprise is the:
A. line
B. function
C. team
D. executive management
E. staff

A 9. The business function in the functional-teamwork concept of management


that deals with activities such as product design, research and development, purchasing,
manufacturing, advertising, marketing, and billing is the:
A. processes function
Management, the Controller, and Cost Accounting 6

B. executive function
C. resources function
D. staff
E. human interrelations function

E 10. Pronouncements of the Cost Accounting Standards Board adhere to the


concept of:
A. indirect costing
B. common costing
C. direct costing
D. standard costing
E. full costing

B 11. The professional certification developed by the IMA indicating professional


competence in the management accounting field is the:
A. CIA
B. CMA
C. CA
D. CPA
E. CPM
Management, the Controller, and Cost Accounting 7

C 12. The plans that are sufficiently detailed to permit the preparation of budgeted
financial statements for the entity as of a future date are:
A. strategic plans
B. medium-range plans
C. short-range plans
D. long-range plans
E. none of the above

B 13. All of the following are organizations in the private sector that influence the
development of cost accounting theory and practice except:
A. FEI
B. IRS
C. AICPA
D. FASB
E. IMA

D 14. Budgeting plays an important role in influencing individual and group


behavior at all of the following stages of the management process, except:
A. setting goals
B. motivating desired performance
C. evaluating performance
D. computing bonuses
E. suggesting when corrective action should be taken

B 15. The functional-teamwork concept of management is structured to emphasize


all of the following except:
A. human interrelations
B. accountability
Management, the Controller, and Cost Accounting 8

C. resources
D. processes
E. none of the above

E 16. All of the following are examples of non-value-added activities except:


A. retrieving
B. handling
C. expediting
D. reworking
E. assembling

D 17. The department that has the responsibility for the financial administration of
a company is:
A. Tax
B. Controller's
C. Cost
D. Treasury
E. Internal Audit

D 18. The collection, presentation, and analysis of cost data should help
management accomplish all of the following tasks except:
A. control the physical quantities of inventory
B. determine company costs and profits for an accounting period
C. choose from among two or more alternatives that will increase
revenues
D. conform to FASB reporting requirements for pensions
E. establish costing methods and procedures that permit cost reductions
Management, the Controller, and Cost Accounting 9

D 19. In an attempt to resolve an ethical conflict in a publicly-held corporation, if


the accountant has unsuccessfully gone to the board of directors, the next step is to:
A. go to the company president
B. go back to middle management to garner support
C. report the problem to the SEC
D. resign
E. none of the above

C 20. An organizational concept recognizing that all positions or functional divisions


can be categorized into two groups is:
A. functional-teamwork concept
B. processes function
C. line-staff concept
D. matching concept
E. resources function

A 21. In an attempt to resolve an ethical conflict when the immediate superior is


involved, an accountant should first:
A. go to the next higher level of management
B. report the problem to the SEC
C. resign
D. go to the company president
E. none of the above

C 22. The Standards of Ethical Conduct for Management Accountants presents


fifteen responsibilities of the management accountant that encompass all of the following
categories except:
A. competence
Management, the Controller, and Cost Accounting 10

B. confidentiality
C. dependability
D. integrity
E. objectivity
Management, the Controller, and Cost Accounting 5

Chapter 2

COST CONCEPTS AND THE COST


ACCOUNTING INFORMATION SYSTEM

MULTIPLE CHOICE

Question Nos. 14, 15, 17, and 20-26 are AICPA adapted.
Question Nos. 13, 18, and 19 are ICMA adapted.
Question No. 16 is CIA adapted.

C 1. A cost accounting information system necessarily should accomplish all of the


following except:
A. reflect the division of authority so that individual managers can be held
accountable
B. provide management with information that facilitates prompt identification of
activities needing attention
C. be more sophisticated than is required by legal, regulatory, and contractual
requirements
D. be tailored to give the most efficient blend of sophistication and simplicity
E. focus management's attention

5
Management, the Controller, and Cost Accounting 6

A 2. Cost classifications are based on the relationship of costs to all of the following
except:
A. ledger accounts
B. accounting periods
C. products
D. volume of production
E. manufacturing departments

B 3. Direct materials and direct labor are considered to be:


A. selling expenses
B. prime costs
C. administrative expenses
D. conversion costs
E. factory overhead

E 4. Depreciation on factory buildings and equipment is classified as:


A. selling expense
B. administrative expense
C. direct labor
D. indirect materials
E. factory overhead

6
Cost Concepts and the Cost Accounting Information System 7

A 5. A typical marketing expense is:


A. freight out
B. indirect labor
C. audit fees
D. uncollectible accounts expense
E. direct labor

E 6. A typical indirect labor cost for a manufacturer is:


A. sales office salaries
B. freight out
C. factory insurance
D. sales commissions
E. materials handling

D 7. Usually, a cost easy to assign accurately to a specific operating department is


a:
A. standard cost
B. common cost
C. fixed cost
D. variable cost
E. joint cost

C 8. In constructing a chart of accounts, all of the following guidelines should be


adhered to except:
A. using numbers rather than letters in coding the accounts
B. dividing charts into balance sheet accounts and income statement
accounts
C. using account titles that reflect a maximum level of detail about each item
Cost Concepts and the Cost Accounting Information System 8

D. giving maximum information with a minimum of supplementary analysis


E. providing sufficient classification to enable cost assignment to responsible
managers

C 9. An expense that is likely to contain both fixed and variable components is:
A. security guard wages
B. supplies
C. heat, light, and power
D. small tools
E. taxes on real estate

C 10. A type of employee whose wages are not a component of indirect labor is a(n):
A. inspector
B. supervisor
C. assembler
D. maintenance worker
E. shop clerk
Cost Concepts and the Cost Accounting Information System 9

B 11. Pitino Company has a beginning inventory of direct materials on March 1 of


$30,000 and an ending inventory on March 31 of $36,000. The following
additional manufacturing cost data were available for the month of March:

Direct materials purchased............................................................$84,000


Direct labor................................................................................... 60,000
Factory overhead.......................................................................... 80,000

During March, prime cost added to production was:


A. $140,000
B. $138,000
C. $144,000
D. $150,000
E. none of the above

SUPPORTING CALCULATION:

$84,000 + $60,000 - ($36,000 - $30,000) = $138,000

C 12. Pitino Company has a beginning inventory of direct materials on March 1 of


$30,000 and an ending inventory on March 31 of $36,000. The following
additional manufacturing cost data were available for the month of March:

Direct materials purchased............................................................$84,000


Direct labor................................................................................... 60,000
Factory overhead.......................................................................... 80,000
Cost Concepts and the Cost Accounting Information System 10

During March, conversion cost added to production was:


A. $80,000
B. $144,000
C. $140,000
D. $138,000
E. none of the above

SUPPORTING CALCULATION: $60,000 + $80,000 = $140,000

A 13. The term "variable costs" refers to:


A. all costs whose total amounts change in proportion to changes in activity
within a relevant range
B. all costs that are likely to respond to the amount of attention devoted to
them by a specified manager
C. all costs that are associated with marketing, shipping, warehousing, and
billing activities
D. all costs that do not change in total for a given period and relevant range,
but become progressively smaller on a per-unit basis as volume increases
E. all manufacturing costs incurred to produce units of output
Cost Concepts and the Cost Accounting Information System 11

C 14. The following statement that best describes a fixed cost is:
A. it may change in total when such change depends on production within
the relevant range
B. it increases on a per-unit basis as production increases
C. it decreases on a per-unit basis as production increases
D. it may change in total when such change is related to changes in
production
E. it is constant per unit of production

A 15. The term "relevant range" as used in cost accounting means the range over
which:
A. cost relationships are valid
B. production may vary
C. relevant costs are incurred
D. costs may fluctuate
E. none of the above

B 16. When the number of units manufactured increases, the most significant change
in average unit cost will be reflected as:
A. a decrease in the variable element
B. a decrease in the nonvariable element
C. an increase in the semivariable element
D. an increase in the variable element
E. an increase in the nonvariable element

C 17. Within a relevant range, the amount of variable cost per unit:
A. moves in the same direction as fixed cost per unit
B. differs at each production level
Cost Concepts and the Cost Accounting Information System 12

C. remains constant at each production level


D. increases as production increases
E. decreases as production increases

B 18. The term "prime costs" refers to:


A. the sum of direct labor costs and all factory overhead costs
B. the sum of direct materials costs and direct labor costs
C. manufacturing costs incurred to produce units of output
D. all costs associated with manufacturing other than direct labor and direct
materials costs
E. cost standards that are predetermined and should be attained

B 19. The term "conversion costs" refers to:


A. costs that are associated with marketing, shipping, warehousing, and billing
activities
B. the sum of direct labor costs and all factory overhead costs
C. the sum of direct materials costs and direct labor costs
D. manufacturing costs incurred to produce units of output
E. all costs associated with manufacturing other than direct labor costs and
direct materials costs
Cost Concepts and the Cost Accounting Information System 13

A 20. Direct labor is a:

Conversion Cost Manufacturing Cost Prime Cost


A. Yes Yes Yes
B. No Yes Yes
C. No No No
D. No No Yes
E. Yes Yes No

C 21. A factory overhead cost:


A. is a direct cost
B. is a prime cost
C. can be a variable cost or a fixed cost
D. can only be a fixed cost
E. includes all factory labor

A 22. Prime cost and conversion cost share what common element of total cost?
A. direct labor
B. commercial expense
C. variable overhead
D. fixed overhead
E. direct materials

E 23. Factory overhead includes:


A. indirect materials but not indirect labor
B. indirect labor but not indirect materials
C. prime costs
D. all manufacturing costs
Cost Concepts and the Cost Accounting Information System 14

E. all manufacturing costs, except direct materials and direct labor

C 24. Indirect materials are a(n):


A. fixed cost
B. irrelevant cost
C. factory overhead cost
D. direct cost
E. prime cost

C 25. Wages of the security guard for a small plant are an example of:

Fixed Factory
Indirect Labor Overhead
A. No Yes
B. No No
C. Yes Yes
D. Yes No
E. none of the above
Cost Concepts and the Cost Accounting Information System 15

B 26. Wages paid to factory machine operators of a manufacturing plant are an


element of:

Prime Cost Conversion Cost


A. Yes No
B. Yes Yes
C. No No
D. No Yes
E. none of the above

E 27. Common costs are:


A. costs that occur when the production of one product is possible only if one
or more other products are manufactured at the same time
B. intended to benefit future periods
C. variable in direct proportion to the level of production
D. chargeable directly to the product
E. costs of facilities or services employed by two or more operations

D 28. Joint costs are:


A. direct costs
B. costs of facilities or services employed by two or more operations
C. revenue expenditures
D. incurred when the production of one product is possible only if other
products are produced at the same time
E. always variable
Cost Concepts and the Cost Accounting Information System 16

B 29. All of the following are examples of nonfinancial performance measures except:
A. the number of defective units produced
B. the gross margin on a product line income statement
C. hours of machine downtime
D. number of days on schedule
E. weight of scrap material produced

E 30. Reasons for the increased attention being given to nonfinancial performance
measures include:
A. dissatisfaction with exclusive reliance on financial measures
B. dissatisfaction with financial measures of plant utilization
C. dissatisfaction with financial measures of processing efficiency
D. dissatisfaction with the slow pace at which a company's data processing
system can modify traditional financial measures
E. all of the above

E 31. Of the following items, a cost object is:


A. a unit of product
B. a customer order
C. a project
D. a division of the company
E. all of the above
Cost Concepts and the Cost Accounting Information System 17

E 32. General corporate-level costs, such as bond interest and taxes, would be
readily traceable to:
A. each unit of product
B. each division of the company
C. each batch of production
D. all units of product ever produced
E. none of the above

D 33. A revenue expenditure is one that:


A. varies with the volume of production
B. is intended to benefit future periods
C. is reported as an asset
D. benefits the current period only
E. remains the same in total as production changes

B 34. An example of a cost that is irrelevant to a future decision is a(n):


A. differential cost
B. sunk cost
C. out-of-pocket cost
D. opportunity cost
E. variable cost

A 35. The measures in a balanced scorecard’s growth and learning perspective


attempt to report on:
A. three kinds of intangible resources: human capital, information, and the
alignment of incentives
B. the organization’s most important work—work in which the organization
must excel in order to successful
Cost Concepts and the Cost Accounting Information System 18

C. the final results that are most important to the owners of the organization,
and the rates of improvement in those results
D. the extent to which the organization is creating and sustaining desirable
customer relationships
E. all of the above

E 36. An example of a cost that is irrelevant to a future decision is a(n):


A. communicating plans
B. focusing attention
C. implementing strategy
D. monitoring progress toward strategic objectives
E. all of the above

A 37. When a balanced scorecard represents a series of predictions telling how


management intends for the organization to succeed, the predictions are in a
sequence that begins with:
A. growth and learning
B. innovation
C. customer service
D. customer satisfaction
E. investment opportunity

A 38. Balanced scorecards are called “balanced” because they report:


A. both leading and lagging measures
B. both balance sheets and income statements
C. both historical costs and replacements costs
D. both original costs and book values
E. all of the above
Cost Concepts and the Cost Accounting Information System 19

PROBLEMS

PROBLEM

1.
Identification of Variable, Fixed, and Semivariable Costs. Place a check mark in the
appropriate column to indicate whether the following costs are variable, fixed, or
semivariable.

Item .......................................................... Variable Fixed Semivariable


1. Small tools.........................................
2. Patent amortization............................
3. Health and accident insurance...........
4. Heat, light, and power........................
5. Straight-line depreciation....................
6. Maintenance of buildings and
grounds..............................................
7. Royalties............................................
8. Materials handling..............................
9. Property and liability insurance...........
10. Maintenance of factory equipment......

SOLUTION

1. Variable
2. Fixed
Cost Concepts and the Cost Accounting Information System 20

3. Semivariable
4. Semivariable
5. Fixed
6. Fixed
7. Variable
8. Variable
9. Fixed
10. Semivariable
Cost Concepts and the Cost Accounting Information System 21

PROBLEM

2.
Classification of Costs. Place a check mark in the appropriate column to indicate the
proper classification of each of the following costs.

Other
Indirect Admini-
Indirect Indirect Factory Marketing strative
Item Materials Labor Costs Expenses Expenses
1. Factory heat, light,
and power....................................
2. Advertising...................................
3. Wages of stockroom clerk............
4. Freight out...................................
5. Oil for machines...........................
6. Salary of vice president
of human relations.......................
7. Legal expenses............................
8. Salary of the factory manager......
9. Employer payroll taxes on
controller's salary.........................
10. Idle time due to assembly
line breakdown.............................

SOLUTION
Cost Concepts and the Cost Accounting Information System 22

1. Other indirect factory costs


2. Marketing expenses
3. Indirect labor
4. Marketing expenses
5. Indirect materials
6. Administrative expenses
7. Administrative expenses
8. Indirect labor
9. Administrative expenses
10. Indirect labor

PROBLEM

3.
Fixed and Variable Costs. In 19A, the Lin Company had sales of $2,500,000, with
$1,250,000 variable and $900,000 fixed costs. In 19B, sales are expected to decrease
10% and the fixed costs are not expected to change.

Required: Determine Lin Company's expected operating income or loss for 19B.
Cost Concepts and the Cost Accounting Information System 23

SOLUTION

Sales ($2,500,000 x 90%)...................................................... $


2,250,000
Less: Variable costs
($1,250,000 x 90%)................................................. $1,125,000
Fixed costs...................................................................... 900,000
2,025,000

Operating income........................................................................ $
225,000

PROBLEM

4.
Determination of per Unit Total Costs. The estimated unit costs for Hoteling Industries,
when operating at a production and sales level of 10,000 units, are as follows:

Cost Item Estimated Unit Cost


Direct materials........................... $15
Direct labor................................. 10
Variable factory overhead............ 8
Fixed factory overhead................ 5
Variable marketing...................... 4
Fixed marketing........................... 3

Required:
Cost Concepts and the Cost Accounting Information System 24

(1) Identify the estimated conversion cost per unit.


(2) Identify the estimated prime cost per unit.
(3) Determine the estimated total variable cost per unit.
(4) Compute the total cost that would be incurred during a month with a production level
of 10,000 units and a sales level of 12,000 units.

SOLUTION

(1) $10 + $8 + $5 = $23 conversion cost per unit


(2) $15 + $10 = $25 prime cost per unit
(3) $15 + $10 + $8 + $4 = $37 variable cost per unit
(4) [ ( $15 + $10 + $8 + $5 + $3 ) x 10,000 ] + ( $4 x 12,000 )
= $410,000 + $48,000 = $458,000 total cost

PROBLEM

5.
Components of Manufacturing Cost. Myerson Inc. produces video cameras. The direct
labor cost of one camera is $200, and the total manufacturing cost is $650. The
overhead cost of one camera is two-thirds as large as its conversion cost.
Cost Concepts and the Cost Accounting Information System 25

Required:

(1) Compute the conversion cost per unit.


(2) Determine the factory overhead cost per unit.
(3) Determine the direct materials cost per unit.

SOLUTION

(1) Let x = Conversion cost per unit


x = $200 + 2/3 x
1/3 x = $200
x = $600
(2) Factory overhead = 2/3 x $600 = $400
(3) Direct materials = $650 - $200 - $400 = $50

PROBLEM

6.
Identification of financial measures, customer measures, internal business process measures, and growth
& learning measures for a balanced scorecard. Place a check mark in the appropriate column to indicate
which one of the four perspectives of a typical balanced scorecard is most likely to contain each of the
following measures.
Internal
Business Growth
&
Item Financial Customer Process
Learning

1. Number of shipments rejected


Cost Concepts and the Cost Accounting Information System 26

by customers. . . . . . . . . . . . . .
2. Number of defective units removed
from work in process. . . . . . . . .
3. Percentage of employees who’ve
completed all training relevant
to their duties. . . . . . . . . . . . . .
4. Growth rate of sales revenue . . . .
5. Percentage of employees whose
cash bonuses depend on
achieving scorecard objectives . . .
6. Income from continuing
operations, before income taxes . .
7. Percentage of employees who
receive all information their job
requires, on time and error-free . .
8. Market share in targeted market
segment . . . . . . . . . . . . . . . . .
9. Number of new products launched
10. Number of patents received . . . . .
Cost Concepts and the Cost Accounting Information System 27

SOLUTION

1. customer
2. internal business process
3. growth & learning
4. financial
5. growth & learning
6. financial
7. growth & learning
8. customer
9. internal business process
10. internal business process
Cost Concepts and the Cost Accounting Information System 16

Chapter 3

COST BEHAVIOR ANALYSIS

MULTIPLE CHOICE

Question Nos. 12-14 and 20-25 are AICPA adapted.


Question Nos. 16-19 and 28 are ICMA adapted.
Question Nos. 15, 26, and 28 are CIA adapted.

D 1. Expenses that require a series of payments over a long period of timesuch as long-term debt and
lease rentalsare frequently known as:
A. programmed fixed expenses
B. avoidable expenses
C. variable expenses
D. committed fixed expenses
E. normal capacity expenses

C 2. A mathematical technique used to fit a straight line to a set of plotted points is:
A. integral calculus
B. the EOQ model
C. the method of least squares
D. linear programming
E. PERT network analysis

E 3. One advantage of using multiple regression analysis is that:


A. computations are simplified
B. only two data points need be considered
C. a two-dimensional graph may be used to show cost relationships
D. costs may be grouped into one independent variable
E. the effects of several variables on costs may be analyzed

B 4. The coefficient of determination indicates:


A. causal relationships among costs and other factors
B. the percentage of explained variance in the dependent variable
C. the linear relationship between two variables
D. whether several variables fluctuate
E. the size of the standard deviation

16
Cost Behavior Analysis 17

E 5. Hoyden Co. developed the following equation to predict certain components of its budget for the
coming period:

Costs = $50,000 + ($5 x direct labor hours)

The $5 would approximate:


A. total cost
B. direct labor rate per hour
C. fixed cost per direct labor hour
D. the coefficient of determination
E. variable costs per direct labor hour

E 6. When cost relationships are linear, total variable manufacturing costs will vary in proportion to
changes in:
A. machine hours
B. direct labor hours
C. total material cost
D. total overhead cost
E. volume of production

B 7. The term "relevant range" as used in cost accounting means the range over which:
A. relevant costs are incurred
B. cost relationships are valid
C. costs may fluctuate
D. sales volume fluctuates
E. production may vary

E 8. Within a relevant range, the amount of fixed cost per unit:


A. differs at each production level on a per-unit basis
B. remains constant in total
C. decreases as production increases on a per-unit basis
D. increases as production decreases on a per-unit basis
E. all of the above

C 9. The following relationships pertain to a year's budgeted activity for Buckeye Company:

High Low
Direct labor hours................................................................ 400,000 300,000
Total costs............................................................................ $154,000 $129,000

What are the budgeted fixed costs for the year?


A. $100,000
B. $25,000
C. $54,000
D. $75,000
E. none of the above
Cost Behavior Analysis 18

SUPPORTING CALCULATION:

High..................................................................................... $ 154,000 400,000


Low...................................................................................... 129,000 300,000
Difference............................................................................ $ 25,000 100,000

Variable rate = $25,000  100,000 = $.25/direct labor hour


Fixed cost = $154,000 - $.25(400,000) = $54,000

B 10. Maintenance expenses of a company are to be analyzed for purposes of constructing a flexible
budget. Examination of past records disclosed the following costs and volume measures:

High Low
Cost per month.................................................................... $39,200 $32,000
Machine hours..................................................................... 24,000 15,000

Using the high-low method of analysis, the estimated variable cost per machine hour is:
A. $12.50
B. $0.80
C. $0.08
D. $1.25
E. none of the above

SUPPORTING CALCULATION:

High..................................................................................... $ 39,200 24,000


Low...................................................................................... 32,000 15,000
Difference............................................................................ $ 7,200 9,000

Variable rate = $7,200 / 9,000 = $.80/machine hour

D 11. A company allocates its variable factory overhead based on direct labor hours. During the past three
months, the actual direct labor hours and the total factory overhead allocated were as follows:

October November December


Direct labor hours...................................... 2,500 3,000 5,000
Total factory
overhead allocated............................. $80,000 $75,000 $100,000

Based upon this information, the estimated variable cost per direct labor hour was:
A. $.125
B. $12.50
C. $.08
D. $8
E. none of the above
Cost Behavior Analysis 19

SUPPORTING CALCULATION:

High..................................................................................... $ 100,000 5,000


Low...................................................................................... 75,000 2,500
Difference............................................................................ $ 25,000 2,500

Variable rate = $20,000  2,500 = $8.00/direct labor hour

A 12. The technique that can be used to determine the variable and fixed portions of a company's costs is:
A. scattergraph method
B. poisson analysis
C. linear programming
D. game theory
E. queuing theory

A 13. The number of variables used in simple regression analysis is:


A. two
B. three
C. more than three
D. three or less
E. one

C 14. Multiple regression analysis:


A. is not a sampling technique
B. involves the use of independent variables only
C. assumes that the independent variables are not correlated
D. establishes a cause-and-effect relationship
E. all of the above

E 15. For a simple regression-analysis model that is used to allocate factory overhead, an internal auditor
finds that the intersection of the line of best fit for the overhead allocation on the y-axis is $50,000.
The slope of the trend line is .20. The independent variable, factory wages, amounts to $900,000 for
the month. What is the estimated amount of factory overhead to be allocated for the month?
A. $910,000
B. $950,000
C. $ 50,000
D. $180,000
E. $230,000

SUPPORTING CALCULATION:

Factory overhead = $50,000 + .2($900,000) = $230,000


Cost Behavior Analysis 20

A 16. As a result of analyzing the relationship of total factory overhead to changes in machine hours, the
following relationship was found:

y bar = $1,000 + $2 x bar

This equation was probably found by using the mathematical techniques called:
A. simple regression analysis
B. dynamic programming
C. linear programming
D. multiple regression analysis
E. none of the above

A 17. As a result of analyzing the relationship of total factory overhead to changes in machine hours, the
following relationship was found:

y bar = $1,000 + $2 x bar

The y bar in the equation is an estimate of:


A. total factory overhead
B. total fixed costs
C. total machine costs
D. total variable costs
E. none of the above

C 18. As a result of analyzing the relationship of total factory overhead to changes in machine hours, the
following relationship was found:

y bar = $1,000 + $2 x bar

The $2 in the equation is an estimate of:


A. fixed costs per machine hour
B. total fixed costs
C. variable costs per machine hour
D. total variable costs
E. none of the above

D 19. As a result of analyzing the relationship of total factory overhead to changes in machine hours, the
following relationship was found:

y bar = $1,000 + $2 x bar

The use of such a relationship of total factory overhead to changes in machine hours is said to be
valid only within the relevant range, which means:
A. within the range of reasonableness as judged by the department supervisor
B. within the budget allowance for overhead
C. within a reasonable dollar amount for machine costs
D. within the range of observations of the analysis
E. none of the above
Cost Behavior Analysis 21

C 20. A measure of the extent to which two variables are related linearly is referred to as:
A. sensitivity analysis
B. input-output analysis
C. coefficient of correlation
D. cause-effect ratio
E. cost-benefit analysis

C 21. The appropriate range for the coefficient of correlation (r) is:
A. -infinity  r  infinity
B. 0r1
C. -1  r  1
D. -100  r  100
E. none of the above

A 22. The covariation between two variables, such as direct labor hours and electricity expense, can best
be measured by:
A. correlation analysis
B. simple regression analysis
C. multiple regression analysis
D. high-low method
E. scattergraph method

B 23. The quantitative method that will separate a semivariable cost into its fixed and variable components
with the highest degree of precision is:
A. simplex method
B. least squares method
C. scattergraph method
D. account analysis
E. high-low method

A 24. If the coefficient of correlation between two variables is zero, a scatter diagram of these variables
would appear as:
A. random points
B. a least squares line that slopes up to the right
C. a least squares line that slopes down to the right
D. under this condition, a scatter diagram could not be plotted on a graph
E. none of the above

D 25. Multiple regression analysis involves the use of:

Dependent Independent
Variables Variables
A. 1 none
B. 1> 1
C. 1> 1>
D. 1 1>
Cost Behavior Analysis 22

C 26. A company using regression analysis to correlate income to a variety of sales indicators found that
the relationship between the number of sales managers in a territory and net income for the territory
had a correlation coefficient of -1. The best description of this situation is:
A. that more sales managers should be hired
B. imperfect negative correlation
C. perfect inverse correlation
D. no correlation
E. perfect positive correlation

B 27. The correlation coefficient that indicates the weakest linear association between two variables is:
A. -0.73
B. -0.11
C. 0.12
D. 0.35
E. 0.72

B 28. If regression was applied to the data shown in Figure 3-1, the coefficients of correlation and

determination would indicate the existence of a:

A. low linear relationship, high explained variation ratio


B. high inverse linear relationship, high explained variation ratio
C. high direct linear relationship, high explained variation ratio
D. high inverse linear relationship, low explained variation ratio
E. none of the above

A 29. Omitting important variables from the multiple regression is referred to as a(n):
A. specification error
B. autocorrelation
C. confidence loss
D. homoscedastic error
E. heteroscedastic error
Cost Behavior Analysis 23

E 30. When two or more independent variables are correlated with one another, the condition is referred to
as:
A. serial correlation
B. autocorrelation
C. heteroscedacity
D. homoscedacity
E. multicollinearity
Cost Behavior Analysis 24

A 31. A large value for standard error of the estimate indicates that:
A. the actual cost will likely vary greatly from the estimated cost as portrayed by the regression
line
B. the actual cost will be greater than the estimate cost as portrayed by the regression line
C. the actual cost will be less than the estimate cost as portrayed by the regression line
D. the actual cost will likely vary little from the estimated cost as portrayed by the regression
line
E. none of the above

D 32. The confidence interval represents:


A. the percentage of variance in the dependent variable as explained by the independent variable
B. the measure of the extent to which variables are related linearly
C. the standard deviation about the regression line
D. a range of values within which the dependent variable is expected to fall a certain percentage
of the time
E. none of the above

C 33. When the distribution of observations around the regression line is uniform for all values of the
independent variable, it is:
A. heteroscedastic
B. serially correlated
C. homoscedastic
D. autocorrelated
E. none of the above

E 34. Expenses that are fixed at management's discretion at a certain level for the period are referred to as:
A. committed fixed costs
B. mixed costs
C. opportunity costs
D. sunk costs
E. programmed fixed costs

A 35. The separation of fixed and variable costs is necessary for all of the following purposes except:
A. absorption costing and net income analysis
B. direct costing and contribution margin analysis
C. break-even and cost-volume-profit analysis
D. differential and comparative cost analysis
E. capital budgeting analysis
Cost Behavior Analysis 25

PROBLEMS

PROBLEM

1.
High and Low Points Method. A controller is interested in analyzing the fixed and variable costs of indirect
labor as related to direct labor hours. The following data have been accumulated:

Indirect Direct Labor


Month Labor Cost Hours
March......................................................................................................... $2,880 425
April........................................................................................................... 3,256 545
May............................................................................................................ 2,820 440
June............................................................................................................ 3,225 560
July............................................................................................................. 3,200 540
August........................................................................................................ 3,200 495

Required: Determine the amount of the fixed portion of indirect labor expense and the variable rate for indirect
labor expense, using the high and low points method. (Round the variable rate to three decimal places and the
fixed cost to the nearest whole dollar.)

SOLUTION

Indirect Direct Labor


Labor Cost Hours
High............................................................................................................ $ 3,225 560
Low............................................................................................................ 2,880 425
Difference.................................................................................................. $ 345 135

Variable rate = $345  135 = $2.556 per direct labor hour


Fixed cost = $3,225 - ($2.556 x 560) = $1,794

PROBLEM

2.
Fixed, Variable, and Semivariable Production Costs. Ibus Instruments Co. developed the following regression
equations to indicate costs at various activity levels:

Direct labor = $4 per unit


Materials = $3 per unit
Supervision = $5,000
Power = $300 + $.25 per unit + $.50 per machine hour
Factory supplies = $250 + $.75 per unit
Depreciationequipment = $1 per machine hour
Depreciationbuilding = $10,000

During the next period, the company anticipates production of 20,000 units and usage of 3,000 machine hours.

Required: Prepare a schedule of the production costs to be incurred during the next period.
Cost Behavior Analysis 26

SOLUTION

Production costs:
Direct labor......................................................................................... $ 80,000
Direct materials.................................................................................. 60,000
Overhead to be incurred:
Supervision......................................................................................... $ 5,000
Power [$300 + ($.25 x 20,000 units) +
($.50 x 3,000 machine hours)]................................................... 6,800
Factory supplies [$250 + ($.75 x 20,000 units)]....................................... 15,250
Depreciationequipment........................................................................... 3,000
Depreciationbuilding............................................................................... 10,000 40,050
Total production cost................................................................................. $ 180,050

PROBLEM

3.
Statistical Scattergraph. Dale Company management is interested in determining the fixed and variable
components of electricity expense, a semivariable cost, as measured against machine hours. Data for the first
eight months of the current year follow:

Machine Electricity
Month Hours Cost
January....................................................................................................... 4,500 $650
February..................................................................................................... 4,750 600
March......................................................................................................... 5,000 750
April........................................................................................................... 5,500 700
May............................................................................................................ 7,250 900
June............................................................................................................ 7,500 800
July............................................................................................................. 6,750 825
August........................................................................................................ 5,250 725

Required: Graph the data provided and determine the total fixed cost and the variable cost per machine hour for
electricity. (Round estimates to the nearest cent.)

SOLUTION

Average cost ($5,950  8)......................................................................... $743.75


Fixed cost per month (from graph)........................................................... 200.00
Average total variable cost........................................................................ $543.75
$543.75
= $ . 0935 variable cost per machine hour
$46,500 ÷ 8
Cost Behavior Analysis 27
Cost Behavior Analysis 28

PROBLEM

4.
Method of Least Squares. The management of Rainbow Inc. would like to separate the fixed and variable
components of electricity as measured against machine hours in one of its plants. Data collected over the most
recent six months follow:

Electricity Machine
Month Cost Hours
January....................................................................................................... $1,100 4,500
February..................................................................................................... 1,110 4,700
March......................................................................................................... 1,050 4,100
April........................................................................................................... 1,200 5,000
May............................................................................................................ 1,060 4,000
June............................................................................................................ 1,120 4,600

Required: Using the method of least squares, compute the fixed cost and the variable cost rate for electricity
expense. (Round estimates to the nearest cent.)

SOLUTION

(1) (2) (3) (4) (5) (6)


Electricity Cost Machine Activity
Month Cost Deviation Hours Deviation (4) Squared (4) x (2)
January.............................. $1,100 (7) 4,500 17 289 (119)
February............................ 1,110 3 4,700 217 47,089 651
March................................ 1,050 (57) 4,100 (383) 146,689 21,831
April.................................. 1,200 93 5,000 517 267,289 48,081
May................................... 1,060 (47) 4,000 (483) 233,289 22,701
June................................... 1,120 13 4,600 117 13,689 1,521
$6,640 (2)* 26,900 2* 708,334 94,666

y bar = Σy  n = $6,640  6 = $1,107

x bar = Σx  n = $26,900  6 = $4,483

*rounding difference

Column (6) 94,666


Variable rate = = = $ . 13
Column (5) 708,334
Fixed cost = $1,107 - ($.13)(4,483) = $524.21
Cost Behavior Analysis 29

PROBLEM

5.
Coefficients of Correlation and Determination. The president of Scranton Steel Co. has prepared the following
data so that an assessment may be made for developing a regression analysis of smelting costs:

Year Smelting Costs Direct Labor Hours Kilograms of Iron Smelted


19_1 $12,000 2,100 50.2
19_2 12,900 1,800 55.6
19_3 13,500 2,250 60.0
19_4 12,750 2,400 54.0
19_5 14,100 2,250 64.4
Total $65,250 10,800 284.2

Required: Compute the coefficient of correlation (r) and the coefficient of determination (r2) for each of the
independent variables. (Round to three decimal places.)

Note to instructor: It may be helpful to provide students with the following equation:

Σ [( x i⋅ x bar yi ¿ bar
r= bar )2 y i ¿ bar )2 ¿ ¿ ¿
square root x i ¿

SOLUTION

DIRECT LABOR HOURS

(1) (2) (3) (4) (5) (6) (7)


Difference Difference
from from
Average Direct Average
Smelting of Labor of 2,160
Costs $13,050 Hours Hours (4) Squared (4) x (2) (2) Squared
$12,000 (1,050) 2,100 (60) 3,600 63,000 1,102,500
12,900 (150) 1,800 (360) 129,600 54,000 22,500
13,500 450 2,250 90 8,100 40,500 202,500
12,750 (300) 2,400 240 57,600 (72,000) 90,000
14,100 1,050 2,250 90 8,100 94,500 1,102,500
$65,250 0 10,800 0 207,000 180,000 2,520,000

Column 6 total
r=
square root [(Column 5 total )(Column 7 total )]

$180,000
r=
square root
Cost Behavior Analysis 30

$180,000
r =
square root

$180,000
r =
$722,246KILOGRAMS OF IRON SMELTED
(8) (9) (10) (11)
Difference
Kilograms of from Average
IronrSmelted
= + . 249of 56.84 (9) Squared (9) x (2)
50.2 (6.64) 44.0896 $6,972
55.6 (1.24) 1.5376 186
60.0 3.16 9.9856 1,422
2
54.0 (2.84) 8.0656 852
r64.4 = . 062 7.56 unTra lai 57.1536
em niem vui khi duoc gan 7,938
ben em, tra lai em loi yeu
284.2 0.00 120.8320 $17,370

Column 11 total
r=
square root [(Column 10 total )(Column 7 total )]

$17,370
r=
square root

$17,370
r=
square root

$17,370
r=
$17,450

r = .995

r 2 = .990
Cost Behavior Analysis 31
Cost Behavior Analysis 32

PROBLEM

6.
Standard Error of the Estimate and Confidence Interval Estimation. The production supervisor of Lyle Inc.
would like to know the range of electricity cost that should be expected about 95 percent of the time at the 15,000
direct labor hour level of activity. The least squares estimate of electricity cost at that level of activity is $750.
The least squares parameter estimates (i.e., the estimates of fixed cost and the variable cost rate) were derived
from a sample of data for a recent 12-month period. The direct labor hour average for the sample period is
13,000, and the direct labor hour deviations from its average squared and summed (Σ(xi-xi)2) is 80,000,000. The
prediction error squared (Σ(yi-yi)2) over the sample period is $40,850.

Required:

Compute:
(1) the standard error of the estimate
(2) the 95 percent confidence interval (Table factor 2.228) estimate for electricity cost at the 15,000 direct labor
hour level of activity

(Round answers to the nearest whole dollar.)

SOLUTION

$40,850
square root ( 12 ⋅ 2 )= square root $4,085 = $64

(1)

2
1 (15,000 ⋅ 13,000)
$750 ±(2.228 )($64 ) square root 1 +
12(+
80,000,000 )
$750 ±(2.228 )($64 ) square root 1.1333
$750 ±(2.228 )($64)(1 .0645)
$750 ± $152
(2)
Cost Behavior Analysis 33

PROBLEM

7.
Method of Least Squares. The data below are found to be highly correlated for Mystic Modem Manufacturing
Corp.:

Fabricating Kilograms of
Costs Materials Used
$15,600 360
18,000 463
17,100 412
21,300 595
19,500 520
$91,500 2,350

Required:

(1) Write an equation reflecting the relationship between fabricating costs and kilograms of materials used,
using the method of least squares.
(2) Determine the standard error of the estimate.
(3) Determine the standard error of the estimate correction factor when direct labor hours are 500.
(4) Determine the coefficient of correlation (r) and the coefficient of determination (r2).

(Round dollar amounts to the nearest cent and unit amounts to four decimal places.)

SOLUTION

(1)

(1) (2) (3) (4) (5) (6) (7)


Difference
from Difference
Average Kilograms from
Fabricating of of Materials Average of
Costs $18,300 Used 470 (4) Squared (4) x (2) (2) Squared
$15,600 (2,700) 360 (110) 12,100 $297,000 $ 7,290,000
18,000 (300) 463 (7) 49 2,100 90,000
17,100 (1,200) 412 (58) 3,364 69,600 1,440,000
21,300 3,000 595 125 15,625 375,000 9,000,000
19,500 1,200 520 50 2,500 60,000 1,440,000
$91,500 0 2,350 0 33,638 $803,700 $19,260,000

Column 6 total $803,700


= = $23. 89 variable rate per kg .
Column 5 total 33,638

y = a + bx
$18,300 = a + ($23.89 x 470)
a = $18,300 - $11,228.30
a = $7,071.70
Equation: y = $7,071.70 + $23.89x
Cost Behavior Analysis 34

(2)

(1) (2) (3) (4) (5)


Kilograms of Prediction Prediction
Materials Fabricating Predicted Error Error Squared
Used Costs Fabricating Costs (2) - (3) (4) Squared
360 $15,600 $15,672 $ (72) $ 5,184
463 18,000 18,133 (133) 17,689
412 17,100 16,914 186 34,596
595 21,300 21,286 14 196
520 19,500 19,495 5 25
2,350 $91,500 $91,500 $ 0 $57,690

Σ(yi y bar)2
( ) ( ) ()
square root
n2 squar e r oot
Column 5 total
n⋅ 2
¿ ¿¿= squar e r o ot
$57,690
5 ⋅2
¿ ¿¿
¿
(3)

1 ( xi⋅ x bar )2
square root 1 +
n
+
(
Σ( x i⋅ x bar )
2 )
1 ( 500 ⋅ 470 )2
= square root 1 +
5
+ ( 33,638 )
(4)
Cost Behavior Analysis 35

(xi⋅x baryi¿bar) 2 $803,70 $803,70 2


r = 2 bar) ¿ ¿ = = .9 85 ¿ ¿r = .9 7¿ ¿
square ro txibar) yi¿ square ro t $804,902
Cost Behavior Analysis 32

Chapter 4

COST SYSTEMS AND COST ACCUMULATION

MULTIPLE CHOICE

Question Nos. 7, 9, 12-14, and 16 are AICPA adapted.


Question Nos. 8, 10, 11, and 15 are CIA adapted.

E 1. The tie-in between general accounts and cost accounts is often discussed with accounting
procedures. An example of a general account is:
A. Materials
B. Work in Process
C. Factory Overhead Control
D. Finished Goods
E. Accumulated Depreciation

C 2. One feature of a standard cost system is that:


A. selection of the cost unit becomes simplified
B. predetermined amounts are ignored
C. an analysis of cost variances is facilitated
D. historical costs are recorded as they are incurred
E. reports are delayed until operations have been performed

A 3. An industry that would most likely use job order costing procedures is:
A. road building
B. fertilizer manufacturing
C. flour milling
D. petroleum refining
E. textile manufacturing

D 4. An industry that would most likely use process costing procedures is:
A. musical instrument manufacturing
B. construction
C. aircraft
D. chemicals
E. office equipment

A 5. Supplies needed for use in the factory are issued on the basis of:
A. materials requisitions
B. time tickets
C. factory overhead analysis sheets
D. clock cards
E. purchase invoices

32
Cost Systems and Cost Accumulation 33

D 6. Finished Goods is debited and Work in Process is credited for a:


A. transfer of materials to the factory
B. return of unused materials from the factory
C. purchase of goods on account
D. transfer of completed production
E. transfer of completed goods out of the factory

B 7. The best cost accumulation procedure to use when many batches, each differing as to product
specifications, are produced is:
A. absorption
B. job order
C. process
D. actual
E. standard

A 8. Job order costs are most useful for:


A. determining the cost of a specific project
B. determining the labor cost involved in production
C. determining inventory valuation using lifo
D. estimating overhead costs
E. controlling indirect costs of future production

E 9. Under a job order cost system, the dollar amount of the entry to transfer the inventory from Finished
Goods to Cost of Goods Sold is the sum of the costs charged to all jobs:
A. completed during the period
B. started in process during the period
C. in process during the period
D. completed and sold during the period
E. sold during the period

A 10. The industry most likely to use job order costing in accounting for costs is:
A. accounting firms
B. textile manufacturer
C. paint manufacturer
D. oil refinery
E. none of the above

A 11. Job order cost accounting systems and process accounting systems differ in the way:
A. costs are traced to cost objects
B. orders are taken and in the number of units in the orders
C. product profitability is determined and compared with planned costs
D. manufacturing processes can be accomplished and in the number of production runs that may
be performed in a year
E. none of the above

D 12. In a job order cost system, the distribution of direct labor costs usually are recorded as an increase
in:
A. Cost of Goods Sold
B. Factory Overhead Control
C. Finished Goods
D. Work in Process
E. none of the above
Cost Systems and Cost Accumulation 34

C 13. Process costing techniques should be used in assigning costs to products:


A. if the product is manufactured on the basis of each order received
B. when production is only partially completed during the accounting period
C. if the product is composed of mass-produced homogeneous units
D. whenever standard costing techniques should not be used
E. none of the above

A 14. A characteristic of a process costing system is:


A. partially processed inventory is restated in terms of completed units
B. costs are accumulated by order
C. it is used by a company manufacturing custom machinery
D. standard costs are not applicable
E. none of the above

C 15. The industry most likely to use process costing in accounting for costs is:
A. road builder
B. electrical contractor
C. airlines
D. automobile repair shop
E. none of the above

B 16. In the computation of manufacturing cost per equivalent unit, the weighted average method of
process costing considers:
A. current costs only
B. current costs plus cost of beginning work in process inventory
C. current costs plus cost of ending work in process inventory
D. current costs less cost of beginning work in process inventory
E. none of the above

B 17. The element of manufacturing cost that supports time tickets is:
A. materials
B. labor
C. factory overhead
D. all of the above
E. none of the above

C 18. The element of manufacturing cost that supports depreciation schedules is:
A. materials
B. labor
C. factory overhead
D. all of the above
E. none of the above

D 19. Work in Process is debited and Materials is credited for:


A. indirect materials requisitioned to production
B. the completion of work in process
C. the sale of completed product
D. direct materials requisitioned to production
E. materials returned to the storeroom
Cost Systems and Cost Accumulation 35

E 20. Factory Overhead Control is debited and Work in Process is credited for:
A. indirect materials requisitioned to production
B. the completion of work in process
C. the sale of completed product
D. direct materials requisitioned to production
E. none of the above

A 21. Products, operations, and processes costed on the basis of predetermined quantities of resources to be
used and predetermined prices of those resources are distinguishing characteristics of which:
A. standard cost system
B. historical cost system
C. process cost system
D. job order cost system
E. backflush cost system

D 22. The tax requirement that certain purchasing and storage costs be allocated to inventory is known as:
A. backflush costing
B. postdeduction
C. just-in-time
D. super absorption
E. none of the above

C 23. The manufacturing systems characterized by short setup times, moderate to low lead times, and very
low direct labor cost is:
A. manual systems
B. fixed automation systems
C. flexible manufacturing systems
D. process cost systems
E. job order cost systems

E 24. The cost accounting system noted for its lack of detailed tracking of work in process during the
accounting period is:
A. process costing
B. job order costing
C. standard costing
D. actual costing
E. backflush costing

D 25. Ziffel Company had the following account balances and results from operations for the month of
July: direct materials consumed, $10,400; direct labor, $8,000; factory overhead, $8,800; July 1,
work in process inventory, $2,400; July 31, work in process inventory, $1,800; finished goods
inventory, July 1, $1,200; finished goods inventory, July 31, $1,000. The total manufacturing cost
for the month of July was:
A. $27,800
B. $28,000
C. $18,400
D. $27,200
E. none of the above

SUPPORTING CALCULATION: $10,400 + $8,000 + $8,800 = $27,200


Cost Systems and Cost Accumulation 36

C 26. Ziffel Company had the following account balances and results from operations for the month of
July: direct materials consumed, $10,400; direct labor, $8,000; factory overhead, $8,800; July 1,
work in process inventory, $2,400; July 31, work in process inventory, $1,800; finished goods
inventory, July 1, $1,200; finished goods inventory, July 31, $1,000. The cost of goods
manufactured was:
A. $27,200
B. $28,000
C. $27,800
D. $26,600
E. none of the above

SUPPORTING CALCULATION: $27,200 + $2,400 - $1,800 = $27,800

B 27. Ziffel Company had the following account balances and results from operations for the month of
July: direct materials consumed, $10,400; direct labor, $8,000; factory overhead, $8,800; July 1,
work in process inventory, $2,400; July 31, work in process inventory, $1,800; finished goods
inventory, July 1, $1,200; finished goods inventory, July 31, $1,000. The cost of goods sold was:
A. $27,200
B. $28,000
C. $27,800
D. $27,600
E. none of the above

SUPPORTING CALCULATION: $27,800 + $1,200 - $1,000 = $28,000

A 28. A cost system where only the variable manufacturing costs are allocated to production is:
A. direct costing
B. prime costing
C. absorption costing
D. standard costing
E. none of the above

D 29. A cost system where all manufacturing cost elements are allocated to production is:
A. direct costing
B. prime costing
C. standard costing
D. full absorption costing
E. none of the above

C 30. A cost system where only direct material and direct labor costs are allocated to production is:
A. direct costing
B. standard costing
C. prime costing
D. full absorption costing
E. none of the above
Cost Systems and Cost Accumulation 37

B 31. The manufacturing systems that are characterized by very high setup times, moderate lead times, and
high direct labor cost are:
A. flexible manufacturing systems
B. fixed automation systems
C. manual systems
D. backflush systems
E. none of the above
Cost Systems and Cost Accumulation 38

PROBLEMS

PROBLEM

1.
Computation of Total Manufacturing Cost, Cost of Goods Manufactured, and Cost of Goods Sold. During
the past year, the Rocco Company incurred these costs: direct labor, $2,500,000; factory overhead, $4,000,000;
and direct materials purchases, $1,500,000. Inventories were costed as follows:

Beginning Ending
Finished goods........................................................................................... $250,000 $300,000
Work in process......................................................................................... 450,000 550,000
Materials.................................................................................................... 75,000 125,000

Required:

(1) Calculate total manufacturing cost for the year.


(2) Calculate the cost of goods manufactured for the year.
(3) Calculate the cost of goods sold for the year.

SOLUTION

(1) Direct materials:


Materials inventory, beginning................................................ $ 75,000
Purchases.................................................................................. 1,500,000
Materials available for use....................................................... $ 1,575,000
Less raw materials inventory, ending...................................... 125,000
Direct materials consumed....................................................... $ 1,450,000
Direct labor..................................................................................... 2,500,000
Factory overhead............................................................................. 4,000,000
Total manufacturing costs.............................................................. $ 7,950,000

(2) Total manufacturing costs [from (1)]............................................. $ 7,950,000


Add work in process inventory, beginning.............................. 450,000
$ 8,400,000
Less work in process inventory, ending................................... 550,000
Cost of goods manufactured........................................................... $ 7,850,000

(3) Cost of goods manufactured [from (2)].......................................... $ 7,850,000


Add finished goods inventory, beginning................................ 250,000
Cost of goods available for sale..................................................... $ 8,100,000
Less finished goods inventory, ending..................................... 300,000
Cost of goods sold........................................................................... $ 7,800,000
Cost Systems and Cost Accumulation 39

PROBLEM

2.
Journal Entries for the Cost Accounting Cycle. On January 1, the ledger of the Phinney Furniture Company
contained, among other accounts, the following: Finished Goods, $25,000; Work in Process, $30,000; Materials,
$15,000. During January, the following transactions were completed:

(a) Materials were purchased at a cost of $28,000.


(b) Direct materials in the amount of $21,000 were issued from the storeroom.
(c) Storeroom requisitions for indirect materials and supplies amounted to $3,200.
(d) The total payroll for January amounted to $31,000, including marketing salaries of $7,500 and
administrative salaries of $5,500. Labor time tickets show that $15,500 of the labor cost was direct labor.
(e) Various factory overhead costs were incurred for $12,000 on account.
(f) Total factory overhead is charged to the work in process account.
(g) Cost of production completed in January totaled $58,000, and finished goods in the shipping room on
January 31 totaled $18,000.
(h) Customers to whom shipments were made during the month were billed for $88,000. (Also record entry for
cost of goods sold.)

Required: Prepare journal entries for the transactions, including the recording, payment, and distribution of the
payroll.

SOLUTION

(a) Materials.......................................................................................... 28,000


Accounts Payable..................................................................... 28,000

(b) Work in Process.............................................................................. 21,000


Materials................................................................................... 21,000

(c) Factory Overhead Control.............................................................. 3,200


Materials................................................................................... 3,200

(d) Payroll............................................................................................. 31,000


Accrued Payroll........................................................................ 31,000

Accrued Payroll.............................................................................. 31,000


Cash.......................................................................................... 31,000

Work in Process.............................................................................. 15,500


Factory Overhead Control.............................................................. 2,500
Marketing Expenses Control.......................................................... 7,500
Administrative Expenses Control................................................... 5,500
Payroll....................................................................................... 31,000

(e) Factory Overhead Control.............................................................. 12,000


Accounts Payable..................................................................... 12,000

(f) Work in Process.............................................................................. 17,700


Factory Overhead Control........................................................ 17,700
Cost Systems and Cost Accumulation 40

(g) Finished Goods............................................................................... 58,000


Work in Process........................................................................ 58,000

(h) Accounts Receivable...................................................................... 88,000


Sales.......................................................................................... 88,000

Cost of Goods Sold (25,000 + 58,000 - 18,000)............................ 65,000


Finished Goods......................................................................... 65,000

PROBLEM

3.
Cost of Goods Manufactured Statement. Cuervo Company manufacturers file cabinets made to consumer
specifications. The following information was available at the beginning of March:

Materials inventory................................................................................... $12,800


Work in process inventory........................................................................ 4,700
Finished goods inventory.......................................................................... 2,300

During March, materials costing $26,000 were purchased, direct labor cost totaled $19,300, and factory overhead
was $12,500 (including $2,500 of indirect materials). March 31 inventories were:

Materials inventory................................................................................... $13,300


Work in process inventory........................................................................ 6,800
Finished goods inventory.......................................................................... 2,800

Required: Prepare a cost of goods manufactured statement for March, 19--. (AICPA adapted)

SOLUTION

Cuervo Company
Cost of Goods Manufactured Statement
For the Month Ended March 31, 19--

Direct materials:
Materials inventory, March 1......................................... $ 12,800
Purchases........................................................................ 26,000
Materials available for use............................................. $ 38,800
Less:...........................................Indirect materials used $ 2,500
Materials inventory, March 31................................. 13,300 15,800
Direct materials consumed............................................. $ 23,000
Direct labor........................................................................... 19,300
Factory overhead.................................................................. 12,500
Total manufacturing cost...................................................... $ 54,800
Add work in process inventory, March 1............................. 4,700
$ 59,500
Less work in process inventory, March 31.......................... 6,800
Cost of goods manufactured................................................. $ 52,700
Cost Systems and Cost Accumulation 41

PROBLEM

4.
Income Statement Relationships. The following data are available for three companies at the end of their fiscal
years:

Company Alpha:
Finished goods, April 1................................................................... $ 400,000
Cost of goods manufactured...................................................... 2,600,000
Sales........................................................................................... 3,500,000
Gross profit on sales.................................................................. 35%
Finished goods inventory, March 31........................................ ?

Company Beta:
Freight in.......................................................................................... $ 12,000
Purchases returns and allowances............................................. 22,000
Marketing expense.................................................................... 85,000
Finished goods, December 31................................................... 65,000
Cost of goods sold..................................................................... 550,000
Cost of goods available for sale................................................ ?

Company Chi:
Gross profit...................................................................................... $ 264,000
Cost of goods manufactured...................................................... 612,000
Finished goods, January 1......................................................... 34,000
Finished goods, December 31................................................... 26,000
Work in process, January 1....................................................... 18,000
Work in process, December 31................................................. 12,000
Sales........................................................................................... ?

Required: Determine the amounts indicated by the question marks. (AICPA adapted)
Cost Systems and Cost Accumulation 42

SOLUTION

Company Alpha:
Sales................................................................................................. $ 3,500,000
Cost of goods sold:
Finished goods inventory, April 1............................................ $ 400,000
Cost of goods manufactured...................................................... 2,600,000
Cost of goods available for sale................................................ $ 3,000,000
Finished goods inventory, March 31........................................ 725,000
Less cost of goods sold.............................................................. 2,275,000
Gross profit (20% of sales).............................................................. $ 1,225,000

Company Beta:
Cost of goods available for sale...................................................... $ 615,000
Less finished goods ending inventory............................................. 65,000
Cost of goods sold........................................................................... $ 550,000

Company Chi:
Sales................................................................................................. $ 884,000
Cost of goods sold:
Cost of goods manufactured...................................................... $ 612,000
Add beginning finished goods inventory.................................. 34,000
Cost of goods available for sale................................................ $ 646,000
Less ending finished goods inventory....................................... 26,000
Less cost of goods sold.............................................................. 620,000
Gross profit...................................................................................... $ 264,000

PROBLEM

5.
Cost of Goods Manufactured; Prime and Conversion Costs. Wyoming Company's purchases of materials
during June totaled $25,000, and the cost of goods sold for June was $130,000. Factory overhead was 200% of
direct labor cost. Other information pertaining to Wyoming Company's inventories and production for June is as
follows:

Inventories Beginning Ending


Finished goods....................................................................................... $42,500 $39,000
Work in process..................................................................................... 15,500 17,000
Materials................................................................................................ 5,000 8,500

Required:

(1) Prepare a schedule of cost of goods manufactured.


(2) Compute the prime cost charged to Work in Process.
(3) Compute the conversion cost charged to Work in Process. (AICPA adapted)
Cost Systems and Cost Accumulation 43

SOLUTION

(1) Wyoming Company


Schedule of Cost of Goods Manufactured
For Month Ended June 30, 19--

Work in process, June 1......................................................................... $ 15,500


Production costs:
Direct materials............................................................................... 21,500 **
Direct labor...................................................................................... 35,500
Factory overhead............................................................................. 71,000 128,000
$ 143,500
Less work in process, June 30............................................................... 17,000
Cost of goods manufactured.................................................................. $ 126,500 *

Let x = direct labor


3x = $106,500
x = $35,500 direct labor
2x = $71,000 factory overhead

(2) Prime cost:


Direct materials [from (1)]..................................................... $ 21,500
Direct labor [from (1)]........................................................... 35,500
$ 57,000
(3) Conversion cost:
Direct labor [from (1)]........................................................... $ 35,500
Factory overhead [from (1)]................................................... 71,000
$ 106,500 ***

* Cost of goods sold ($130,000) + ending finished goods inventory ($39,000) - beginning finished goods
inventory ($42,500) = $126,500.

** Purchases of materials during June ($25,000) + beginning materials inventory ($5,000) - ending materials
inventory ($8,500) = $21,500.

*** Production costs for June ($128,000) - direct materials ($21,500) = direct labor and factory overhead
($106,500).

PROBLEM

6.
Cost of Goods Manufactured and Sold. For May, Jimbo Inc. had cost of goods manufactured equal to $90,000;
direct materials used $30,000; cost of goods sold, $100,000; direct labor, $38,000; purchases of materials,
$40,000; cost of goods available for sale, $125,000; and total factory labor, $48,000. Work in process was
$25,000 on May 1 and $15,000 on May 31. The company uses a single materials account for direct and indirect
materials.
Cost Systems and Cost Accumulation 44

Required: Prepare the following:

(1) A cost of goods sold statement. For brevity, show single-line items for factory overhead and direct materials
used.
(2) Summary general journal entries to record:

(a) purchase of materials on account


(b) use of materials, including direct materials of 1,000
(c) accrual of factory payroll, including indirect labor of $10,000 (use a payroll clearing account)
(d) distribution of factory labor cost
(e) transfer of completed work to finished goods
(f) sales on account, at a markup equal to 100% of production cost

SOLUTION

(1) Jimbo Inc.


Cost of Goods Sold Statement
For Month Ended May 31, 19--
(in thousands)

Direct materials consumed.................................................................... $ 30


Direct labor............................................................................................ 38
Factory overhead................................................................................... 12
Total manufacturing cost [Note (a)]..................................................... $ 80
Add work in process inventory, May 1................................................. 25
$ 105
Less work in process inventory, May 31.............................................. 15
Cost of goods manufactured.................................................................. $ 90
Add finished goods inventory, May 1 [Note (b)]................................. 35
Cost of goods available for sale............................................................ $ 125
Less finished goods inventory, May 31 [Note (c)]............................... 25
Cost of goods sold................................................................................. $ 100

Note (a): Cost of goods manufactured............................................... $ 90


Add work in process, ending.............................................. 15
$ 105
Less work in process, beginning......................................... 25
Equals total manufacturing cost.......................................... $ 80

Note (b): Cost of goods available for sale.......................................... $ 125


Less cost of goods manufactured........................................ 90
Equals finished goods, beginning....................................... $ 35

Note (c): Cost of goods available for sale.......................................... $ 125


Less cost of goods sold....................................................... 100
Equals finished goods, ending............................................ $ 25
Cost Systems and Cost Accumulation 45

(2)

(a) Materials....................................................................................... 40,000


Accounts Payable................................................................... 40,000

(b) Work in Process............................................................................ 30,000


Factory Overhead Control............................................................ 1,000
Materials................................................................................. 31,000

(c) Payroll ($38,000 + $10,000)........................................................ 48,000


Accrued Payroll...................................................................... 48,000

(d) Work in Process............................................................................ 38,000


Factory Overhead Control............................................................ 10,000
Payroll..................................................................................... 48,000

(e) Finished Goods............................................................................. 90,000


Work in Process...................................................................... 90,000

(f) Accounts Receivable.................................................................... 200,000


Sales [$100,000 + (100% of $100,000)]................................ 200,000

Cost of Goods Sold....................................................................... 100,000


Finished Goods....................................................................... 100,000
Cost Systems and Cost Accumulation 46

JOB ORDER COSTING


MULTIPLE CHOICE
Question Nos. 4, 5, 9-13, and 18 are AICPA adapted.
Question Nos. 7, 8, and 14-17 are ICMA adapted.

A 1. Under job order cost accumulation, the factory overhead control account controls:
A. factory overhead analysis sheets
B. all general ledger subsidiary accounts
C. job order cost sheets
D. cost reports by processes
E. materials inventories
B 2. Supplies needed for use in the factory are issued on the basis of:
A. job cost sheets
B. materials requisitions
C. time tickets
D. factory overhead analysis sheets
E. clock cards
B 3. Finished Goods is debited and Work in Process is credited for a:
A. transfer of completed goods out of the factory
B. transfer of completed production to the finished goods storeroom
C. purchase of goods on account
D. transfer of materials to the factory
E. return of unused materials from the factory
A 4. In job order costing, when materials are returned to the storekeeper that were previously issued to
the factory for cleaning supplies, the journal entry should be made to:
A. Materials
Factory Overhead
B. Materials
Work in Process
C. Purchases Returns
Work in Process
D. Work in Process
Materials
E. Factory Overhead
Work in Process

46
Job Order Costing 47

A 5. Under a job order cost system, the dollar amount of the entry to transfer the inventory from Work in
Process to Finished Goods is the sum of the costs charged to all jobs:
A. completed during the period
B. started in process during the period
C. in process during the period
D. completed and sold during the period
E. none of the above
B 6. When a manufacturing company has a highly automated plant producing many different products,
probably the most appropriate basis of applying factory overhead costs to Work in Process is:
A. units processed
B. machine hours
C. direct labor hours
D. direct labor dollars
E. none of the above
A 7. Cherokee Company applies factory overhead on the basis of direct labor hours. Budget and actual
data for direct labor and overhead for the year are as follows:
Budget Actual
Direct labor hours............................................................................. 600,000 650,000
Factory overhead costs..................................................................... $720,000 $760,000
The factory overhead for Cherokee for the year is:
A. overapplied by $20,000
B. overapplied by $40,000
C. underapplied by $20,000
D. underapplied by $40,000
E. neither underapplied nor overapplied

SUPPORTING CALCULATION:

$720,000
= $1. 20 _ 650,000
600,000

= $780,000 (applied )⋅ $760,000 (actual ) = $20,000 (overapplied)


Job Order Costing 48

C 8. At the end of the year, Paola Company had the following account balances after applied factory
overhead had been closed to Factory Overhead Control:
Factory Overhead Control........................................................................................ $ 1,000 CR
Cost of Goods Sold................................................................................................... 980,000 DR
Work in Process........................................................................................................ 38,000 DR
Finished Goods.......................................................................................................... 82,000 DR
The most common treatment of the balance in Factory Overhead Control would be to:
A. carry it as a deferred credit on the balance sheet
B. report it as miscellaneous operating revenue on the income statement
C. credit it to Cost of Goods Sold
D. prorate it between Work in Process and Finished Goods
E. prorate it among Work in Process, Finished Goods, and Cost of Goods Sold
B 9. Overapplied factory overhead would result if:
A. the plant were operated at less than normal capacity
B. factory overhead costs incurred were less than costs charged to production
C. factory overhead costs incurred were unreasonably large in relation to units produced
D. factory overhead costs incurred were greater than costs charged to production
E. a firm incurred a significant amount of overhead
A 10. The Waitkins Company estimated Department A's overhead at $255,000 for the period based on an
estimated volume of 100,000 direct labor hours. At the end of the period, the factory overhead
control account for Department A had a balance of $265,500; actual direct labor hours were 105,000.
What was the over- or under-applied overhead for the period?
A. $2,250
B. $(2,250)
C. $15,000
D. $(15,000)
E. $(5,000)
SUPPORTING CALCULATION:

$255,000
= $2 .55 _ 105,000 = $267,750 (applied )⋅ $265,500 (actual )
100,000

= $2,250 (overapplied )
Job Order Costing 49

D 11. Howell Corporation has a job order cost system. The following debits (credits) appeared in Work in
Process for the month of July:
July 1, balance............................................................................................................... $ 12,000
July 31, direct materials................................................................................................. 40,000
July 31, direct labor....................................................................................................... 30,000
July 31, factory overhead.............................................................................................. 27,000
July 31, to finished goods.............................................................................................. (100,000)
Howell applies overhead to production at a predetermined rate of 90% based on the direct labor cost.
Job 1040, the only job still in process at the end of July, has been charged with factory overhead of
$2,250. What was the amount of direct materials charged to Job 1040?
A. $6,750
B. $2,250
C. $2,500
D. $4,250
E. $9,000

SUPPORTING CALCULATION:

$2,250
Direct materials = $9,000 ⋅ ⋅ $2,250 = $4,250
.9
Job 1040 = $12,000 + $40,000 + $30,000 + $27,000 - $100,000 = $9,000

E 12. Valentino Corporation makes aluminum fasteners. Among Valentino's 19-- manufacturing costs
were:
Wages and salaries:
Machine operators................................................................................................. $80,000
Factory supervisors................................................................................................ 30,000
Machine mechanics............................................................................................... 20,000

Direct labor amounted to:


A. $50,000
B. $100,000
C. $110,000
D. $130,000
E. none of the above
Job Order Costing 50

B 13. Rudolpho Corporation makes aluminum fasteners. Among Rudolpho's 19-- manufacturing costs
were:

Materials and supplies:


Aluminum.................................................................................................................... $400,000
Machine parts............................................................................................................... 18,000
Lubricants for machines.............................................................................................. 5,000
Direct materials amounted to:
A. $23,000
B. $400,000
C. $405,000
D. $418,000
E. $423,000
C 14. Selected cost data (in thousands) concerning the past fiscal year's operations of the Moscow
Manufacturing Company are presented below.

Inventories
Beginning Ending
Materials........................................................................................... $75 $ 85
Work in process................................................................................ 80 30
Finished goods.................................................................................. 90 110

Materials used, $326


Total manufacturing costs charged to production during the year (including direct materials, direct
labor, and factory overhead applied at the rate of 60% of direct labor cost), $686
Cost of goods available for sale, $826
Selling and general expenses, $25
The cost of direct materials purchased during the year amounted to:
A. $360
B. $316
C. $336
D. $411
E. none of the above

SUPPORTING CALCULATION: $326 + $85 - $75 = $336


Job Order Costing 51

C 15. Selected cost data (in thousands) concerning the past fiscal year's operations of the Moscow
Manufacturing Company are presented below.

Inventories
Beginning Ending
Materials........................................................................................... $75 $ 85
Work in process................................................................................ 80 30
Finished goods.................................................................................. 90 110

Materials used, $326


Total manufacturing costs charged to production during the year (including direct materials, direct
labor, and factory overhead applied at the rate of 60% of direct labor cost), $686
Cost of goods available for sale, $826
Selling and general expenses, $25

Direct labor costs charged to production during the year amounted to:
A. $216
B. $135
C. $225
D. $360
E. none of the above

SUPPORTING CALCULATION: $686 = $326 + x + .6x


x = $225

A 16. Selected cost data (in thousands) concerning the past fiscal year's operations of the Moscow
Manufacturing Company are presented below.
Inventories
Beginning Ending
Materials........................................................................................... $75 $ 85
Work in process................................................................................ 80 30
Finished goods.................................................................................. 90 110

Materials used, $326


Total manufacturing costs charged to production during the year (including direct materials, direct
labor, and factory overhead applied at the rate of 60% of direct labor cost), $686
Cost of goods available for sale, $826
Selling and general expenses, $25

The cost of goods manufactured during the year was:


A. $736
B. $716
C. $636
D. $766
E. none of the above

SUPPORTING CALCULATION: $80 + $686 - $30 = $7


Job Order Costing 52

A 17. Selected cost data (in thousands) concerning the past fiscal year's operations of the Moscow
Manufacturing Company are presented below.
Inventories
Beginning Ending
Materials........................................................................................... $75 $ 85
Work in process................................................................................ 80 30
Finished goods.................................................................................. 90 110

Materials used, $326


Total manufacturing costs charged to production during the year (including direct materials, direct
labor, and factory overhead applied at the rate of 60% of direct labor cost), $686
Cost of goods available for sale, $826
Selling and general expenses, $25
The cost of goods sold during the year was:
A. $716
B. $691
C. $801
D. $736
E. none of the above

SUPPORTING CALCULATION: $90 + $736 - $110 = $716


A 18. J. D. Doonesbury Company manufactures tools to customer specifications. The following data
pertain to Job 1501 for April:
Direct materials used....................................................................................................... $ 4,200
Direct labor hours worked............................................................................................... 300
Direct labor rate per hour................................................................................................ $ 8.00
Machine hours used......................................................................................................... 200
Applied factory overhead rate per machine hour........................................................... $ 15.00
What is the total manufacturing cost recorded on Job 1501 for April?
A. $9,600
B. $10,300
C. $11,100
D. $5,400
E. $8,800

SUPPORTING CALCULATION: $4,200 + (300 x $8) + (200 x $15) = $9,600

C 19. In service businesses using job order costing, the most commonly used base for applying overhead to
jobs is:
A. machine hours
B. direct materials consumed
C. direct labor cost
D. meals, travel, and entertainment
E. none of the above
Job Order Costing 53

A 20. In service businesses using job order costing, the hourly rate used to charge costs to a job usually
includes:
A. both labor and overhead cost
B. labor cost only
C. overhead cost only
D. labor, overhead, and miscellaneous costs
E. none of the above

A 21. Work in Process is debited and Materials is credited for:


A. the issuance of direct materials into production
B. the issuance of indirect materials into production
C. the return of materials to the storeroom
D. the application of materials overhead
E. none of the above

B 22. Factory Overhead Control is debited and Payroll is credited for:


A. the recording of payroll
B. the distribution of indirect labor costs
C. the distribution of direct labor costs
D. the distribution of withholding taxes
E. none of the above

A 23. Applied Factory Overhead is debited and Factory Overhead is credited to:
A. close the estimated overhead account to actual overhead
B. record the actual factory overhead for the period
C. charge estimated overhead to all jobs worked on during the period
D. to record overapplied overhead for the period
E. none of the above

C 24. The best overhead allocation base to use in a labor-intensive manufacturing environment probably
would be:
A. materials cost
B. machine hours
C. direct labor hours
D. units of production
E. none of the above

D 25. Finished Goods is debited and Cost of Goods Sold is credited for:
A. transfer of completed goods to the customer
B. sale of a customer order
C. return of materials to the supplier
D. return of goods by the customer
E. none of the above
Job Order Costing 54

PROBLEMS
1.
Job Order Cost Schedule. Winkel Woodcrafters produces special-order wood products. The company uses job
order costing for pricing and cost accumulation purposes. The following costs were incurred on two recent jobs:

Cost Item........................................................ Job Pine-20 Job Birch-10


Direct materials:
Issued........................................................ $6,500 $8,000
Returned................................................... 500 0
Indirect materials used.................................. 500 400
Direct labor.................................................... $9,000 $15,000
Direct labor rate............................................. $9 per hour $10 per hour
Overhead application rate............................. $10 per direct labor hour $15 per direct labor hour

The company adds a 50% markup on cost in determining the amount to charge for each job.

Required: Prepare a schedule showing the cost and the amount to be charged for each job.

SOLUTION

Job Pine-20 Job Birch -10


Direct materials....................................................................................... $ 6,000 $ 8,000
Direct labor.............................................................................................. 9,000 15,000
Factory overhead applied ....................................................................... 10,000 22,500
Total................................................................................................... $ 25,000 $ 45,500
Allowance for profit and other costs...................................................... 12,500 22,750
Amount to be charged............................................................................. $ 37,500 $ 68,250

PROBLEM
2. Job Order Cost Sheet; Over- or Underapplied Overhead. During June, the following transactions took
place at the Cassandran Corp.

June 3 Purchased materials, $30,000.


5 Requisitioned materials from inventory, $20,000 (75% of these were direct; 25% were indirect).
Direct materials of $3,000 and indirect materials of $1,000 were for Job 00-1. The remainder were
for Job 00-2.
7 For Job 00-2, returned $150 of direct materials and $200 of indirect materials.
8 Recorded liabilities for payroll: direct labor, $15,000 and indirect labor, $5,000. Of the direct labor
cost, 60% was for Job 00-1; the remainder was for Job 00-2.
10 Incurred other factory overhead costs, $20,000 (all applicable to Jobs 00-1 and 00-2).
14 Applied overhead at the rate of 200% of direct labor cost to Jobs 00-1 and 00-2, which were
completed and transferred to finished goods account today.
Job Order Costing 55

Required: Assuming that Jobs 00-1 and 00-2 were the only jobs during the period and that all overhead (as
recorded above) is the total applicable overhead for these projects:

(1) Prepare a job order cost sheet for each job.


(2) Determine the difference between applied and actual overhead for the month.

SOLUTION

(1)
Job 00-1 Job 00-2
Materials........................................................................................................................ $ 3,000 $ 11,850
Labor.............................................................................................................................. 9,000 6,000
Overhead applied........................................................................................................... 18,000 12,000
Total cost........................................................................................................................ $ 30,000 $ 29,850

(2) Analysis of Factory Overhead


Incurred:
Indirect materials..................................................................................................... $ 4,800
Indirect labor........................................................................................................... 5,000
Other overhead incurred.......................................................................................... 20,000 $ 29,800
Applied:
Job 00-1................................................................................................................... $ 18,000
Job 00-2................................................................................................................... 12,000 30,000
Amount overapplied ..................................................................................................... $ (200 )

PROBLEM

3.
Job Order Cycle Entries. The following completed cost sheets were prepared for three jobs that were in
production during April in the Special Order Division of Byron Company:

Job 097 Job 781 Job 946


Direct materials................................................................................... $ 6,000 $2,700 $4,100
Direct labor......................................................................................... 9,200 7,300 8,200
Applied factory overhead................................................................... 6,900 5,475 6,120
Allowance for commercial expenses and profit................................ 11,050 7,738 9,210

On April 1, Job 097 was 75% complete as to materials, labor, and overhead. It was finished during the month.
The other jobs were started and finished during the month. Jobs 097 and 946 were sold on account at the end of
the month.

Required: Prepare general journal entries to be recorded in April to accumulate these job costs for Work in
Process as well as for Finished Goods and for the sale of the two jobs.
Job Order Costing 56

SOLUTION

Debit Credit
Work in Process........................................................................................................ 8,300 *
Materials............................................................................................................. 8,300

Work in Process........................................................................................................ 17,800 **


Accrued Payroll.................................................................................................. 17,800

Work in Process........................................................................................................ 13,320 ***


Factory Overhead Control.................................................................................. 13,320

Finished Goods.......................................................................................................... 55,995


Work in Process.................................................................................................. 55,995

Cost of Goods Sold................................................................................................... 40,520


Finished Goods................................................................................................... 40,520

Accounts Receivable................................................................................................. 60,780


Sales.................................................................................................................... 60,780

* (.25 x $6,000) + $2,700 + $4,100


** (.25 x $9,200) + $7,300 + $8,200
*** (.25 x $6,900) + $5,475 + $6,120

PROBLEM

4.
Voyager Inc. produces customized vans in a job order shop. On November 1, the following balances appear in the
inventory records:

Finished goods........................................................................................................................................ $179,000


Work in process...................................................................................................................................... 308,000
Materials.................................................................................................................................................. 83,000

The amount in Finished Goods represents $101,000 recorded for Van 175 and $78,000 recorded for Van 177.
The work in process account represents the three vans in process, as follows:

Van 179 Van 180 Van 181


Factory overhead................................................................................ $75,000 $50,000 $25,000
Direct labor......................................................................................... 60,000 40,000 20,000
Direct materials................................................................................... 26,000 7,000 5,000
Job Order Costing 57

The following transactions occurred during November:


(a) Purchased materials on account, $80,000.
(b) Requisitioned $60,000 of materials from inventory: $15,000 applied to Van 180, $25,000 to Van 181, and
$16,000 to Van 182, a new order; the balance was for indirect materials.
(c) Recorded the liability for the payroll and the labor cost distribution in a single entry: total payroll,
$208,750. Of the payroll cost, 10% applied to Van 179, 20% to Van 180, 35% to Van 181, 30% to Van
182, and the remainder to indirect labor.
(d) Paid the payroll.
(e) Applied factory overhead at the rate of 150% of direct labor cost.
(f) Completed Vans 179 and 180.
(g) Sold Vans 175, 177, and 180 at 50% over manufacturing costs.

Required: Prepare general journal entries to record these transactions.

SOLUTION

Debit Credit
(a) Materials.......................................................................................................... 80,000
Accounts Payable.................................................................................... 80,000

(b) Factory Overhead Control............................................................................... 4,000


Work in Process.............................................................................................. 56,000
Materials.................................................................................................. 60,000

(c) Factory Overhead Control............................................................................... 10,437


Work in Process.............................................................................................. 198,313
Accrued Payroll....................................................................................... 208,750

(d) Accrued Payroll............................................................................................... 208,750


Cash......................................................................................................... 208,750

(e) Work in Process.............................................................................................. 297,470


Applied Factory Overhead...................................................................... 297,470

(f) Finished Goods................................................................................................ 429,563


Work in Process....................................................................................... 429,563

(g) Accounts Receivable....................................................................................... 593,063


Sales......................................................................................................... 593,063

Cost of Goods Sold......................................................................................... 395,375


Finished Goods........................................................................................ 395,375
Job Order Costing 58

PROBLEM

5.
Manufacturing Costs. The work in process account of Meyers Company showed:

Work in Process

Materials $22,000 | Finished goods $68,000


Direct labor 37,000 |
Factory overhead 55,500 |

Materials charged to the one job still in process amounted to $5,000. Factory overhead is applied as a
predetermined percentage of direct labor cost.

Required: Compute the following:

(1) The amount of direct labor cost in finished goods.


(2) The amount of factory overhead in finished goods.

SOLUTION

(1) The amount of direct labor in finished goods:


Finished goods............................................................................................................................. $68,000
Materials included in finished goods........................................................................................... 17,000
Direct labor and factory overhead in finished goods.................................................................. $51,000

Factory overhead charged to work in process $55,500


= = 1.5
Direct labor charged to work in process $37,000

Let x = direct labor in finished goods


2.5x = $51,000 direct labor and factory overhead in finished goods
x = $20,400 direct labor in finished goods

(2) The amount of factory overhead in finished goods:

x = $20,400
1.5x = 1.5($20,400)
1.5x = $30,600 factory overhead in finished goods

PROBLEM

6.
Manufacturing Costs. Teddy Company is to submit a bid on the production of 5,500 vases. It is estimated that
the cost of materials will be $8,500, and the cost of direct labor will be $12,000. Factory overhead is applied at
50% of direct labor cost in the Molding Department and at $7.50 per direct labor hour in the Finishing
Department. Of the above direct labor, it is estimated that 500 direct labor hours at a cost of $4,000 will be
required in Finishing. The company wishes a markup of 100% of its total production cost.
Job Order Costing 59

Required: Determine the following:

(1) Estimated cost to produce.


(2) Estimated prime cost.
(3) Estimated conversion cost.
(4) Bid price.

SOLUTION

(1) Materials................................................................................................................................ $ 8,500


Direct labor............................................................................................................................ 12,000
Factory overhead:
Molding (50% x $8,000)................................................................................................. 4,000
Finishing (500 DLH x $7.50).......................................................................................... 3,750
Estimated cost to produce..................................................................................................... $ 28,250

(2) Materials................................................................................................................................ $ 8,500


Direct labor............................................................................................................................ 12,000
Estimated prime cost............................................................................................................. $ 20,500

(3) Direct labor............................................................................................................................ $ 12,000


Factory overhead................................................................................................................... 7,750
Estimated conversion cost..................................................................................................... $ 19,750

(4) Estimated cost to produce..................................................................................................... $ 28,250


Markup ($28,250 x 100%).................................................................................................... 28,250
Bid price................................................................................................................................. $ 56,500

PROBLEM

7.
Flow of Costs Through T Accounts. The Palmer Company had the following inventories at the beginning and
end of July:

July 1 July 31
Materials.................................................................................................................. $20,000 $ 45,000
Work in process....................................................................................................... ? 185,000
Finished goods......................................................................................................... 65,000 115,000

During July, the cost of materials purchased was $160,000 and factory overhead of $125,000 was applied at a rate
of 75% of direct labor cost. July cost of goods sold was $240,000.

Required: Prepare completed T accounts showing the flow of the cost of goods manufactured and sold.
Job Order Costing 60

SOLUTION

Materials Work in Process


Inv. 20,000 | WIP135,000
Inv. 48,333 ** |FG290,000
Purch. 160,000 | Materials 135,000 |
180,000 | Factory |
45,000 | overhd. 125,000 |
| Labor 166,667 |
| 475,000 |
| 185,000 |

Finished Goods Cost of Goods Sold

Inv. 65,000 | CGS 240,000 FG 240,000 |


WIP 290,000* | |
355,000 | |
115,000 | |

CGA-Canada (adapted). Reprinted with permission.

*Beginning inventory + WIP = Ending inventory + CGS


$65,000 + WIP = $115,000 + $240,000
WIP = $290,000

**Beginning WIP + Mfg. costs = Ending WIP + FG


Beginning WIP + $426,667 = $185,000 + $290,000
Beginning WIP = $48,333 C:\WINDOWS\hinhem.scr
Job Order Costing 61

Chapter 6

PROCESS COSTING

MULTIPLE CHOICE

Question Nos. 1, 7-12, 18-23, and 28-32 are AICPA adapted.

B 1. An equivalent unit of material or conversion cost is equal to:


A. the prime cost
B. the amount of material or conversion cost necessary to complete one unit of production
C. a unit of work in process inventory
D. the amount of material or conversion cost necessary to start a unit of production into work in
process
E. 50% of the material or conversion cost of a unit of finished goods inventory, assuming a
linear production pattern

B 2. The product flow format where certain portions of the work are done simultaneously and then
brought together for completion is called:
A. applied
B. parallel
C. standard
D. selective
E. sequential

C 3. An item that does not appear on a cost of production report is:


A. work in processbeginning inventory
B. cumulative costs through the end of departmental production
C. finished goodsending inventory
D. materials used in the department
E. unit costs added by the department

C 4. Goode Manufacturing has three producing departments in its factory. The ending inventory in the
Milling Department consisted of 3,000 units. These units were 60% complete with respect to labor
and factory overhead. Materials are applied at the end of the milling process. Unit costs for the
complete process in the Milling Department are: materials, $1; labor, $2; and factory overhead, $3.
The appropriate unit cost for each unit in the ending inventory is:
A. $2
B. $5
C. $3
D. $6
E. $4

SUPPORTING CALCULATION: 60% ($2 + $3) = $3

61
Process Costing 62

D 5. When added materials in subsequent departments result in an increase of the units produced, the unit
transferred-in costs will:
A. be reclassified as new materials
B. be increased to provide for the additional units
C. be accounted for under the fifo costing method
D. be decreased as they are spread over more units
E. remain unchanged

E 6. Gyro Products transferred 10,000 units to one department. An additional 3,000 units of materials
were added in the department. At the end of the month, 7,000 units were transferred to the next
department. There was no beginning inventory. The costs for units transferred in would be
effectively allocated over:
A. 17,000 units
B. 3,000 units
C. 10,000 units
D. 7,000 units
E. 13,000 units

SUPPORTING CALCULATION: 7,000 units transferred out + 6,000 units in ending inventory
= 13,000 units

E 7. A characteristic of a process costing system is that:


A. costs are accumulated by order
B. it is used by a company manufacturing custom machinery
C. standard costs are not applicable
D. it requires a lot more detailed accounting than does a job order system
E. work in process inventory is restated in terms of completed units

D 8. Transferred-in costs as used in a process cost accounting system are:


A. supervisory salaries that are transferred from an overhead cost center to a production cost
center
B. ending work in process inventory of a previous process that will be used in a succeeding
process
C. labor that is transferred from another department within the same plant instead of hiring
temporary workers from the outside
D. costs of the product of a previous internal process that is subsequently used in a succeeding
internal process
E. none of the above

E 9. In a process costing system, how is the unit cost affected in a production cost report when materials
are added in a department subsequent to the first department and the added materials result in
additional units?
A. The first department's unit cost is increased, but it does not necessitate an adjustment of the
transferred-in unit cost.
B. The first department's unit cost is decreased, but it does not necessitate an adjustment of the
transferred-in unit cost.
C. The first department's unit cost is not affected.
D. The first department's unit cost is increased, which necessitates an adjustment of the
transferred-in unit cost.
E. The first department's unit cost is decreased, which necessitates an adjustment of the
transferred-in unit cost.
Process Costing 63

E 10. Assuming that there was no beginning work in process inventory and the ending work in process
inventory is 50% complete as to conversion costs, the number of equivalent units as to conversion
costs would be:
A. less than the units completed
B. more than the units completed
C. the same as the units placed in process
D. the same as the units completed
E. less than the units placed in process

A 11. An error was made in the computation of the percentage of completion of the current year's ending
work in process inventory. The error resulted in assigning a lower percentage of completion to each
component of the inventory than actually was the case. What is the effect of this error upon:
(1) the computation of equivalent units in total
(2) the computation of costs per equivalent unit
(3) costs assigned to cost of goods completed for the period

(1) (2) (3)


A. understate overstate overstate
B. understate understate overstate
C. overstate understate understate
D. overstate overstate understate
E. none of the above

C 12. Read, Inc. instituted a new process in October. During October, 10,000 units were started in
Department A. Of the units started, 7,000 were transferred to Department B, and 3,000 remained in
work in process at October 31. The work in process at October 31 was 100% complete as to material
costs and 50% complete as to conversion costs. Materials costs of $27,000 and conversion costs of
$39,950 were charged to Department A in October. What were the total costs transferred to
Department B?
A. $46,900
B. $53,600
C. $51,800
D. $57,120
E. none of the above

SUPPORTING CALCULATION:

Materials unit cost = $27,000  (7,000 + 3,000) = $2.70


Conversion unit cost = $39,950  [7,000 + 50%(3,000)] = $4.70
Costs transferred = 7,000($2.70 + $4.70) = $51,800

D 13. In accounting for beginning inventory costs, the method that allows the addition of beginning
inventory costs with costs incurred during the period is referred to as:
A. first-in, first-out
B. addition
C. last-in, first-out
D. average
E. first-in, last-out
Process Costing 64

E 14. Chicago Processing Co. uses the average costing method and reported a beginning inventory of
5,000 units that were 20% complete with respect to materials in one department. During the month,
11,000 units were started; 8,000 units were finished; ending inventory amounted to 8,000 units that
were 60% complete with respect to materials. Total materials cost during the period for work in
process should be spread over:
A. 7,200 units
B. 16,000 units
C. 11,200 units
D. 13,200 units
E. 12,800 units

SUPPORTING CALCULATION: 8,000 + .60(8,000) = 12,800 units

E 15. In determining the cost of goods transferred in from a previous department under the average cost
method:
A. a simple average of unit costs is used
B. beginning inventory costs are separated from costs transferred in during the period
C. a first-in, first-out approach is used
D. equivalent production in ending inventory is separated from other transferred-in costs
E. a weighted average of unit costs is used

E 16. The average and fifo process costing methods differ in that the average method:
A. can be used under any cost flow assumption
B. is much more difficult to apply than the fifo method
C. requires that ending work in process inventory be stated in terms of equivalent units of
production
D. considers the ending work in process inventory only partially complete
E. does not consider the degree of completion of beginning work in process inventory when
computing equivalent units of production

A 17. The first step in applying the average cost method is to:
A. add the beginning work in process costs to the current period's production costs
B. divide the current period's production costs by the equivalent units
C. subtract the beginning work in process costs from the current period's production costs
D. A and B
E. B and C

C 18. Beginning work in process was 60% complete as to conversion costs, and ending work in process
was 45% complete as to conversion costs. The dollar amount of the conversion cost included in
ending work in process (using the average cost method) is determined by multiplying the average
unit conversion costs by what percentage of the total units in ending work in process?
A. 60%
B. 55%
C. 45%
D. 52%
E. 100%
Process Costing 65

C 19. Dover Corporation's production cycle starts in the Mixing Department. The following information is
available for April:

Units
Work in process, April 1 (50% complete)..................................................................... 40,000
Started in April............................................................................................................... 240,000
Work in process, April 30 (60% complete)................................................................... 25,000

Materials are added at the beginning of the process in the Mixing Department. Using the average
cost method, what are the equivalent units of production for the month of April?

Materials Conversion
A. 255,000 255,000
B. 270,000 280,000
C. 280,000 270,000
D. 305,000 275,000
E. 240,000 250,000

SUPPORTING CALCULATION:

Materials = 40,000 + 240,000 = 280,000


Conversion = (280,000 - 25,000) + .6(25,000) = 270,000

B 20. Information concerning Department A of Neeley Company for June is as follows:

Materials
Units Costs
Beginning work in process..................................................................... 17,000 $12,800
Started in June........................................................................................ 82,000 69,700
Units completed..................................................................................... 85,000
Ending work in process.......................................................................... 14,000

All materials are added at the beginning of the process. Using the average cost method, the cost per
equivalent unit for materials is:
A. $0.825
B. $0.833
C. $0.85
D. $0.97
E. $1.01

SUPPORTING CALCULATION: ($12,800 + $69,700)  (85,000 + 14,000) = $.833


Process Costing 66

B 21. Kennedy Company adds materials in the beginning of the process in the Forming Department, which
is the first of two stages of its production cycle. Information concerning the materials used in the
Forming Department in October is as follows:

Materials
Units Costs
Work in process, October 1................................................................... 6,000 $ 3,000
Units started............................................................................................ 50,000 25,560
Units completed and transferred out...................................................... 44,000

Using the average cost method, what was the materials cost of work in process at October 31?
A. $3,000
B. $6,120
C. $3,060
D. $5,520
E. $6,000

SUPPORTING CALCULATION:

($3,000 + $25,560)  (44,000 + 12,000) = $.51


$.51 x 12,000 = $6,120

E 22. Roger Company manufactures Product X in a two-stage production cycle in Departments A and B.
Materials are added at the beginning of the process in Department B. Roger uses the average costing
method. Conversion costs for Department B were 50% complete as to the 6,000 units in beginning
work in process and 75% complete as to the 8,000 units in ending work in process. A total of 12,000
units were completed and transferred out of Department B during February. An analysis of the costs
relating to work in process and production activity in Department B for February follows:

Transferred- Materials Conversion


in Costs Costs Costs
Work in process, February 1:
Costs attached.................................................... $12,000 $2,500 $1,000
February activity:
Costs added........................................................ 29,000 5,500 5,000

The total cost per equivalent unit transferred out for February of Product X, rounded to the nearest
penny, was:
A. $2.82
B. $2.85
C. $2.05
D. $2.75
E. $2.78

SUPPORTING CALCULATION:

Transferred-in costs = $41,000  20,000 = $2.05


Materials cost = $8,000  20,000 = .40
Conversion cost = $6,000  18,000 = .33
$2.78
Process Costing 67

A 23. Simpson Co. adds materials at the beginning of the process in Department M. The following
information pertains to Department M's work in process during April:

Units
Work in process on April 1
(60% complete as to conversion cost).................................................................... 3,000
Started in April............................................................................................................... 25,000
Completed in April......................................................................................................... 20,000
Work in process on April 30
(75% complete as to conversion cost).................................................................... 8,000

Under the average costing method, the equivalent units for conversion cost are:
A. 26,000
B. 25,000
C. 24,000
D. 21,800
E. none of the above

SUPPORTING CALCULATION: 20,000 + .75(8,000) = 26,000

D 24. During March, Quig Company's Department Y equivalent unit product costs, computed under the
average cost method, were as follows:

Materials.................................................. $1
Conversion............................................... 3
Transferred-in.......................................... 5

Materials are introduced at the end of the process in Department Y. There were 4,000 units (40%
complete as to conversion costs) in work in process at March 31. The total costs assigned to the
March 31 work in process inventory should be:
A. $36,000
B. $28,800
C. $27,200
D. $24,800
E. none of the above

SUPPORTING CALCULATION: $5(4,000) + $3(4,000 x .4) = $24,800

The following questions are based on the material in the Appendix to the chapter.

B 25. If a company reports two different unit costs for goods transferred to the next department, it is
reasonable to assume that:
A. the department accounts for lost units at the end of the process
B. a fifo costing method is used
C. lost unit costs are computed separately
D. an average costing method is used
E. errors must have occurred in recording costs
Process Costing 68

C 26. In order to compute equivalent units of production using the fifo method of process costing, work for
the period must be broken down to units:
A. started and completed during the period
B. completed during the period and units in ending inventory
C. completed from beginning inventory, started and completed during the month, and units in
ending inventory
D. started during the period and units transferred out during the period
E. processed during the period and units completed during the period

A 27. The first-in, first-out method of process costing will produce the same cost of goods manufactured
amount as the average cost method when:
A. there is no beginning inventory
B. there is no ending inventory
C. beginning and ending inventories are each 50% complete
D. beginning inventories are 100% complete as to materials
E. goods produced are homogeneous

B 28. The fifo method of process costing differs from the average cost method of process costing in that
fifo:
A. allocates costs based on whole units, but the average cost method uses equivalent units
B. considers the stage of completion of beginning work in process in computing equivalent units
of production, but the average cost method does not
C. does not consider the stage of completion of beginning work in process in computing
equivalent units of production, but the average cost method does
D. is applicable only to those companies using the fifo inventory pricing method, but the average
cost method may be used with any inventory pricing method
E. none of the above

A 29. Connor Company computed the flow of physical units completed for Department M for the month of
March as follows:

Units completed:
From work in process on March 1............................................................................. 15,000
From March production............................................................................................. 45,000
Total..................................................................................................................... 60,000

Materials are added at the beginning of the process. The 12,000 units of work in process at March 31
were 80% complete as to conversion costs. The work in process at March 1 was 60% complete as to
conversion costs. Using the fifo method, the equivalent units for March conversion costs were:
A. 60,600
B. 55,200
C. 57,000
D. 54,600
E. 63,600

SUPPORTING CALCULATION: (15,000 x .4) + 45,000 + (12,000 x .8) = 60,600


Process Costing 69

D 30. The Hilo Company computed the physical flow of units for Department A for the month of April as
follows:

Units completed:
From work in process on April 1............................................................................... 10,000
From April production............................................................................................... 30,000
Total..................................................................................................................... 40,000

Materials are added at the beginning of the process. Units of work in process at April 30 were 8,000.
The work in process at April 1 was 80% complete as to conversion costs, and the work in process at
April 30 was 60% complete as to conversion costs. What are the equivalent units of production for
the month of April using the fifo method?

Materials Conversion Costs


A. 48,000 48,000
B. 40,000 47,600
C. 36,800 38,000
D. 38,000 36,800
E. 48,000 44,800

SUPPORTING CALCULATION:

Materials = 30,000 + 8,000 = 38,000


Conversion = (10,000 x .2) + 30,000 + (8,000 x .6) = 36,800

E 31. Department A is the first stage of Mann Company's production cycle. The following information is
available for conversion costs for the month of April:

............................................................................................................................... Units
Beginning work in process (60% complete)................................................................... 20,000
Started in April................................................................................................................. 340,000
Completed in April and transferred to Department B..................................................... 320,000
Ending work in process (40% complete)........................................................................ 40,000

Using the fifo method, the equivalent units for the conversion cost calculation are:
A. 336,000
B. 360,000
C. 328,000
D. 320,000
E. 324,000

SUPPORTING CALCULATION:

(20,000 x .4) + 300,000 + (40,000 x .4) = 324,000


Process Costing 70

PROBLEMS

PROBLEM

1.
Cost of Production Report. Fort Myers Corporation manufactures a product that is processed in two
departments: Mixing and Cooking. At the beginning and end of May, there were no inventories of unfinished
work. During May, 50,000 units of this product were completed. Materials used during May cost $28,000, of
which one half were used in the Mixing Department and one half were used in the Cooking Department. Direct
labor wages totaled $60,000, with $40,000 applicable to Mixing and $20,000 to Cooking. The amounts for direct
factory overhead incurred for each department and for general factory overhead apportioned to each department
were:

Mixing Cooking
Department Department
Factory overhead incurred............................................................................... $7,500 $9,000
General factory overhead apportioned............................................................ 5,000 6,000

Required: Prepare a partial cost of production report, showing the total cost to be accounted for in each
department.

SOLUTION

Fort Myers Corporation


Partial Cost of Production Report
For May, 19--

Mixing Department Cooking Department


Total Equivalent Unit Total Equivalent Unit
Cost Units Cost Cost Units Cost
Cost from preceding
department.................................... --- 50,000 --- $ 66,500 50,000 $1.33
Cost added by department..................
Materials....................................... $ 14,0001 50,000 $ 0.282 $ 14,000 50,000 $0.28
Labor............................................ 40,000 50,000 0.80 20,000 50,000 0.40
Factory overhead.......................... 12,500 50,000 0.25 15,000 50,000 0.30
Total cost added.................................. $ 66,500 $ 1.33 $ 49,000 $0.98
Total cost to be accounted
for................................................. $ 66,500 $ 1.33 $ 115,500 $2.31
1
$28,000 x 1/2 = $14,000
2
$14,000  50,000 units = $.28
Process Costing 71

PROBLEM

2.
Computation of Equivalent Production. Hanrahan Company uses process costing to account for the costs of its
only product, X. Production takes place in two departmentsSanding and Polishing. On December 31, the
inventory for Product X was as follows:

No unused materials
Work in process
Sanding Department............................................. 800 units (3/4 complete as to labor)
Work in process
Polishing Department........................................... 1,000 units (1/2 complete as to materials and
3/4 complete as to direct labor)
Finished Goods.............................................................. 600 units

Required:

(1) Compute the equivalent units of materials in all inventories combined at December 31.
(2) Compute the equivalent units of the Sanding Department's direct labor in all inventories at December 31.

SOLUTION

(1) Work in processSanding Department.......................................................................................... 800


Work in processPolishing Department (1,000 units x 1/2)......................................................... 500
Finished goods............................................................................................................................... 600
Units of materials in all inventories, Dec. 31............................................................................... 1,900

(2) Work in processSanding Department (800 units x 3/4).............................................................. 600


Work in processPolishing Department........................................................................................ 1,000*
Finished goods............................................................................................................................... 600*
Units of Sanding Dept.'s direct labor in all inventories, December 31........................................ 2,200

* All Sanding Department direct labor would be in all of these units or else they never would have been
transferred.
Process Costing 72

PROBLEM

3.
Calculation of Unit CostsAverage Costing Method. Barcelona Beach Products reports the following data for
the first department in its production process:

Units in process at beginning of period (all materials; 3/4 labor


and factory overhead).................................................................................................................... 5,000
Units started in process............................................................................................................................. 35,000
Units transferred out.................................................................................................................................. 33,000
Units still in process (all materials; 1/2 labor and factory overhead)...................................................... 5,000
Units completed but not yet transferred to Finished Goods..................................................................... 2,000

Related data were:

Work in Process at Added During


Beginning of Period Period
Materials............................................................................................... $100,000 $ 304,000
Labor..................................................................................................... 125,400 407,100
Factory overhead.................................................................................. 173,500 407,750
Total.............................................................................................. $398,900 $ 1,118,850

Required: Using the average costing method:

(1) Compute the unit cost for materials, labor, and factory overhead.
(2) Determine the cost of the work in process ending inventory.

SOLUTION

(1) Materials: ($100,000 + $304,000) / 40,000 units* = $10.10 per unit


Labor: ($125,400 + $407,100) / 37,500 units* = $14.20 per unit
Factory overhead: ($173,500 + $407,750) / 37,500 units = $15.50 per unit

*Equivalent production:
Materials: 33,000 + 2,000 + 5,000 = 40,000 units
Labor and factory overhead: 33,000 + 2,000 + (1/2 x 5,000) = 37,500 units

(2) Units in process at end of period:


Completed and on hand (2,000 x $39.80)......................................................................... $ 79,600
Materials (5,000 units x $10.10)........................................................................................ 50,500
Labor (5,000 units x 1/2 x $14.20).................................................................................... 35,500
Factory overhead (5,000 units x 1/2 x $15.50).................................................................. 38,750
$ 204,350
Process Costing 73

PROBLEM

4.
Journal Entries for Process Cost System. Xavier Corporation uses process costing in its two production
departments. A separate work in process account is kept in the general ledger for each production department.
The following data relate to operations for the month of March:

Beginning Added
Inventory During March
Direct materials cost: Department A $ 5,000 $25,000
Department B 3,000 20,000
Direct labor cost: Department A 6,000 40,000
Department B 4,500 35,000
Applied overhead: Department A 12,000 90,000
Department B 4,500 35,000

During March, 45,000 units with a cost of $5 each were transferred from Department A to Department B, and
40,000 units with a cost of $9 each were transferred from Department B to finished goods inventory.

Required: Prepare the appropriate general journal entries to record the cost charged to the producing departments
during March and the cost of units transferred from Department A to Department B and Department B to finished
goods inventory.

SOLUTION

Work in Process  Department A............................................................................ 25,000


Work in Process  Department B............................................................................ 20,000
Materials........................................................................................................... 45,000

Work in Process  Department A............................................................................ 40,000


Work in Process  Department B............................................................................ 35,000
Payroll.............................................................................................................. 75,000

Work in Process  Department A............................................................................ 90,000


Work in Process  Department B............................................................................ 35,000
Applied Factory Overhead............................................................................... 125,000

Work in Process  Department B............................................................................ 225,000


Work in Process  Department A.................................................................... 225,000

Finished Goods Inventory........................................................................................ 360,000


Work in Process  Department B.................................................................... 360,000
Process Costing 74

PROBLEM

5.
Cost of Production Report, Second Department, Average Costing. Isogen Corporation manufactures a product
in three departments. The product is cut out of lumber in the Cutting Department, then transferred to the Planing
Department where it is shaped and certain parts purchased from outside vendors are added to the unit, and finally
transferred to the Finishing Department where it is primed, painted, and packaged. Since only one product is
manufactured by the company, a process cost system is used. The company adopted the average cost flow
assumption to account for its work in process inventories. Data related to September operations in the Planing
Department follow:

Units in beginning inventory.............................................................................................................. 3,000


Units received from the Cutting Department this period.................................................................. 7,500
Units transferred to Finishing Department this period...................................................................... 8,500
Units in ending inventory (75% materials, 50% labor and overhead................................................ 2,000

Beginning Added
Costs charged to the department: Inventory This Period
Costs from the preceding department.......................................................... $15,500 $63,250
Materials....................................................................................................... 7,800 20,700
Direct labor................................................................................................... 3,200 16,750
Factory overhead.......................................................................................... 9,975 39,900

Required: Prepare a September cost of production report for the Planing Department.
Process Costing 75

SOLUTION

Isogen Corporation
Planing Department
Cost of Production Report
For September, 19--

Quantity Schedule.................................................. Materials Labor Overhead Quantity


Beginning inventory................................................. 3,000
Received from Cutting Department......................... 7,500
10,500

Transferred to Finishing Department....................... 8,500


Ending inventory...................................................... 75% 50% 50% 2,000
10,500

Total Equivalent Unit


Cost Charged to Department................................ Cost Units* Cost
Beginning inventory:
Cost from preceding department....................... $ 15,500
Materials...................................................... 7,800
Labor........................................................... 3,200
Factory overhead......................................... 9,975
Total cost in beginning inventory..................... $ 36,475
Cost added during period:
Cost from preceding department....................... $ 63,250 10,500 $ 7.50
Materials...................................................... 20,700 10,000 2.85
Labor........................................................... 16,750 9,500 2.10
Factory overhead......................................... 39,900 9,500 5.25
Total cost added during period.......................... $ 140,600
Total cost charged to the department....................... $ 177,075 $ 17.70

% Unit Total
Cost Accounted for as Follows Units Complete Cost Cost
Transferred to Finishing
Department...................................... 8,500 100% $17.70 $150,450
Work in process,
ending inventory:
Cost from preceding
department................................. 2,000 100 7.50 $ 15,000
Materials................................... 2,000 75 2.85 4,275
Labor......................................... 2,000 50 2.10 2,100
Factory overhead....................... 2,000 50 5.25 5,250 26,625
Total cost accounted for......................... $177,075

* Total number of equivalent units required in the cost accounted for section determined as follows:

Prior
Dept. Cost Materials Labor Overhead
Equivalent units transferred out............................... 8,500 8,500 8,500 8,500
Equivalent units in ending inventory....................... 2,000 1,500 1,000 1,000
Total equivalent units............................................... 10,500 10,000 9,500 9,500
Process Costing 76

PROBLEM

6.
Cost of Production Report, Increase in Quantity with Added Materials, Average Costing. Carlson Chemical
Company produces a chemical in three departments, Mixing, Blending, and Bottling. Mixing, where the
compounds are added, is the first department. The powder is then transferred to the second department where
water is added to produce a liquid. After water has been added, the chemical is bottled for storage and transported
to customers. A process cost system with an average cost flow assumption is used to account for work in process
inventories. Data related to operations in the Blending Department during the month of October follow:

Units in beginning inventory........................................................................................................................ 2,000


Units received from the Mixing Department this period............................................................................. 4,000
Units added to process in the Blending Department this period................................................................. 12,000
Units transferred to Bottling Department this period.................................................................................. 14,000
Units in ending inventory (100% materials, 40% labor and overhead)...................................................... 4,000

Beginning Added
Costs charged to the department: Inventory This Period
Costs from the preceding department.......................................................... $2,300 $11,200
Materials....................................................................................................... 720 2,520
Direct labor................................................................................................... 1,150 2,750
Factory overhead.......................................................................................... 2,100 5,700

Required: Prepare a cost of production report for the Blending Department.


Process Costing 77

SOLUTION

Carlson Chemical Company


Blending Department
Cost of Production Report
For October, 19--

Quantity Schedule.................................................. Materials Labor Overhead Quantity


Beginning inventory................................................. 2,000
Received from Mixing Department......................... 4,000
Added to process in Blending Department.............. 12,000
18,000

Transferred to Bottling Department......................... 14,000


Ending inventory...................................................... 100% 40% 40% 4,000
18,000

Total Equivalent Unit


Cost Charged to Department................................ Cost Units* Cost
Beginning Inventory:
Cost from preceding department....................... $ 2,300
Materials...................................................... 720
Labor........................................................... 1,150
Factory overhead......................................... 2,100
Total cost in beginning inventory..................... $ 6,270
Cost added during period:
Cost from preceding department....................... $ 11,200 18,000 $ .75
Materials...................................................... 2,520 18,000 .18
Labor........................................................... 2,750 15,600 .25
Factory overhead......................................... 5,700 15,600 .50
Total cost added during period.......................... $ 22,170
Total cost charged to the department....................... $ 28,440 $ 1.68

% Unit Total
Cost Accounted for as Follows............ Units Complete Cost Cost
Transferred to Bottling
Department...................................... 14,000 100% $1.68 $23,520
Work in process,
ending inventory:
Cost from preceding
department................................. 4,000 100 .75 $3,000
Materials................................... 4,000 100 .18 720
Labor......................................... 4,000 40 .25 400
Factory overhead....................... 4,000 40 .50 800 4,920
Total cost accounted for......................... $28,440

* Total number of equivalent units required in the cost accounted for section determined as follows:

Prior
Dept. Cost Materials Labor Overhead
Equivalent units transferred out............................... 14,000 14,000 14,000 14,000
Equivalent units in ending inventory....................... 4,000 4,000 1,600 1,600
Total equivalent units............................................... 18,000 18,000 15,600 15,600
Process Costing 78

The remaining problems are based on material in the chapter Appendix.

PROBLEM

7.
Equivalent Production Schedule. Javis Jam Co. uses fifo costing for its production processes. In December, the
Cooking Department reported the following summary of its activities:

Units in processbeginning inventory


(3/4 materials; 1/2 labor and factory overhead)................................................. 8,000
Units started in process during the period................................................................... 15,000 23,000

Units transferred to next department........................................................................... 19,000


Units still in process (1/2 materials;
1/4 labor and factory overhead).......................................................................... 4,000 23,000

Required: Prepare an equivalent production schedule for materials, labor, and factory overhead in the Cooking
Department using fifo costing.

SOLUTION

Labor and
Factory
Materials Overhead
Units transferred out.................................................................................................... 19,000 19,000
Less beginning inventory (all units)............................................................................ 8,000 8,000

Units started and finished this period.......................................................................... 11,000 11,000


Add beginning inventory (work this period):
Materials (8,000 units x 1/4)............................................................................... 2,000
Labor and factory overhead (8,000 units x 1/2)................................................. 4,000

Add ending inventory:


Materials (4,000 units x 1/2)............................................................................... 2,000
Labor and factory overhead (4,000 units x 1/4)................................................. 1,000
Equivalent production................................................................................................. 15,000 16,000
Process Costing 79

PROBLEM

8.
Cost of Production Report, Second Department, Fifo Costing. Handy Tool Company manufactures a product
in two departments, Shaping and Assembly. The product is cut out of sheet metal, bent to shape, and painted in
the Shaping Department. Then, it is transferred to the Assembly Department where component parts purchased
from outside vendors are added to the unit. A process cost system with a fifo cost flow assumption is used to
account for work in process inventories. Data related to November operations in the Assembly Department
follow:

Units in beginning inventory (90% materials, 80% labor and overhead)................................................. 1,000
Units received from the Shaping Department this period.......................................................................... 3,000
Units transferred to Finished Goods Inventory this period........................................................................ 2,800
Units in ending inventory (50% materials, 40% labor and overhead)....................................................... 1,200

Beginning Added
Costs charged to the department:................................................................................ Inventory This Period
Costs from the preceding department................................................................. $23,600 $29,250
Materials.............................................................................................................. 7,700 13,375
Direct labor.......................................................................................................... 3,500 9,672
Factory overhead................................................................................................. 4,900 16,616

Required: Prepare a November cost of production report on a fifo basis for the Assembly Department.
Process Costing 80

SOLUTION

Handy Tool Corporation


Assembly Department
Cost of Production Report
For November, 19--

Quantity Schedule.................................................. Materials Labor Overhead Quantity


Beginning inventory................................................. 90% 80% 80% 1,000
Received from Shaping Department........................ 3,000
4,000

Transferred to Finishing Department....................... 2,800


Ending inventory...................................................... 50 40 40 1,200
4,000

Total Equivalent Unit


Cost Charged to Department................................ Cost Units* Cost
Beginning inventory:
Cost from preceding department....................... $ 23,600
Materials...................................................... 7,700
Labor........................................................... 3,500
Factory overhead......................................... 4,900
Total cost in beginning inventory..................... $ 39,700
Cost added during period:
Cost from preceding department....................... $ 29,250 3,000 $ 9.75
Materials...................................................... 13,375 2,500 5.35
Labor........................................................... 9,672 2,480 3.90
Factory overhead......................................... 16,616 2,480 6.70
Total cost added during period.......................... $ 68,913
Total cost charged to the department....................... $ 108,613 $ 25.70

% Unit Total
Cost Accounted for as Follows............ Units Complete Cost Cost
Transferred to Finished Goods:
Beginning inventory............................... $39,700
Cost to complete:
Materials................................... 1,000 10% $ 5.35 535
Labor......................................... 1,000 20 3.90 780
Factory overhead....................... 1,000 20 6.70 1,340 $ 42,355
Started and completed this
period........................................ 1,800 100 25.70 46,260
Total cost transferred to
Finished Goods................................ $ 88,615
Work in process,
ending inventory:
Cost from preceding
department................................. 1,200 100% $ 9.75 11,700
Materials................................... 1,200 50 5.35 3,210
Labor......................................... 1,200 40 3.90 1,872
Factory overhead....................... 1,200 40 6.70 3,216 19,998
Total cost accounted for......................... $ 108,613
Process Costing 81

* Number of equivalent units of cost added during the current period determined as follows:

Prior
Dept. Cost Materials Labor Overhead
To complete beginning inventory............................ 0 100 200 200
Started and completed this period........................... 1,800 1,800 1,800 1,800
Ending inventory...................................................... 1,200 600 480 480
Total equivalent units............................................... 3,000 2,500 2,480 2,480
Process Costing 82

Chapter 7

THE COST OF QUALITY AND


ACCOUNTING FOR PRODUCTION LOSSES

MULTIPLE CHOICE

Question Nos. 16, 17, 22, and 23 are AICPA adapted.


Question No. 24 is CIA adapted.

A 1. The quality costs that are associated with materials and products that fail to meet quality standards
and result in manufacturing losses are known as:
A. internal failure costs
B. external failure costs
C. prevention costs
D. appraisal costs
E. none of the above

D 2. The quality costs that are associated with designing, implementing, and maintaining the quality
system are known as:
A. appraisal costs
B. internal failure costs
C. external failure costs
D. prevention costs
E. none of the above

C 3. The quality costs that are incurred to ensure that materials and products meet quality standards are
known as:
A. external failure costs
B. prevention costs
C. appraisal costs
D. internal failure costs
E. none of the above

B 4. The quality costs that are incurred because inferior quality products are shipped to customers are
known as:
A. internal failure costs
B. external failure costs
C. prevention costs
D. appraisal costs
E. none of the above

82
The Cost of Quality and Accounting for Production Losses 83

D 5. All of the following are characteristics of total quality management except:


A. the company's objective for all business activity is to serve its customers
B. top management provides an active leadership role in quality improvement
C. all employees are actively involved in quality improvement
D. the company maintains a loosely defined system of identifying quality problems so as not to
stifle employee creativity
E. the company provides continuous training as well as recognition for achievement

A 6. The best approach to quality improvement is to concentrate on:


A. prevention
B. detection
C. appraisal
D. increased production
E. none of the above

C 7. A mathematical technique used to monitor production quality and reduce product variability is:
A. the method of least squares
B. the statistical scattergraph method
C. statistical process control
D. linear programming
E. none of the above

D 8. Appraisal costs include all of the following except:


A. inspecting and testing materials
B. inspecting products during and after production
C. obtaining information from customers about product satisfaction
D. designing quality into the product and the production process
E. all of the above

B 9. Internal failure costs include all of the following except:


A. the cost of the scrap
B. the cost of warranty repairs and replacements
C. rework
D. downtime due to machine failures
E. all of the above

E 10. All of the following accounts would be acceptable ones to credit at the time scrap is sold except:
A. Scrap Sales
B. Cost of Goods Sold
C. Factory Overhead Control
D. Work in Process
E. all of the above would be acceptable

C 11. Scrap includes all of the following except:


A. the trimmings remaining after processing materials
B. defective materials that cannot be used or returned to the vendor
C. partially or fully completed units that are in some way defective
D. broken parts resulting from employee or machine failures
E. all of the above
The Cost of Quality and Accounting for Production Losses 84

A 12. When spoilage occurs because of some action taken by the customer, the unrecoverable cost of the
spoilage should be charged to:
A. Work in Process
B. Spoiled Goods Inventory
C. Factory Overhead Control
D. Applied Factory Overhead
E. none of the above

C 13. When spoilage occurs because of some internal failure, the unrecoverable cost should be charged to:
A. Work in Process
B. Spoiled Goods Inventory
C. Factory Overhead Control
D. Applied Factory Overhead
E. none of the above

A 14. When rework occurs because of some action taken by the customer, the cost of the rework should be
charged to:
A. Work in Process
B. Spoiled Goods Inventory
C. Factory Overhead Control
D. Applied Factory Overhead
E. none of the above

C 15. When rework occurs because of some internal failure, the cost of the rework should be charged to:
A. Work in Process
B. Spoiled Goods Inventory
C. Factory Overhead Control
D. Applied Factory Overhead
E. none of the above

C 16. Newman Company's Job 1865 for the manufacture of 2,200 coats was completed during August at
the unit costs presented below. Due to an internal failure in the production process, 200 coats were
found to be spoiled during final inspection that were sold to a jobber for $6,000.

Direct materials................................................................................................................. $20


Direct labor........................................................................................................................ 18
Factory overhead............................................................................................................... 18
................................................................................................................................ $56

What would be the unit cost of good coats produced on Job 1865?
A. $57.00
B. $55.00
C. $56.00
D. $58.00
E. none of the above

SUPPORTING CALCULATION: $20 + $18 + $18 = $56


The Cost of Quality and Accounting for Production Losses 85

A 17. During March, Vaughan Company incurred the following costs on Job 009 for the manufacture of
200 motors:

Original cost accumulation:


Direct materials........................................................................................................... $ 660
Direct labor................................................................................................................. 800
Factory overhead (150% of direct labor)................................................................... 1,200
$ 2,660

Direct costs of reworking 10 units:


Direct materials........................................................................................................... $100
Direct labor................................................................................................................. 160
..................................................................................................................................... $260

The rework costs were attributable to the exacting specifications of the customer. What is the cost
per finished unit of Job 009?
A. $15.80
B. $14.60
C. $14.00
D. $13.30
E. none of the above

SUPPORTING CALCULATION:

$2,660 + $260 + (150% x $160) = $3,160  200 = $15.80

C 18. Spoilage occurs as a result of an internal failure in a process cost system. Using average costing, the
number of equivalent units that production costs should be charged to would be based upon:
A. spoiled units
B. units transferred out and spoiled units
C. units transferred out, spoiled units, and units in ending inventory
D. units transferred out and units in ending inventory
E. none of the above

D 19. Spoilage occurs as a result of normal production shrinkage in a process cost system. Using average
costing, the number of equivalent units that production costs should be charged to would be based
upon:
A. spoiled units
B. units transferred out and spoiled units
C. units transferred out, spoiled units, and units in ending inventory
D. units transferred out and units in ending inventory
E. none of the above

C 20. In a process cost system, the cost of spoilage due to an internal production failure should be recorded
as:
A. dr. Work in Process; cr. Finished Goods
B. dr. Work in Process; cr. Factory Overhead Control
C. dr. Factory Overhead Control; cr. Work in Process
D. dr. Materials; cr. Factory Overhead
E. dr. Finished Goods; cr. Work in Process
The Cost of Quality and Accounting for Production Losses 86

B 21. Gyro Products transferred 10,000 units to one department. An additional 3,000 units of materials
were added in the department. At the end of the month, 7,000 units were transferred to finished
goods; while 4,000 units remained in work in process inventory. There was no beginning inventory,
and lost units were a result of normal production shrinkage. The production costs for the period in
this department would be effectively allocated over:
A. 12,000 units
B. 11,000 units
C. 10,000 units
D. 7,000 units
E. 13,000 units

SUPPORTING CALCULATION: 7,000 + 4,000 = 11,000

B 22. In manufacturing its products for the month of March, Leo Co. incurred normal production shrinkage
of $10,000 and spoilage due to internal failure of $12,000. How much spoilage cost should Leo
charge to Factory Overhead Control for the month of March?
A. $22,000
B. $12,000
C. $10,000
D. $0
E. none of the above

C 23. Willis, Inc. instituted a new process in October. During October, 10,000 units were started in
Department A. Of the units started, 1,000 were lost in the process due to normal production
shrinkage, 7,000 were transferred to Department B, and 2,000 remained in work in process at
October 31. The work in process at October 31 was 100% complete as to materials costs and 50%
complete as to conversion costs. Materials costs of $27,000 and conversion costs of $40,000 were
charged to Department A in October. What were the total costs transferred to Department B?
A. $46,900
B. $53,600
C. $56,000
D. $57,120
E. none of the above

SUPPORTING CALCULATION:

Materials: $27,000  (7,000 + 2,000) = $3


Conversion: $40,000  (7,000 + 1,000) = $5
Transferred costs: 7,000 x $8 = $56,000
The Cost of Quality and Accounting for Production Losses 87

D 24. A company that manufactures baseballs begins operations on January 1. Each baseball requires
three elements: a hard plastic core, several yards of twine that are wrapped around the plastic core,
and a piece of leather to cover the baseball. The plastic core is started down a conveyor belt and is
automatically wrapped with the twine to the approximate size of the baseball, at which time the
leather cover is sewn to the wrapped twine. Finished baseballs are inspected, and the ones that are
defective due to internal production failure are pulled out. Defective baseballs cannot be
economically salvaged and are destroyed. Cost and production reports for the first week of
operations are:

Raw material cost................................................................................................................ $ 840


Conversion cost.................................................................................................................... 315
................................................................................................................................... $ 1,155

During the week, 2,100 baseballs were completed; 2,000 passed inspection. There was no ending
work in process. The cost of the spoilage charged to Factory Overhead is:
A. $33
B. $22
C. $1,100
D. $55
E. none of the above

SUPPORTING CALCULATION:

Materials: $840  (2,000 + 100) = $.40


Conversion: $315  (2,000 + 100) = $.15
Spoilage: 100 x $.55 = $55

A 25. In a process cost system, the cost of rework usually is debited to:
A. Factory Overhead Control
B. Applied Factory Overhead
C. Spoiled Goods Inventory
D. Work in Process
E. none of the above

The following questions are based on the Appendix to the chapter:

D 26. If spoilage occurs as a result of an internal failure in a process cost system, using fifo costing, the
number of equivalent units that production costs should be charged to would be based upon:
A. spoiled units
B. units transferred out and spoiled units
C. units transferred out, beginning inventory, and units in ending inventory
D. units transferred out, spoiled units, units in ending inventory, and units in beginning inventory
E. none of the above
The Cost of Quality and Accounting for Production Losses 88

C 27. If spoilage occurs as a result of normal production shrinkage in a process cost system, using fifo
costing, the number of equivalent units that production costs should be charged to would be based
on:
A. spoiled units
B. units transferred out and spoiled units
C. units transferred out, beginning inventory, and units in ending inventory
D. units transferred out, spoiled units, units in ending inventory, and units in beginning inventory
E. none of the above

B 28. Primo Products transferred 15,000 units to one department. An additional 5,000 units were in
beginning inventory in the department. At the end of the month, 12,000 units were transferred to the
next department, 6,000 units remained in work in process, 40% complete as to conversion costs and
the remaining units were lost at the 75% stage of conversion. Beginning inventory was 60%
complete as to conversion costs and lost units were the result of internal failure. The equivalent
units of conversion cost using fifo costing is:
A. 14,400
B. 12,900
C. 13,900
D. 13,400
E. none of the above

SUPPORTING CALCULATION:

Equivalent units in beginning inventory (40% x 5,000).................................................. 2,000


Equivalent units started and completed during period
(12,000 - 5,000)......................................................................................................... 7,000
Equivalent units in ending inventory (40% x 6,000)........................................................ 2,400
Equivalent units of spoilage (75% x 2,000)..................................................................... 1,500
Total equivalent units........................................................................................................ 12,900

A 29. Primo Products transferred 15,000 units to one department. An additional 5,000 units were added in
the department. At the end of the month, 12,000 units were transferred to the next department, 6,000
units remained in work in process, 40% complete as to conversion costs and the remaining units
were lost at the 75% stage of conversion. Beginning inventory was 60% complete as to conversion
costs, and lost units were the result of normal production shrinkage. The equivalent units of
conversion cost using fifo is:
A. 11,400
B. 14,400
C. 12,900
D. 13,400
E. none of the above

SUPPORTING CALCULATION:

Equivalent units in beginning inventory (40% x 5,000).................................................. 2,000


Equivalent units started and completed during period
(12,000 - 5,000)......................................................................................................... 7,000
Equivalent units in ending inventory (40% x 6,000)........................................................ 2,400
Total equivalent units........................................................................................................ 11,400
The Cost of Quality and Accounting for Production Losses 89

PROBLEMS

PROBLEM

1.
Journal Entries for Scrap. Munoz Metal Products accumulates metal shavings from the shop floor and sells
them periodically to a nearby scrap dealer. Scrap sales, on account, for the period just ended total $2,300.

Required: Indicate the journal entries when:

(1)The scrap sales are viewed as additional revenue.


(2)The scrap sales are viewed as a reduction of the cost of goods sold during the period.
(3)The scrap sales are viewed as a reduction of factory overhead.
(4)The scrap sales are traceable to individual jobs and are viewed as a reduction in the cost of materials used on
the jobs.

SOLUTION

(1) Accounts Receivable.................................................................................... 2,300


Scrap Sales (or Other Income).............................................................. 2,300

(2) Accounts Receivable.................................................................................... 2,300


Cost of Goods Sold............................................................................... 2,300

(3) Accounts Receivable.................................................................................... 2,300


Factory Overhead Control.................................................................... 2,300

(4) Accounts Receivable.................................................................................... 2,300


Work in Process.................................................................................... 2,300

PROBLEM

2.
Spoilage in a Job Order Cost System. Walker Inc. manufactures custom wood products. During the current
period, an order for 2,000 workbenches was begun on Job 1994. After the job was completed, the benches were
inspected and 100 units were determined to be defective. The customer has agreed to accept the order with only
1,900 units instead of the quantity originally ordered. The spoiled units can be sold as seconds for $25 each.
Spoiled goods are kept in a separate inventory account from finished goods. Total costs charged to
Job 1994 follow:

Materials ....................................................................................................................................... $ 5,100


Labor (200 hours x $15 per hour)...................................................................................................... 3,000
Factory overhead ($9.50 per labor hour)........................................................................................... 1,900
Total cost charged to Job 1994.......................................................................................................... $ 10,000

Custom jobs are marked up 150 percent on cost.


The Cost of Quality and Accounting for Production Losses 90

Required:

(1) Assuming that the defective units were the result of an internal failure (i.e., an employee error or a
machine failure), prepare the appropriate general journal entries to record the transfer of the defective
units to a separate inventory account and the completion and shipment of Job 1994 to the customer.
(2) Assuming that the defective units were the result of a change in design specified by the customer after the
units were completed, prepare the appropriate general journal entries to record the transfer of the
defective units to the separate inventory account and the completion and shipment of Job 1994 to the
customer.

SOLUTION

(1) Spoiled Goods Inventory (10 units x $25 salvage)...................................... 250


Factory Overhead Control............................................................................ 250
Work in Process (10 units x $50* cost)................................................ 500

Cost of Goods Sold....................................................................................... 9,500


Work in Process ($10,000 - 500).......................................................... 9,500

Accounts Receivable ($9,500 x 150%)........................................................ 14,250


Sales....................................................................................................... 14,250

(2) Spoiled Goods Inventory (10 units x $25 salvage)...................................... 250


Work in Process.................................................................................... 250

Cost of Goods Sold....................................................................................... 9,750


Work in Process ($10,000 - $250)........................................................ 9,750

Accounts Receivable ($9,750 x 150%)........................................................ 14,625


Sales....................................................................................................... 14,625

*$10,000 total job cost


= $50 per unit
200 units on job
The Cost of Quality and Accounting for Production Losses 91

PROBLEM

3.
Entries for Charging Rework Costs Caused by Internal Failure and by Change in Customer Specification.
Albany Appliances manufactured 100 microwave ovens in a recent production run and discovered that 10 ovens
were defective and required reworking as follows:

Rework cost per unit:


Materials........................................................................................................................................... $ 10
Labor................................................................................................................................................. 25
Factory overhead.............................................................................................................................. 25
Total.......................................................................................................................................... $ 60
Normal production cost per unit:
Materials........................................................................................................................................... $ 50
Labor................................................................................................................................................. 75
Factory overhead.............................................................................................................................. 75
Total.......................................................................................................................................... $ 200

Required:

(1) Prepare the journal entries to record (a) the normal production costs, (b) the rework costs, and (c) the
transfer of the job costs to Finished Goods assuming that rework costs were caused by an internal failure.
(2) Prepare the same journal entries as in (1), assuming that rework costs were caused by a change in
customer specifications.

SOLUTION

Debit Credit
(1) (a) Work in Process................................................................................. 20,000
Materials..................................................................................... 5,000
Payroll......................................................................................... 7,500
Applied Factory Overhead......................................................... 7,500

(b) Factory Overhead Control ($60 x 10)............................................... 600


Materials..................................................................................... 100
Payroll......................................................................................... 250
Applied Factory Overhead......................................................... 250

(c) Finished Goods ($200 x 100)............................................................ 20,000


Work in Process.......................................................................... 20,000

(2) (a) Same as first entry in (1) (a) above.

(b) Work in Process................................................................................. 600


Materials..................................................................................... 100
Payroll......................................................................................... 250
Applied Factory Overhead......................................................... 250

(c) Finished Goods.................................................................................. 20,600


Work in Process.......................................................................... 20,600
The Cost of Quality and Accounting for Production Losses 92

PROBLEM

4.
Computation of Equivalent Units With Production Losses. Potter Paint Company manufactures paint in three
departments using a process cost system with an average cost flow assumption. Selected cost and production data
for the Blending Department, the second department in the production process, for the month just ended, are as
follows:

Units in beginning work in process......................................................................................................... 5,000


Units received from Mixing Department................................................................................................ 25,000
Units transferred to Finishing Department............................................................................................. 20,000
Units in ending work in process.............................................................................................................. 7,000
Units spoiled due to internal failure........................................................................................................ 3,000

Work in process, beginning inventory:


Cost from preceding department...................................................................................................... $ 4,200
Materials........................................................................................................................................... 1,960
Labor................................................................................................................................................. 895
Factory overhead.............................................................................................................................. 685
Costs added during the period:
From preceding department............................................................................................................. $ 15,900
Materials........................................................................................................................................... 8,775
Labor................................................................................................................................................. 4,550
Factory overhead.............................................................................................................................. 3,770

The paint is inspected at the end of the process in the Blending Department to detect any spoiled batches. Ending
inventory is 75% complete as to materials and 25% complete as to conversion costs.

Required:

(1) Compute the equivalent units of production for each cost element in the Blending Department for the
month just ended.
(2) Determine the average cost per equivalent unit for each cost element.

SOLUTION

(1)...................................................................... From
Preceding
Department Materials Labor Overhead
Equivalent units transferred out........................... 20,000 20,000 20,000 20,000
Equivalent units in ending inventory................... 7,000 5,250 1,750 1,750
Equivalent units of spoilage................................. 3,000 3,000 3,000 3,000
Total equivalent units........................................... 30,000 28,250 24,750 24,750
The Cost of Quality and Accounting for Production Losses 93

(2)...................................................................... From
Preceding
Department Materials Labor Overhead

Cost in beginning inventory................................. $ 4,200 $ 1,960 $ 895 $ 685


Cost added during the period............................... 15,900 8,775 4,550 3,770
Total cost to be accounted for.............................. $ 20,100 $ 10,735 $ 5,445 $ 4,455
Divide by equivalent units.................................... 30,000 28,250 24,750 24,750
Cost per equivalent units...................................... $ .67 $ .38 $ .22 $ .18

PROBLEM

5.
Spoilage With a Salvage Value in a Process Cost System Using an Average Cost Flow Assumption. Carter
Company manufactures a single product in two departments, Cutting and Finishing. Units of a product are started
in the Cutting Department and then transferred to the Finishing Department where they are completed. Units are
inspected at the 80% stage of completion in the Finishing Department. Good units are transferred to finished
goods inventory when completed and spoiled units are transferred to a separate inventory account. Spoiled units
are inventoried at their salvage value of $3 each, and the unrecoverable cost of spoilage, which was caused by an
internal failure, should be charged to the appropriate account.
Materials are added at the beginning of the production process. At the end of June, 2,000 units were still in
process in the Finishing Department, 100% complete as to materials and 60% complete as to conversion costs.
During July, 20,000 units were transferred from the Cutting Department to the Finishing Department and 15,000
were transferred from the Finishing Department to finished goods inventory. At the end of July, the Finishing
Department still had 4,000 units in process, 100% complete as to materials and 20% complete as to conversion
costs. Cost data related to July operations in the Finishing Department follow:

Beginning Added
Costs charged to the department:................................................................................. Inventory This Period
Cost from preceding department........................................................................... $6,050 $54,450
Materials................................................................................................................ 3,410 30,690
Labor...................................................................................................................... 1,638 14,742
Factory overhead................................................................................................... 2,184 19,656

Required: Complete the following cost of production report for the Finishing Department based on the data
presented for July, assuming the company uses a process cost system with average costing to account for its
production.
The Cost of Quality and Accounting for Production Losses 94

SOLUTION

Carter Corporation
Finishing Department
Cost of Production Report
For July, 19--

Quantity Schedule Materials Labor Overhead Quantity


Beginning inventory................................................. 2,000
Received from Cutting Department......................... 20,000
22,000

Transferred to finished goods................................... 15,000


Ending inventory...................................................... 100% 20% 20% 4,000
Spoiled in process..................................................... 100% 80% 80% 3,000
22,000

Total Equivalent Unit


Cost Charged to Department Cost Units* Cost
Beginning inventory:
Cost from preceding department....................... $ 6,050
Materials............................................................ 3,410
Labor. ............................................................... 1,638
Factory overhead................................................ 2,184
Total cost in beginning inventory............... $ 13,282

Cost added during period:


Cost from preceding department....................... $ 54,450 22,000 $2.75
Materials............................................................ 30,690 22,000 1.55
Labor. . ............................................................... 14,74218,200 .90
Factory overhead................................................ 19,656 18,200 1.20
Total cost added during period................... $ 119,538
Total cost charged to the department....................... $ 132,820 $6.40
The Cost of Quality and Accounting for Production Losses 95

% Unit Total
Cost Accounted for as Follows Units Complete Cost Cost
Transferred to
finished goods.................................. 15,000 100% $ 6.40 $96,000
Transferred to spoiled goods
inventory at salvage value............... 3,000 3.00 9,000
Charge to factory overhead
for spoilage:
Cost from preceding
department................................ 3,000 100% $ 2.75 $ 8,250
Materials.......................................... 3,000 100% 1.55 4,650
Labor. . ...................................3,000 80% .90 2,160
Factory overhead............................. 3,000 80% 1.20 2,880
$ 17,940
Less salvage value of
spoiled units.............................. 3,000 3.00 9,000 8,940
Work in process,
ending inventory:
Cost from preceding
department................................ 4,000 100% $ 2.75 11,000
Materials.......................................... 4,000 100% 1.55 6,200
Labor. . ...................................4,000 20% .90 720
Factory overhead............................. 4,000 20% 1.20 960 18,880
Total cost accounted for........................ $132,820

* Total number of equivalent units required in the cost accounted for section determined as follows:

Prior
Dept. Cost Materials Labor Overhead
Equivalent units transferred out............................... 15,000 15,000 15,000 15,000
Equivalent units in ending inventory....................... 4,000 4,000 800 800
Equivalent units of spoilage..................................... 3,000 3,000 2,400 2,400
Total equivalent units............................................... 22,000 22,000 18,200 18,200
The Cost of Quality and Accounting for Production Losses 96

PROBLEM

6.
Production Shrinkage in a Process Cost System Using an Average Cost Flow Assumption. Carrera Chemical
Inc. uses a process cost system with an average cost flow assumption to account for the production of its only
product. The product is manufactured in two departments. Units of product are started in the Cooking
Department and then transferred to the Blending Department where they are completed. Because of the intense
heat applied in the Cooking Department, some of the production volume is lost to evaporation. Labor and
overhead are treated as one element of cost in the Cooking Department (i.e., conversion cost). Data related to
May operations in the Cooking Department follow:

Units in beginning inventory.................................................................................................................... 10,000


Units started in process this period........................................................................................................... 45,000
Units transferred to the Blending Department this period....................................................................... 40,000
Units in ending inventory (100% materials, 40% conversion cost)......................................................... 9,000

Beginning Added
Costs charged to the department:................................................................................. Inventory This Period
Materials................................................................................................................. $4,375 $11,795
Conversion cost...................................................................................................... 2,975 6,181

Required: Prepare a cost of production report for the Cooking Department based on the data presented for May.
The Cost of Quality and Accounting for Production Losses 97

SOLUTION

Carrera Chemical Inc.


Cooking Department
Cost of Production Report
For May, 19--

Conversion
Quantity Schedule..................................................................... Materials Cost Quantity
Beginning inventory................................................................... 10,000
Started in process this period..................................................... 45,000
55,000

Transferred to Blending Department......................................... 40,000


Ending inventory........................................................................ 100% 40% 9,000
Lost in process............................................................................ 6,000
55,000

Total Equivalent Unit


Cost Charged to Department.................................................. Cost Units* Cost
Beginning inventory:
Materials............................................................................... $ 4,375
Conversion cost.................................................................... 2,975
Total cost in beginning inventory................................. 7,350
Cost added during period:
Materials............................................................................... $ 11,795 49,000 $.33
Conversion cost.................................................................... 6,181 43,600 .21
Total cost added during period..................................... $ 17,976
Total cost charged to the department......................................... $ 25,326 $.54

% Unit Total
Cost Accounted for as Follows Units Complete Cost Cost
Transferred to Blending
Department...................................... 40,000 100% $.54 $21,600
Work in process,
ending inventory:
Materials.......................................... 9,000 100% $.33 $2,970
Conversion cost............................... 9,000 40% .21 756 3,726
Total cost accounted for........................ $25,326

* Total number of equivalent units required in the cost accounted for section determined as follows:

Conversion
Materials Cost
Equivalent units transferred out................................................................................... 40,000 40,000
Equivalent units in ending inventory........................................................................... 9,000 3,600
Total equivalent units................................................................................................... 49,000 43,600
The Cost of Quality and Accounting for Production Losses 98

This problem is based on material presented in the Appendix to the chapter.

PROBLEM

7.
Spoilage With a Salvage Value in a Process Cost System With a Fifo Cost Flow Assumption. School Craft
Petroleum Company uses a process cost system with a fifo cost flow assumption to account for production, which
is manufactured in two departments. Units of product are started in the Cracking Department and then transferred
to the Refining Department where they are completed. Units are inspected at the end of the production process in
the Refining Department. Good units are transferred to finished goods inventory and spoiled units are transferred
to a separate inventory account. Spoiled units are inventoried at their salvage value of $8 each, and the
unrecoverable cost of spoilage resulting from an internal production failure is charged to the appropriate account.
Data related to September operations in the Refining Department follow:

Units in beginning inventory (60% materials, 30% labor, 30% overhead)............................................. 2,800
Units received from Cracking Department this period............................................................................ 8,400
Units transferred to the finished goods inventory this period.................................................................. 7,600
Units transferred to special inventory account this period....................................................................... 1,100
Units in ending inventory (100% materials, 50% labor, 50% overhead)................................................ 2,500

Beginning Added
Costs charged to the department:................................................................................. Inventory This Period
Cost from preceding department........................................................................... $17,889 $68,040
Materials................................................................................................................. 2,733 11,900
Labor...................................................................................................................... 7,278 30,063
Factory overhead.................................................................................................... 12,350 51,016

Required: Prepare a cost of production report for the Refining Department based on the data presented for
September.
The Cost of Quality and Accounting for Production Losses 99

SOLUTION

School Craft Petroleum Company


Refining Department
Cost of Production Report
For September, 19--

Quantity Schedule Materials Labor Overhead Quantity


Beginning inventory................................................. 60% 30% 30% 2,800
Received from Cracking Department...................... 8,400
11,200

Transferred to finished goods................................... 7,600


Ending inventory...................................................... 100% 50% 50% 2,500
Spoiled in process..................................................... 100% 100% 100% 1,100
11,200

Total Equivalent Unit


Cost Charged to Department Cost Units* Cost
Beginning inventory:
Cost from preceding department....................... $ 17,889
Materials............................................................ 2,733
Labor ............................................................... 7,278
Factory overhead................................................ 12,350
Total cost in beginning inventory............... $ 40,250
Cost added during current period:
Cost from preceding department....................... $ 68,040 8,400 $ 8.10
Materials............................................................ 11,900 9,520 1.25
Labor ............................................................... 30,063 9,110 3.30
Factory overhead................................................ 51,016 9,110 5.60
Total cost added during period................... $ 161,019
Total cost charged to the department....................... $ 201,269 $ 18.25
The Cost of Quality and Accounting for Production Losses 100

% Unit Total
Cost Accounted for as Follows Units Complete Cost Cost
Transferred to finished goods:
From beginning inventory................. $ 40,250
Cost to complete this
period:
Materials............................... 2,800 40% $ 1.25 $ 1,400
Labor..................................... 2,800 70% 3.30 6,468
Factory overhead.................. 2,800 70% 5.60 10,976 $ 59,094
Started and completed
this period.................................... 4,800 100% $ 18.25 87,600
Total cost transferred to
Finishing Department................. $ 146,694
Transferred to spoiled goods
inventory at salvage value................. 1,100 $ 8.00 8,800
Charged to factory overhead for
spoilage:
Cost of completed spoiled
units............................................. 1,100 100% $ 18.25 $ 20,075
Less salvage value of spoiled
units............................................. 1,100 8.00 8,800 11,275
Work in process,
ending inventory:
Cost from preceding
department................................... 2,500 100% $ 8.10 $ 20,250
Materials............................................ 2,500 100% 1.25 3,125
Labor. .............................................. 2,500 50% 3.30 4,125
Factory overhead............................... 2,500 50% 5.60 7,000 34,500
Total cost accounted for........................... $ 201,269

* Number of equivalent units of cost added during the current period determined as follows:

Prior
Dept. Cost Materials Labor Overhead
To complete beginning inventory...................... 0 1,120 1,960 1,960
Started and completed this period...................... 4,800 4,800 4,800 4,800
Ending inventory................................................. 2,500 2,500 1,250 1,250
Spoiled units........................................................ 1,100 1,100 1,100 1,100
Total equivalent units......................................... 8,400 9,520 9,110 9,110
The Cost of Quality and Accounting for Production Losses 101

Chapter 8

COSTING BY-PRODUCTS AND JOINT PRODUCTS

MULTIPLE CHOICE

Question Nos. 7, 10, 12-19, and 22 are AICPA adapted.


Question No. 25 is ICMA adapted.
Question Nos. 11, 20, 23, and 24 are CIA adapted.

B 1. The allocation of joint costs to individual products is useful primarily for purposes of:
A. determining whether to produce one of the joint products
B. inventory costing
C. determining the best market price
D. deciding whether to sell at the split-off point
E. evaluating whether an output is a main product or a by-product

B 2. The method used for the allocation of joint costs to products is important:
A. only in the minds of accountants
B. because profits will be affected when ending inventories change from the beginning of the
period
C. because its validity for justifying prices before regulatory authorities is unquestioned
D. because profit margins differ when the relative sales value method is used
E. for income determination when inventories are nonexistent

A 3. In a joint production process, a by-product is also described as:


A. a simultaneously produced product of relatively low value
B. a form of main product with controllable production proportions
C. waste
D. products of low value recovered at the end of a production process
E. a product with no value contribution to help offset production costs

D 4. All of the following are methods of costing by-products except the:


A. market value method
B. recognition of net revenue method
C. recognition of gross revenue method
D. average unit cost method
E. replacement cost method

101
Costing By-Products and Joint Products 102

E 5. Reporting revenue from by-product sales on the income statement as additional sales revenue:
A. allocates costs to by-products on the basis of quantities produced
B. reduces the main product cost by the estimated market value of the by-product
C. credits main product costs only when the by-product is used in further production
D. allocates a proper share of production costs to the by-product
E. overstates ending inventory costs of the main product

E 6. All of the following are methods of allocating joint production costs except the:
A. market value method
B. quantitative unit method
C. average unit cost method
D. average cost method
E. recognition of net revenue method

D 7. Tobin Company manufactures products S and T from a joint process. The market value at split-off
was $50,000 for 6,000 units of Product S and $50,000 for 2,000 units of Product T. Assuming that
the portion of the total joint cost properly allocated to Product S using the market value method was
$30,000, the total joint cost was:
A. $40,000
B. $42,500
C. $45,000
D. $60,000
E. $75,000

SUPPORTING CALCULATION:

$50,000
= .5
$50,000 + $50,000

$30,000
= $60,000
.5

C 8. Costs to be incurred after the split-off point are most useful for:
A. adjusting inequities in the joint cost allocation procedure
B. determining the levels of joint production
C. assessing the desirability of further processing
D. setting the mix of output products
E. assessing sales realization values for allocating joint costs accurately
Costing By-Products and Joint Products 103

D 9. Alphabet Company manufactures Products A and B from a joint process that also yields a by-
product, X. Alphabet accounts for the revenues from its by-product sales as a deduction from the
cost of goods sold of its main products. Additional information is as follows:

A B X Total
Units produced................................ 15,000 9,000 6,000 30,000
Joint costs....................................... $ 264,000
Market value at split-off................. $290,000 $150,000 $ 10,000 $450,000

Assuming that joint product costs are allocated using the market value at the split-off approach, the
joint cost allocated to Product B would be:
A. $136,540
B. $79,200
C. $88,000
D. $86,591
E. $99,000

SUPPORTING CALCULATION:

$150,000
_ ( $264,000 ⋅ $10,000 ) = $86,591
$290,000 + $150,000

D 10. If a company obtains two salable products from the refining of one ore, the refining process should
be accounted for as a(n):
A. reduction process
B. depletion process
C. mixed cost process
D. joint process
E. extractive process

A 11. The assignment of raw material costs to the major end products resulting from refining a barrel of
crude oil is best described as:
A. joint costing
B. differential costing
C. incremental costing
D. variable costing
E. indirect costing

B 12. The following components of production that can be allocated as joint costs when a single
manufacturing process produces several salable products are:
A. indirect production costs only
B. materials, labor, and overhead
C. materials and labor only
D. labor and overhead only
E. overhead and materials only
Costing By-Products and Joint Products 104

A 13. The following statement that best describes a by-product is:


A. a product that usually produces a small amount of revenue when compared to the main
product's revenue
B. a product that does not bear any portion of the joint processing costs
C. a product that is produced from material that would otherwise be scrap
D. a product that has a lower unit selling price than the main product
E. a product created along with the main product whose sales value does not cover its cost of
production

B 14. Relative sales value at split-off is used to allocate:


Cost Beyond
Split-Off Joint Costs
A. yes no
B. no yes
C. no no
D. sometimes never
E. yes yes

B 15. The following is acceptable regarding the allocation of joint product costs to a by-product:

None Allocated Some Portion Allocated


A. not acceptable not acceptable
B. acceptable acceptable
C. acceptable not acceptable
D. sometimes acceptable never acceptable
E. not acceptable acceptable

D 16. Idaho Corporation manufactures liquid chemicals A and B from a joint process. Joint costs are
allocated on the basis of relative market value at split-off. It costs $4,560 to process 500 gallons of
Product A and 1,000 gallons of Product B to the split-off point. The market value at split-off is $10
per gallon for Product A and $14 for Product B. Product B requires an additional process beyond
split-off at a cost of $2 per gallon before it can be sold. What is Idaho's cost to produce 1,000 gallons
of Product B?
A. $5,040
B. $4,360
C. $4,860
D. $5,360
E. $3,360

SUPPORTING CALCULATION:

( $14,000$14,000
+ $5,000
_ $4,560 ) + ($2 _ 1,000 ) = $5,360
Costing By-Products and Joint Products 105

C 17. Harry Corp. manufactures Products J, K, L, and M from a joint process. Additional information is as
follows:

Market If Processed Further


Units Value at Additional Market
Product Produced Split-Off Costs Value
J 6,000 $ 80,000 $ 7,500 $ 90,000
K 5,000 60,000 6,000 70,000
L 4,000 40,000 4,000 50,000
M 3,000 20,000 2,500 30,000
18,000 $ 200,000 $ 20,000 $ 240,000

Assuming that total joint costs of $160,000 were allocated using the market value at split-off
approach, what joint costs were allocated to each product?

J K L M
A. $53,333 $44,444 $35,556 $26,667
B. $60,000 $46,667 $33,333 $20,000
C. $64,000 $48,000 $32,000 $16,000
D. $60,000 $48,000 $32,000 $20,000
E. $40,000 $40,000 $40,000 $40,000

SUPPORTING CALCULATION:

J: 40% x $160,000 = $64,000


K: 30% x $160,000 = $48,000
L: 20% x $160,000 = $32,000
M: 10% x $160,000 = $16,000

E 18. Cayan Company manufactures three main products, F, G, and W, from a joint process. Joint costs
are allocated on the basis of relative market value at split-off. Additional information for June
production activity follows:

F G W Total
Units produced................................. 50,000 40,000 10,000 100,000
Joint costs......................................... ? ? ? $450,000
Market value at split-off.................. $ 420,000 $ 270,000 $60,000 $750,000
Additional costs if
processed further....................... $ 88,000 $ 30,000 $12,000 $130,000
Market value if
processed further....................... $ 538,000 $ 320,000 $87,000 $945,000

Assuming that the 10,000 units of W were processed further and sold for $87,000, what was Cayan's
gross profit on this sale?
A. $75,000
B. $51,000
C. $21,000
D. $28,500
E. $39,000
Costing By-Products and Joint Products 106

SUPPORTING CALCULATION:

Sales:........................................................................................................ $87,000
Cost of Goods Sold:
Joint Costs........................................................................................ $36,000
Separable Costs................................................................................ 12,000 48,000
Gross Profit.............................................................................................. $39,000

B 19. A company manufactures two joint products at a joint cost of $1,000. These products can be sold at
split-off, or when further processed at an additional cost, sold as higher quality items. The decision
to sell at split-off or further process should be based on the:
A. allocation of the $1,000 joint cost using the quantitative unit measure
B. assumption that the $1,000 joint cost is irrelevant
C. allocation of the $1,000 joint cost using the relative sales value approach
D. assumption that the $1,000 joint cost must be allocated using a physical-measure approach
E. allocation of the $1,000 joint cost using any equitable and rational allocation basis

D 20. The characteristic that is most often used to distinguish a product as either a joint product or a by-
product is the:
A. amount of labor used in processing the product
B. amount of separable product costs that are incurred in processing
C. amount (i.e., weight, inches, etc.) of the product produced in the manufacturing process
D. relative sales value of the products produced in the process
E. none of the above

A 21. A company processes raw material into products F1, F2, and F3. Each ton of raw material produces
five units of F1, two units of F2, and three units of F3. Joint processing costs to the split-off point are
$15 per ton. Further processing results in the following per unit figures:

F1 F2 F3
Additional processing costs per unit........................... $28 $30 $25
Selling price per unit................................................... 30 35 35

If joint costs are allocated by the net realizable value of finished product, what proportion of joint
costs should be allocated to F1?
A. 20%
B. 30%
C. 33 1/3%
D. 50%
E. none of the above

SUPPORTING CALCULATION:

($2 _ 5 )
= 20%
($2 _ 5 ) + ($5 _ 2 ) + ($10 _ 3 )
Costing By-Products and Joint Products 107

B 22. Jeffrey Co. manufactures Products A and B from a joint process. Market value at split-off was
$700,000 for 10,000 units of A, and $300,000 for 15,000 units of B. Using the market value at split-
off approach, joint costs properly allocated to A were $140,000. Total joint costs were:
A. $98,000
B. $200,000
C. $233,333
D. $350,000
E. none of the above

SUPPORTING CALCULATION:

$700,000
= .70
$700,000 + $300,000

$140,000
= $200,000
.70

C 23. A company produces three main joint products and one by-product. The by-product's relative market
value is quite low compared to that of the main products. The preferable accounting for the by-
product's net realizable value is as:
A. an addition to the revenues of the other products allocated on their respective net realizable
values
B. revenue in the period in which it is sold
C. a reduction in the joint cost to be allocated to the three main products
D. a separate net realizable value upon which to allocate some of the joint costs
E. none of the above

C 24. A company manufactures Products X and Y using a joint process. The joint processing costs are
$10,000. Products X and Y can be sold at split-off for $12,000 and $8,000 respectively. After split-
off, Product X is processed further at a cost of $5,000 and sold for $21,000, whereas Product Y is
sold without further processing. If the company uses the market value method for allocating joint
costs, the joint cost allocated to X is:
A. $4,000
B. $5,000
C. $6,000
D. $6,667
E. none of the above

SUPPORTING CALCULATION:

$12,000
_ $10,000 = $6,000
$12,000 + $8,000
Costing By-Products and Joint Products 108

D 25. The Hovart Corporation manufactures two products out of a joint processCompod and Ultrasene.
The joint (common) costs incurred are $250,000 for a standard production run that generates 120,000
gallons of Compod and 80,000 gallons of Ultrasene. Compod sells for $2.00 per gallon, while
Ultrasene sells for $3.25 per gallon. If there are no additional processing costs incurred after the
split-off point, the amount of joint cost of each production run allocated to Compod by the
quantitative unit method is:
A. $100,000
B. $120,000
C. $130,000
D. $150,000
E. some amount other than those given above

SUPPORTING CALCULATION:

120,000
_ $250,000 = $150,000
120,000 + 80,000

A 26. Ace Company produced 20,000 units of Clubs, 15,000 units of Diamonds, and 10,000 units of
Hearts. If the company uses the average unit cost method of allocating joint production costs, which
were $120,000 for the period, the joint costs allocated to Diamonds would be:
A. $40,000
B. $20,000
C. $80,000
D. $45,000
E. none of the above

SUPPORTING CALCULATION:

15,000
_ $120,000 = $40,000
20,000 + 15,000 + 10,000

C 27. A company uses the weighted average method to assign joint products. Weight factors used to
assign joint costs to its three joint products were: Product A, 4 points; Product B, 7 points; and
Product C, 8 points. Units produced were: Product A, 10,000; Product B, 5,000; and Product C,
3,125. The amount of the joint costs of $100,000 that would be allocated to Product C are:
A. $42,105
B. $17,241
C. $25,000
D. $30,000
E. none of the above

SUPPORTING CALCULATION:

( 3,125 _ 8 )
_ $100,000 = $25,000
(10,000 _ 4 ) + (5,000 _ 7 ) + ( 3,125 _ 8 )
Costing By-Products and Joint Products 109

E 28. The two standards in the Standards of Ethical Conduct for Management Accountants that pertain
most specifically to consideration of joint costs allocation are:
A. competence and confidentiality
B. confidentiality and integrity
C. competence and integrity
D. confidentiality and objectivity
E. none of the above
Costing By-Products and Joint Products 110

PROBLEMS

PROBLEM

1.
Consideration of By-Product in Net Income Determination. Harvard Products Co. manufactures two
productsYalies and Brownies. The Brownies are a by-product from its regular process. During the year, 10,000
Yalies were sold at $8 each. The total production cost was $5 per unit of Yalies, and marketing and administrative
expenses totaled $20,000. There were no beginning inventories, but ending inventories amounted to 1,000 units.
From the sale of Brownies, the company received $12,000, which was recorded as additional revenue from sales.

Required: Prepare an income statement showing the operating income for the year.

SOLUTION

Harvard Products Co.


Income Statement
For Year Ended December 31, 19--

Sales: Main product (10,000 Yalies @ $8)................................................................. $80,000


By-product (Brownies)................................................................................................. 12,000
Total sales..................................................................................................................... $92,000
Cost of goods sold:
Total production cost (12,000 units1 @ $5)......................................................... $60,000
Ending inventory (1,000 units @ $5)................................................................... 5,000 55,000
Gross profit................................................................................................................... $37,000
Marketing and administrative expenses...................................................................... 20,000
Operating income......................................................................................................... $17,000
1
Sales + Ending inventory - Beginning inventory = Production
10,000 + 1,000 - 0 = 11,000
Costing By-Products and Joint Products 111

PROBLEM

2.
By-Product Sales in Net Income Determination. Galaxy Flavorings Company produces tea bags. As part of the
manufacturing process, the tea leaves are separated from the stalks and stems. The tea leaves are sold as the main
product, while the stalks and stems are sold as the by-product for use in nursery mulch. During May, the company
processed 25,000 boxes of tea bags at a unit cost of $.75. Beginning inventory consisted of 2,000 boxes at a unit
cost of $.70 per box. During May, 20,000 boxes were sold for $1.75 each. The company also sold 500 pounds of
stalks and stems at a total price of $850. Marketing and administrative expenses amounted to $12,000.

Required: Prepare an income statement showing the operating income for May, assuming that the revenue from
the company's by-product sales is deducted from the production costs. (Show unit costs for the ending inventory
using the average cost method rounded to three decimal places.)

SOLUTION

Galaxy Flavorings Company


Income Statement
For Month Ended May 31, 19--

Sales: Main product (20,000 boxes @ $1.75).................................. $ 35,000


Cost of goods sold:
Beginning inventory (2,000 boxes @ $.70)............................... $ 1,400
Total production cost (25,000 boxes @ $.75)........................... $18,750
Revenue from sales of by-product............................................. (850) 17,900
Cost of goods available for sale................................................. $ 19,300
Ending inventory (7,000 boxes @ $.746 1)................................. 5,222 14,078
Gross profit........................................................................................ $ 20,922
Marketing and administrative expenses............................................ 12,000
Operating income............................................................................... $ 8,922
1
(25,000 x $.75) + (2,000 x $.70)/(20,000 + 7,000) =
($18,750 + $1,400)/27,000 =
$20,150/27,000 = $.746
Costing By-Products and Joint Products 112

PROBLEM

3.
Determination of Ending Inventory; Hypothetical Market Value Method. Macho Inc. manufactures two
beveragesRed Eye and Tornado. The production process is such that both beverages are jointly processed in the
Basic Blending Department. At the end of the basic blending process, Red Eye is sold at $10 per gallon, but
Tornado must be processed at a further cost of $7 per gallon before it can be sold at $15 per gallon. In June, the
total joint cost amounted to $96,000, while 5,000 gallons of Red Eye and 12,500 gallons of Tornado were
produced. There were no beginning inventories. At the end of June, there were 1,500 gallons of Red Eye and
2,000 gallons of Tornado on hand.

Required: Calculate the ending inventory costs for Red Eye and Tornado, using the hypothetical market value
method.

SOLUTION

Ending
Inventory Unit Costs Total
Product (Units) (per Schedule) Costs
Red Eye........................................................................................ 1,500 $ 6.40 $ 9,600
Tornado........................................................................................ 2,000 12.12 24,240
Ending inventory.................................................................. $ 33,840

Ultimate Ultimate Processing )


Market Value Units Market Costs After )
Product per Unit Produced Value Split-Off )
Red Eye............................................................... $10 5,000 $ 50,000 0 )
Tornado............................................................... 15 12,500 187,500 $87,5001 )
$ 237,500 $87,500 )

( Total
( Hypothetical Joint Cost Production
( Market Value Allocation2 Cost Unit Cost
( $ 50,000 $32,000 $ 32,000 $ 6.40
( 100,000 64,000 151,500 12.12
( $ 150,000 $96,000 $ 183,500
1
12,500 units x $7 = $87,500
2
$96,000/$150,000 = 64%, percentage to allocate joint cost
Costing By-Products and Joint Products 113

PROBLEM

4.
Joint Cost AllocationMarket Value and Weighted Average Methods. Texarkana Oil Co. produces three joint
products: gasoline, kerosene, and naphtha. Total joint production cost for May was $59,500. The units produced
and unit sales prices at the split-off point were:

Product Units Unit Sales Price


Gasoline................................................................................................................ 10,000 $5
Kerosene................................................................................................................ 15,000 4
Naphtha................................................................................................................. 20,000 3

In determining costs by the weighted average method, each unit is weighted as follows:

Product Per Unit Weighting


Gasoline................................................................................................................................. 10.3
Kerosene................................................................................................................................ 5
Naphtha................................................................................................................................. 3

Required: Allocate the production cost, using:

(1) The market value method


(2) The weighted average method

SOLUTION

(1)
Unit Total
Sales Market
Product Units Price Value Allocation 1
Gasoline.................................................................. 10,000 $5 $ 50,000 $ 17,500
Kerosene................................................................. 15,000 4 60,000 21,000
Naphtha.................................................................. 20,000 3 60,000 21,000
$ 170,000 $ 59,500
1
$59,500/$170,000 = 35%

(2)
Per Unit Weighted
Product Units Weighting Units Allocation 2
Gasoline.................................................................. 10,000 10.3 103,000 $ 25,750
Kerosene................................................................. 15,000 5 75,000 18,750
Naphtha.................................................................. 20,000 3 60,000
.................................................................... 15,000
238,000 $ 59,500
2
$59,500/238,000 = $.25 per weighted unit
Costing By-Products and Joint Products 114

PROBLEM

5.
Market Value Method for By-Products. Flores Inc. manufactures one main product and two by-products. Data
for July are:

Main By-Product By-Product


Product A B Total
Sales.....................................................$150,000 $12,000 $ 7,000 $169,000
Manufacturing cost before
separation.................................................... 75,000
Manufacturing cost after
separation.................................................... 23,000 2,200 1,800 27,000
Marketing and administrative
expense....................................................... 12,000 1,500 1,100 14,600

Profit allowed for By-Product A is 15% of sales and for By-Product B is 20% of sales.

Required:

(1) Calculate the manufacturing cost before separation that is to be charged to By-Products A and B.
(2) Prepare an income statement detailing sales and costs for each product.

SOLUTION

(1) A B
Sales............................................................................................................................... $ 12,000 $7,000
Manufacturing cost after separation.............................................................................. $ 2,200 $1,800
Marketing and administrative expenses........................................................................ 1,500 1,100
Profit allowance (A, 15%; B, 20%).............................................................................. 1,800 1,400
$ 5,500 $4,300
Manufacturing cost before separation........................................................................... $ 6,500 $2,700

(2) Flores Inc.


Income Statement
For July, 19--

Main
Product A B Total
Sales..................................................................... $ 150,000 $ 12,000 $7,000 $ 169,000
Cost of goods sold:
Before separation......................................... $ 65,800 $ 6,500 $2,700 $ 75,000
After separation........................................... 23,000 2,200 1,800 27,000
$ 88,800 $ 8,700 $4,500 $ 102,000
Gross profit $ 61,200 $ 3,300 $2,500 $ 67,000
Less marketing and
administrative expenses 12,000 1,500 1,100 14,600
Profit from operations $ 49,200 $ 1,800 $1,400 $ 52,400
Costing By-Products and Joint Products 115

PROBLEM

6.
Joint Cost Analysis for Managerial Decisions. The Conga Company produced three products, C, O, and N, as
the result of joint processing which cost $51,700.

C O N
Units produced................................................................... 22,000 17,500 11,750
Separable processing costs................................................ $ 33,000 $ 24,875 $ 31,375
Unit sales price.................................................................. $ 5.50 $ 7.25 $ 8.50

Required:

(1) Allocate the joint cost to the three products using the market value method.
(2) Suppose that Product O could be sold at the split-off point for $5.00. Would that be a good idea? Show
calculations.

SOLUTION

(1) Ultimate Ultimate )


Market Value Units Market )
Product per Unit Produced Value )
C.......................................................... $5.50 22,000 $121,000 )
O.......................................................... 7.25 17,500 126,875 )
N.......................................................... 8.50 11,750 99,875 )
$347,750 )

( Processing Hypothetical Apportionment


( Cost After Market of Joint
( Split-Off Value Production Cost
( $33,000 $ 88,000 $17,600*
( 24,875 102,000 20,400
( 31,375 68,500 13,700
( $89,250 $ 258,500 $51,700

* 51,700  $258,500 = .20; $88,000 x .20 = 17,600

(2)
Differential revenue [17,500 x (7.25 - 5.00)]............................................................................................. $39,375
Differential cost.......................................................................................................................................... 24,875
Net effect of separable processing.............................................................................................................. $14,500

Conclusion: Based on the information given, O should be processed beyond the split-off point.
Costing By-Products and Joint Products 116

Chapter 9

MATERIALS: CONTROLLING, COSTING, AND PLANNING

MULTIPLE CHOICE

Question Nos. 23-27, 30, 31, and 38-40 are AICPA adapted.
Question No. 22 is CIA adapted.

A 1. The cycle of materials procurement and use includes all of the following steps except for:
A. determining the cost of goods sold
B. the production budget
C. preparing the receiving report
D. maintaining the materials ledger
E. engineering to determine materials specifications

E 2. In a well-controlled materials system, the Purchasing Department performs all of the following
activities except the:
A. placing of purchase orders with suppliers
B. receiving of purchase requisitions
C. maintaining of information on market prices for goods used
D. preparation of purchase orders
E. approving and checking of invoices

B 3. The purchase requisition is a document used to:


A. initiate the return of merchandise to the vendor
B. inform the purchasing agent of a need for a materials item
C. initiate payment for merchandise received
D. inform the Purchasing Department of a receipt of goods
E. authorize the vendor to supply merchandise or materials

B 4. The expense that theoretically is not a correct part of inventory cost is:
A. freight-in
B. freight-out
C. inspection costs
D. accounting costs for materials received
E. purchasing costs

C 5. Theoretically, cash discounts permitted on purchased raw materials should be:


A. added to other income, whether taken or not
B. added to other income, only if taken
C. deducted from inventory, whether taken or not
D. deducted from inventory, only if taken
E. none of the above

116
Materials: Controlling, Costing, and Planning 117

E 6. The materials requisition:


A. is the list of materials requirements for each step in the production sequence
B. informs the purchasing agent of the quantity and kind of materials needed
C. contracts for quantities to be delivered
D. certifies quantities received and reports results of inspection and testing
E. authorizes the storeroom to deliver types and quantities of materials to a given department

C 7. The purchase order:


A. is the list of materials requirements for each step in the production sequence
B. informs the purchasing agent of the quantity and kind of materials needed
C. contracts for quantities to be delivered
D. certifies quantities received and reports results of inspection and testing
E. authorizes the storeroom to deliver types and quantities of materials to a given department

A 8. The bill of materials:


A. is the list of materials requirements for each step in the production sequence
B. informs the purchasing agent of the quantity and kind of materials needed
C. contracts for quantities to be delivered
D. certifies quantities received and reports results of inspection and testing
E. authorizes the storeroom to deliver types and quantities of materials to a given department

D 9. The receiving report:


A. is the list of materials requirements for each step in the production sequence
B. informs the purchasing agent of the quantity and kind of materials needed
C. contracts for quantities to be delivered
D. certifies quantities received and reports results of inspection and testing
E. authorizes the storeroom to deliver types and quantities of materials to a given department

D 10. The purchasing department performs all of the following functions except:
A. receives purchase requisitions for materials, supplies, and equipment
B. keeps informed concerning sources of supply, prices, and delivery schedules
C. prepares and places purchase orders
D. compares quantities received with the suppliers' packing list
E. arranges for the reporting among the purchasing, receiving, and accounting departments

C 11. The purchase requisition may originate with all of the following except:
A. a storeroom employee
B. a materials record clerk
C. a receiving department clerk
D. a research, engineering, or other department employee who needs materials of a special nature
E. a computer

B 12. The receiving department does all of the following except:


A. unloads and unpacks incoming materials
B. keeps informed concerning sources of supply, prices, and delivery schedules
C. matches materials received with descriptions on purchase orders
D. arranges for inspection, when necessary
E. routes accepted materials to the appropriate departments
Materials: Controlling, Costing, and Planning 118

A 13. A cost of having too few items on hand in inventory is:


A. frequent stockouts
B. excessive insurance costs
C. payment of additional warehouse space
D. spoilage costs
E. costs of obsolescence

B 14. Of the following, the expense that is not relevant to determining the most economic quantity to order
is:
A. additional costs to store inventory
B. rental of warehouse space under a ten-year lease
C. interest expense of financing purchases
D. spoilage costs
E. variable costs of placing an order

B 15. A company has been ordering more than the economic order quantity. This would result in:
A. more frequent order points
B. carrying costs greater than order costs
C. equal safety stock costs and carrying costs
D. carrying costs less than order costs
E. insufficient safety stock costs

B 16. Annual demand for squash racquets is 50,000 units, and carrying costs amount to $2 per unit. Order
costs for the company amount to $5. The optimum order quantity in units for squash racquets is
(rounded to the nearest unit):
A. 191
B. 500
C. 250
D. 100
E. 625

SUPPORTING CALCULATION:

square root (2$2_ 50,000 _ $5 ) = 500


E 17. A company orders 10,000 units (a one-year supply) of Zap at one time. Zap costs $1 per unit, and
order costs amount to $500 each time an order is placed. The costs to carry Zap in inventory amount
to 20% of the materials cost. For an entire year, the inventory carrying costs and order costs are:
A. $2,000
B. $200
C. $500
D. $1,000
E. $1,500

SUPPORTING CALCULATION:

10,000 _ $500 $1 _ . 20 _ 10,000


+ = $1,500
10,000 2
Materials: Controlling, Costing, and Planning 119

B 18. If the average lead time and usage figures are used for determining the order point, then the
probability of a stockout is:
A. .005%
B. 50%
C. 5%
D. 100%
E. 2.5%

A 19. There are 1,000 Trolls in stock, and 1,500 are due in from orders that were placed previously. The
company sells Trolls at the rate of 100 per day and finds that it takes an average of 20 days for an
order to be received. Because usage and lead times are known with certainty and because the
company has determined that an order must be placed now, the desired safety stock quantity must be
equal to:
A. 500 units
B. 1,000 units
C. 2,500 units
D. 100 units
E. 1,500 units

SUPPORTING CALCULATION:

1,000 + 1,500 = (100 x 20) + SSQ

SSQ = 500

B 20. The use of quantitative models can be modified to improve the management of inventory by:
A. including only fixed costs in the EOQ analysis
B. employing a minimum safety stock level because delivery time and inventory usage rates may
vary
C. purchasing inventory only once a year to save on ordering cost
D. purchasing inventory monthly to save on carrying cost
E. eliminating semivariable costs from any consideration in the EOQ analysis because of the
difficulty of estimating those costs

C 21. An inventory control technique that reviews quantities on hand periodically and orders sufficient
quantities to bring inventory up to a desired level expressed as a number of days' or weeks' supply is
the:
A. two-bin method
B. ABC inventory control method
C. order cycling method
D. min-max method
E. automatic order point system

B 22. The factor that need not be considered when calculating an inventory economic order quantity
(EOQ) is:
A. annual sales of a product
B. safety stock level
C. order-placing costs
D. storage costs
E. risk of inventory obsolescence and deterioration
Materials: Controlling, Costing, and Planning 120

B 23. Brad Company has correctly computed its economic order quantity as 500 units. However,
management would rather order in quantities of 600 units. How will Brad's total annual purchase
order cost and total annual carrying cost for an order quantity of 600 units compare to the respective
amounts for an order quantity of 500 units?
A. higher purchase order cost and lower carrying cost
B. lower purchase order cost and higher carrying cost
C. higher purchase order cost and higher carrying cost
D. lower purchase order cost and lower carrying cost
E. none of the above

A 24. Carter Company buys a certain part for its manufacturing process for $20 a part and needs 10,000
parts a year. It costs $3 a year to carry one of these parts in inventory. The cost of placing a
purchase order for these parts is $15. Assuming that the parts will be required evenly throughout the

2 _ 10,000 _ 15
A.
3

10,000 _ 3
B.
15

10,000 _ 15
C.
3

2 _ 10,000 _ 3
D.
15

E. none of the above


year, the formula for the economic order quantity is the square root of:
A 25. For its economic order quantity model, a company has a $10 cost of placing an order and a $2
annual cost of carrying one unit in stock. If the cost of placing an order increases by 20%, the
annual cost of carrying one unit in stock increases by 25%, and all other considerations remain
constant, the economic order quantity will:
A. decrease
B. increase
C. remain unchanged
D. either increase or decrease, depending on the reorder point
E. either increase or decrease, depending on the safety stock

C 26. For inventory management, ignoring safety stocks, a valid computation of the reorder point is:
A. order costs plus carrying costs
B. the square root of the anticipated demand during lead time
C. the anticipated demand per day during lead time times lead time in days
D. the economic order quantity
Materials: Controlling, Costing, and Planning 121

E. the economic order quantity times the anticipated demand during lead time
Materials: Controlling, Costing, and Planning 122

C 27. The Cappalari Company wishes to determine the amount of safety stock that it should maintain for
Product D to result in the lowest cost. The following information is available:

Stockout cost............................................................................................... $ 80 per occurrence


Carrying cost of safety stock...................................................................... $ 2 per unit
Number of purchase orders......................................................................... 5 per year

The options available to Cappalari are as follows:

Units of Probability of Running


Safety Stock out of Safety Stock
10 50%
30 30%
50 10%
55 5%

The number of units of safety stock that will result in the lowest cost is:
A. 30
B. 50
C. 55
D. 10
E. none of the above

SUPPORTING CALCULATION:

Safety Expected Stockout Carrying Stockout and


Stock Stockouts Cost Cost Carrying Cost
10 2.5 $200 $ 20 $220
30 1.5 120 60 180
50 .5 40 100 140
55 .25 20 110 130

B 28. The following information is available for Odyssey Company's Material Y:

Annual usage in units.......................................................................................................... 10,000


Working days per year........................................................................................................ 250
Normal lead time in working days...................................................................................... 30
Maximum lead time in working days................................................................................. 70

Assuming that the units of Material Y will be required evenly throughout the year, the order point
would be:
A. 2,000
B. 2,800
C. 2,105
D. 1,200
E. 1,600

SUPPORTING CALCULATION:

[(10,000  250) x 30] + [(70 - 30) x 40]


30 (10,000  250) + 40 (70 - 30) = 2,800
Materials: Controlling, Costing, and Planning 123

A 29. The following information relates to Hudson Company's Material A:

Annual usage in units.......................................................................................................... 7,200


Working days per year........................................................................................................ 240
Normal lead time in working days...................................................................................... 20
Maximum lead time in working days................................................................................. 45

Assuming that the units of Material A will be required evenly throughout the year, the safety stock
and order point would be:

Safety Stock Order Point


A. 750 1,350
B. 600 750
C. 600 1,350
D. 750 600
E. none of the above

SUPPORTING CALCULATION:

Safety Stock: (7,200  240) (45 - 20) = 750


Order Point: 20 (7,200  240) + 750 = 1,350

C 30. Penguin Company manufactures winter jackets. Setup costs are $2.00. Penguin manufactures 4,000
jackets evenly throughout the year. Using the economic order quantity approach, the optimal
production run would be 200 when the cost of carrying one jacket in inventory for one year is:
A. $0.10
B. $0.20
C. $0.40
D. $0.05
E. none of the above

SUPPORTING CALCULATION:

2 _ 4,000 _ $2
square root ( CC )
CC
Materials: Controlling, Costing, and Planning 124

A 31. The following data refer to various annual costs relating to the inventory of a single-product
company:

Unit transportation-in on purchases.................................................................................... $ .20


Storage per unit.................................................................................................................... .12
Insurance per unit................................................................................................................ 10
Annual interest foregone from alternate investment of funds............................................ $ 800
Annual number of units required........................................................................................ 10,000

What is the annual carrying cost per unit?


A. $.30
B. $.42
C. $.50
D. $.32
E. $.22

SUPPORTING CALCULATION:

$800
$ .12 + $ .10 + = $ . 30
10,000

D 32. Bliss Company has an order point at 1,400 units, usage during normal lead time of 600 units, and an
EOQ of 2,000 units. Its maximum inventory, assuming normal lead time and usage, would be:
A. 3,400 units
B. 2,000 units
C. 1,200 units
D. 2,800 units
E. 4,000 units

SUPPORTING CALCULATION:

(1,400 - 600) + 2,000 = 2,800

A 33. The inventory model that follows the concept that 80% of the value of an inventory is in 20% of the
inventory items is the:
A. ABC plan
B. economic order quantity (EOQ) model
C. just-in-time inventory system
D. materials requirements planning (MRP) system
E. zero inventory model

B 34. The materials control method that is based on the premise that the quantities of most stock items are
subject to definable limits is the:
A. cycle review method
B. min-max method
C. two-bin method
D. ABC plan
E. none of the above
Materials: Controlling, Costing, and Planning 125

C 35. The materials control method that is based on physical observation that an order point has been
reached is the:
A. cycle review method
B. min-max method
C. two-bin method
D. ABC plan
E. none of the above

The following questions are based on the Appendix to the chapter:

C 36. If the cost of goods sold computed when inventory is costed using the fifo method is less than when
using the lifo method:
A. prices decreased
B. prices remained unchanged
C. prices increased
D. price trend cannot be determined from the information given
E. prices went up and down

A 37. The method of inventory pricing that best approximates specific identification of the actual flow of
costs and units in most manufacturing situations is:
A. first-in, first-out
B. last-in, first-out
C. base stock
D. average cost
E. none of the above

D 38. The following information was available from the inventory records of the Anthony Company for
January 19X7:

Unit Total
Units Cost Cost
Balance at January 1, 19X7........................................ 2,000 $ 9.775 $19,550
Purchases:
January 6, 19X7................................................... 1,500 10.300 15,450
January 26, 19X7................................................. 3,400 10.750 36,550
Sales:
January 7, 19X7................................................... 1,800
January 31, 19X7................................................. 3,200
Balance at January 31, 19X7...................................... 1,900

Assuming that Anthony maintains perpetual inventory records, what should be the inventory at
January 31, 19X7, using the average cost inventory method rounded to the nearest dollar?
A. $19,998
B. $19,523
C. $19,703
D. $19,950
E. none of the above

SUPPORTING CALCULATION:

(1,700 _ $10 ) + ( 3,400 _ $10 .75 )


1,900 _ = $19,950
5,100
Materials: Controlling, Costing, and Planning 126

D 39. The following information was available from the inventory records of the Anthony Company for
January 19X7:

Unit Total
Units Cost Cost
Balance at January 1, 19X7........................................ 2,000 $ 9.775 $19,550
Purchases:
January 6, 19X7................................................... 1,500 10.300 15,450
January 26, 19X7................................................. 3,400 10.750 36,550
Sales:
January 7, 19X7................................................... 1,800
January 31, 19X7................................................. 3,200
Balance at January 31, 19X7...................................... 1,900

Assuming that Anthony does not maintain perpetual inventory records, what should be the inventory
at January 31, 19X7, using the average cost inventory method rounded to the nearest dollar?
A. $19,950
B. $19,998
C. $19,523
D. $19,702
E. none of the above

SUPPORTING CALCULATION:

$71,550
_ 1,900 = $19,702
6,900

A 40. In a period of rising prices, using which of the following inventory cost flow methods would result in
the highest ending inventory?
A. fifo
B. average cost
C. weighted average cost
D. moving average cost
E. lifo

A 41. The inventory cost flow method that involves computations based on broad inventory pools of
similar items is:
A. dollar-value lifo
B. average cost
C. moving average
D. fifo
E. regular quantity of goods lifo
Materials: Controlling, Costing, and Planning 127

PROBLEMS

PROBLEM

1.
Applied Acquisition Costs. James Company Inc. records incoming materials at invoice price less cash discounts
plus applied receiving and handling cost. For product Beta, the following data are available:

Budgeted for Actual Cost


the Month for the Month
Freight-in and cartage-in................................................................................... $ 3,800 $ 3,750
Purchasing Department cost.............................................................................. 7,150 7,075
Receiving Department cost............................................................................... 5,825 5,850
Storage and handling......................................................................................... 6,130 6,100
Testing, spoilage, and rejects............................................................................ 3,345 3,850
Total................................................................................................................... $ 26,250 $ 26,625

The purchasing budget shows estimated net purchases of $175,000 for the month. Actual invoices net of discounts
total $173,500 for the month.

Required:

(1) Determine the applied acquisition costing rate for the month.
(2) Determine the amount of applied cost added to materials purchased during the month.
(3) Indicate the amount of and the possible disposition of the variance.

SOLUTION

Budgeted acquisition cost $26,250


= = 15% applied acquisition costing rate for the month
Budgeted purchases $175,000
(1)
(2) $173,500 net purchases x 15% applied acquisition costing rate = 26,025 applied cost added to materials
purchased during the month
(3) The underapplied acquisition cost of $600 (26,625 actual cost - 26,025 applied cost) should be debited to
Cost of Goods Sold or prorated to Cost of Goods Sold and inventories.
Materials: Controlling, Costing, and Planning 128

PROBLEM

2.
Determination of Optimal Order Quantity. Micro Corp. uses 1,000 units of Chip annually in its production.
Order costs consist of $10 for placing a long-distance call to make the order and $40 for delivering the order by
truck to the company warehouse. Each Chip costs $100, and the carrying costs are estimated at 15.625% of the
inventory cost.

Required:

(1) Compute the economic order quantity for Chip and the total order costs and carrying costs for the year.
(2) Determine the best order quantity if Chip is purchased only in multiples of 25 units. (Round answers to the
nearest whole dollar.)

SOLUTION

2 _ RU _ CO
(1) EOQ = square root ( CU _ CC )
2 _ 1,000 _ $50
= square root ( $100 _ .15625 )
= square root 6,400

= 80

RU _ CO 1,000 _ $50
= = $625 order cost
EOQ 80

‰ EOQ _ CU _ CC = ‰ _ 80 _ $100 _ .15625 = $625 carrying cost

(2) The best order quantity is 75. By process of elimination, try both 75 units and 100 units:
Materials: Controlling, Costing, and Planning 129

RU _ CO 1,000 _ $50
= = $667 order cost
75 75

 EOQ x CU x CC =  x 75 x $100 x .15625 = 586 carrying cost


$ 1,253 total cost

Additional computations:
Order quantity at 100:
Order cost.................................................................................................................... $ 500
Carrying cost............................................................................................................... 781
Total................................................................................................................................... $ 1,281
Materials: Controlling, Costing, and Planning 130

PROBLEM

3.
Order Point, Inventory Levels, Ordering Cost. Charleston Company has developed the following data to assist
in controlling one of its inventory items:

Economic order quantity.............................................................................................. 1000 liters


Average daily use......................................................................................................... 100 liters
Maximum daily use...................................................................................................... 120 liters
Working days per year................................................................................................. 250 days
Safety stock................................................................................................................... 140 liters
Cost of carrying inventory........................................................................................... $1.00 per liter per year
Lead time .................................................................................................................... 7 working days

Required: Compute the following:

(1) Order point


(2) Average inventory
(3) Maximum inventory assuming normal lead time and usage
(4) Cost of placing one order

SOLUTION

(1) Order point: 140 + (100 x 7 days) = 840 liters


(2) Average inventory: 140 + (1000/2) = 640 liters
(3) Normal maximum inventory: 140 + 1000 = 1140 liters
(4) Cost of placing one order (CO):

2 _ RU _ CO
EOQ = square root ( CU _ CC% )
2 _ 25000 _ CO
1,000 = square root ( $1 )
1,000 = square root ($50,000 _ CO)

1,000,000 = $50,000 _ CO

CO = $20
Materials: Controlling, Costing, and Planning 131

PROBLEM

4.
Cost Saving by Use of EOQ. Warner Co. uses 6,000 units of material per year at a cost of $4 per unit. Carrying
costs are estimated to be $1.125 per unit per year, and order costs amount to $60 per order. As an incentive to its
customers, Warner will extend quantity discounts according to the following schedule:

Minimum List Net


Order Price Discount Price
500 $4 2% $3.92
1,000 4 4 3.84
2,000 4 6 3.76

Required:

(1) Determine the economic order quantity (ignoring quantity discounts) and the total annual order cost,
carrying cost, and materials costs at EOQ (considering quantity discounts).
(2) Compute the annual order cost, carrying cost, materials cost, and total cost at each discount level. (Round
to the nearest dollar.)
(3) Identify the order size, choosing from one of the three discount levels, that will minimize the total cost.

SOLUTION

2 _ 6,000 _ $60
(1) EOQ = square root ( $1 . 125 )
= 800 units per order

Order cost + Carrying cost + Materials cost = Total cost

(6,000/800 x $60) + (800/2 x $1.125) + (6,000 x $3.92) = $450 + $450 + $23,520 = $24,420

(2) Size of order........................................................................... 500 1,000 2,000


Number of orders per year..................................................... 12 6 3
Average inventory.................................................................. 250 500 1,000

Order cost............................................................................... $ 720 $ 360 $ 180


Carrying cost ($1.125 per unit).............................................. 281 563 1,125
Materials cost:
6,000 x $3.92................................................................... 23,520
6,000 x $3.84................................................................... 23,040
6,000 x $3.76................................................................... 22,560
Total cost................................................................. $ 24,521 $ 23,963 $ 23,865

(3) The order size that will minimize the total cost is 2,000 units.
Materials: Controlling, Costing, and Planning 132

PROBLEM

5.
Determination of Optimal Size of a Production Run. Georgia Corp. produces fireworks in various forms. A
cardboard tube, Part No. A86-E, is manufactured rather than ordered from an outside supplier. The company
estimates that its need each year for this tube is 4,800 gross and that variable manufacturing costs are $60 per
gross. Setup costs amount to $162 per production run, and storage costs are equal to 5% of variable
manufacturing costs.

Required:

(1) Determine the optimal size of a production run and the total annual setup cost and total carrying cost at that
size.
(2) Determine the optimal size of a production run, the total annual setup cost, and the total carrying cost,
assuming that storage space is limited to 400 units.

SOLUTION

Total an ual setup 4,80 720


( )( )
¿ cost plus total ¿ ¿¿ = $1,080 + $1,080 ¿¿ = $2,160 ¿
car ying cost
720 2
(2) The optimal size would have to be 400 units because total costs at any lot size below 400 units are greater.
Costs at this size are:

Total an ual setup 4,80 40


( )( )
¿ cost plus total ¿ ¿¿
car ying cost
40 2
= $1,94 + $60 ¿¿ = $2,54 ¿
Materials: Controlling, Costing, and Planning 133

PROBLEM

6.
Safety Stock. Jefferson & Sons Inc. would like to determine the safety stock to maintain for a product so that the
lowest combination of stockout cost and carrying cost would result. Each stockout will cost $100; the carrying
cost for each safety stock unit will be $2; the product will be ordered ten times a year. The following probabilities
of running out of stock during an order period are associated with various safety stock levels:

Safety Stock Level Probability of Stockout


25 units 50%
50 25
75 10
100 5

Required: Determine the combined stockout and safety stock carrying cost associated with each level and the
recommended level of safety stock.
(AICPA adapted)

SOLUTION

Safety Annual Probability Expected Cost Annual )


Stock Number x of = Annual x per = Stockout )
Level of Orders Stockout Stockouts Stockout Cost )
)
25 10 .5 5 $100 $500 )
50 10 .25 2.5 100 250 )
75 10 .1 1 100 100 )
100 10 .05 .5 100 50 )

( Annual Safety Stock Annual


( + Carrying Cost = Combined
( ($2 per Unit) Cost
(
( $ 50 $550
( 100 350
( 150 250
( 200 250

The recommended level of safety stock is either 75 units or 100 units, since each have the same combined cost.
Materials: Controlling, Costing, and Planning 134

The following problem is based on the Appendix to the chapter.

PROBLEM

7.
Materials Costing Methods Using Materials Ledger Card. A company that uses a perpetual inventory system
had the following transactions for Material 999 during July:

July 1 Beginning balance: 2,800 units @ $12.00 per unit


4 Issued 1,200 units
6 Received 1,000 units @ $13.30 per unit
8 Issued 1,000 units
14 Received 400 units @ $14.00 per unit
17 Issued 800 units
20 Received 500 units @ $14.16 per unit

Required: Prepare a materials ledger card for Material 999, using (1) fifo costing, (2) lifo costing, and (3) average
costing.

SOLUTION

(1) Received Issued )


Unit Unit )
Date Units Cost Cost Units Cost Cost )
July 1 )
4 1,200 $12.00 $14,400 )
6 1,000 $13.30 $13,300 )
8 1,000 12.00 12,000 )
14 400 14.00 5,600 )
17 600 12.00 7,200 )
200 13.30 2,660 )
20 500 14.16 7,080 )

( Balance
( Unit
( Units Cost Amount
( 2,800 $12.00 $33,600
( 1,600 12.00 19,200
( 1,600 12.00 $ 19,200
( 1,000 13.30 13,300 32,500
( 600 12.00 $ 7,200
( 1,000 13.30 13,300 20,500
( 600 12.00 $ 7,200
( 1,000 13.30 13,300
( 400 14.00 5,600 26,100
( 800 13.30 $ 10,640
( 400 14.00 5,600 16,240
( 800 13.30 $ 10,640
( 400 14.00 5,600
( 500 14.16 7,080 23,320
Materials: Controlling, Costing, and Planning 135

(2) Received Issued )


Unit Unit )
Date Units Cost Cost Units Cost Cost )
July 1 )
4 1,200 $12.00 $14,400 )
6 1,000 $13.30 $13,300 )
8 1,000 13.30 13,300 )
14 400 14.00 5,600 )
17 400 14.00 5,600 )
400 12.00 4,800 )
20 500 14.16 7,080 )

( Balance
( Unit
( Units Cost Amount
( 2,800 $12.00 $33,600
( 1,600 12.00 19,200
( 1,600 12.00 $ 19,200
( 1,000 13.30 13,300 32,500
( 1,600 12.00 19,200
( 1,600 12.00 $ 19,200
( 400 14.00 5,600 24,800
( 1,200 12.00 14,400
( 1,200 12.00 $ 14,400
( 500 14.16 7,080 21,480

(3) Received Issued )


Unit Unit )
Date Units Cost Cost Units Cost Cost )
July 1 )
4 1,200 $12.00 $14,400 )
6 1,000 $13.30 $13,300 )
8 1,000 12.50 12,500 )
14 400 14.00 5,600 )
17 800 12.80 10,240 )
20 500 14.16 7,080 )

( Balance
( Unit
( Units Cost Amount
( 2,800 $12.00 $33,600
( 1,600 12.00 19,200
( 2,600 12.50 32,500
( 1,600 12.50 20,000
( 2,000 12.80 25,600
( 1,200 12.80 15,360
( 1,700 13.20 22,440
Materials: Controlling, Costing, and Planning 207

Chapter 15

BUDGETING: PROFITS, SALES, COSTS, AND EXPENSES

MULTIPLE CHOICE

Question Nos. 11-16, 21, and 22 are ICMA adapted.

A 1. Short-range budgets must be considered in conjunction with long-range plans in order to:
A. find the best short-range budget
B. obtain systematic feedback
C. predict the future
D. coordinate risk and return evaluations
E. eliminate risk

C 2. The background for long-range plans is formed by all of the following items except:
A. population growth
B. personal consumption expenditures
C. precise future product costs
D. indexes of industrial production
E. economic factors and market trends

A 3. In setting profit objectives, management must consider all of the following items except:
A. indexes of industrial production
B. sales volume required to meet all costs, dividends, and retained earnings requirements
C. sales volume attainable in the present plant
D. the break-even point
E. profit or loss for given sales volume levels

A 4. The procedure for setting profit objectives in which management specifies a given rate of return that
it seeks to realize in the long run by means of planning toward that end is the:
A. a priori method
B. ad hoc method
C. pragmatic method
D. theoretical method
E. a posteriori method

207
Budgeting: Profits, Sales, Costs, and Expenses 208

C 5. Social impacts on the management planning process include all of the following except:
A. nonrenewable resource consumption
B. public safety
C. income taxation
D. impact of company products on health
E. environmental pollution

E 6. A budget that contains summaries of the sales, manufacturing, and expense budgets is a:
A. budgeted cost of goods manufactured and sold statement
B. sales budget
C. production budget
D. factory overhead budget
E. budgeted income statement

C 7. The principal functions of the budget committee include all of the following except:
A. reviewing individual budget estimates
B. deciding on general policies
C. enforcing budgeted standards
D. analyzing budget reports
E. suggesting revisions to budget estimates

D 8. In planning for future sales, the type of data most likely to be found in trade association
publicationsor from the trade associations themselveswould be the:
A. unemployment rate
B. general economic conditions
C. company's potential market share
D. industry's volume of sales
E. company's past sales by product line

C 9. A company that has inventory on hand at the beginning of a budget period and that has determined
its desired sales and ending inventory levels uses the following formula to figure the amount of
production required:
A. Production = Beginning Inventory + Ending Inventory - Sales
B. Production = Sales - Beginning Inventory - Ending Inventory
C. Production = Sales - Beginning Inventory + Ending Inventory
D. Production = Sales - Beginning Inventory
E. Production = Sales + Beginning Inventory - Ending Inventory

E 10. For budget purposes, the most useful cost classification method is the:
A. significant variance system
B. dollar value classification
C. variability classification
D. natural classification
E. departmental classification
Budgeting: Profits, Sales, Costs, and Expenses 209

E 11. The goals and objectives upon which an annual profit plan is based should be limited to:
A. financial measures, such as net income, return on investment, and earnings per share
B. quantitative measures, such as growth in unit sales, number of employees, and manufacturing
capacity
C. qualitative measures of organizational activity, such as product innovation leadership, product
quality levels, and product safety
D. the financial and quantitative measures
E. a combination of financial, quantitative, and qualitative measures

B 12. The primary role of the budget committee is to:


A. justify the budget to the executive committee of the board of directors
B. decide on general policies, compile the budget, and manage the budget process
C. force the final profit plan to conform to top-management goals
D. settle disputes among operating executives during the development of the annual operating
plan
E. develop the annual profit plan by selecting the alternatives to be adopted from the suggestions
submitted by the various operating segments

E 13. When an organization prepares a forecast, it:


A. consolidates the plans of the separate requests into one overall plan
B. presents the plan for a range of activity so that the plan can be adjusted for changes in activity
levels
C. classifies budget requests by activity and estimates the benefits arising from each activity
D. divides the activities of individual responsibility centers into a series of packages that are
ranked ordinally
E. presents a statement of expectations for a period of time but does not present a firm
commitment

D 14. A distinction between forecasting and planning:


A. is that forecasting relies exclusively on statistical techniques while planning does not
B. is not valid because they are synonymous
C. arises because they are based upon different assumptions about economic events
D. is that a plan can be prepared on the basis of a forecast
E. is that forecasting is a management activity while planning is a technical activity

C 15. A continuous budget:


A. is used only in process manufacturing companies
B. works best for a company that can reliably forecast events a year or more into the future
C. is a plan that is revised monthly or quarterly
D. is an annual plan that is part of a five-year plan
E. is a plan devised by a full-time planning staff
Budgeting: Profits, Sales, Costs, and Expenses 210

B 16. Ying Company plans to sell 200,000 units of finished product in October and anticipates a growth
rate in sales of 5% per month. The desired monthly ending inventory in units of finished product is
80% of the next month's estimated sales. There are 150,000 finished units in the inventory on
September 30.

Ying's production requirement in units of finished product for the three-month period ending
December 31 is:
A. 664,000
B. 665,720
C. 630,000
D. 712,025
E. none of the above

SUPPORTING CALCULATION:

Production = Sales + Ending inventory - Beginning inventory


[200,000 + (200,000 x 1.05) + (200,000 x 1.05 2)] + (200,000 x 1.053 x .8) - 150,000
= 665,720

E 17. In setting profit objectives, management needs to consider:


A. return on capital employed
B. profit or loss resulting from a given volume of sales
C. sales volume that the present operating capacity can produce
D. operating capacity necessary to attain the profit objectives
E. all of the above

C 18. All of the following have been found to be good motivators for a company's personnel except:
A. a system of employee support through coaching, counseling, and career planning
B. a system that not only considers company objectives, but also employees' skills and capacities
C. a pay incentive system based on increased productivity
D. a system of communication that allows employees to query their superiors with trust and
honest communication
E. a system of promotion that generates and sustains employee faith in its validity and judgment

A 19. The plan that serves as a check on the accuracy of all other budgets is the:
A. budgeted balance sheet
B. treasurer's budget
C. sales budget
D. credit rating budget
E. forecast cash flow statement

E 20. If estimated sales and ending inventory in units are 50,000 and 12,000, respectively; and the amount
of required production is 54,000 units, the beginning inventory in units would be:
A. 2,000
B. 0
C. 16,000
D. 4,000
E. none of the above
Budgeting: Profits, Sales, Costs, and Expenses 211

SUPPORTING CALCULATION:

Production = Sales + Ending inventory - Beginning inventory


54,000 = 50,000 + 12,000 - 8,000

C 21. The Husker Company's sales budget shows quarterly sales for the next year as follows:

Quarter 1..................................................................................................................... 10,000 units


Quarter 2..................................................................................................................... 8,000 units
Quarter 3..................................................................................................................... 12,000 units
Quarter 4..................................................................................................................... 14,000 units

Company policy is to have a finished goods inventory at the end of each quarter equal to 20% of the
next quarter's sales. Budgeted production for the second quarter of the next year would be:
A. 7,200 units
B. 8,000 units
C. 8,800 units
D. 8,400 units
E. some amount other than those given above

SUPPORTING CALCULATION:

Sales + Ending inventory - Beginning inventory = Production


8,000 + (.2 x 12,000) - (.2 x 8,000) = 8,800

A 22. The Erica Corporation's budget calls for the following production:

Quarter 1..................................................................................................................... 45,000 units


Quarter 2..................................................................................................................... 38,000 units
Quarter 3..................................................................................................................... 34,000 units
Quarter 4..................................................................................................................... 48,000 units

Each unit of product requires three pounds of direct material. The company's policy is to begin each
quarter with an inventory of direct materials equal to 30% of that quarter's direct material
requirements. Budgeted direct materials purchases for the third quarter would be:
A. 114,600 pounds
B. 89,400 pounds
C. 38,200 pounds
D. 29,800 pounds
E. some amount other than those given

SUPPORTING CALCULATION:

Production + Ending inventory - Beginning inventory = Purchases


(34,000 x 3) + (.3 x 48,000 x 3) - (.3 x 34,000 x 3) = 114,600
Budgeting: Profits, Sales, Costs, and Expenses 212

E 23. A company's profit plan consists of:


A. a detailed operating budget
B. long- and short-range income statements
C. balance sheets
D. cash budgets
E. all of the above

E 24. The procedure for setting profit objectives in which the determination of profit objectives is
subordinated to the planning, and the objectives emerge as the product of the planning itself is the:
A. a priori method
B. practical method
C. pragmatic method
D. theoretical method
E. a posteriori method

C 25. The procedure for setting profit objectives in which management uses a profit standard that has been
empirically tested and sanctioned by experience is the:
A. a priori method
B. practical method
C. pragmatic method
D. theoretical method
E. a posteriori method
Budgeting: Profits, Sales, Costs, and Expenses 213

PROBLEMS

PROBLEM

1.
Production, Inventory, and Working Capital Requirements. Pronto Products prepares a budget forecast of its
needs for the coming year. The current year's data and estimates for the coming year are presented below for the
three styles of electric can openers sold by the company.

Unit Current Year


Can Opener Price Sales Ending Inv. Sales Estimates
Quick-Lid............................................. $40 8,000 units 1,200 units 20,000 units
Easy-Open............................................ 30 10,000 1,500 26,000
Pry-Off................................................. 20 12,000 1,800 30,000

Next year's estimates are prepared by salespeople who management believes are very optimistic. Therefore,
predictions of sales levels should be reduced by 25% to be realistic. In addition, the company requires an ending
inventory equal to 10% of sales.

Required:

(1) Compute predicted unit sales for each type of can opener and the production required to provide for sales
and inventory needs.
(2) Compute the dollar revenues expected to be obtained for each can opener.
(3) Compute the working capital required if the cost to produce each can opener is 55% of the sales price and
if the company requires working capital equal to 15% of total production cost. (Show computations and
round to the nearest dollar.)

SOLUTION

(1)
Less Plus
Predicted Beginning Ending Production
Can Opener Unit Sales Inventory Inventory Required
Quick-Lid........................................ 15,000 (20,000 x .75) 1,200 1,500 15,300
Easy-Open....................................... 19,500 (26,000 x .75) 1,500 1,950 19,950
Pry-Off............................................ 22,500 (30,000 x .75) 1,800 2,250 22,950

(2)
Can Opener Unit Sales Unit Price Total Sales
Quick-Lid................................................................................... 15,000 $40 $ 600,000
Easy-Open.................................................................................. 19,500 30 585,000
Pry-Off........................................................................................ 22,500 20 450,000
$ 1,635,000
Budgeting: Profits, Sales, Costs, and Expenses 214

(3)
Production Cost (55%
Units x Unit Sales Price
Can Opener Produced x Units Required)
Quick-Lid..................................................................................... 15,300 $336,600
Easy-Open.................................................................................... 19,950 329,175
Pry-Off.......................................................................................... 22,950 252,450
Total production cost........................................................... $918,225

15% x $918,225 = $137,734 working capital required

PROBLEM

2.
Sales and Production Budgets; Labor Requirements. Farkel Fabricators is in the process of preparing its
budget for the coming year. The following data are provided:

Beginning inventory....................................................................................................................... 15,000 units


Estimated sales............................................................................................................................... 175,000 units
Desired ending inventory............................................................................................................... 20,000 units
Estimated production losses due to spoilage................................................................................. 5,000 units
Units produced per direct labor hour............................................................................................. 5 units

Each employee works a total of 2,000 hours per year. A supervisor is required for every five employees. Since
fractional employees and supervisors are not available, the number of employees and supervisors to be employed
must always be rounded to the next highest number whenever it is a fraction.
Each unit will yield a revenue of $5, while each unit produced (including spoiled units) costs $1.50.

Required:

(1) Prepare the production budget in units for the coming year.
(2) Determine the number of direct labor employees and supervisors required for the coming year. (Show
supporting computations.)

SOLUTION

(1)
Production for:
Current sales........................................................................................................................................ 175,000
Spoiled goods...................................................................................................................................... 5,000
Ending inventory................................................................................................................................. 20,000
Total units required.................................................................................................................................... 200,000
Provided by beginning inventory.............................................................................................................. (15,000)
Current production..................................................................................................................................... 185,000
Budgeting: Profits, Sales, Costs, and Expenses 215

(2)
Production required/Units per employee hour = Employee hours required
185,000/5 = 37,000

Employee hours required/Annual hours per employee = Direct labor employees required
37,000/2,000 = 18.5 or 19 employees

Employees/Ratio of employees to supervisors = Supervisors required


19/5 = 3.8 or 4 supervisors

PROBLEM

3.
Sales Forecast; Budgeted Income Statement. The management of Podunk Pottery Co. would like to earn 20%
on its invested capital of $4,000,000. The company estimates sales of 100,000 pots during the coming year
ending December 31. Sales commissions are paid at the rate of 10% of the sales price. Other expenses are as
follows:

Variable manufacturing expenses.................................................................................... 30% of sales


Fixed manufacturing expenses......................................................................................... $ 100,000
Fixed general and administrative expenses..................................................................... $ 25,000

Required:

(1) Compute the dollar amount of target net income.


(2) Prepare a budgeted income statement for the coming year.

SOLUTION

(1) The net income must equal 20% of $4,000,000, or $800,000.

(2) Podunk Pottery Company


Budgeted Income Statement
For Year Ending December 31, 19--

Sales............................................................................................................................ $ 1,541,667
Less cost of goods sold:
Variable manufacturing expenses...................................................................... $ 462,500
Fixed manufacturing expenses........................................................................... 100,000 562,500
Gross profit................................................................................................................. $ 979,167
Sales commissions...................................................................................................... $ 154,167
Fixed general and administrative expenses............................................................... 25,000 179,167
Net income.................................................................................................................. $ 800,000
Budgeting: Profits, Sales, Costs, and Expenses 216

PROBLEM

4.
Production, Materials and Manufacturing Budget. Dink Products Inc. prepared the following figures as a basis
for its 19B budget:

Estimated Sales Price Required


Product Expected Sales per Unit Materials per Unit
X Y
Bens.................................................... 40,000 units $ 9.00 2 lbs. 4 lbs.
Bimmer............................................... 20,000 12.00 4 lbs. 1 lb.

Estimated inventories at the beginning and desired quantities at the end of 19B are:

Purchase
Material Beginning Ending Price per Pound
X................................................................... 5,000 lbs. 6,000 lbs. $1.20
Y................................................................... 6,000 7,500 .60

Direct Labor
Hours Per
Product Beginning Ending 1,000 Units
Bens.............................................................. 3,000 units 2,500 units 150
Bimmer......................................................... 1,000 2,000 375

The direct labor cost is budgeted at $16 per hour and variable factory overhead at $12 per hour of direct labor.
Fixed factory overhead, estimated to be $120,000, is a joint cost and is not allocated to specific products in
developing the manufacturing budget for internal management use.

Required:

(1) Prepare the production budget.


(2) Prepare the purchases budget.
(3) Prepare the manufacturing budget by product and in total.
Budgeting: Profits, Sales, Costs, and Expenses 217

SOLUTION

(1) Bens Bimmer


Units required to meet sales budget............................................................................ 40,000 20,000
Add desired ending inventories................................................................................... 2,500 2,000
Total units required...................................................................................................... 42,500 22,000
Less estimated beginning inventories.......................................................................... 3,000 1,000
Planned production...................................................................................................... 39,500 21,000

(2) Material X Material Y


(in Pounds) (in Pounds)
Bens.............................................................................................................................. 79,000 158,000
Bimmer......................................................................................................................... 84,000 21,000
163,000 179,000
Add desired ending inventories................................................................................... 6,000 7,500
169,000 186,500
Less estimated beginning inventories.......................................................................... 5,000 6,000
Budgeted quantities of materials purchased................................................................ 164,000 180,500
Budgeted purchase price per pound............................................................................ $ 1.20 $ .60
Budgeted dollar amounts of materials purchased....................................................... $ 196,800 $ 108,300

(3) Manufacturing Budget

Bens Bimmer Total


Materials:
X: 39,500 x 2 x $1.20............................................................. $ 94,800 $ 94,800
21,000 x 4 x $1.20............................................................. $ 100,800 100,800
Y: 39,500 x 4 x $ .60............................................................. 94,800 94,800
21,000 x 1 x $ .60............................................................. 12,600 12,600
$ 189,600 $ 113,400 $ 303,000
Direct labor:
39.5 x 150 x $16....................................................................... $ 94,800 $ 94,800
21 x 375 x 16............................................................................ $ 126,000 126,000
$ 94,800 $ 126,000 $ 220,800
Factory overheadvariable:
39.5 x 150 x $12....................................................................... $ 71,100 $ 71,100
21 x 375 x $12.......................................................................... $ 94,500 94,500
$ 71,100 $ 94,500 $ 165,600
Total variable manufacturing cost................................................... $ 355,500 $ 333,900 $ 689,400
Fixed manufacturing cost................................................................ 120,000
Total manufacturing cost................................................................. $ 809,400
Budgeting: Profits, Sales, Costs, and Expenses 218

PROBLEM

5.
Projected Income Statement and Balance Sheet. The 19B forecast for Elenko Company appears below in the
form of a prospective trial balance:

Elenko Co.
Prospective Trial Balance
December 31, 19B

Cash.............................................................................................................................. 5,000
Accounts receivable..................................................................................................... 15,000
Inventory (1/1/19B, 30,000 units)............................................................................... 6,000
Plant and equipment..................................................................................................... 200,000
Accumulated depreciation........................................................................................... 20,000
Accounts payable......................................................................................................... 10,000
Notes payable (due in 5 yrs.)....................................................................................... 30,000
Common stock............................................................................................................. 50,000
Retained earnings......................................................................................................... 48,000
Sales............................................................................................................................. 200,000
Materials....................................................................................................................... 10,000
Direct labor.................................................................................................................. 20,000
Variable factory overhead............................................................................................ 15,000
Fixed factory overhead................................................................................................ 25,000
Marketing expenses..................................................................................................... 30,000
General and administrative expenses.......................................................................... 18,000
Income tax payable...................................................................................................... ?
Dividends..................................................................................................................... 14,000
358,000 358,000

Adjustments for the change in inventory and for income tax (at 30%) have not been made. The scheduled
production for 19B is 280,000 units, while the sales volume will reach 300,000 units. A full-cost first-in, first-out
inventory system is used.

Required:

(1) Prepare a prospective statement of income and retained earnings for 19B, including the computation of
the cost of the ending inventory.
(2) Prepare a prospective balance sheet for 19B.
Budgeting: Profits, Sales, Costs, and Expenses 219

SOLUTION

(1) Elenko Co.


Prospective Statement of Income and Retained Earnings
For Year Ending December 31, 19B

Revenue:
Sales...................................................................................................................... $ 200,000
Expenses:
Cost of goods manufactured and sold:
Materials.......................................................................................................... $ 10,000
Direct labor...................................................................................................... 20,000
Variable factory overhead............................................................................... 15,000
Fixed factory overhead.................................................................................... 25,000
$ 70,000
Beginning inventory........................................................................................ 6,000
$ 76,000
Ending inventory............................................................................................. 2,5001 73,500
Gross profit................................................................................................................... $ 126,500
Marketing expenses..................................................................................................... $ 30,000
General and administrative expenses.......................................................................... 18,000 48,000
Income before income tax........................................................................................... $ 78,500
Income tax (30%)......................................................................................................... 23,550
Net income................................................................................................................... $ 54,950
Beginning retained earnings........................................................................................ 48,000
$ 102,950
Less dividends.............................................................................................................. 14,000
Ending retained earnings............................................................................................. $ 88,950
1
Inventory:
Units:
Beginning inventory........................................................................................ 30,000
Deducted from inventory................................................................................ 20,000
Ending inventory............................................................................................. 10,000

Cost:
Cost per unit: $70,000 manufacturing cost/280,000 units = $.25
Cost of ending inventory: 10,000 units x $.25 = $2,500
Budgeting: Profits, Sales, Costs, and Expenses 220

(2) Elenko Co.


Prospective Balance Sheet
December 31, 19B

Assets
Current assets:
Cash..................................................................................................................... $ 5,000
Accounts receivable............................................................................................ 15,000
Inventory.............................................................................................................. 2,500 $ 22,500
Plant and equipment................................................................................................... $ 200,000
Less accumulated depreciation.................................................................................. 20,000 180,000
Total assets.......................................................................................................... $ 202,500

Liabilities and Shareholders' Equity


Current liabilities:
Accounts payable................................................................................................. $ 10,000
Income tax payable.............................................................................................. 10,350 $ 20,350
Long-term liabilities:
Notes payable...................................................................................................... 30,000
Shareholders' equity:
Common stock..................................................................................................... 50,000
Retained earnings................................................................................................ 102,150
Total liabilities and shareholders' equity............................................................ $ 202,500

PROBLEM

6.
Budgeted Cost of Goods Manufactured and Sold Statement. WKRP, Inc., with $50,000,000 of par stock
outstanding, plans to budget earnings of 10%, before income tax, on this stock. The Marketing Department
budgets sales at $40,000,000. The budget director approves the sales budget and expenses as follows:

Marketing............................................................................................................................................. 20% of sales


Administrative...................................................................................................................................... 10%

Labor is expected to be 50% of the total manufacturing cost; materials issued for the budgeted production will
cost $12,500,000; therefore, any savings in manufacturing cost will have to be in factory overhead. Inventories
are to be as follows:

Beginning of Year End of Year


Finished goods............................................................................................ $8,000,000 $10,000,000
Work in process.......................................................................................... 1,000,000 3,000,000
Materials...................................................................................................... 5,000,000 4,000,000

Required: Prepare the budgeted cost of goods manufactured and sold statement, showing the budgeted purchases
of materials and the adjustments for inventories of materials, work in process, and finished goods.
Budgeting: Profits, Sales, Costs, and Expenses 221

SOLUTION

WKRP, Inc.
Budgeted Cost of Goods Manufactured and Sold Statement
For Year Ending December 31, 19--

Materials:
Beginning inventory........................................................................... $ 5,000,000
Purchases............................................................................................ 11,500,000 5
Materials available for use................................................................. $ 16,500,000
Less ending inventory........................................................................ 4,000,000
Cost of materials used........................................................................ $ 12,500,000
Labor.......................................................................................................... 13,500,000
Factory overhead....................................................................................... 1,000,0004
Total manufacturing cost........................................................................... $ 27,000,0003
Add beginning work in process inventory................................................ 1,000,000
$ 28,000,000
Deduct ending work in process inventory................................................. 3,000,000
Cost of goods manufactured...................................................................... $ 25,000,0002
Add beginning finished goods inventory.................................................. 8,000,000
Cost of goods available for sale................................................................ $ 33,000,000
Deduct ending finished goods inventory.................................................. 10,000,000
Cost of goods sold..................................................................................... $ 23,000,0001
1
Earnings (10% of $50,000,000 = 5,000,000)............................................ 12.5 % of sales
Marketing and administrative expenses..................................................... 30
42.5 % of sales
Cost of goods sold ($23,000,000)............................................................... 57.5
100 % of sales

2
Cost of goods Ending finished Beginning finished Cost of goods
sold + goods inventory - goods inventory = manufactured
$23,000,000 + $10,000,000 - $8,000,000 = $25,000,000

3
Costs of goods Ending work in Beginning work in Total manufacturing
manufactured + process inventory - process inventory = cost (materials, labor,
and factory overhead)
$25,000,000 + $3,000,000 - $1,000,000 = $27,000,000
4
Total Labor (50% of Cost of materials Factory overhead
manufacturing - manufacturing cost) - used =
cost
$27,000,000 - $13,500,000 - $12,500,000 = $1,000,000
5
Cost of Ending materials Beginning Materials purchases
materials used + inventory - materials inventory =
$12,500,000 + $4,000,000 - $5,000,000 = $11,500,000
Budgeting: Profits, Sales, Costs, and Expenses 197

Chapter 14

ACTIVITY ACCOUNTING: ACTIVITY-BASED COSTING AND


ACTIVITY-BASED MANAGEMENT

MULTIPLE CHOICE

A 1. A base used to allocate the cost of a resource to the different activities using that resource is a(n):
A. resource driver
B. activity driver
C. final cost object
D. driver
E. none of the above

B 2. A base used to allocate the cost of products, customers, or other final cost objects is a(n):
A. resource driver
B. activity driver
C. final cost object
D. driver
E. none of the above

B 3. Examples of activities at the batch level of costs include:


A. cutting, painting, and packaging
B. scheduling, setting up, and moving
C. designing, changing, and advertising
D. heating, lighting, and security
E. none of the above

C 4. Examples of activities at the product level of costs include:


A. cutting, painting, and packaging
B. scheduling, setting up, and moving
C. designing, changing, and advertising
D. heating, lighting, and security
E. none of the above

D 5. Examples of activities at the plant level of costs include:


A. cutting, painting, and packaging
B. scheduling, setting up, and moving
C. designing, changing, and advertising
D. heating, lighting, and security
E. none of the above

197
Activity Accounting: Activity-Based Costing and Activity-Based Management 198

A 6. Examples of activities at the unit level of costs include:


A. cutting, painting, and packaging
B. scheduling, setting up, and moving
C. designing, changing, and advertising
D. heating, lighting, and security
E. none of the above

A 7. Examples of unit-level costs are:


A. portions of electricity and indirect materials
B. salaries of schedulers and setup personnel
C. salaries of designers and programmers
D. depreciation and insurance on buildings
E. none of the above

C 8. Examples of product-level costs are:


A. portions of electricity and indirect materials
B. salaries of schedulers and setup personnel
C. salaries of designers and programmers
D. depreciation and insurance on buildings
E. none of the above

D 9. Examples of plant-level costs are:


A. portions of electricity and indirect materials
B. salaries of schedulers and setup personnel
C. salaries of designers and programmers
D. depreciation and insurance on buildings
E. none of the above

B 10. Examples of batch-level costs are:


A. portions of electricity and indirect materials
B. salaries of schedulers and setup personnel
C. salaries of designers and programmers
D. depreciation and insurance on buildings
E. none of the above

A 11. Examples of unit-level activity drivers include:


A. units of output and direct labor hours
B. number of batches and material moves
C. number of products and design changes
D. square footage occupied
E. all of the above

B 12. Examples of batch-level activity drivers include:


A. units of output and direct labor hours
B. number of batches and material moves
C. number of products and design changes
D. square footage occupied
E. all of the above
Activity Accounting: Activity-Based Costing and Activity-Based Management 199

C 13. Examples of product-level activity drivers include:


A. units of output and direct labor hours
B. number of batches and material moves
C. number of products and design changes
D. square footage occupied
E. all of the above

D 14. Examples of plant-level activity drivers include:


A. units of output and direct labor hours
B. number of batches and material moves
C. number of products and design changes
D. square footage occupied
E. all of the above

A 15. Unit-level costs are costs that:


A. inevitably increase whenever a unit is produced
B. are caused by the number of batches produced and sold
C. are incurred to support the number of different products produced
D. are incurred to sustain capacity at a production site
E. none of the above

D 16. Plant-level costs are costs that:


A. inevitably increase whenever a unit is produced
B. are caused by the number of batches produced and sold
C. are incurred to support the number of different products produced
D. are incurred to sustain capacity at a production site
E. none of the above

E 17. Unit-level drivers are:


A. inversely proportional to the volume of output
B. measures of activities that vary with the number of batches produced and sold
C. measures of activity that vary with the number of different products produced and sold
D. for assigning plant-level costs
E. none of the above

A 18. Traditional costing systems are characterized by their use of which of the following measures as
bases for allocating overhead to output:
A. unit-level drivers
B. batch-level drivers
C. product-level drivers
D. plant-level drivers
E. none of the above

E 19. ABC systems are characterized by their use of which of the following measures as bases for
allocating overhead to output:
A. unit-level drivers
B. batch-level drivers
C. product-level drivers
D. plant-level drivers
E. all of the above
Activity Accounting: Activity-Based Costing and Activity-Based Management 200

A 20. All of the following are distinctions that usually exist between traditional and ABC costing systems,
except that:
A. the number of overhead cost pools tends to be lower in ABC systems
B. the number of allocation bases tend to be higher in ABC systems
C. costs within an ABC cost pool tend to be more homogeneous than the costs within a
traditional system's cost pool
D. all ABC systems are two-stage costing systems, while traditional systems may be one- or two-
stage
E. all of the above are distinctions

D 21. All of the following are distinctions that usually exist between traditional and ABC costing systems,
except that:
A. the number of overhead cost pools tends to be higher in ABC systems
B. the number of allocation bases tend to be higher in ABC systems
C. costs within an ABC cost pool tend to be more homogeneous than the costs within a
traditional system's cost pool
D. all ABC systems are one-stage costing systems, while traditional systems may be one- or two-
stage
E. all of the above are distinctions

B 22. Compared to an ABC system, a traditional costing system reports:


A. a lower unit cost for high-volume products and a higher unit cost for low-volume products
B. a higher unit cost for high-volume products and a lower unit cost for low-volume products
C. the same unit costs for high- and low-volume products as does an ABC system
D. either higher or lower unit cost for high-volume products than an ABC system depending
upon the level of fixed costs
E. none of the above

C 23. Activity-based management (ABM) is:


A. a costing system in which multiple overhead cost pools are allocated using bases that include
one or more nonvolume related factors
B. a base used to allocate the cost of a resource to the different activities using it
C. the use of information obtained from ABC to make improvements in the firm
D. a base used to allocate the cost of an activity to products and customers
E. none of the above

A 24. All of the following are ways that activities can be managed to achieve improvements in a process,
except:
A. activity induction
B. activity elimination
C. activity selection
D. activity sharing
E. all of the above are ways in which activities may be managed

C 25. All of the following are examples of non-value-added activities except:


A. ordering
B. receiving
C. assembling
D. inspections
E. setting up
Activity Accounting: Activity-Based Costing and Activity-Based Management 201

PROBLEMS

PROBLEM

1.
Levels of Activity Drivers

Required: Each of the following is a potential activity driver. Identify the most likely level of each activity
driver by writing U for a unit-level driver, B for a batch-level driver, and P for a product-level driver.

1.Number of setups
2.Number of work orders
3.Machine hours
4.Pounds of product
5.Number of part numbers
6.Design hours
7.Number of design changes
8.Marketing promotions
9.Direct materials dollars
10.Loads of materials moved

SOLUTION

1. B 2. B 3. U 4. U 5. P
6. P 7. P 8. P 9. U 10. B

PROBLEM

2.
Distortion of Batch-Level Costs. Maupin Company's existing cost system accumulates all overhead in a single
cost pool and allocates it based on direct labor hours. Last year, overhead costs totaled $1,500,000, and Product A
used 3,000 of the 30,000 total direct labor hours. An ABC study revealed that of the total overhead cost for last
year, $100,000 represented batch-level costs; these batch-level costs are driven by work orders; and a total of 500
work orders were issued, of which 25 were for Product A.

Required: With respect to batch-level costs only, calculate the existing cost system's direction and amount of cost
distortion for Product A.

SOLUTION

The existing system allocated 3,000/30,000 = 10% of all overhead to Product A last year; but A accounted for
only 25/500 = 5% of batch-level activity. So, with respect to batch-level costs only, the existing system
overstated A's cost last year by a total of:

(10% - 5%) x $100,000 = $5,000 overstatement


Activity Accounting: Activity-Based Costing and Activity-Based Management 202

PROBLEM

3.
Value-Added and Non-Value-Added Activities. Sequential Company's sole product, a unique end table made
from lumber, is produced and sold in the following sequence of steps:

(a) wood received and inspected at receiving dock


(b) wood moved to stores inventory
(c) wood moved to Cutting Department
(d) wood cut to size
(e) moved to Planing Department
(f) placed in queue to await planing
(g) wood smoothed and shaped
(h) moved to Inspection Department
(i) inspected
(j) moved to in-process storage area
(k) moved to Assembly Department
(l) various parts of the table are assembled
(m) placed on hand truck to await material handler
(n) moved to Staining Department
(o) tables stained
(p) moved to Inspection Department
(q) inspected
(r) moved to Shipping Department
(s) tables shipped

Required: Which of the steps add value to the product?

SOLUTION

Activities (d), (g), (l), and (o) are the only ones which add value.
Activity Accounting: Activity-Based Costing and Activity-Based Management 203

PROBLEM

4.
Allocation Rates and Driver Rates. The Barre Division of Scranton Company manufactures many high-volume
products and many low-volume products. Selected information follows for Barre's most recent year of operations:

Indirect costs:
Machine related:
Machine operation........................................................................................................................... $ 75,000
Machine setup.................................................................................................................................. 50,000
Total machine overhead........................................................................................................... $ 125,000

Materials related:
Materials handling........................................................................................................................... $ 45,000
Other materials-related.................................................................................................................... 60,000
Total materials overhead.......................................................................................................... $ 105,000
Other overhead................................................................................................................................. $ 190,000
Total overhead................................................................................................................................. $ 420,000

Machine hours......................................................................................................................................... 10,000


Pounds of materials................................................................................................................................. 50,000
Setup hours.............................................................................................................................................. 1,000
Purchase orders........................................................................................................................................ 1,200
Direct labor hours.................................................................................................................................... 25,000

Barre's existing cost system allocates all machine-related overhead based on machine hours and all the remaining
overhead based on direct labor hours. However, a recent study determined that machine setup costs and material
handling costs are primarily related to setup hours, and other materials-related costs are primarily related to the
number of purchase orders issued. Barre does not keep significant materials inventories on hand.

Required:

(1) Calculate the two overhead rates in Barre's existing cost system for the most recent year.
(2) Calculate the overhead (driver) rates that an ABC system would use for the most recent year, making only
the changes suggested by the results of the recent study.
Activity Accounting: Activity-Based Costing and Activity-Based Management 204

SOLUTION

$125,000 of machine-related overhead


= $12 .50 per machine hour
10,000 machine hours

$295,000 of remaining overhead costs


= $11. 80 per direct labor hour
25,000 DLH
(1)

$75,000 of machine operation overhead


= $7 .50 per machine hour
10,000 machine hours

$50,000 of machine-setup overhead + $45,000 of materials handling overhead


= $95 per setup hour
1,000 setup hours
¿
$60,000 of other materials-related cost
= $50 per purchase order
1,200 purchase orders

$190,000 of ¿
= $7 .60 per direct labor hour
25,000 DLH
(2)
Activity Accounting: Activity-Based Costing and Activity-Based Management 205

PROBLEM

5.
Comparison of ABC and Traditional Costing; Two Products. Blaine Company produces two products, Nifty
and So-So, and uses a costing system in which all overhead is accumulated in a single cost pool and allocated
based on machine hours. Blaine's management has decided to implement ABC because a cost study has revealed
significant amounts of overhead cost related to setup activity and design activity. The number of setups and the
number of design hours will be the activity drivers for the two new cost pools, and machine hours will continue as
the base for allocating the remaining overhead. Selected information follows for Blaine Company's most recent
year of operations:

Nifty So-So Total


Units produced................................................................................ 500 15,500 16,000

Direct material cost:


Per unit..................................................................................... $ 200 $ 20
Total......................................................................................... $100,000 $ 310,000 $ 410,000

Machine hours................................................................................. 3,000 47,000 50,000


Direct labor cost.............................................................................. $ 50,000 $ 350,000 $ 400,000
Setups............................................................................................... 120 80 200
Design hours.................................................................................... 6,000 4,000 10,000

Overhead:
Setup-related............................................................................ $ 250,000
Design-related.......................................................................... 350,000
Other......................................................................................... 900,000
Total overhead......................................................................... $ 1,500,000

Required:

(1) Calculate the total and per-unit costs reported for the two products by the existing costing system.
(2) Calculate the total and per-unit costs reported for the two products by the ABC system.
Activity Accounting: Activity-Based Costing and Activity-Based Management 206

SOLUTION

(1) Blaine Company


Product Costs from Existing Cost System

Overhead Rate:
$1,500,000 of overhead divided by 50,000 machine hours = $30 per machine hour

Nifty So-So Total


Direct material................................................................................. $100,000 $ 310,000 $ 410,000
Direct labor...................................................................................... 50,000 350,000 400,000
Overhead:
$30 x 3,000.............................................................................. 90,000
$30 x 47,000............................................................................ 1,410,000 1,500,000
Total cost......................................................................................... $240,000 $ 2,070,000 $ 2,310,000
Units produced................................................................................ 500 15,500
Cost per unit.................................................................................... $ 480.00 $ 133.55

(2) Blaine Company


Product Costs from Activity-Based Costing System

Overhead Rate:
$250,000 setup-related costs divided by 200 setups = 1,250 per setup
$350,000 design-related costs divided by 10,000 design hours = $35 per design hour
$900,000 of other overhead divided by 50,000 machine hours = $18 per machine hour

Nifty So-So Total


Direct material................................................................................. $ 100,000 $ 310,000 $ 410,000
Direct labor...................................................................................... 50,000 350,000 400,000
Overhead:
$1,250 x 120 setups................................................................. 150,000
$1,250 x 80 setups................................................................... 100,000 250,000
$35 x 6,000 design hours......................................................... 210,000
$35 x 4,000 design hours......................................................... 140,000 350,000
$18 x 3,000 machine hours...................................................... 54,000
$18 x 47,000 machine hours.................................................... 846,000 900,000
Total cost......................................................................................... $ 564,000 $ 1,746,000 $
2,310,000
Units produced................................................................................ 500 15,500
Cost per unit.................................................................................... $ 1,128.00 $ 112.65
Activity Accounting: Activity-Based Costing and Activity-Based Management 134

Chapter 10

JUST-IN-TIME AND BACKFLUSHING

MULTIPLE CHOICE

D 1. One of the requirements for a JIT system to be successful is:


A. cyclical production
B. adequate inventory stock
C. coupling it with job order costing
D. high quality and balanced work loads
E. all of the above

A 2. All of the following are terms used to describe the JIT effort to reduce inventories of work in process
and raw materials, except:
A. backflush production
B. stockless production
C. lean production
D. ZIP production
E. none of the above are appropriate terms

C 3. The JIT production ideal is a batch size of:


A. one hundred
B. ten
C. one
D. zero
E. none of the above

C 4. The objective of reducing inventory to zero is possible if all of the following conditions are present,
except:
A. low or insignificant setup costs
B. minimum lead times
C. long setup times
D. balanced and level work loads
E. no interruptions due to stockouts

D 5. The continuing reduction of inventories is achieved by all of the following steps except:
A. inventories are reduced until a problem is discovered
B. once the problem is defined the inventory level is increased to keep the system operating
smoothly
C. the problem is analyzed and practical ways are identified to reduce it
D. once the problem is removed, the inventory level is increased until another problem is
discovered
E. all of the above steps are required

134
Just-in-Time and Backflushing 135

B 6. In a JIT system, velocity is inversely related to:


A. backflushing
B. throughput time
C. acceleration
D. zero inventory production
E. none of the above

D 7. If 500 units are produced per day and 2,000 units are in process at any time, the throughput time is:
A. 1/2 day
B. 1/4 day
C. two days
D. four days
E. none of the above

SUPPORTING CALCULATION:

2,000
= 4 days
500

D 8. In a JIT system, if the rate of output is doubled while the number of units in process is cut in half,
then the speed of the system has been:
A. reduced by 25%
B. doubled
C. reduced by 50%
D. quadrupled
E. none of the above

A 9. Of the following, the only activity that adds value to a product is:
A. processing time
B. moving time
C. waiting time
D. inspection time
E. all of the above

B 10. If the annual carrying cost percentage is 30% and average work in process is $300,000 and
management plans to use JIT to double the velocity of work in process without changing total annual
output, the savings in annual carrying costs will be:
A. $90,000
B. $45,000
C. $150,000
D. $180,000
E. none of the above

SUPPORTING CALCULATION:

30% x 1/2 x $300,000 = $45,000


Just-in-Time and Backflushing 136

A 11. If Step 1 in a production process processes each unit and sends it to await Step 2, and 500 units are
waiting between Steps 1 and 2, how many defective units might Step 1 produce before the problem
is detected in Step 2?
A. 500
B. an unlimited number
C. 250
D. 1,000
E. none of the above

C 12. Assume that a company plans a reduction in work in process levels of 50% and has an annual
inventory carrying cost of 20% and a past average cost of work in process of $75,000. The 50%
reduction in work in process would be expected to produce annual savings of:
A. $37,500
B. $15,000
C. $7,500
D. $3,750
E. none of the above

SUPPORTING CALCULATION:

50% x 20% x $75,000 = $7,500

E 13. Alpha Company has 10 work stations where work in process is held, 100 average units in work in
process per station, an average cost of a unit in work in process of $75, and an annual inventory
carrying cost of 20%. If Alpha plans a 50% reduction in work in process levels, the expected annual
savings in carrying costs would be:
A. $37,500
B. $15,000
C. $30,000
D. $3,750
E. none of the above

SUPPORTING CALCULATION:

50% x 20% x (10 x 100 x $75) = $7,500

B 14. Beta Company has an average dollar loss per defective unit of $25, a planned reduction in number of
defective units produced per out-of-control condition of 5, and the number of out-of-control
conditions not discovered immediately is 250. The expected savings in cost of defects would be:
A. $1,250
B. $31,250
C. $6,250
D. $125
E. none of the above

SUPPORTING CALCULATION:

$25 x 5 x 250 = $31,250


Just-in-Time and Backflushing 137

B 15. Beta Company has an average dollar loss per defective unit of $25, a planned reduction in work in
process levels of 50%, and an average number of units in work in process per station of 100.
Assume that the total number of instances in which some work station goes out of control limits and
produces defects is expected to be 500 annually and that in half those instances the out-of-control
condition is not discovered immediately and enters 10% of the units produced. The expected annual
savings in cost of defective units would be:
A. $1,250
B. $31,250
C. $6,250
D. $125
E. none of the above

SUPPORTING CALCULATION:

$25 x (50% x 100 x 10%) x (1/2 x 500) = $31,250

D 16. The costs to be offset against the savings from lower work in process levels in a JIT system include
all of the following, except:
A. handling a larger number of small batches of work in process
B. the higher probability of shutdowns due to the smaller safety stock
C. the possibility that setup costs cannot be reduced enough to offset the larger number of setups
D. the possibility of customer dissatisfaction due to slower response time to orders
E. all of the above

A 17. Advantages that result from reducing raw materials inventory include all of the following except:
A. a decreased possibility of not being able to produce a unit when required
B. a need for less storage space
C. a reduced risk of obsolescence
D. a reduced risk of damaged materials
E. all of the above are advantages

C 18. Under a JIT approach to purchasing, the ideal number of vendors for each material is:
A. two
B. less than six
C. one
D. as many as can supply quality goods
E. none of the above

E 19. All of the following are obstacles to JIT purchasing, except:


A. the layout of the production process
B. the frequency of schedule changes
C. the attitudes of purchasing agents and suppliers
D. the distance from suppliers
E. all of the above are obstacles
Just-in-Time and Backflushing 138

B 20. JIT purchasing eliminates all of the following documents, except:


A. purchase requisitions
B. blanket purchase orders
C. receiving reports
D. materials requisitions
E. all of the above are eliminated

B 21. All of the following statements apply to a JIT work cell except that:
A. a cell is responsible for the entire production of a product or part
B. every worker in the cell specializes in a single task
C. a cell's workers may be evaluated and rewarded as a team
D. all workers in a cell are responsible for product quality
E. all of the above statements apply

A 22. All of the following are JIT performance measures, except:


A. capacity utilization
B. cycle time efficiency
C. inventory turnover
D. unscheduled maintenance downtime
E. number of defects

E 23. The cost accounting system that is noted for its lack of detailed tracking of work in process during
the accounting period is:
A. process costing
B. job order costing
C. standard costing
D. actual costing
E. backflush costing

A 24. The cost accounting system that would be most apt to use a single inventory account entitled Raw
and In Process (RIP) would be:
A. backflush costing
B. process costing
C. job order costing
D. historical costing
E. standard costing

D 25. To backflush materials cost from Raw and In Process (RIP) to Finished Goods, the calculation would
be:
A. materials in ending RIP inventory plus materials received during the period minus materials
in the beginning RIP inventory
B. materials in ending finished goods inventory plus materials cost transferred from RIP minus
materials in beginning finished goods inventory
C. materials in beginning finished goods inventory plus materials cost transferred from RIP
minus materials in ending finished goods inventory
D. materials in beginning RIP inventory plus materials received during the period minus
materials in ending RIP inventory
E. none of the above
Just-in-Time and Backflushing 139

B 26. Cheeta Company has materials cost in the June 1 Raw and In Process of $10,000, materials received
during June of $205,000 and materials cost in the June 30 Raw and In Process of $12,500. The
amount to be backflushed from Raw and In Process to Finished Goods at the end of June would be:
A. $215,000
B. $202,500
C. $207,500
D. $217,500
E. none of the above

D 27. In backflush costing, if the conversion cost in the Raw and In Process was $500 on July 1 and $1,000
on July 31, the account to be credited at the end of July for the $500 increase would be:
A. Raw and In Process
B. Finished Goods
C. Raw Materials
D. Cost of Goods Sold
E. none of the above

A 28. In backflush costing, if the conversion cost in Raw and In Process was $1,000 on March 1 and $400
on March 31, the account to be credited for the $600 decrease would be:
A. Raw and In Process
B. Finished Goods
C. Raw Materials
D. Cost of Goods Sold
E. none of the above
Just-in-Time and Backflushing 140

PROBLEMS

PROBLEM

1.
Cost Savings From Smaller Inventory. Automated Assembly Company maintains a WIP inventory at each of
15 work stations, and the average size of the inventory is 200 units per station. The physical flow of units into and
out of each WIP location is first-in, first-out. The total number of instances in which some work station goes out
of its control limits is expected to be 100 during the coming year. In 80% of these instances, the out-of-control
condition is expected to be discovered immediately by the operator at that station; in the other 20% of these
instances, a defect will enter 10% of the units produced. These defective units enter WIP between stations, where
they will be discovered by the next station's operator. Every out-of-control condition is corrected as soon as it is
discovered. The average cost of a unit in WIP is $40, and the average loss from an out-of-control condition is $20
per defective unit produced. The annual cost of carrying WIP is 33% of the cost of the inventory.
Management plans to reduce the number of units held at every work station by 50%. The rate of final
output will be unchanged, and no other changes will be made in the system.

Required:

(1) Calculate the expected carrying cost savings from the change planned by the management.
(2) Calculate the expected savings in cost of defects if the changes are implemented.

SOLUTION

(1) Carrying cost savings = 33% x reduction in average cost of WIP


= 33% x 50% x past average cost of WIP
= .33 x .5 x (15 x 200 x $40)
= $19,800
(2) Savings in cost of defects = $20 x reduction in the number of defective units
= $20 x (50% x 200 x 10%) x (.20 x 100)
= $20 x 10 x 20
= $4,000

PROBLEM

2.
Inventory Size, Velocity, and Lead Time. Probtype Incorporated requires an average lead time of 45 days on
customer orders that require parts not kept in stock. When such a customer order is received, the parts order is
placed with a vendor immediately by telephone, and the parts are received in an average of 21 days. The parts are
inspected and put into production an average of three days after receipt. The average time spent in production is
16 days. After production is completed, the order goes through final inspection in two days and arrives at the
customer's site after an additional three days, on average.
Management plans to leave the rate of final output unchanged, induce vendors to reduce their total lead
time by one-third, and reduce the average size of WIP to one-fourth of its present level.
Just-in-Time and Backflushing 141

Required: Assuming management's plans are implemented successfully, calculate the average lead time on
customer orders that require parts not kept in stock.

SOLUTION

The average lead time will be 26 days, calculated as follows:

Reduction of vendor lead time = 1/3 x 21 days = 7 days

Reduction of time in WIP = 3/4 of present time in WIP


= 3/4 x 16 days
= 12 days

New lead time = present lead time - reductions


= 45 days - (7 days + 12 days)
= 26 days

PROBLEM

3.
Comparison of Process Costing and Backflushing; Unit Cost Calculations. BF Company had 35 units in
process, 50% converted, at the beginning of a recent, typical month; the conversion cost component of this
beginning inventory was $525. There were 40 units in process, 50% converted, at the end of the month. During
the month, 5,000 units were completed and transferred to finished goods, and conversion costs of $250,000 were
incurred.

Required:

(1) Carrying calculations to three decimal places, find the conversion cost per unit for the month:

(a) by the average cost method as used in process costing.

(b) by dividing the total conversion cost incurred during the month by the number of units completed
during the month (do not calculate equivalent units).

(c) by dividing the total conversion cost incurred during the month by the number of units started
during the month.

(2) Using the three unit costs from Requirement (1), calculate three amounts for the total conversion cost of
the ending inventory of work in process to the nearest dollar.

(3) In light of the results of Requirement (2), which of the three methods of calculating unit conversion cost
would you recommend for the purpose of inventory costing, 1(a), 1(b), or 1(c)? Why?
Just-in-Time and Backflushing 142

SOLUTION

(1) (a) Equivalent production = 5,000 + (.50 x 40 ) = 5,020 units

$250,525
= $49. 905 per unit
5,020

$250,000
(b) = $50 per unit
5,000

(c) Units started = 5,000 + 40 - 35 = 5,005

$250,000
= $49. 950 per unit
5,005

(2) 40 x .50 x 49.905 = 998


40 x .50 x 50.000 = 1,000
40 x .50 x 49.950 = 999

(3) Considering that the results of Requirement (2) were within two dollars of each other, then method 1(b)
would be recommended because of its ease and simplicity.

PROBLEM

4.
Backflush Costing With a Finished Goods Account. The LanFat Manufacturing Company uses a Raw and In
Process (RIP) inventory account and expenses all conversion costs to the cost of goods sold account. At the end
of each month, all inventories are counted, their conversion cost components are estimated, and inventory account
balances are adjusted accordingly. Raw material cost is backflushed from RIP to Finished Goods. The following
information is for the month of August:

Beginning balance for RIP account, including $4,800 of conversion cost........................................... $ 43,500
Raw materials received on credit........................................................................................................... 680,000
Ending RIP inventory per physical count, including $5,300 conversion
cost estimate.................................................................................................................................... 47,200

Required: Prepare all journal entries involving the RIP account.


Just-in-Time and Backflushing 143

SOLUTION

Journal entries involving the RIP account are:

Raw and In Process...................................................................................................... 680,000


Accounts Payable.................................................................................................. 680,000

This is a summary entry for all receipts of raw materials during the period. As direct materials are used, no entry
is needed, because they remain a part of RIP.

Finished Goods............................................................................................................. 676,800


Raw and In Process............................................................................................... 676,800

This entry backflushes material cost from RIP to Finished Goods. This is a postdeduction. The calculation is:

Material in August 1 RIP balance................................................................................ $ 38,700


Material received during August................................................................................. 680,000
$ 718,700
Material in August 31 RIP, per physical count........................................................... 41,900
Amount to be backflushed........................................................................................... $ 676,800

Raw and In Process...................................................................................................... 500


Cost of Goods Sold................................................................................................ 500

Conversion cost in RIP is adjusted from the $4,800 of August 1 to the $5,300 estimate at August 31. The
offsetting entry is made to Cost of Goods Sold, where all conversion costs were charged during August.

PROBLEM

5.
Backflush Costing With No Finished Goods Account. The ATM Manufacturing Company produces only for
customer order, and most work is shipped within twenty-four hours of the receipt of an order. ATM uses a Raw
and In Process (RIP) inventory account and expenses all conversion costs to the cost of goods sold account. At
the end of each month, inventory is counted, its conversion cost component is estimated, and the RIP account
balance is adjusted accordingly. Raw material cost is backflushed from RIP to Cost of Goods Sold. The
following information is for the month of June:

Beginning balance of RIP account, including $900 of conversion cost............................................... $ 8,500


Raw materials received on credit........................................................................................................... 187,000
Ending RIP inventory per physical count, including $1,100 conversion
cost estimate.................................................................................................................................... 7,900

Required: Prepare all journal entries involving the RIP account.


Just-in-Time and Backflushing 144

SOLUTION

Journal entries involving the RIP account are:

Raw and In Process...................................................................................................... 187,000


Accounts Payable.................................................................................................. 187,000

This is a summary entry for all receipts of raw materials during the period. As direct materials are used, no entry
is needed because they remain a part of RIP.

Cost of Goods Sold....................................................................................................... 187,800


Raw and In Process............................................................................................... 187,800

This entry backflushes material cost from RIP to Cost of Goods Sold. This is a postdeduction. The calculation is:

Material in June 1 RIP balance.................................................................................... $ 7,600


Material received during June...................................................................................... 187,000
$ 194,600
Material in June 30 RIP, per physical count................................................................ 6,800
Amount to be backflushed........................................................................................... $ 187,800

Raw and In Process...................................................................................................... 200


Cost of Goods Sold................................................................................................ 200

Conversion cost in RIP is adjusted from the $900 of June 1 to the $1,100 estimate at June 30. The offsetting entry
is made to Cost of Goods Sold, where all conversion costs were charged during June.

PROBLEM

6.
Backflush Costing; Entries in RIP and Finished Goods. The Clifton Manufacturing Company has a cycle time
of 1.5 days, uses a Raw and In Process (RIP) account, and charges all conversion costs to Cost of Goods Sold. At
the end of each month, all inventories are counted, their conversion cost components are estimated, and inventory
account balances are adjusted. Raw material cost is backflushed from RIP to Finished Goods. The following
information is for May:

Beginning balance of RIP account, including $600 of conversion cost............................................... $ 5,500


Beginning balance of finished goods account, including $2,000 of
conversion cost................................................................................................................................ 6,000
Raw materials received on credit........................................................................................................... 173,000
Ending RIP inventory per physical count, including $850 conversion
cost estimate.................................................................................................................................... 6,200
Ending finished goods inventory per physical count, including $1,550
conversion cost estimate.................................................................................................................. 4,900

Required: Prepare all the journal entries that involve the RIP account and/or the finished goods account.
Just-in-Time and Backflushing 145

SOLUTION

Raw and In Process...................................................................................................... 173,000


Accounts Payable.................................................................................................. 173,000

Finished Goods............................................................................................................. 172,550


Raw and In Process............................................................................................... 172,550

To backflush material cost from RIP to Finished Goods. This is a postdeduction. The calculation is:

Material in May 1 RIP balance.................................................................................... $ 4,900


Material received during May...................................................................................... 173,000
$ 177,900
Material in May 31 RIP, per physical count................................................................ 5,350
Amount to be backflushed........................................................................................... $ 172,550

Cost of Goods Sold....................................................................................................... 173,200


Finished Goods...................................................................................................... 173,200

To backflush material cost from Finished Goods to Cost of Goods Sold. This is a postdeduction. The calculation
is:

Material in May 1 finished goods................................................................................ $ 4,000


Material backflushed from RIP.................................................................................... 172,550
$ 176,550
Material in May 31 finished goods, per physical count.............................................. 3,350
Amount to be backflushed........................................................................................... $ 173,200

Cost of Goods Sold....................................................................................................... 200


Raw and In Process...................................................................................................... 250
Finished Goods...................................................................................................... 450

Conversion cost in RIP is adjusted from $600 of May 1 to the $850 estimate at May 31. Conversion cost in
Finished Goods is adjusted from the $2,000 at May 1 to the $1,550 estimate at May 31.
Just-in-Time and Backflushing 172

Chapter 13

FACTORY OVERHEAD: DEPARTMENTALIZATION

MULTIPLE CHOICE

Question Nos. 10, 11-14, and 26 are AICPA adapted.


Question Nos. 15-28 and 23-25 are CIA adapted.

C 1. A department that would be classified as a producing department is:


A. Production Control
B. Utilities
C. Finishing
D. Medical
E. Shipping

B 2. A department that would be classified as a service department is:


A. Refining
B. Receiving
C. Mixing
D. Assembly
E. Finishing

A 3. In determining the right method for allocating equipment depreciation to departments, the best
recommendation is to:
A. use the cost of equipment in the department as a basis for allocation
B. allocate on the basis of square footage used in a given department
C. charge the amounts to General Plant
D. use algebraic techniques
E. allocate on the basis of companywide rates

A 4. The most reasonable basis for allocating worker's compensation insurance is:
A. departmental payroll
B. building depreciation
C. kilowatt-hours
D. number of employees
E. materials used

E 5. A company is attempting to allocate the costs of electricity in various departments. The variable
portion of electricity expense is to be allocated using kilowatt-hours. The information needed in
order to allocate the fixed portion of the current period's electricity expense is:
A. rated horsepower of equipment
B. number of machines in each department
C. estimated materials consumption
D. number of employees
E. square footage in each department

172
Factory Overhead: Departmentalization 173

E 6. The method for allocating service department costs that requires the least clerical work is:
A. use of square footage in each department
B. step method
C. allocation to other service departments only
D. simultaneous method
E. direct method

E 7. Rapid Falls Corp. has three producing departments, A, B, and C, with 50, 30, and 20 employees,
respectively, in each department. Factory payroll costs other than direct labor are accumulated in a
Payroll Department account and are assigned to producing departments on the basis of number of
employees. The total payroll in each department was: A, $300,000; B, $275,000; C, $325,000; and
Payroll, $50,000. Other costs accumulated in the Payroll Department amounted to $200,000. The
amount of Payroll Department costs chargeable to Department C is:
A. $125,000
B. $100,000
C. $40,000
D. $10,000
E. $50,000

SUPPORTING CALCULATION:

$50,000 + $200,000
= $2,500/employee
50 + 30 + 20

E 8. The following statement that best describes cost allocation is:


A. a company, as a general rule, should allocate indirect costs randomly or based on an "ability-
to-bear" criterion
B. a company can affect total income the most strongly by using the algebraic method of
allocating indirect costs
C. a company can maximize or minimize total company income by selecting different bases on
which to allocate indirect costs
D. a company should select an allocation base to raise or lower reported income on given
products
E. a company's total income will remain unchanged no matter how indirect costs are allocated
Factory Overhead: Departmentalization 174

D 9. Carmichael Manufacturing Company has two production departments (Fabrication and Assembly)
and three service departments (General Factory Administration, Factory Maintenance, and Factory
Cafeteria). A summary of the year's costs and other data for each department prior to allocation of
service department costs appears below.

General Factory )
Fabrication Assembly Administration )
Labor costs.............................................. $1,950,000 $ 2,050,000 $90,000 )
Material costs.......................................... $3,130,000 $ 950,000 --- )
Overhead................................................. $1,650,000 $ 1,850,000 $70,000 )
Direct labor hours................................... 562,500 437,500 31,000 )
Number of employees............................. 280 200 12 )
Square footage occupied......................... 88,000 72,000 1,750 )

( Factory Factory
( Maintenance Cafeteria
( $82,100 $87,000
( $65,000 $91,000
( $56,100 $62,000
( 27,000 42,000
( 8 20
( 2,000 4,800

The costs of the General Factory Administration Department, Factory Maintenance Department, and
Factory Cafeteria are allocated on the basis of direct labor hours, square footage occupied, and
number of employees, respectively. There are no manufacturing overhead variances.
Assuming that Carmichael elects to distribute service department costs under the direct method
of cost allocation, the amount of Factory Maintenance Department costs that would be allocated to
the Fabrication Department is (round all final calculations to the nearest dollar):
A. $106,091
B. $91,440
C. $0
D. $111,760
E. none of the above

SUPPORTING CALCULATION:

$82,100 + $65,000 + $56,100


= $1 . 27 _ 88,000 = $111,760
88,000 + 72,000
Factory Overhead: Departmentalization 175

A 10. Carmichael Manufacturing Company has two production departments (Fabrication and Assembly)
and three service departments (General Factory Administration, Factory Maintenance, and Factory
Cafeteria). A summary of the year's costs and other data for each department prior to allocation of
service department costs appears below.

General Factory )
Fabrication Assembly Administration )
Labor costs.............................................. $1,950,000 $2,050,000 $90,000 )
Material costs.......................................... $3,130,000 950,000 --- )
Overhead................................................. $1,650,000 $1,850,000 $70,000 )
Direct labor hours................................... 562,500 437,500 31,000 )
Number of employees............................. 280 200 12 )
Square footage occupied......................... 88,000 72,000 1,750 )

( Factory Factory
( Maintenance Cafeteria
( $82,100 $87,000
( $65,000 $91,000
( $56,100 $62,000
( 27,000 42,000
( 8 20
( 2,000 4,800

The costs of the General Factory Administration Department, Factory Maintenance Department, and
Factory Cafeteria are allocated on the basis of direct labor hours, square footage occupied, and
number of employees, respectively.
The amount of General Factory Administration Department costs that would be allocated to the
Assembly Department under the direct method is (round all final calculations to the nearest dollar):
A. $70,000
B. $90,000
C. $0
D. $63,636
E. none of the above

SUPPORTING CALCULATION:

$90,000 + $70,000
= $ . 16 _ 437,500 = $70,000
562,500 + 437,500
Factory Overhead: Departmentalization 176

B 11. Carmichael Manufacturing Company has two production departments (Fabrication and Assembly)
and three service departments (General Factory Administration, Factory Maintenance, and Factory
Cafeteria). A summary of the year's costs and other data for each department prior to allocation of
service department costs appears below.

General Factory )
Fabrication Assembly Administration )
Labor costs.............................................. $1,950,000 $2,050,000 $90,000 )
Material costs.......................................... $3,130,000 $950,000 --- )
Overhead................................................. $1,650,000 $1,850,000 $70,000 )
Direct labor hours................................... 562,500 437,500 31,000 )
Number of employees............................. 280 200 12 )
Square footage occupied......................... 88,000 72,000 1,750 )

( Factory Factory
( Maintenance Cafeteria
( $82,100 $87,000
( $65,000 $91,000
( $56,100 $62,000
( 27,000 42,000
( 8 20
( 2,000 4,800

The costs of the General Factory Administration Department, Factory Maintenance Department, and
Factory Cafeteria are allocated on the basis of direct labor hours, square footage occupied, and
number of employees, respectively.
Assuming that Carmichael elects to distribute service department costs to other service
departments using the step method of cost allocation and that the order of distribution is based on the
dollar amount of costs originating in the service departments, how much of the total Factory
Cafeteria cost would be allocated to the Factory Maintenance Department? (Round all final
calculations to the nearest dollar.)
A. $96,000
B. $3,840
C. $6,124
D. $0
E. none of the above

SUPPORTING CALCULATION:

$87,000 + $91,000 + $62,000


= $480 / employee
280 + 200 + 12 + 8
Factory Overhead: Departmentalization 177

B 12. Carmichael Manufacturing Company has two production departments (Fabrication and Assembly)
and three service departments (General Factory Administration, Factory Maintenance, and Factory
Cafeteria). A summary of the year's costs and other data for each department prior to allocation of
service department costs appears below.

General Factory )
Fabrication Assembly Administration )
Labor costs.............................................. $1,950,000 $2,050,000 $90,000 )
Material costs.......................................... $3,130,000 $950,000 --- )
Overhead................................................. $1,650,000 $1,850,000 $70,000 )
Direct labor hours................................... 562,500 437,500 31,000 )
Number of employees............................. 280 200 12 )
Square footage occupied......................... 88,000 72,000 1,750 )

( Factory Factory
( Maintenance Cafeteria
( $82,100 $87,000
( $65,000 $91,000
( $56,100 $62,000
( 27,000 42,000
( 8 20
( 2,000 4,800

The costs of the General Factory Administration Department, Factory Maintenance Department, and
Factory Cafeteria are allocated on the basis of direct labor hours, square footage occupied, and
number of employees, respectively.
How much of the Factory Maintenance Department costs would be allocated to the Factory
Cafeteria under the step method, assuming that the order of distribution is based on the dollar
amount of costs originating in the service departments? (Round all final calculations to the nearest
dollar.)
A. $148,910
B. $0
C. $5,787
D. $5,856
E. none of the above

SUPPORTING CALCULATION:

Factory Cafeteria costs........................................................................................................ $240,000


Factory Maintenance costs.................................................................................................. $203,300
 Factory Cafeteria already closed out.
Factory Overhead: Departmentalization 178

A 13. Acie Company has two service departments and three production departments, each producing a
separate product. For a number of years, Acie has allocated the costs of the service departments to
the production departments on the basis of the annual sales dollars. In a recent audit report, the
internal auditor stated that the distribution of service department costs on the basis of annual sales
dollars would lead to serious inequities. It was recommended that maintenance and engineering
service hours be used as a better service cost allocation basis. For illustration purposes, the
following information was appended to the audit report:

Service Departments )
Maintenance Engineering )
Maintenance hours used....................................... 400 )
Engineering hours used........................................ 400 )
Department direct costs........................................ $12,000 $54,000 )

( Production Departments
( Department A Department B Department C
( 800 200 200
( 800 400 400
( $80,000 $90,000 $50,000

Using the simultaneous method, what would be the total Engineering Department cost after
allocation of interservice department costs, but before allocation to the Maintenance and Production
Departments?
A. $60,000
B. $57,000
C. $12,000
D. $54,000
E. none of the above

SUPPORTING CALCULATION:

Maintenance = $12,000 + .2E


Engineering = $54,000 + .25M
E = $54,000 + .25($12,000 + .2E)
E = $54,000 + $3,000 + .05E
.95E = $57,000
E = $60,000
Factory Overhead: Departmentalization 179

D 14. Acie Company has two service departments and three production departments, each producing a
separate product. For a number of years, Acie has allocated the costs of the service departments to
the production departments on the basis of the annual sales dollars. In a recent audit report, the
internal auditor stated that the distribution of service department costs on the basis of annual sales
dollars would lead to serious inequities. It was recommended that maintenance and engineering
service hours be used as a better service cost allocation basis. For illustration purposes, the
following information was appended to the audit report:

Service Departments )
Maintenance Engineering )
Maintenance hours used....................................... 400 )
Engineering hours used........................................ 400 )
Department direct costs........................................ $12,000 $54,000 )

( Production Departments
( Department A Department B Department C
( 800 200 200
( 800 400 400
( $80,000 $90,000 $50,000

Using the simultaneous method, what would be the total Maintenance Department cost after
allocation of interservice department costs, but before allocation to the Engineering and Production
Departments?
A. $72,000
B. $12,000
C. $60,000
D. $24,000
E. none of the above

SUPPORTING CALCULATION:

Maintenance = $12,000 + .2E


Engineering = $54,000 + .25M
M = $12,000 + .2E
M = $12,000 + .2($54,000 + .25M)
M = $12,000 + $10,800 + .05M
.95M = $22,800
M = $24,000
Factory Overhead: Departmentalization 180

C 15. Acie Company has two service departments and three production departments, each producing a
separate product. For a number of years, Acie has allocated the costs of the service departments to
the production departments on the basis of the annual sales dollars. In a recent audit report, the
internal auditor stated that the distribution of service department costs on the basis of annual sales
dollars would lead to serious inequities. It was recommended that maintenance and engineering
service hours be used as a better service cost allocation basis. For illustration purposes, the
following information was appended to the audit report:

Service Departments )
Maintenance Engineering )
Maintenance hours used....................................... 400 )
Engineering hours used........................................ 400 )
Department direct costs........................................ $12,000 $54,000 )

( Production Departments
( Department A Department B Department C
( 800 200 200
( 800 400 400
( $80,000 $90,000 $50,000

Using the step method of cost allocation, what amount of maintenance cost would be allocated to
Department A, assuming that the service departments are distributed in the order of total dollars of
direct departmental costs?
A. $0
B. $25,500
C. $15,200
D. $3,187.50
E. none of the above

SUPPORTING CALCULATION:

Maintenance = $12,000 + .2($54,000) = $22,800


Department A = 800/1,200 x $22,800 = $15,200
Factory Overhead: Departmentalization 181

B 16. Acie Company has two service departments and three production departments, each producing a
separate product. For a number of years, Acie has allocated the costs of the service departments to
the production departments on the basis of the annual sales dollars. In a recent audit report, the
internal auditor stated that the distribution of service department costs on the basis of annual sales
dollars would lead to serious inequities. It was recommended that maintenance and engineering
service hours be used as a better service cost allocation basis. For illustration purposes, the
following information was appended to the audit report:

Service Departments )
Maintenance Engineering )
Maintenance hours used....................................... 400 )
Engineering hours used........................................ 400 )
Department direct costs........................................ $12,000 $54,000 )

( Production Departments
( Department A Department B Department C
( 800 200 200
( 800 400 400
( $80,000 $90,000 $50,000

Using the step method of cost allocation, what amount of engineering cost would be allocated
directly to Department A, assuming that the service departments are distributed in the order of total
dollars of direct departmental costs?
A. $11,400
B. $21,600
C. $10,800
D. $22,800
E. none of the above

SUPPORTING CALCULATION:

800/2,000 x $54,000 = $21,600

E 17. A factor to be considered in deciding the kinds of departments required for establishing accurate
departmental overhead rates with which to control costs is:
A. location of operations, processes, and machinery
B. responsibilities for production and costs
C. number of departments or cost centers
D. similarity of operations, procedures, and machinery in each department
E. all of the above

E 18. Services available for the benefit of producing departments and other service departments can be
organized by:
A. establishing a separate service department for each function
B. combining several functions into one department
C. placing service costs in a department called "general factory cost pool"
D. none of the above
E. all of the above
Factory Overhead: Departmentalization 182

B 19. Entities that have practiced departmentalization for many years, by grouping their activities into
categories such as occupancy, sales promotion, purchasing, and delivery are:
A. hospitals
B. retail stores
C. banks
D. insurance companies
E. colleges

A 20. An automotive company has three divisions. One division manufactures new replacement parts for
automobiles; another rebuilds engines; and the third does repair and overhaul work on a line of
trucks. All three divisions use the services of a central payroll department. The best method of
allocating the cost of the payroll department to the various operating divisions is:
A. total labor hours incurred in the divisions
B. value of production in the divisions
C. direct materials costs incurred in the divisions
D. machine hours used in the divisions
E. none of the above

B 21. The Janitorial Department provides cleaning services to all departments of a large store.
Management wishes to allocate the janitorial costs to the various departments that benefit from the
service. The most reasonable allocation base for janitorial costs would be:
A. sales of each department
B. square footage of each department
C. number of employees in each department
D. total direct costs of each department before any allocations
E. none of the above

C 22. A hospital has a $100,000 expected utility bill this year. The Janitorial, Accounting, and Orderlies
Departments are service functions to the Operating, Hospital Rooms, and Laboratories Departments.
Floor space assigned to each department is:

Department...................................................................................................... Square Footage


Janitorial.......................................................................................................... 1,000
Accounting...................................................................................................... 2,000
Orderlies.......................................................................................................... 7,000
Operating......................................................................................................... 4,000
Hospital Rooms............................................................................................... 30,000
Laboratories..................................................................................................... 6,000
.............................................................................................................. 50,000

How much of the $100,000 will eventually become the Hospital Rooms Department total costs,
assuming use of the direct method of allocation based on square footage?
A. $60,000
B. $72,000
C. $75,000
D. $80,000
E. none of the above
Factory Overhead: Departmentalization 183

SUPPORTING CALCULATION:

30,000
_ $100,000 = $75,000
40,000

C 23. Serpent Corp. distributes service department overhead costs directly to producing departments
without allocation to the other service department. Information for the month of June is as follows:

Service Departments
Maintenance Utilities
Overhead costs incurred........................................................... $20,000 $10,000
Service provided to department:
Maintenance...................................................................... -- 10%
Utilities............................................................................. 20% --
ProducingA..................................................................... 40% 30%
ProducingB..................................................................... 40% 60%
Totals........................................................................................ 100% 100%

The amount of Maintenance Department costs distributed to ProducingA Department for June was:
A. $8,000
B. $8,800
C. $10,000
D. $11,000
E. none of the above

SUPPORTING CALCULATION:

40%
_ $20,000 = $10,000
80%

D 24. Multiple overhead rates are most commonly used when:


A. production consists of long runs of a single product
B. the company has more than one production department
C. manufacturing operations are labor intensive
D. production consists of a diverse product line
E. none of the above

B 25. An example of a nonvolume-related overhead base would be:


A. direct materials cost
B. number of setups
C. machine hours
D. direct labor dollars
E. none of the above
Factory Overhead: Departmentalization 184

C 26. An example of a department that would be a prime candidate for multiple overhead rates would be
one whose overhead was primarily:
A. labor driven
B. machine related
C. caused by setups and production design changes
D. materials related
E. none of the above
Factory Overhead: Departmentalization 185

PROBLEMS

PROBLEM

1.
Overhead Allocation and Rates. To determine an overhead application rate for its Machining and Assembly
Departments, the management of Knight Co. requested the following overhead cost data for June:

Machining Assembly
Item Department Department Total
Number of employees................................................................ 60 40 100
Square footage............................................................................ 15,000 10,000 25,000
Monthly average wage per employee
(direct and indirect)............................................................ $ 2,000 $ 2,500 --
Overhead directly chargeable to
department (excluding indirect labor)............................... $ 90,000 $ 75,000 $165,000
Materials used............................................................................. 60,000 90,000 150,000
Factory rent................................................................................. ? ? 33,000
Other building costs.................................................................... ? ? 60,000
Payroll Department cost............................................................. ? ? 18,000
Freight-in and other Receiving
Department costs................................................................ ? ? 75,000

In each department, 80% of the employees are direct laborers. Overhead is charged to production on the basis of
direct labor dollars. The allocation basis for other data is as follows: all building costs, square footage; Payroll
Department cost, number of employees; freight-in and other Receiving Department costs, materials used.

Required:

(1) Compute the total overhead chargeable to the Machining and Assembly Departments.
(2) Compute the overhead application rate as a percentage of direct labor cost for each department. (Round to
the nearest whole percent.)
Factory Overhead: Departmentalization 186

SOLUTION

(1) Machining Assembly


Department Department
Overhead directly chargeable............................................................................. $ 90,000 $ 75,000
Indirect labor:
20% x 60 x $2,000........................................................................................ 24,000
20% x 40 x $2,500........................................................................................ 20,000

Factory rent:
15,000
$33,000 x ---------........................................................................................ 19,800
25,000

10,000
$33,000 x ---------........................................................................................ 13,200
25,000

Other building costs:


15,000
$60,000 x ---------........................................................................................ 36,000
25,000

10,000
$60,000 x ---------........................................................................................ 24,000
25,000

Payroll Department cost:


60
$18,000 x ------............................................................................................ 10,800
100

40
$18,000 x ------............................................................................................ 7,200
100

Freight-in and other Receiving Department costs:


$60,000
$75,000 x -----------...................................................................................... 30,000
$150,000

$90,000
$75,000 x -----------...................................................................................... 45,000
$150,000
Total overhead..................................................................................................... $ 210,600 $ 184,400

(2)
Direct labor costs:
80% x 60 x $2,000........................................................................................ $ 96,000
80% x 40 x $2,500........................................................................................ $ 80,000

Overhead as a percentage of direct labor cost:


$210,600/$96,000......................................................................................... 219%
$184,400/$80,000......................................................................................... 231%
Factory Overhead: Departmentalization 187

PROBLEM

2.
Overhead Application; Correction of Net Profit (or Loss). Pomeroy Printers Inc. uses job order costing.
Printers' wages are charged to direct labor, while typesetters' wages are charged to overhead and comprise 30% of
applied overhead. Overhead is applied at the rate of 150% of direct labor cost. During July, only two jobs were
started and completed. Relevant data from these jobs were:

Item Job 1776 Job 1865


Materials cost...................................................................................................... $ 5,000 $ 3,000
Direct labor.......................................................................................................... 10,000 8,000
Overhead applied................................................................................................. 15,000 12,000
Total cost of job............................................................................................ $ 30,000 $ 23,000
Selling price......................................................................................................... 30,000 35,000
Gross profit from job.................................................................................... $ 0 $ 12,000

Management determines that the typesetters' wages should be a direct labor cost and that Job 1776 required 1/3 of
the total typesetting cost incurred, while Job 1865 required 2/3.

Required:

(1) Determine the total typesetters' wages for July.


(2) Determine the corrected direct labor costs for Jobs 1776 and 1865.
(3) Determine the correct gross profit (or loss) for each job. (Round the new overhead rate to the nearest
whole percent and the total overhead to the nearest dollar.)

SOLUTION

(1) $8,100 [30% x ($15,000 + $12,000)]

(2) Job 1776 Job 1865


$ 10,000 $ 8,000
+ 2,700 (1/3 x $8,100) + 5,400 (2/3 x $8,100)
$ 12,700 $ 13,400

(3)
Item Job 1776 Job 1865
Materials cost...................................................................................................... $ 5,000 $ 3,000
Direct labor.......................................................................................................... 12,700 13,400
Overhead applied 1............................................................................................... 9,144 9,648
Total cost of job............................................................................................ $ 26,844 $ 26,048
Selling price......................................................................................................... 30,000 35,000
Gross profit from job.................................................................................... $ 3,156 $ 8,952

1
Total overhead $18,900
= = 72 . 41% or 72% overhead applied
Direct labor cost $12,700 + $13,400
Factory Overhead: Departmentalization 188

PROBLEM

3.
Overhead Distribution Via Direct Method. Geo-trig Inc. has three producing departments (Sine, Cosine, and
Tangent) and two service departments (Rhombus and Triangle). Data that summarize overhead activity for
January are:

Producing Departments Service Departments


Sine Cosine Tangent Rhombus Triangle
Total overhead before service
department allocations........................ $50,000 $80,000 $30,000 $40,000 $20,000
Square footage
occupied.............................................. 3,000 4,000 3,000 1,000 1,500
Number of employees............................... 50 30 20 10 10

Rhombus costs are distributed on the basis of square footage occupied, while Triangle costs are distributed on the
basis of number of employees. The direct method is used for allocating service department costs to producing
departments.

Required: Prepare a schedule indicating the detailed components of overhead costs for the producing and service
departments, including the directly assigned and allocated overhead.
Factory Overhead: Departmentalization 189

SOLUTION

Producing Departments Service Departments


Sine Cosine Tangent Rhombus Triangle
Total overhead before service
department allocations........................ $50,000 $ 80,000 $30,000 $40,000 $20,000
Allocation of Rhombus costs:
(Base = square footage)
3,000
Sine: ---------- x $40,000..................... 12,000 -- -- (12,000)
10,0001

4,000
Cosine: --------- x $40,000.................. -- 16,000 -- (16,000)
10,000

3,000
Tangent: --------- x $40,000................ -- -- 12,000 (12,000)
10,000

Allocation of Triangle costs:


(Base = number of employees)
50
Sine: ------ x $20,000......................... 10,000 -- -- (10,000)
1002

30
Cosine: ---- x $20,000......................... -- 6,000 -- (6,000)
100

20
Tangent: ------ x $20,000.................... -- -- 4,000 (4,000)
100

Total overhead........................................... $72,000 $102,000 $46,000


1
Denominator = 3,000 + 4,000 + 3,000 = 10,000 square feet
or 30% + 40% + 30%
2
Denominator = 50 + 30 + 20 = 100 employees
or 50% + 30% + 20%
Factory Overhead: Departmentalization 190

PROBLEM

4.
Distribution of Direct and Indirect Overhead Costs to Producing Departments. Chaing Chemical Co.
operates with three producing departmentsBlending, Testing, and Terminal. The overhead items and amounts for
the period, along with the bases for their allocation, are listed below.

Item Amount Allocation Basis


Building depreciation......................................................................... $ 24,000 Square footage
Janitorial cost...................................................................................... 33,000 Square footage
Materials receiving cost...................................................................... 48,000 Materials usage
Payroll Department cost..................................................................... 126,000 Number of employees
Power................................................................................................... 75,000 Horsepower of equipment

Other relevant data are:


Blending Testing Terminal
Department Department Department Total
Number of employees......................................... 25 40 19 84
Direct labor hours................................................ 62,000 104,000 54,000 220,000
Horsepower of equipment................................... 60,000 15,000 5,000 80,000
Kilowatt-hours..................................................... 4,000 1,000 6,000 11,000
Square footage..................................................... 2,000 2,000 2,000 6,000
Directly chargeable overhead cost..................... $ 125,000 $ 75,000 $87,500 $287,500
Direct materials.................................................. $ 75,000 $ 25,000 -- $100,000

Required: Prepare the overhead distribution for each producing department, including the detail for each item of
allocated overhead and the overhead rate based on direct labor hours for each department (rounded to the nearest
cent).
Factory Overhead: Departmentalization 191

SOLUTION

Blending Testing Terminal


Department Department Department
Directly chargeable cost................................................................ $125,000 $ 75,000 $ 87,500

Building depreciation:
2,000
$24,000 x ---------.................................................................... 8,000 8,000 8,000
6,000

Janitorial cost:
2,000
$33,000 x ---------.................................................................... 11,000 11,000 11,000
6,000

Materials receiving cost:


$75,000
$48,000 x ------------................................................................ 36,000
$100,000

$25,000
$48,000 x ------------................................................................ -- 12,000 --
$100,000

Payroll Department cost:


25
$126,000 x ----......................................................................... 37,500
84

40
$126,000 x ----......................................................................... -- 60,000 --
84

19
$126,000 x ----......................................................................... -- -- 28,500
84

Power:
60,000
$75,000 x ---------.................................................................... 56,250
80,000

15,000
$75,000 x ---------.................................................................... -- 14,063 --
80,000

5,000
$75,000 x ---------.................................................................... -- -- 4,688
80,000

Total overhead............................................................................... $273,750 $180,063 $139,688

Overhead rate per direct labor hour.............................................. 4.42 1.73 2.59


Factory Overhead: Departmentalization 192

PROBLEM

5.
Overhead Allocation Via the Step Method. Granny's Nut Co. operates with three producing departments
(Cutting, Dividing, and Shelling that are serviced by two service departments Equipment Maintenance and
General Plant). Costs are allocated using the step method with the service department servicing the greatest
number of other departments allocated first. General Plant is allocated on the basis of square footage and
Equipment Maintenance is allocated on the basis of direct labor hours. Relevant May data are:

Producing Departments Service Departments


Equipment General
Cutting Dividing Shelling Maintenance Plant
Overhead before
allocation of service
department costs................................. $105,000 $93,000 $87,000 $56,000 $30,000
Square footage........................................... 8,000 12,000 6,000 4,000 --
Machine hours used................................... 6,000 2,000 7,000 -- --
Direct labor used....................................... 5,000 6,000 9,000 -- --

Required: Prepare a schedule indicating the allocation of service department costs to producing departments and
the rate per machine hour for applying overhead in each producing department. (Round to the nearest cent.)
Factory Overhead: Departmentalization 193

SOLUTION

Producing Departments Service Departments


Equipment General
Cutting Dividing Shelling Maintenance Plant
Overhead before
allocation of service
department costs................................. $105,000 $93,000 $ 87,000 $56,000 $30,000
Allocation of service
department costs:

General Plant:

8,000
--------- x $30,000...................................... 8,000 -- -- -- (8,000)
30,000

12,000
--------- x $30,000...................................... -- 12,000 -- -- (12,000)
30,000

6,000
--------- x $30,000...................................... -- -- 6,000 -- (6,000)
30,000

4,000
--------- x $30,000...................................... -- -- -- 4,000 (4,000)
30,000

Equipment Maintenance:

5,000
--------- x $60,000...................................... 15,000 -- -- (15,000) --
20,000

6,000
--------- x $60,000...................................... -- 18,000 -- (18,000) --
20,000

9,000
--------- x $60,000...................................... -- -- 27,000 (27,000) --
20,000

Total overhead........................................... $128,000 $123,000 $120,000

Machine hours........................................... 6,000 2,000 7,000


Overhead application rate......................... $21.33 $61.50 17.14
Factory Overhead: Departmentalization 194

PROBLEM

6.
Overhead Distribution Via the Simultaneous Method. Orleans Corp. operates two producing departments, C
and D, and two service departments, E and F. The overhead before allocation of service department costs,
together with the usage of services from the service departments, is:

Overhead Before
Allocation of Service Services Provided by
Department Department Costs E F
Producing:
C.............................................................................. $18,000 30% --
D.............................................................................. 29,000 30% 80%
Service:
E.............................................................................. 8,000 -- 20%
F.............................................................................. 1,400 40% --
$56,400

Required: Prepare the overhead distribution, using the simultaneous method to allocate the service departments'
costs to the producing departments.

SOLUTION

Let: E = $8,000 + .2F


F = $1,400 + .4E
Substituting: E = $8,000 + .2($1,400 + .4E)
.92E = $8,280
E = $9,000
Substituting: F = $1,400 + .4E
= $1,400 + .4($9,000)
= $5,000

Distribution of Overhead
Producing Departments Service Departments
C D E F
Overhead before
allocation of service
department costs................................... $18,000 $29,000 $ 8,000 $ 1,400
Distribution of Department E:
$9,000 x 30%........................................ 2,700 2,700 -- --
$9,000 x 40%........................................ -- -- -- 3,600
Total distributed.......................................... -- -- (9,000) --
Distribution of Department F:
$5,000 x 80%........................................ -- 4,000 -- --
$5,000 x 20%........................................ -- -- 1,000 --
Total distributed.......................................... -- -- -- (5,000)
Overhead after distribution......................... $20,700 $35,700 $ 0 $ 0

Proof: $20,700 + $35,700 = $56,400 total


Factory Overhead: Departmentalization 195

PROBLEM

7.
Multiple Overhead Rates. American Manufacturing Inc. (AMI) has a diverse product line with some jobs
requiring much labor and little machine use, and others requiring the opposite mix. Because no single base for a
predetermined overhead rate will provide AMI management with reliable product cost information, overhead is
classified into two cost pools, and two predetermined overhead rates are used. For 19A, it is estimated that total
overhead costs will consist of $200,000 of overhead related to the expenditure of direct labor dollars and $800,000
of overhead related to machine usage. Total machine usage is expected to be 40,000 hours for the year, and total
direct labor dollars are expected to be $400,000.
Job 711 required $1,500 of direct materials, 60 hours of labor at $15 per hour, and 5 hours of machine time.
Job 727 required $2,500 of direct materials, 45 hours of labor at $15 per hour, and 35 hours of machine time.

Required:

(1) Calculate AMI's predetermined overhead rates for 19A.


(2) Determine the total cost of Job 711.
(3) Determine the total cost of Job 727.
(4) If AMI had used a single predetermined overhead rate based on direct labor dollars to apply all overhead
costs, what would have been the predetermined rate?
(5) Based on your computations in (1) and (4) above and considering the two jobs in (2) and (3) above, what
would be the competitive implications of using the single predetermined overhead rate and quoting prices
at cost plus a small markup?
Factory Overhead: Departmentalization 196

SOLUTION

(1) The dual predetermined overhead rates are:

$200,000
= $ . 50 per direct labor dollar
$400,000 direct labor dollars

and

$800,000
= $20 per machine hour
40,000 machine hours

(2) Job 711


Direct material...................................................................................................................................... $ 1,500
Direct labor (60 x $15)......................................................................................................................... 900
Applied overhead:
$900 x $.50 = 450
5 x $ 20 = 100.................................................................................................................. 550
Total...................................................................................................................................................... $ 2,950

(3) Job 727


Direct material...................................................................................................................................... $ 2,500.00
Direct labor (45 x $15)......................................................................................................................... 675.00
Applied overhead:
$675 x $.50 = 337.50
35 x $ 20 = 700.00............................................................................................................. 1,037.50
Total...................................................................................................................................................... $ 4,212.50

(4) A single predetermined overhead rate based on direct labor dollars would be:

$200,000 + $800,000
= $2. 50 per direct labor dollar
$400,000 direct labor dollars

(5) The competitive implications of a single overhead rate are that on jobs requiring much labor and little
machine time (e.g., Job 711), AMI will compute its costs at too high a level and will, therefore, quote too
high a price to the customer. These jobs will probably be lost to competitors who know their costs better.
On jobs requiring much machine time and little labor (e.g., Job 727), AMI will calculate its costs at too
low a level and will, therefore, quote too low a price, but will generate less profit than expected or
perhaps even a loss.
Factory Overhead: Departmentalization 237

Chapter 17

RESPONSIBILITY ACCOUNTING AND REPORTING

MULTIPLE CHOICE

Question Nos. 8, 10, 11-13, and 16-24 are AICPA adapted.


Question No. 21 is ICMA adapted.
Question No. 9 is CIA adapted.

E 1. Internal reports prepared under the responsibility accounting approach should highlight:
A. cost properly allocable to the cost center under generally accepted accounting principles
B. fixed cost of production
C. variable cost of production
D. conversion cost
E. controllable cost

C 2. A company has three producing departments and one service department. Due to a scheduling error
in the service department, an unfavorable variance was created. A sound responsibility accounting
system would dictate that the variance be:
A. ignored
B. allocated to producing departments, but not on the same basis as ordinary charges for use of
the service
C. charged to the service department causing the variance and not allocated to other departments
D. allocated to both producing and service departments
E. allocated to producing departments on the basis of usage

A 3. The control of service department costs at the source level is accomplished by means of:
A. predetermining service requirements in user departments
B. allocating service usage on the basis of priority of need
C. limiting the number of hours of service used
D. organizing maintenance labor
E. limiting the number of hours of service provided

B 4. The rate used to distribute service hours to recipient departments is denoted by all of the following
terms except:
A. sold-hour rate
B. burden rate
C. charging rate
D. transfer rate
E. billing rate

237
Responsibility Accounting and Reporting 238

E 5. The cost item least likely to appear in a performance report based on responsibility accounting
techniques for the supervisor of an assembly line in a large manufacturing situation is:
A. materials
B. repairs and maintenance
C. direct labor
D. other indirect labor
E. supervisor's salary

D 6. Responsibility reports should possess all of the following characteristics except:


A. being issued with regularity
B. fitting the organization chart
C. being consistent in form and content each time they are issued
D. being stated only in dollars for operating management
E. comparing budgeted with actual figures

D 7. Controllable costs are:


A. costs that fluctuate in total in response to small changes in the rate of capacity utilization
B. costs that will be unaffected by current managerial decisions
C. costs that management decides to incur in the current period to enable the company to achieve
objectives other than filling customers' orders
D. costs that are likely to respond to the amount of attention devoted to them by a specified
manager
E. costs that are governed mainly by past decisions that established present levels of operating
and organizational capacity and that change slowly only in response to changes in capacity

D 8. An accounting system in which the operations of the business are broken down into cost centers and
the control function of a supervisor or manager is emphasized is:
A. control accounting
B. budgetary accounting
C. absorption accounting
D. responsibility accounting
E. operations-research accounting

C 9. In a responsibility accounting system, costs are classified into categories on the basis of:
A. prime and overhead costs
B. administrative and nonadministrative costs
C. controllable and noncontrollable costs
D. direct and indirect costs
E. fixed and variable costs

C 10. When used for performance evaluation, periodic internal reports based on a responsibility accounting
system should not:
A. distinguish between controllable and noncontrollable costs
B. be related to the organization chart
C. include allocated fixed overhead
D. include variances between actual and budgeted controllable costs
E. all of the above
Responsibility Accounting and Reporting 239

D 11. The most desirable measure of departmental performance for evaluating the departmental manager is
departmental:
A. contribution to indirect expenses
B. revenue less departmental variable expenses
C. revenue less departmental fixed expenses
D. revenue less controllable departmental expenses
E. net income

D 12. Internal reports prepared under the responsibility accounting approach should be limited to which of
the following costs?
A. only costs properly allocable to the cost center under generally accepted accounting principles
B. only variable costs of production
C. only conversion costs
D. only controllable costs
E. all of the above

B 13. Of most relevance in deciding how or which costs should be assigned to the responsibility center is
the degree of:
A. variability
B. controllability
C. avoidability
D. causality
E. linearity

C 14. A company's only service department provides the following data:

Monthly Service Hours Actual


Service Center Budget Available Monthly Expense
Carpenter Shop $40,000 1,600 $47,800

It serves three producing departments that show the following budgeted and actual cost and
service-hours data:

Estimated Actual
Services Required Services Used

Department No. Carpenter Shop Carpenter Shop


1...................................................................... 350 hrs. 600 hrs.
2...................................................................... 800 hrs. 750 hrs.
3...................................................................... 450 hrs. 650 hrs.

The sold-hour rate for the carpenter shop is:


A. $29.88
B. $20.00
C. $25.00
D. $23.90
E. none of the above

SUPPORTING CALCULATION:

$40,000  1,600 = $25


Responsibility Accounting and Reporting 240

A 15. A company's only service department provides the following data:

Monthly Service Hours Actual


Service Center Budget Available Monthly Expense
Carpenter Shop $40,000 1,600 $47,800

It serves three producing departments that show the following budgeted and actual cost and
service-hours data:

Estimated Actual
Services Required Services Used

Department No. Carpenter Shop Carpenter Shop


1...................................................................... 350 hrs. 600 hrs.
2...................................................................... 800 hrs. 750 hrs.
3...................................................................... 450 hrs. 650 hrs.

The spending variance for the carpenter shop, assuming that 80% of the budgeted expense is fixed,
is:
A. $5,800 unfav.
B. $7,800 unfav.
C. $5,800 fav.
D. $7,800 fav.
E. none of the above

SUPPORTING CALCULATION:

Actual factory overhead......................................................................... $ 47,800


Budget allowance:
Variable ($5 x 2,000)...................................................................... 10,000
Fixed (80% x $40,000)................................................................... 32,000 42,000
Spending variance.................................................................................. $ 5,800

B 16. The primary difference between a fixed (static) budget and a variable (flexible) budget is that a fixed
budget:
A. cannot be changed after the period begins; while a variable budget can be changed after the
period begins
B. is a plan for a single level of sales (or other measure of activity); while a variable budget
consists of several plans, one for each of several levels of sales (or other measure of activity)
C. includes only fixed costs; while variable budget includes only variable costs
D. is concerned only with future acquisitions of fixed assets; while a variable budget is
concerned with expenses that vary with sales
E. none of the above
Responsibility Accounting and Reporting 241

C 17. A flexible budget is:


A. appropriate for control of direct materials and direct labor but not for control of factory
overhead
B. not appropriate when costs and expenses are affected by fluctuations in volume limits
C. appropriate for any relevant level of activity
D. appropriate for control of factory overhead but not for control of direct materials and direct
labor
E. none of the above

B 18. A flexible budget is appropriate for a:

Direct Labor Marketing


Budget Budget
A. yes no
B. yes yes
C. no no
D. no yes

C 19. If a company wishes to establish a factory overhead budget system in which estimated costs can be
derived directly from estimates of activity levels, it should prepare a:
A. discretionary budget
B. fixed budget
C. flexible budget
D. capital budget
E. cash budget

C 20. Flexible budgeting is a reporting system wherein the:


A. statements included in the budget report vary from period to period
B. budget standards may be adjusted at will
C. planned level of activity is adjusted to the actual level of activity before the variance report is
prepared
D. reporting dates vary according to the levels of activity reported upon
E. none of the above

B 21. Flintstone Company uses flexible budgeting for cost control. Flintstone produced 10,800 units of a
product during March, incurring indirect material costs of $13,000. Its static budget for the year
reflected variable indirect material costs of $180,000 at a production volume of 144,000 units. A
flexible budget for March production would reflect indirect material costs of:
A. $13,000
B. $13,500
C. $13,975
D. $11,700
E. none of the above

SUPPORTING CALCULATION:

($180,000  144,000) x 10,800 = $13,500


Responsibility Accounting and Reporting 242

B 22. A company uses a two-way analysis for overhead variances: spending and idle capacity. The idle
capacity variance is based on the:
A. variable overhead application rate
B. fixed overhead application rate
C. semivariable overhead application rate
D. total overhead application rate
E. volume of total expenses at various activity levels

B 23. In analyzing factory overhead variances, an idle capacity variance is the difference between the:
A. master budget application rate and the flexible budget application rate times actual hours
worked
B. budget allowance for actual units produced for the period and the amount of applied factory
overhead
C. actual amount spent for factory overhead items during the period and the amount applied
during the period
D. actual factory overhead incurred and the budget allowance estimated for the capacity used
E. amount shown in the flexible budget and the amount shown in the master budget

B 24. The spending variance for variable overhead based on direct labor hours is the difference between
the actual variable overhead cost and the variable overhead cost that should have been incurred for
the actual hours worked. This variance results from:
A. differences caused by variations in production volume
B. price and quantity differences for overhead costs
C. differences caused by variations in sales volume
D. price differences for overhead costs
E. quantity differences for overhead costs

C 25. In the traditional view of responsibility accounting where individuals are evaluated rather than
operating systems, all of the following dysfunctional results may occur, except:
A. managers tend to take actions that are self-serving rather than beneficial to the company as a
whole
B. managers concentrate on meeting the budget rather than the best level of performance that can
be achieved
C. managers tend to focus their attention on long-run targets and ignore the short-term needs of
the company
D. many competent managers leave the company
E. all of the above may occur

D 26. All of the following are reasons why responsibility reports are of limited use to managers in helping
them to control costs, except:
A. most responsibility accounting systems improperly base allowable budgets on volume-based
measures of activity
B. control data available in a responsibility reporting system are too aggregated to be useful
C. control data available to managers are not easily interpreted by all operating managers
D. control data available to managers is too timely to be precise
E. all of the above are reasons
Responsibility Accounting and Reporting 243

PROBLEMS

PROBLEM

1.
Costs Allocated to Producing Departments; Variance Analysis. Starsky Inc. has two departments providing
service to its producing departmentsthe Building Services Department and the General Plant Department.
Relevant data for June are:

Building Services General Plant


Department Department
Budgeted fixed overhead................................. $50,000 $100,000
Variable overhead............................................ $25 per service hour $15 per direct labor hour
Normal activity level....................................... 10,000 hours per month 50,000 direct labor hours
June activity..................................................... 12,000 hours 45,000 direct labor hours
Actual department costs.................................. $358,000 $755,000

Required:

(1)Compute the predetermined billing rates used for allocating each service department's costs at normal activity.
(2)Compute the costs allocated to the producing departments from each service department, using the
predetermined rates.
(3)Compute the spending and idle capacity variances for each service department.

SOLUTION

(1)Building Services Department: [$50,000 + ($25 x 10,000 hrs.)]/10,000 hrs. = $300,000/10,000 hrs. = $30 per
service hour
General Plant Department: [$100,000 + ($15 x 50,000 hrs.)]/50,000 hrs. = $850,000/50,000 hrs. = $17 per
direct labor hour
(2)Building Services Department: 12,000 hrs. x $30 = $360,000
General Plant Department: 45,000 hrs. x $17 = $765,000
(3)
Building Services General Plant
Department Department
Actual overhead........................................ $ 358,000 $ 755,000
Less overhead allowed for
capacity achieved:
Fixed................................................... $ 50,000 $100,000
Variable
($25 x 12,000 hrs.)...................... 300,000 350,000
($15 x 45,000 hrs.)...................... 675,000 775,000
Spending variance..................................... $ 8,000 unfav. $ (20,000) fav.
Overhead allowed for capacity
achieved............................................. $ 350,000 $ 775,000
Less overhead applied
[from (2)]............................................ 360,000 765,000
Idle capacity variance............................... $ (10,000) fav. $ 10,000 unfav.
Responsibility Accounting and Reporting 244

PROBLEM

2.
Variable Cost Rate; Over- or Underdistributed Variable Cost. Greco Gear Co. has two producing
departmentsAssembly and Finishingand one service departmentUtilities. Allocation of fixed service
department costs is based on readiness-to-serve capacity provided for each department. Variable service
department costs are charged on the basis of actual consumption. These costs are distributed to departments at a
predetermined rate based on variable costs at capacity. Present relevant data are:

Producing Departments
Assembly Finishing
Power consumption (based on kilowatt-hours this month)....................................... 35,000 56,000
Maximum kilowatt-hours required............................................................................ 40,000 60,000

Utilities Department
Budgeted fixed cost (this month)............................................................................... $25,000
Budgeted variable cost at capacity............................................................................ 10,000
Actual variable cost (this month)............................................................................... 8,550

Required:

(1) Compute the rate per kwh used to distribute variable cost.
(2) Compute the distribution of fixed and variable Utilities Department costs for the month.
(3) Compute the over- or underdistributed variable cost and explain what kind of variance it is and who is
responsible for the variance.

SOLUTION

(1) Budgeted variable cost at capacity/Capacity provided = $10,000/(40,000 kwh + 60,000 kwh) = $.10 kwh
for distribution of variable costs
(2)
Producing Departments
Assembly Finishing
Fixed cost distribution:
$25,000 x 40,000 kwh/100,000 kwh................................................................... $10,000
$25,000 x 60,000 kwh/100,000 kwh................................................................... $15,000
Variable cost distribution:
$.10 per kwh x 35,000 kwh................................................................................. 3,500
$.10 per kwh x 56,000 kwh................................................................................. 5,600
Total cost distributed.................................................................................................. $13,500 $20,600
Responsibility Accounting and Reporting 245

(3)
Over- or underdistributed variable cost:
Total variable cost............................................................................................... $8,550
Cost distributed:
Assembly Department.................................................................................. $3,500
Finishing Department................................................................................... 5,600 9,100
Overdistributed cost............................................................................................. $ (550)

Because all of the fixed cost was billed to user departments on the basis of maximum capacity available, there is
no idle capacity variance. The entire variance is a spending variance. The manager of the Utilities Department is
responsible for controlling variable cost; therefore, this variance should appear on the manager's monthly
performance report.

PROBLEM

3.
Over- or Underdistributed Cost; Variance Analysis. Watergate Hotel provides the following data on overhead
costs for its Room Service Division:

Budgeted departmental expenses:


Variable expense.................................................................................................. $ 26,000
Fixed expense...................................................................................................... 15,000
Total departmental expense (direct)............................................................. $ 41,000
Budgeted distributed costs from other departments:
Personnel Department (fixed)............................................................................. 7,000
Food Service Department (variable)................................................................... 32,000
Total departmental overhead........................................................................ $ 80,000

Distribution rate (based on 10,000 calls)................................................................... $ 8 per call to


room service
Actual data for the current period:
Calls to room service........................................................................................... 11,000
Fixed expense...................................................................................................... $ 14,500
Variable expense.................................................................................................. 26,000
Distributed cost:
Personnel Department................................................................................... 7,500
Food Service Department............................................................................. 39,000

Required:

(1) Determine the departmental over- or underdistributed cost.


(2) Determine the spending and idle capacity variances for the Room Service Division's costs, plus the
spending variances as distributed from the other departments. (Round all answers to two decimal places.)
Responsibility Accounting and Reporting 246

SOLUTION

(1)
Cost incurred:
Fixed expense...................................................................... $14,500
Variable expense................................................................. 26,000
Personnel Department cost.................................................. 7,500
Food Service Department cost............................................ 39,000 87,000
Distributed cost (11,000 calls @ $8)......................................... $ 88,000
Overdistributed cost................................................................... $ (1,000)

(2)
Overhead incurred in Room Service Division........................... $40,500
Spending variance............................................................... $ (3,100) fav.

Overhead expected at 11,000 calls:


Fixed ................................................................................... $15,000
Variable: $26,000/10,000 x 11,000.................................... 28,600 43,600
Idle capacity variance.................................................... (1,500) fav.
Applied overhead:
$41,000/10,000 x 11,000..................................................... 45,100
Overabsorbed overhead.............................................................. $ (4,600) fav.
Overhead distributed from other departments:
Personnel Department (fixed):
Actual............................................................................. $ 7,500
Estimated........................................................................ 7,000
Spending variance..................................................... $ 500 unfav.

Food Service Department (variable):


Actual distributed cost................................................... $ 39,000
Cost expected at capacity attained................................. 35,2001
Spending variance..................................................... $ 3,800 unfav.
Total variances from other departments............................. $ 4,300 unfav.
1
$32,000/10,000 x 11,000 = $35,200
Responsibility Accounting and Reporting 247

PROBLEM

4.
Responsibility Report. In April, the vice president of sales of Petro Products asks the controller to prepare a
responsibility report for the performance evaluation of the manager of its Division Y, which is organized into
Sections A and B.

The following cost items related to the operation of Division Y for the month of May, 19-- are presented by the
controller:

Actual Budgeted
Item Cost Cost
Division Y costs:
Staff wages............................................................................................................ $ 20,000 $ 18,500
Supplies................................................................................................................. 6,000 4,800
Manager's salary.................................................................................................... 8,000 6,400
Other expenses...................................................................................................... 15,000 13,400
Total Division Y cost..................................................................................... $ 49,000 $ 43,100

Administration cost allocable to Division Y............................................................... $ 17,000 $ 14,500


Unit outputDivision Y............................................................................................... 10,000 10,000

Section A costs:
Supervisor's salarySection A.............................................................................. 8,000 9,500
Employees' wagesSection A:
Juracek............................................................................................................ 2,000 1,900
Molloy............................................................................................................. 3,500 3,600
Nienaber.......................................................................................................... 3,300 3,250
Oats................................................................................................................. 4,100 4,050
Peterson........................................................................................................... 5,800 5,650
Washington..................................................................................................... 5,000 5,000
Materials costSection A...................................................................................... 4,500 5,200
Indirect laborSection A....................................................................................... 7,800 7,300
Other overhead costsSection A........................................................................... 18,000 19,600
Total Section A costs...................................................................................... $ 62,000 $ 65,050

Section B costs:
Supervisor's salarySection B............................................................................... $ 7,000 $ 7,500
Employees' wagesSection B:
Laurie.............................................................................................................. 4,400 4,350
Potash.............................................................................................................. 3,600 3,800
Tillman............................................................................................................ 2,100 2,050
Other overhead costsSection B........................................................................... 15,000 14,500
Total Section B costs...................................................................................... $ 32,100 $ 32,200

Required: Prepare a responsibility report for the month of May in a format suitable for evaluating the
performance of Division Y's manager.
Responsibility Accounting and Reporting 248

SOLUTION

Petro Products
Responsibility Report
Manager, Division Y
For May, 19--

Actual Over- Under-


Cost Item Cost Budgeted Cost
Division Y costs:
Staff wages................................................................................................. $ 20,000 $ 1,500 U
Supplies...................................................................................................... 6,000 1,200 U
Manager's salary......................................................................................... 8,000 1,600 U
Other expenses........................................................................................... 15,000 1,600 U
Section A cost................................................................................................... 62,000 (3,050) F
Section B cost.................................................................................................... 32,100 (100) F
Total............................................................................................................ $ 143,100 $ 2,750 U

PROBLEM

5.
Flexible Budget. At normal capacity, Boulder Products Corp. manufactures 10,000 trail bikes. At that level, unit
variable costs for the Assembly Department are:

Direct materials............................................................................................................................................ $ 30
Direct labor................................................................................................................................................... 60
Indirect labor................................................................................................................................................ 30
Repairs and maintenance............................................................................................................................. 15
General factory expenses............................................................................................................................. 15
$ 150

Fixed expenses are $150,000 for indirect labor, $175,000 for repairs and maintenance, and $80,000 for general
factory.

Required: Prepare a flexible budget for the Assembly Department at 70%, 80%, 90%, and 100% of normal
capacity.
Responsibility Accounting and Reporting 249

SOLUTION

Assembly Department
Flexible Budget

Percentage of capacity...................................... 70% 80% 90% 100%


Units.................................................................. 7,000 8,000 9,000
10,000
Variable cost:
Direct materials................................................. $ 210,000 $ 240,000 $ 270,000 $ 300,000
Direct labor........................................................ 420,000 480,000 540,000 600,000
Indirect labor..................................................... 210,000 240,000 270,000 300,000
Repairs and maintenance.................................. 105,000 120,000 135,000 150,000
General factory expenses.................................. 105,000 120,000 135,000 150,000
Total variable cost....................................... $ 1,050,000 $ 1,200,000 $ 1,350,000 $ 1,500,000

Fixed cost:
Indirect labor..................................................... 150,000 150,000 150,000 150,000
Repairs and maintenance.................................. 175,000 175,000 175,000 175,000
General factory.................................................. 80,000 80,000 80,000 80,000
Total fixed cost............................................ $ 405,000 $ 405,000 $ 405,000 $ 405,000
Total cost........................................................... $ 1,455,000 $ 1,605,000 $ 1,755,000 $ 1,905,000

PROBLEM

6.
Overhead Analysis; Report to Supervisors. The cost and operating data on June factory overhead for
Department 711 are as follows:

Budgeted Actual
Factory Factory
Overhead Overhead
Variable departmental overhead:
Supplies......................................................................................................................... $ 4,000 $ 3,400
Repairs and maintenance.............................................................................................. 1,600 1,400
Indirect labor................................................................................................................. 8,000 7,400
Power and light.............................................................................................................. 2,400 2,000
Heat................................................................................................................................ 800 600
Subtotal.................................................................................................................... $ 16,800 $ 14,800

Fixed departmental overhead:


Building expense........................................................................................................... $ 1,600 $ 1,520
Depreciationmachinery............................................................................................... 4,800 4,800
Property tax and insurance............................................................................................ 800 760
Subtotal.................................................................................................................... $ 7,200 $ 7,080
Total.................................................................................................................... $ 24,000 $ 21,880

Operating data:
Normal capacity hours.................................................................................................. 4,000
Factory overhead rate per hour..................................................................................... $6
Actual hours.................................................................................................................. 3,600
Responsibility Accounting and Reporting 250

Required: Prepare a departmental report for the supervisor of Department 711 that shows the departmental
spending variance for each item of factory overhead and includes a single idle capacity variance.

SOLUTION

Spending
Variance
Original Budget Actual (Unfav./
Budget Allowance Cost Fav.)
Capacity hours..................................................... 4,000 3,600 3,600
Variable costs:
Supplies................................................................ $ 4,000 $ 3,600 $ 3,400 $ (200)
Repairs and maintenance.................................... 1,600 1,440 1,400 (40)
Indirect labor....................................................... 8,000 7,200 7,400 200
Power and light.................................................... 2,400 2,160 2,000 (160)
Heat...................................................................... 800 720 600 (120)
Subtotal......................................................... $ 16,800 $ 15,120 $ 14,800
Fixed costs:
Building expense................................................. $ 1,600 $ 1,600 $ 1,520 (80)
Depreciationmachinery..................................... 4,800 4,800 4,800 0
Property tax and insurance.................................. 800 800 760 (40)
Subtotal......................................................... $ 7,200 $ 7,200 $ 7,080
Total costs.............................................. $ 24,000 $ 22,320 $ 21,880 $ (440)
Applied factory overhead.................................... 21,600
Idle capacity variance.......................................... $ 720 unfav.

Actual factory overhead...................................... $ 21,880


Applied factory overhead.................................... 21,600
Underapplied factory overhead........................... $ 280

Spending variance............................................... $ (440)


Idle capacity variance.......................................... 720
Underapplied factory overhead........................... $ 280
Responsibility Accounting and Reporting 146

Chapter 11

LABOR: CONTROLLING AND ACCOUNTING FOR COSTS

MULTIPLE CHOICE

Question Nos. 11-12 are AICPA adapted.


Question No. 13 is ICMA adapted.
Question No. 14 is CIA adapted.

C 1. To check the accuracy of hours worked, one would ordinarily compare clock cards with:
A. employee earnings records
B. personnel records
C. job tickets
D. labor variance reports
E. time recorded in the payroll journal

D 2. Uno Manufacturing Corporation has found that the production of a certain product is subject to an
80% learning curve. Production is in lots of 100 units, with 8 hours required for the first lot each
time the product is manufactured. The total time to produce 400 units is:
A. 19.52 hours
B. 24 hours
C. 32 hours
D. 20.48 hours
E. 25.6 hours

SUPPORTING CALCULATION:

Lot Cumulative Average/Lot Total Time


1 8.00 8.00
2 6.40 12.80
4 5.12 20.48

D 3. A company started a new process and during the first week found that the number of units produced
was considerably less than standard. As time progressed, the company noted that production
increased until it reached the standard level several weeks later. After that, there was little
improvement in production rates, which was probably a result of:
A. installation of a standard cost system
B. better incentive plans
C. a labor efficiency variance
D. the learning phenomenon
E. education of newly hired employees

146
Labor: Controlling and Accounting for Costs 147

A 4. A ratio that is employed in connection with the productivity and performance standard to measure
the operating achievement of an operation is the:
A. productivity-efficiency ratio
B. physical output per labor-hour ratio
C. base-rate ratio
D. fringe-cost ratio
E. performance-report ratio

B 5. The document that is used to secure information as to the type of work performed is the:
A. labor voucher
B. time ticket
C. daily efficiency report
D. clock card
E. requisition

E 6. The incentive program that bases an employee's bonus on meeting an objective stated in terms of
time per output unit is:
A. group learning-curve plan
B. 100-percent group bonus plan
C. straight piecework plan
D. Emerson efficiency system
E. 100-percent bonus plan

A 7. An example of a fringe benefit is:


A. the employer portion of FICA tax
B. the direct labor wage rate
C. the straight rate for overtime hours
D. withheld taxes
E. the employee portion of FICA tax

E 8. The company division that is responsible for recording the direct labor cost on the appropriate
production reports and the indirect labor cost on the departmental cost analysis sheets is:
A. Timekeeping Department
B. Production Planning Department
C. Personnel Department
D. Payroll Department
E. Cost Department

B 9. To curtail the wage-price spiral requires that:


A. wage increases must be greater than unit cost increases
B. productivity increases must be greater than or equal to wage increases
C. unit cost increases must be greater than wage increases
D. direct labor cost increases must be in the form of fringe benefits
E. aggregate labor cost increases must not be in the form of fringe benefits
Labor: Controlling and Accounting for Costs 148

D 10. An 80% learning curve was in effect for a certain industry. The first time the task was performed, it
required a time of 800 minutes. When the task was performed for the eighth time, the cumulative
average time per task, rounded to the nearest minute, equaled:
A. 6,400 minutes
B. 800 minutes
C. 512 minutes
D. 410 minutes
E. 3,277 minutes

SUPPORTING CALCULATION:

Task Cumulative Average Time/Task


1 800
2 640 (800 x .8)
4 512 (640 x .8)
8 409.6 (512 x .8)

D 11. The Webb Company's new process will be carried out in one department. The production process
has an expected learning curve of 80%. The costs subject to the learning effect for the first batch
produced by the process were $10,000. Using the simplest form of the learning function, the
cumulative average cost after the sixteenth batch is:
A. $10,000.00
B. $5,120.00
C. $3,276.80
D. $4,096.00
E. $8,000.00

SUPPORTING CALCULATION:

Batch Cumulative Average Cost


1 $10,000
2 $8,000 ($10,000 x .8)
4 $6,400 ($8,000 x .8)
8 $5,120 ($6,400 x .8)
16 $4,096 ($5,120 x .8)

D 12. If a firm is considering the use of learning-curve analysis in the determination of labor cost standards
for a new product, it should be advised that this technique generally is most relevant to situations in
which the production time per unit decreases as additional units are produced and the unit cost:
A. increases or decreases unpredictably
B. increases slightly
C. increases substantially
D. decreases
E. does not change
Labor: Controlling and Accounting for Costs 149

B 13. A construction company has just completed a bridge over the Snake River. This is the first bridge the
company ever built, and it required 100 weeks to complete. Now having hired a bridge construction
crew with some experience, the company would like to continue building bridges. Because of the
investment in heavy machinery needed continuously by this crew, the company believes it would
have to bring the average construction time to less than one year (52 weeks) per bridge to earn a
sufficient return on investment. The average construction time will follow an 80% learning curve.
To bring the average construction time (over all bridges constructed) below one year per bridge, the
crew would have to build approximately:
A. 3 additional bridges
B. 7 additional bridges
C. 8 additional bridges
D. 15 additional bridges
E. 2 additional bridges

SUPPORTING CALCULATION:

Bridges Cumulative Average/Bridge


1 100
2 80 (100 x .8)
4 64 (80 x .8)
8 51 (64 x .8)
 8-1 = 7

E 14. Which of the following may be scheduled in production planning by the use of learning curves?
A. subassembly production
B. delivery dates of finished products
C. labor assignments
D. purchases of materials
E. all of the above

E 15. The assumption(s) that characterizes better human resource management is:
A. there is a vast pool of ideas in the workforce waiting to be tapped
B. people who do the work are best qualified to improve it
C. decision making should be pushed down to the lowest level possible
D. worker participation increases commitment to company objectives
E. all of the above

E 16. To be successful, an incentive wage plan must:


A. provide for proportionately more pay for output above standard
B. set fair standards so that extra effort will result in bonus pay
C. result in immediate reward every payday
D. be applicable to situations in which a worker can increase output
E. all of the above

B 17. The incentive wage plan in which the production standard is computed in minutes per piece and is
then translated into money per piece is the:
A. double-time plan
B. straight piecework plan
C. 100-percent bonus plan
D. group bonus plan
E. none of the above
Labor: Controlling and Accounting for Costs 150

D 18. An incentive wage plan under which a worker's production is divided by the standard quantity,
resulting in an efficiency ratio by which the base wage rate is multiplied, is the:
A. group bonus plan
B. straight commission plan
C. straight piecework plan
D. 100-percent bonus plan
E. none of the above

B 19. The department responsible for recruiting and employment procedures, training programs, job
descriptions, and job evaluations is the:
A. Payroll Department
B. Personnel Department
C. Cost Department
D. Production Planning Department
E. Timekeeping Department

E 20. The department responsible for work scheduling, release of job orders to the producing departments,
and the dispatching of work in the factory is the:
A. Timekeeping Department
B. Payroll Department
C. Personnel Department
D. Cost Department
E. Production Planning Department

C 21. The department that supervises, controls, and collects the clock card and job ticket is the:
A. Cost Department
B. Production Planning Department
C. Timekeeping Department
D. Payroll Department
E. Personnel Department

E 22. The department that records the job classification, department, and wage rate for each employee is
the:
A. Personnel Department
B. Cost Department
C. Production Planning Department
D. Timekeeping Department
E. Payroll Department

A 23. The incentive wage plan in which employee suggestions are the heart of the plan is:
A. organizational incentive plan
B. straight piecework plan
C. 100-percent bonus plan
D. group bonus plan
E. all of the above
Labor: Controlling and Accounting for Costs 151

D 24. The incentive wage plan in which the central theme is that all employees have the capacity to make
valuable contributions to an organization is:
A. group bonus plan
B. 100-percent bonus plan
C. straight piecework plan
D. gainsharing plan
E. none of the above

E 25. The incentive wage plan in which the company sets a predetermined formula and if improvement
above a certain amount occurs, all employees including management participate in the bonus is the:
A. Taylor plan
B. Halsey plan
C. Gantt plan
D. Emerson plan
E. Scanlon plan

A 26. The learning-curve formula is:


A. y = axb
B. y = abx
C. x = aby
D. a = bx + y
E. none of the above

B 27. In highly automated manufacturing where direct labor is small relative to other production costs and
not easily traceable to specific jobs, direct labor costs may be charged directly to:
A. Income Summary
B. Factory Overhead Control
C. Work in Process
D. Cost of Goods Sold
E. none of the above

D 28. Symbols that can be processed electronically to identify numbers, letters, or special characters are:
A. clock cards
B. optical scanners
C. time tickets
D. bar codes
E. none of the above

The following questions are based on the Appendix to the chapter:

E 29. The law establishing the requirement that a company must pay overtime premiums to certain
workers is the:
A. Social Security Act
B. Cost Accounting Standards Act
C. Employment Retirement Income Security Act
D. Overtime Hours Act
E. Fair Labor Standards Act
Labor: Controlling and Accounting for Costs 152

E 30. When scheduling delays occur throughout the week and results in a specific job being completed
during an overtime shift, the overtime premium is charged to:
A. Accrued Overtime Premium Receivable
B. Work in Process
C. the job worked on during the overtime premium
D. an extraordinary loss account
E. Factory Overhead Control

D 31. An employee is paid a base rate of $800 per week for 52 weeks. The employee is entitled to a two-
week vacation each year. Factory Overhead Control is debited each week for accrued vacation pay
of:
A. $15.38
B. $30.77
C. $20
D. $32
E. $16

SUPPORTING CALCULATION:

800 _ 2
= $32
50

E 32. To spread the cost of vacation pay over production throughout the year, the weekly payroll entry
would include a debit to which of the following accounts for the vacation pay portion of the entry?
A. Payroll
B. Liability for Vacation Pay
C. Cash
D. Work in Process
E. Factory Overhead Control

A 33. If an employee earns $10 per hour and receives time-and-a-half for hours worked in excess of 40 per
week, in a week when 45 hours were worked the overtime premium would be:
A. $25
B. $50
C. $10
D. $5
E. none of the above

SUPPORTING CALCULATION:

(1/2 x $10) (45 - 40) = $25


Labor: Controlling and Accounting for Costs 153

A 34. In accordance with the Federal Wage and Hour Law, employee payroll records must show all of the
following except:
A. the breakdown between debits to Work in Process and to Factory Overhead Control
B. total extra pay for overtime worked each week
C. total daily or weekly earnings at straight time
D. total wages paid during each pay period
E. the basis on which wages are paid

B 35. To spread the cost of bonus payments over production throughout the year, the weekly payroll entry
would include a debit to which of the following accounts for the bonus pay portion of the entry?
A. Cash
B. Factory Overhead Control
C. Work in Process
D. Payroll
E. Liability for Bonus Pay

B 36. FASB Statement No. 43 requires an employer to accrue a liability for employees' rights to receive
compensation for future absences when all of the following conditions are met, except:
A. the amount can be reasonably estimated
B. the accrual is for future sick pay benefits in advance of employee absences
C. the rights are attributable to employees' services already rendered
D. the rights vest or accumulate
E. payment is probable

B 37. Every employer subject to unemployment taxes must keep records of all of the following except:
A. the amount of contributions paid into each state unemployment compensation fund
B. the accounting method used to record the tax accrual
C. all information required to be shown on the tax return
D. the total amount of remuneration paid to each employee during the year
E. the total amount of remuneration that constitutes taxable wages

C 38. Cafeteria plans provide for:


A. free meals for all employees who have to work overtime
B. the employer picking up the employee's portion of the FICA tax
C. a specified dollar amount to be apportioned to defray the cost of various available benefits
D. cost of living allowances tied to the Consumer Price Index
E. none of the above

A 39. Vacation pay accruals are allowed for income tax purposes when:
A. it is a vested plan and the amounts are paid within two and one-half months of year's end
B. the rights are attributed to employee services already rendered
C. payment is probable
D. the amount can be reasonably estimated
E. the pay relates to a government contract
Labor: Controlling and Accounting for Costs 154

PROBLEMS

PROBLEM

1.
Labor Costs Under Straight Piecework Plan. The following labor data for the past week were prepared for B.
Masterson, an employee of Boot Hill Corp.:

Day Units Produced Hours Worked


Monday.............................................................................................. 110 8
Tuesday............................................................................................. 125 8
Wednesday........................................................................................ 120 8
Thursday............................................................................................ 135 8
Friday................................................................................................. 130 8

Masterson's wage rate is $15 per hour, and the standard production rate is 15 units per hour.

Required: Determine the daily wages for Masterson and the labor cost per unit for units produced during each day
of the week, assuming that the company is on a straight piecework incentive wage plan and that a worker is
guaranteed a wage of $15 per hour. (Round the unit labor cost to two decimal places.)

SOLUTION

Excess Units Unit Labor


Day Produced Bonus Regular Pay Total Pay Cost
Monday............................ -- -- $120 $120 $1.09
Tuesday............................ 5 $ 5 120 125 1.00
Wednesday....................... -- -- 120 120 1.00
Thursday.......................... 15 15 120 135 1.00
Friday............................... 10 10 120 130 1.00

Base wage rate/standard production rate


= $15 per hour/15 units per hour
= $1 labor cost per unit for units produced each day
Labor: Controlling and Accounting for Costs 155

PROBLEM

2.
Labor Cost Under 100-Percent Bonus Plan. B. Parker, an employee of B. Robber and Company, submitted the
following data for work performed last week:

Units Produced
Day Each Day
Monday............................................................................................................................. 22
Tuesday............................................................................................................................. 24
Wednesday........................................................................................................................ 30
Thursday............................................................................................................................ 21
Friday................................................................................................................................ 27

During the week, Parker worked 8 hours each day and was paid a flat hourly wage of $10, plus a bonus based on
the 100% bonus plan. Standard production is 3 units per hour. The bonus is computed on a daily basis.

Required: Prepare a report for Parker, showing daily earnings, the daily efficiency ratio, and the labor cost per
unit produced each day. (Round labor cost per unit to two decimal places.)

SOLUTION

Daily Daily Labor Cost per Unit


Day Earnings Efficiency Ratio Produced Each Day
Monday............................................................ $ 80 .9167 $3.64
Tuesday............................................................ 80 1.0000 3.33
Wednesday....................................................... 100 1.2500 3.33
Thursday.......................................................... 80 .8750 3.81
Friday............................................................... 90 1.1250 3.33
Labor: Controlling and Accounting for Costs 156

PROBLEM

3.
Effect of Wage Increase on Higher Productivity; Pricing a Unit of Output. Walo Widget Inc. is in the process
of completing labor negotiations for the coming year. Part of these negotiations call for an increase in the base
wage rate for direct labor from $10 to $12 per hour, with a corresponding increase in fringe benefits. At present,
fringe benefits amount to 35% of total wages, and this percentage will remain unchanged with the new contract.
The present labor standards call for 8 direct labor hours per unit of output. Other conversion costs amount to $40
per unit, of which 75% is for variable costs. Materials costs amount to $8 per unit. Administrative costs are fixed
and amount to $10 per unit at the present production level. Products are sold with a gross margin of 30% on sales.

Required:

(1) Compute the current selling price of a unit of output.


(2) Compute the new selling price to be charged if there is no increase in productivity as a result of the new
labor contract.
(3) Compute the selling price to be charged if the new labor contract were accompanied by a 20% increase in
productivity.

(Round all computations to the nearest whole cent.)

SOLUTION

(1) (2) (3)


With Wage With Wage and 20%
Present Increase Productivity Increase
Production costs:
Direct labor cost....................................................... $ 80.001 $ 96.002 $ 80.043
Fringe benefits......................................................... 28.004 33.605 28.016
Variable cost............................................................ 30.007 30.00 30.00
Fixed cost................................................................. 10.008 10.00 8.339
Materials cost........................................................... 8.00 8.00 8.00
Total......................................................................... $ 156.00 $ 177.60 $ 154.38

Selling price 10................................................................ $ 222.86 $ 253.71 $ 220.54


1
$10 per hour x 8 hours = $80 per unit
2
$12 per hour x 8 hours = $96 per unit
3
$12 per hour x (8 hours/1.20 units) = $12 per hour x 6.67 hours = $80.04 per unit
4
35% x $80 unit direct labor cost = $28 per unit
5
35% x $96 unit direct labor cost = $33.60 per unit
6
35% x $80.04 unit direct labor cost = $28.01 per unit
7
75% x $40 = $30 per unit
8
25% x $40 = $10 per unit
9
$10/1.20 units = $8.33 per unit
10
Production costs/(1 - .30 gross profit ratio) = Selling price
Present: $156/.70 = $222.86
With wage increase: $177.60/.70 = $253.71
With wage and 20% productivity increase: $154.38/.70 = $220.54
Labor: Controlling and Accounting for Costs 157

PROBLEM

4.
Learning Curve Effect on Total Cost. Armstrong-Glenn (A-G) Inc. is preparing to bid on the construction of
seven additional rocket carrier frames for launching communication satellites. Under a special contract, the
company has already built one frame with the following costs:

Materials.............................................................................................................................................. $ 800,000
Labor (60,000 hrs.).............................................................................................................................. 750,000
Variable overhead:
50% of direct labor cost................................................................................................................ 375,000
On the basis of materials used...................................................................................................... 150,000
Total..................................................................................................................................................... $ 2,075,000

Variable overhead based on materials used represents materials storage cost. For seven frames, this cost would be
$1,050,000. The company was informed that the maximum acceptable bid is $2,000,000 per unit. However, A-G
will not place a bid unless it can recover its costs plus a $600,000 gross profit per frame. An 80% learning curve
is in effect.

Required:

(1) Determine the total direct labor hours required for all eight frames.
(2) Determine the total cost for the seven frames covered by the new bid.
(3) Determine the profit (or loss) per unit if a bid of $2,000,000 per frame is offered. (Round all amounts to
the nearest whole dollar.)
(4) Should A-G accept the contract at a bid price of $2,000,000 per frame?

SOLUTION

(1) Accumulated Number of Accumulated Average Time


Times Task Is Performed per Task Unit (in Hours)
1 60,000
2 48,000 (60,000 x .8)
4 38,400 (48,000 x .8)
8 30,720 (38,400 x .8)

8 units x 30,720 average hours per unit


= 245,760 total direct labor hours required

(2) Cost for 7 frames covered by bid:


Materials (7 units @ $800,000)...................................................................................... $ 5,600,000
Labor (245,760 total hours - 60,000 hours for first unit)
x (12.50 per hr.)........................................................................................................ 2,322,000
Variable overhead: 50% of $2,322,000.......................................................................... 1,161,000
Materials storage as given............................................................................................... 1,050,000
Total................................................................................................................................. $ 10,133,000

(3) Bid price per unit............................................................................................................. $ 2,000,000


Per-unit cost ($10,133,000/7)......................................................................................... 1,447,571
Gross profit per unit........................................................................................................ $ 552,429

(4) No. The profit per unit will be less than the required profit per unit by $47,571 ($552,429 - $600,000
required profit).
Labor: Controlling and Accounting for Costs 158

PROBLEM

5.
100-Percent Group Bonus Plan. The Assembly Department of the Gladdon Company employs 10 workers on an
8-hour shift at $15 per hour. Production for the second week of May shows: Monday, 350 units; Tuesday, 400
units; Wednesday, 425 units; Thursday, 440 units; Friday, 390 units. The company has recently installed a group
100-percent bonus system with standard production for the group of 50 units per hour. The bonus is computed
each day. The controller asks that an analysis of the week's production costs be made.

Required: Prepare a schedule showing the daily earnings in the department and the unit labor cost. (Round unit
costs to three decimal places.)

SOLUTION

Standard Hours Regular )


Units for Units Actual Group Bonus (Hrs. )
Day.......................Produced Produced Hours Wage Saved @ $15) ) Monday
.......................................350 70 80 $1,200 $ 0 )
Tuesday............................... 400 80 80 1,200 0 )
Wednesday.......................... 425 85 80 1,200 75 )
Thursday.............................. 440 88 80 1,200 120 )
Friday.................................. 390 78 80 1,200 0 )

( Total Labor
( Group Cost per
( Earnings Unit
( $1,200 $3.429
( 1,200 3.000
( 1,275 3.000
( 1,320 3.000
( 1,200 3.077

PROBLEM

6.
Organizational (Gainsharing) Plan. The Humanistic Company employs an organizational incentive plan for its
entire manufacturing facility. For the year 19B, 850 employees were eligible, and each participated equally.
The plan provides for a gainsharing pool totalling 30% of the value of wages being saved. The saving is
computed by determining the prior year's productivity ratio (standard hours for work done divided by total actual
direct and indirect labor hours). This ratio (rounded to six decimal places) is then divided into the standard hours
for the work being done during the current year. The resulting figure is compared to current year's actual direct
and indirect labor hours.

19B 19A
Standard hours for work done...................................................................................... 776,000 725,000
Total actual direct and indirect labor hours................................................................ 1,695,000 1,650,000

The 19B average hourly pay plus labor fringe benefits was $21.
Labor: Controlling and Accounting for Costs 159

Required: Compute the gainsharing incentive, rounded to the nearest dollar, in total and per employee.

SOLUTION

19A productivity ratio =


725,000  1,650,000 = .439394

Hours needed for 19B production at 19A productivity ratio =


776,000  .439394........................................................................................................................ 1,766,069
Less total actual direct and indirect labor hours................................................................................. 1,695,000
Hours saved................................................................................................................................... 71,069

Value of wages saved = 71,069 x $21 = $1,492,449

Employee gainsharing incentive total =


$1,492,449 x 30% = $447,735

Gainsharing incentive per employee =


$447,735  850 employees = $527

The following problem is based on the Appendix to the chapter:

PROBLEM

7.
Fringe Benefits. A production worker earns $3,000 a month, and the company pays one month's salary as a
bonus at the end of the year. The worker is also entitled to a half-month paid vacation, and the company pays
$5,000 a year into a pension fund for the worker. Bonus, vacation pay, and fringe benefits are charged to
production during the 11 1/2 months that the employee is at work. The federal and state unemployment insurance
tax rates are .8% and 5.4%, respectively. The employer's share of FICA tax is 7.5%. All labor-related fringe
benefits for production workers are treated as factory overhead.

Required: Prepare the journal entries to record the February payroll distribution and the cost of fringe benefits.

SOLUTION

Work in Process.............................................................................................................. 3,000.00


Payroll....................................................................................................................... 3,000.00

Factory Overhead............................................................................................................ 1,237.08


FICA Tax Payable ($3,000 x .075).......................................................................... 225.00
Federal Unemployment Tax Payable ($3,000 x .008)............................................. 24.00
State Unemployment Tax Payable ($3,000 x .054)................................................. 162.00
Liability for Bonus ($3,000/11.5)............................................................................ 260.87
Liability for Vacation Pay [(1/2 x $3,000)/11.5]..................................................... 130.43
Liability for Pensions ($5,000/11.5)........................................................................ 434.78
Labor: Controlling and Accounting for Costs 160

Chapter 12

FACTORY OVERHEAD: PLANNED, ACTUAL, AND APPLIED

MULTIPLE CHOICE

Question Nos. 8 and 13-15 are AICPA adapted.


Question Nos. 10 and 12 are ICMA adapted.
Question Nos. 9, 11, and 21 are CIA adapted.

E 1. All of the following phrases are used as alternate terminology for "factory overhead" except:
A. manufacturing expense
B. indirect manufacturing cost
C. factory expense
D. factory burden
E. other expense

C 2. The component of per-unit costs that remains constant as the production level varies is:
A. general and administrative expenses
B. commercial expenses
C. variable factory overhead
D. fixed factory overhead
E. heat, light, and power

D 3. To express factory overhead as a percentage of direct materials dollars, estimated factory overhead is
divided by estimated:
A. machine hours
B. normal capacity
C. units of materials used
D. materials cost
E. materials requisition usage

E 4. Estimated factory overhead is $600,000, and the hours usage of machinery is expected to be
150,000. Factory overhead is applied at the rate of $10 per direct labor hour. The wage rate for
direct labor is $6 per hour, and the total number of estimated direct labor hours for the period is:
A. 100,000
B. 150,000
C. 300,000
D. 600,000
E. 60,000

160
Factory Overhead: Planned, Actual, and Applied 161

SUPPORTING CALCULATION:

$600,000
= $10/ DLHR
x

x = 60,000

B 5. An objection to the use of a factory overhead rate based on direct labor dollars is that:
A. these items are difficult to measure
B. a job is charged with more overhead when a highly paid operator works on the job than when
a low-paid operator performs the work
C. overhead is allocated in relation to units produced by workers
D. overhead rates will be distributed inequitably when there are no wage differentials in the
department
E. costs of applying this method are excessive

D 6. A company expects to produce an average of 75,000 units per year, but last year production equaled
60,000 units. For the coming year, estimated production is 90,000 units. Estimated overhead costs
are $900,000, and overhead is applied at the rate of $10 per unit. The company bases its overhead
rates on:
A. theoretical (engineering) capacity
B. a short-term planning approach
C. historical capacity costs
D. expected actual capacity
E. normal capacity

C 7. Direct costing differs from absorption costing in that:


A. direct materials and direct labor do not become a part of product cost under direct costing
B. the variable portion of overhead cost does not become a part of product cost under direct
costing
C. the fixed portion of overhead cost does not become a part of product cost under direct costing
D. marketing and administrative expenses become a part of product cost under direct costing
E. direct costing does not differ from absorption costing

E 8. Application rates for factory overhead best reflect anticipated fluctuations in sales over a cycle of
years when they are computed under the concept of:
A. practical capacity
B. expected actual capacity
C. theoretical capacity
D. maximum capacity
E. normal capacity

D 9. Underapplied factory overhead related to a significant decrease in production should be charged to:
A. Finished Goods Inventory
B. Cost of Goods Sold
C. Work in Process Inventory and Finished Goods Inventory
D. Work in Process Inventory, Finished Goods Inventory, and Cost of Goods Sold
E. Work in Process Inventory
Factory Overhead: Planned, Actual, and Applied 162

B 10. Brownfield Company applies factory overhead on the basis of direct labor hours. Budget and actual
data for direct labor and overhead for the year are as follows:

..................................................................................................... Budget Actual


Direct labor hours................................................................................... 600,000 550,000
Factory overhead costs........................................................................... $720,000 $640,000

The factory overhead for Brownfield for the year is:


A. underapplied by $40,000
B. overapplied by $20,000
C. overapplied by $40,000
D. underapplied by $20,000
E. neither underapplied nor overapplied

SUPPORTING CALCULATION:

$640,000 ⋅ ( $720,000
600,000
_ 550,000 ) = ($20,000 )

D 11. A company manufactures plastic products for the home and restaurant market. The company also
does contract work for other customers and utilizes a job order costing system. The flexible budget
covering next year's expected range of activity is:

Direct labor hours..................................................... 50,000 80,000 110,000


Machine hours.......................................................... 40,000 64,000 88,000
Variable overhead costs........................................... $100,000 $160,000 $220,000
Fixed overhead costs................................................ 150,000 150,000 150,000
Total overhead costs................................................. $250,000 $310,000 $370,000

A predetermined overhead rate based on direct labor hours at expected actual capacity is used to
apply total overhead. Management has estimated that 100,000 direct labor hours will be used next
year. The predetermined overhead rate per direct labor hour to be used to apply total overhead to
individual jobs next year is:
A. $3.70
B. $3.88
C. $3.36
D. $3.50
E. none of the above

SUPPORTING CALCULATION:

$310,000 + $2(100,000 - 80,000)

$350,000
= = $3 .50
100,000
Factory Overhead: Planned, Actual, and Applied 163

C 12. At the end of the last fiscal year, Tiger Company had the following account balances:

Overapplied Overhead........................................................................................................ $ 1,000


Cost of Goods Sold............................................................................................................. 980,000
Work in Process Inventory................................................................................................. 38,000
Finished Goods Inventory................................................................................................... 82,000

The most common treatment of the Overapplied Overhead would be to:


A. carry it as a deferred credit on the balance sheet
B. report it as a miscellaneous operating revenue on the income statement
C. credit it to Cost of Goods Sold
D. prorate it between Work in Process Inventory and Finished Goods Inventory
E. prorate it among Work in Process Inventory, Finished Goods Inventory, and Cost of Goods
Sold

A 13. Overapplied factory overhead would result if:


A. factory overhead costs incurred were less than costs charged to production
B. factory overhead costs incurred were unreasonably large in relation to units produced
C. factory overhead costs incurred were greater than costs charged to production
D. theoretical capacity were used in computing the overhead rate
E. the plant were operating at less than normal capacity

D 14. Clyde Company found that the differences in product costs resulting from the application of
predetermined overhead rates rather than actual overhead rates were very significant when actual
production was substantially less than planned production. The most likely explanation is that:
A. costs of overhead were substantially less than anticipated
B. overhead was composed chiefly of variable costs
C. several products were produced simultaneously
D. fixed factory overhead was a significant cost
E. costs of overhead items were substantially higher than anticipated

A 15. Avery Co. uses a predetermined factory overhead rate based on direct labor hours. For the month of
October, Avery's budgeted overhead was $300,000 based on a budgeted volume of 100,000 direct
labor hours. Actual overhead amounted to $325,000 with actual direct labor hours totaling 110,000.
How much was the overapplied or underapplied overhead?
A. $5,000 overapplied
B. $5,000 underapplied
C. $30,000 overapplied
D. $30,000 underapplied
E. none of the above

SUPPORTING CALCULATION:

$300,000
(110,000)⋅ $325,000 = $5,000
100,000
Factory Overhead: Planned, Actual, and Applied 164

A 16. The absolute maximum capacity possible under the best conceivable operating conditions is a
description of which type of activity level used in the computation of overhead rates?
A. theoretical
B. normal
C. practical
D. expected actual
E. currently attainable (expected)

B 17. All of the following are terms used to describe the phenomenon measured in the denominator of an
overhead rate, except the:
A. base
B. overhead cost base
C. overhead rate base
D. overhead allocation base
E. all of the above are acceptable terms

B 18. The budget for a given factory overhead cost during a given period was $80,000. The actual cost for
the period was $72,000. Considering these facts, it can be said that the plant manager has done a
better-than-expected job in controlling the cost if:
A. the cost is a discretionary fixed cost and actual production equaled budgeted production
B. the cost is variable and actual production equaled budgeted production
C. the cost is variable and actual production was 90% of budgeted production
D. the cost is fixed and the actual production was less than budgeted production
E. the cost is variable and actual production was 80% of budgeted production

B 19. In highly automated manufacturing, all of the following may be appropriate bases for factory
overhead application except:
A. machine hours
B. direct labor hours
C. number of setups
D. number of inspections
E. movement of materials

E 20. The transactions-base approach to overhead application gives particular consideration to:
A. the amount of direct labor cost
B. the number of machine hours
C. overhead costs that are not driven by volume of output
D. special, high-volume production items
E. homogeneous production processes

D 21. When the amount of overapplied factory overhead is significant, the entry to close Overapplied
Factory Overhead will most likely require:
A. a debit to Cost of Goods Sold
B. debits to Cost of Goods Sold, Finished Goods Inventory, and Work in Process Inventory
C. a credit to Cost of Goods Sold
D. credits to Cost of Goods Sold, Finished Goods Inventory, and Work in Process Inventory
E. none of the above
Factory Overhead: Planned, Actual, and Applied 165

A 22. The type of activity level that results when theoretical capacity is reduced by allowances for
unavoidable interruptions is:
A. practical capacity
B. expected actual capacity
C. normal capacity
D. excess capacity
E. none of the above

D 23. The condition that results either from greater productive capacity than the company could ever hope
to use or from an imbalance in equipment or machinery is termed:
A. theoretical capacity
B. practical capacity
C. idle capacity
D. excess capacity
E. none of the above

B 24. The method of product costing in which only variable overhead is included in the overhead rate is:
A. absorption costing
B. direct costing
C. conventional costing
D. full costing
E. none of the above

C 25. All of the following are names for the product costing method in which both fixed and variable costs
are included in overhead rates, except:
A. absorption costing
B. conventional costing
C. direct costing
D. full costing
E. all of the above
Factory Overhead: Planned, Actual, and Applied 166

PROBLEMS

PROBLEM

1.
Determining Overhead Rate; Expected Actual Capacity Method. Desmond Corp. estimates that its production
for the coming year will be 10,000 widgets, which is 80% of normal capacity, with the following unit costs:
materials, $40; direct labor, $60. Direct labor is paid at the rate of $24 per hour. The widget shaper, the most
expensive piece of machinery, must be run for 20 minutes to produce one widget. Total estimated overhead is
expected to consist of $400,000 for variable overhead and $400,000 for fixed overhead.

Required: Compute the overhead rate for each of the following bases, using the expected actual capacity activity
level:

(1)physical output
(2)materials cost
(3)direct labor cost
(4)direct labor hours
(5)machine hours

(Round all amounts to the nearest whole number.)

SOLUTION

Estimated overhead (EOH ) $800,000


(1) = = $80 overhead per widget
Estimated units of production 10,000 widgets

EOH $800,000
(2) _ 100 = _ 100 = 200% of materials cost
Estimated materials cost $40 _ 10,000 widgets

EOH $800,000
(3) _ 100 = _ 100 = 133% of direct labor cost
Estimated direct labor cost $60 _ 10,000 widgets

EOH $800,000
(4) = = $32 per direct labor hour
Estimated direct labor hours 25,000 hours 1

1
Labor cost per widget $60
= = 2.5 direct labor hours per widget
Labor widget rate $24

2.5 direct labor hours per widget x 10,000 widgets = 25,000 estimated direct labor hours
Factory Overhead: Planned, Actual, and Applied 167

EOH $800,000
(5) = = $240 per machine hour
Estimated machine hours 3,333 hours 1

1
20 minutes per widget
= 1/3 machine hour per widget
60 minutes per hour

1/3 hour per widget x 10,000 widgets = 3,333 estimated machine hours

NOTE: Since the following problem is identical to Problem 1, except for the activity level used, do not test on
both problems at the same time.

PROBLEM

2.
Determining Overhead Rate; Normal Capacity Method. Desmond Corp. estimates that its production for the
coming year will be 10,000 widgets, which is 80% of normal capacity, with the following unit costs: materials,
$40; direct labor, $60. Direct labor is paid at the rate of $24 per hour. The widget shaper, the most expensive
piece of machinery, must be run for 20 minutes to produce one widget. Total estimated overhead is expected to
consist of $400,000 for variable overhead and $400,000 for fixed overhead.

Required: Compute the overhead rate for each of the following bases, using the normal capacity activity level:

(1)physical output
(2)materials cost
(3)direct labor cost
(4)direct labor hours
(5)machine hours

(Round answers to the nearest whole dollar or percentage.)

SOLUTION
Factory Overhead: Planned, Actual, and Applied 168

$800,000
(1) = $64 overhead per widget
12,500 widgets

$800,000
(2) _ 100 = 160% of materials cost
($40 _ 12,500 widgets )

$800,000
(3) _ 100 = 107% of direct labor cost
($60 _ 12,500 widgets )

$800,000
(4) = $25 .60 or $26 per direct labor hour
(2.5 hours _ 12,500 widgets )
Factory Overhead: Planned, Actual, and Applied 169

$800,000
(5) = $192 per machine hour
(1/3 hour _ 12,500 widgets )

PROBLEM

3.
Factory Overhead Application. St. Louis Sounds Inc. manufactures audio equipment. The company estimates
the following costs at normal capacity and other items for the coming period:

Direct materials................................................................................................... $300,000


Direct labor.......................................................................................................... 520,000
Factory overhead (fixed)..................................................................................... 300,000
Factory overhead (variable)................................................................................ 240,000

Normal capacity................................................................................................... 100,000 direct labor hours


Expected production............................................................................................ 80,000 direct labor hours

Required: Compute the overhead application rate for fixed, variable, and total overhead per direct labor hour,
using both the normal capacity and the expected actual capacity activity levels.

SOLUTION

Overhead per Direct Labor Hour

At Expected
Overhead Actual Capacity At Normal Capacity

$300,000 $300,000
Fixed............................................... ----------------- = $3.75 ------------------ = $3.00
80,000 DLH 100,000 DLH

$192,000 $240,000
Variable........................................... ----------------- = 2.40 ------------------ = 2.40
80,000 DLH 100,000 DLH
Total................................................ $6.15 $5.40
Factory Overhead: Planned, Actual, and Applied 170

PROBLEM

4.
Overhead Analysis. Data for the past two years for J&J Corp. are:

19A 19B
Units produced.............................................................................................................. 10,000 11,000
Overhead applied per unit............................................................................................. $ 15 $ 18
Actual overhead:
Fixed....................................................................................................................... 50,000 55,000
Variable.................................................................................................................. 95,000 150,000
Estimated overhead:
Fixed....................................................................................................................... 50,000 56,000
Variable.................................................................................................................. 130,000 142,000

The company determines overhead rates based on estimated units to be produced.

Required:

(1) Determine the estimated units of production used to obtain the overhead allocation rates in 19A and 19B.
(2) Determine the over- or underapplied factory overhead for each of the two years.

SOLUTION
Factory Overhead: Planned, Actual, and Applied 171

Estimated overhead
(1) = Overhead per unit
Estimated units of production

$50,000 + $130,000
19 A
x

$15x = $180,000

x = 12,000 Estimated units of production

$56,000 + $142,000
19B
x

$18x = $198,000

x = 11,000 Estimated units of production


(2)

19A: Applied Factory Overhead (10,000 x $15)............................................................................ $ 150,000


Actual Factory Overhead....................................................................................................... 145,000
Overapplied Factory Overhead.............................................................................................. $ 5,000

19B: Actual Factory Overhead....................................................................................................... $ 205,000


Applied Factory Overhead (11,000 x $18)............................................................................ 198,000
Underapplied Factory Overhead............................................................................................ $ 7,000
Factory Overhead: Planned, Actual, and Applied 172

PROBLEM

5.
Entries for Factory Overhead. Blend Rite Inc. assembles and sells electric mixers. All parts are purchased and
labor is paid on the basis of $22 per mixer assembled. The cost of the parts per mixer totals $20. As the company
handles only this one product, the unit cost basis for applying factory overhead is used. Estimated factory
overhead for the coming period, based on a production of 40,000 mixers, is as follows:

Indirect materials................................................................................................................................. $ 60,000


Indirect labor........................................................................................................................................ 180,000
Light and power................................................................................................................................... 45,000
Depreciation......................................................................................................................................... 35,000
Miscellaneous...................................................................................................................................... 16,000

During the period, 42,000 mixers were assembled and actual factory overhead was $355,000. These units were
completed but not yet transferred to the finished goods storeroom.

Required:

(1) Prepare journal entries to record the above information, including the entry to close the balance in the
applied overhead account to the actual overhead account.
(2) Determine the amount of over- or underapplied factory overhead.

SOLUTION

(1) Work in Process............................................................................................. 840,000


Materials................................................................................................. 840,000

Work in Process............................................................................................. 924,000


Payroll.................................................................................................... 924,000

Factory Overhead Control............................................................................. 355,000


Materials, Payroll, Accruals, and Various Credits................................ 355,000

Work in Process............................................................................................. 352,800


Factory Overhead Applied..................................................................... 352,800

Factory Overhead Applied............................................................................ 352,800


Factory Overhead Control...................................................................... 352,800

Estimated factory overhead $336,000


Overhead rate: = = $8 .40 factory overhead rate per mixer
Estimated production 40,000

(2) Underapplied factory overhead: $355,000 - $352,800 = $2,200


Factory Overhead: Planned, Actual, and Applied 173

PROBLEM

6.
Disposition of Over- or Underapplied Overhead. The following information is available concerning the
inventory and cost of goods sold accounts of PGA Company at the end of the most recent year:

Work in Finished Cost of Goods


Process Goods Sold
Direct material............................................................................ $ 5,000 $ 8,000 $ 11,000
Direct labor................................................................................. 6,000 15,000 15,000
Applied overhead........................................................................ 4,000 12,000 24,000
Year-end balance........................................................................ $ 15,000 $ 35,000 $ 50,000

Applied overhead has already been closed to Factory Overhead Control.

Required:

Give the journal entry required to close Factory Overhead Control, assuming:
(1) Overapplied overhead of $10,000 is to be allocated to inventories and Cost of Goods Sold in proportion to
the balances in those accounts.
(2) Underapplied overhead of $10,000 is to be allocated to inventories and Cost of Goods Sold in proportion
to the amounts of applied overhead contained in those accounts.

SOLUTION

Requirement (1) Requirement (2)


Account Percentage of Applied Percentage of
Balance Total Overhead Total
Work in Process............................................. $ 15,000 15% $ 4,000 10%
Finished Goods.............................................. 35,000 35% 12,000 30%
Cost of Goods Sold........................................ 50,000 50% 24,000 60%
Total........................................................ $ 100,000 100% $ 40,000 100%

(1) Factory Overhead Control............................................................................... 10,000


Work in Process....................................................................................... 1,500
Finished Goods........................................................................................ 3,500
Cost of Goods Sold.................................................................................. 5,000

(2) Work in Process.............................................................................................. 1,000


Finished Goods................................................................................................ 3,000
Cost of Goods Sold......................................................................................... 6,000
Factory Overhead Control....................................................................... 10,000
Factory Overhead: Planned, Actual, and Applied 222

Chapter 16

BUDGETING: CAPITAL EXPENDITURES, RESEARCH AND


DEVELOPMENT EXPENDITURES, AND CASH; PERT/COST

MULTIPLE CHOICE

Question Nos. 12-15 and 17-19 are AICPA adapted.


Question Nos. 11, 21-22, and 25 are ICMA adapted.
Question Nos. 10, 16, 20, 23, and 24 are CIA adapted.

C 1. In a Program Evaluation and Review Technique system (PERT), reducing total time can be
accomplished only by:
A. adding another shift
B. shortening a slack path
C. shortening the critical path
D. working overtime
E. using sensitivity analysis

C 2. The type of research a company undertakes for modifying existing finished goods so as to enhance
or at least maintain its competitive position by providing better quality or performance is known as:
A. basic research
B. safety, health, and convenience research
C. product improvement
D. new product development
E. capacity improvement

C 3. A useful and absolutely essential tool that is used by management to determine payments for bond
requirements, income tax installments, and pension and retirement funds is the:
A. production budget
B. projected or forecast income statement
C. cash budget
D. expense budget
E. capital expenditures budget

D 4. At the beginning of a budget period, prepaid rent was $3,000. Rent expense for the period is
expected to equal $18,000, while prepaid rent at the end of the period is expected to equal $2,000.
The cash required for the rent payments is:
A. $19,000
B. $18,000
C. $20,000
D. $17,000
E. $23,000

222
Budgeting: Capital, Research and Development Expenditures, and Cash; PERT/Cost 223

SUPPORTING CALCULATION:

$18,000 - ($3,000 - $2,000) = $17,000

D 5. In preparing a cash budget, the data concerning cash requirements for dividends and loans is most
likely found in the:
A. expense budget
B. sales budget
C. plant and equipment budget
D. treasurer's budget
E. budgeted balance sheet

D 6. The planning method whose major use is in the determination of the longest time duration for the
completion of an entire project is:
A. probabilistic budgets
B. the fiscal responsibility system
C. zero-base budgeting
D. PERT or CPM
E. PPBS

B 7. The planning procedure that is used principally in governmental and nonprofit agencies and requires
a manager to justify an entire budget rather than just budget increases is:
A. cash forecasting
B. zero-base budgeting
C. the fiscal responsibility system
D. PERT
E. PPBS

B 8. In using the PERT system and estimating the expected time for each activity, the formula requires
that the optimistic time value be given a weighting of:
A. 1/4
B. 1
C. 4
D. 6
E. 1/2

C 9. The estimated times for the completion of an activity are: optimistic, 2 days; most likely, 6 days;
and pessimistic, 16 days. The expected time would then be:
A. 6 days
B. 8 days
C. 7 days
D. 16 days
E. none of the above

SUPPORTING CALCULATION:

2 + 4 (6) + 16
=7
6
Budgeting: Capital, Research and Development Expenditures, and Cash; PERT/Cost 224

E 10. Zero-base budgeting:


A. emphasizes the relationship of effort to projected annual revenues
B. involves the review of changes made to an organization's original budget
C. does not provide a projection of annual expenditures
D. is a method peculiar to budgeting by program
E. involves the review of each cost component from a cost/benefit perspective

A 11. A budget system referred to as the "planning, programming, budgeting system (PPBS)":
A. classifies budget requests by activity and estimates the benefits arising from each activity
B. presents the plan for a range of activity so that the plan can be adjusted for changes in activity
levels
C. drops the current month or quarter and adds a future month or a future quarter as the current
month or quarter is completed
D. consolidates the plans of the separate requests into one overall plan
E. divides the activities of individual responsibility centers into a series of packages that are
ranked ordinally

A 12. The E. Mundo Company is preparing its cash budget for the month of May. The following
information is available concerning its accounts receivable:

Estimated credit sales for May.................................................................................... $ 200,000


Actual credit sales for April......................................................................................... $ 150,000
Estimated collections in May for credit sales in May................................................. 20%
Estimated collections in May for credit sales in April................................................ 70%
Estimated collections in May for credit sales prior to April....................................... $ 20,000
Estimated write-offs in May for uncollectible credit sales......................................... $ 8,000
Estimated provision for bad debts in May for credit sales in May............................. $ 7,000

The estimated cash receipts from accounts receivable collections in May are:
A. $165,000
B. $157,000
C. $158,000
D. $150,000
E. none of the above

SUPPORTING CALCULATION:

($200,000 x .2) + ($150,000 x .7) + $20,000 = $165,000


Budgeting: Capital, Research and Development Expenditures, and Cash; PERT/Cost 225

B 13. Schmidlap Company is preparing its cash budget for the month of April. The following information
is available concerning its inventories:

Inventories at beginning of April................................................................................. $ 90,000


Estimated purchases for April...................................................................................... 440,000
Estimated cost of goods sold for April........................................................................ 450,000
Estimated payments in April for purchases in March................................................. 75,000
Estimated payments in April for purchases prior to March........................................ 30,000
Estimated payments in April for purchases in April................................................... 75%

The estimated cash disbursements for inventories in April are:


A. $411,250
B. $435,000
C. $405,000
D. $442,500
E. none of the above

SUPPORTING CALCULATION:

$75,000 + $30,000 + ($440,000 x .75) = $435,000

D 14. Shula, Inc. is preparing its cash budget for the month of November. The following information is
available concerning its inventories:

Inventories at beginning of November........................................................................ $ 180,000


Estimated cost of goods sold for November................................................................ 900,000
Estimated inventories at end of November................................................................. 160,000
Estimated payments in November for purchases prior to November......................... 225,000
Estimated payments in November for purchases in November.................................. 80%

The estimated cash disbursements for inventories in November are:


A. $704,000
B. $1,057,000
C. $945,000
D. $929,000
E. none of the above

SUPPORTING CALCULATION:

($880,000* x .8) + 225,000 = $929,000


*$180,000 + x - $160,000 = $900,000
x = $880,000

D 15. A formal diagram of the interrelationships of complex time series of activities is:
A. linear programming
B. Poisson distribution models
C. Monte Carlo models
D. PERT
E. the method of least squares
Budgeting: Capital, Research and Development Expenditures, and Cash; PERT/Cost 226

D 16. The most appropriate technique for determining the longest time required to complete a particular
project would be:
A. integer programming
B. game theory
C. queuing theory
D. Program Evaluation and Review Technique (PERT)
E. regression analysis

E 17. Program Evaluation and Review Technique (PERT) is a system that uses:
A. probabilistic budgets
B. least squares method
C. linear programming
D. economic order quantity formula
E. network analysis and critical path methods (CPM)

D 18. In a Program Evaluation and Review Technique (PERT) system, activities along the critical path:
A. intersect at a corner point described by the feasible area
B. may be delayed without affecting completion time
C. follow the line of best fit
D. have a slack of zero
E. have a positive slack

A 19. The quantitative technique that would be most useful for analyzing the interrelationships of time and
activities to discover potential bottlenecks is:
A. Program Evaluation and Review Technique (PERT)
B. regression analysis
C. probabilistic budgeting
D. queuing theory
E. linear programming

E 20. The use of PERT or CPM might apply when planning for:
A. the installation of a new computer system
B. the development of a new product
C. the construction of a new office building
D. project development
E. all of the above

D 21. A factory has several small construction and repair projects for the maintenance crew to perform.
There are a limited number of painters, woodworkers, and electricians. The method that will help
provide the fastest completion of all jobs is:
A. transportation algorithms
B. queuing theory
C. time-series analysis
D. PERT/CPM analysis
E. linear programming
Budgeting: Capital, Research and Development Expenditures, and Cash; PERT/Cost 227

B 22. Critical Path Method (CPM) is a technique for analyzing, planning, and scheduling large, complex
projects by determining the critical path from a single time estimate for each event in a project. The
critical path:
A. is the shortest time path from the first event to the last event for a project
B. is the longest time path from the first event to the last event for a project
C. is the maximum amount of time an activity may be delayed without delaying the total project
beyond its target time
D. is the earliest starting time an activity for a project can begin
E. is the pessimistic time estimate for an activity of a project

C 23. The network shown in Figure 16-1 was developed by using the Program Evaluation and Review
Technique (PERT) to aid in scheduling the development of a new product. The critical path is:
A. 1-3-6
B. 1-2-4-6
C. 1-2-5-6
D. 1-3-4-6
E. 1-4-6

SUPPORTING CALCULATION:

1 - 3 - 6 = 14
1 - 3 - 4 - 6 = 13
1 - 4 - 6 = 12
1 - 2 - 4 - 6 = 11
1 - 2 - 5 - 6 = 15
Budgeting: Capital, Research and Development Expenditures, and Cash; PERT/Cost 228

D 24. Using the following data, compute the cash financing needs or excess cash to invest.

Cash balance, beginning.............................................................................................. $ 20,000


Collections from customers......................................................................................... 150,000
Disbursements:
For direct materials............................................................................................... 25,000
For other costs and expenses................................................................................ 30,000
For payroll............................................................................................................. 75,000
For income taxes................................................................................................... 6,000
For machinery purchase........................................................................................ 30,000
Minimum cash balance desired.................................................................................... 20,000

A. excess cash$4,000
B. excess cash$14,000
C. financing need$10,000
D. financing need$16,000
E. none of the above

SUPPORTING CALCULATION:

$20,000 + $150,000 - $25,000 - $30,000 - $75,000 - $6,000 - $30,000 - $20,000


= ($16,000)

A 25. CMR is a retail mail-order firm that currently uses a central collection system that requires all
checks to be sent to its Boston headquarters. An average of five days is required for mailed checks
to be received, four days for CMR to process them, and one-and-a-half days for the checks to clear
through the bank. A proposed lock-box system would reduce the mail and process time to three
days, and the check clearing time to one day. CMR has an average daily collection of $100,000. If
CMR should adopt the lock-box system, its average cash balance would increase by:
A. $650,000
B. $250,000
C. $800,000
D. $400,000
E. none of the above

SUPPORTING CALCULATION:

Mail and processing savings = (5 + 4 - 3) x $100,000 = $ 600,000


Clearing savings = (1.5 - 1) x $100,000 = 50,000
= $ 650,000

D 26. The research and development budget is considered best for:


A. balancing the research and development program
B. coordinating the program with the company's other projects
C. checking certain phases of nonfinancial planning
D. all of the above
E. none of the above
Budgeting: Capital, Research and Development Expenditures, and Cash; PERT/Cost 229

B 27. The only research and development costs that should be expensed in the period incurred are those
that are:
A. conducted for others
B. unique to chemical manufacturers
C. unique to extractive industries
D. incurred by government regulated enterprise
E. none of the above should be expensed

A 28. The treasurer's budget indicates cash requirements for all of the following, except:
A. commercial expenses
B. dividends
C. interest on bonus
D. donations
E. income tax

B 29. Benefits of a computerized budgeting process include all of the following except:
A. shortening the planning cycle
B. reducing the need for planning
C. time to reconsider planning assumptions
D. operating analysis capability
E. plans can be updated continuously

C 30. Prospective financial information should include all of the following except:
A. a description of what management intends to present
B. a summary of significant assumptions
C. an auditor's opinion
D. a caveat that the expected results may not be achieved
E. a format similar to the historical financial statements
Budgeting: Capital, Research and Development Expenditures, and Cash; PERT/Cost 230

PROBLEMS

PROBLEM

1.
Cash Receipts Budget. Astro Co. bills its customers for sales on account at the end of each month, with terms of
2/10/EOM, n/45. Fifty percent of credit sales are paid within the discount period, while 30% are paid at the end
of the next period. Fifteen are paid at the end of the second following month, but these customers pay a 2%
service charge on any balance due. Receivables are recorded at gross. The following data are given for the last
two months and for the next two months:

Last Two Months Next Two Months


August September October November
Cash sales............................................................ $20,400 $18,000 $46,200 $31,500
Credit sales.......................................................... 40,000 90,000 60,000 52,000
Other receipts...................................................... 2,000   4,600

Required: Prepare a cash receipts budget for the two-month period, October and November.

SOLUTION

Astro Co.
Cash Receipts Budget
For October-November, 19--

October November
Cash sales....................................................................................................................... $ 46,200 $31,500
Other receipts.................................................................................................................  4,600
Credit sales:
August:
Late (15% x $40,000)...................................................................................... 6,000 
Service charge (2% x $6,000).......................................................................... 120 
September:
Discount taken.................................................................................................. 44,1001 
Current (30% x $90,000)................................................................................. 27,000 
Late (15% x $90,000)......................................................................................  13,500
Service charge (2% x $13,500)........................................................................  270
October:
Discount taken..................................................................................................  29,4002
Current (30% x $60,000).................................................................................  18,000
Total receipts........................................................................................................... $123,420 $97,270
1 2
$90,000 x 50% = $45,000 gross $60,000 x 50% = $30,000 gross
$45,000 x 2% = $900 discount $30,000 x 2% = $600 discount
$45,000 - $900 = $44,100 net $30,000 - $600 = $29,400 net
Budgeting: Capital, Research and Development Expenditures, and Cash; PERT/Cost 231

PROBLEM

2.
Four-Month Cash Budget. The management of Island Novelties Co. is preparing a cash budget for the next
four-month period. Relevant data for this budget are:

March April May June


Credit sales..................................................... $60,000 $55,000 $90,000 $75,000

Credit sales were $40,000 in January and $48,000 in February. In July, credit sales are estimated at $105,000.
Collections on account are made at the rate of 75% in the month following the sale and 20% in the second month
following the sale.
Variable expenses other than purchases are equal to 30% of sales; 75% of both variable expenses and
purchases are paid in the month incurred, while 25% are paid in the next month. Cost of goods sold is equal to
50% of sales, and purchases are made so that the ending inventory is maintained at a level equal to 60% of the
needs for the next month's sales. Fixed expenses are $3,000 per month.

Required: Prepare a cash budget for the four-month period, March through June, indicating the net increase (or
decrease) in the cash balance for each month.

SOLUTION

Island Novelties Co.


Cash Budget
For March-June, 19--

March April May June


Receipts from sales in:
January...................................................... $ 8,000   
February.................................................... 36,000 $ 9,600  
March........................................................  45,000 $ 12,000 
April..........................................................   41,250 $ 11,000
May...........................................................    67,500
Total receipts....................................... $ 44,000 $ 54,600 $ 53,250 $ 78,500

Disbursements for:
Variable expenses:
February.............................................. $ 3,600   
March.................................................. 13,500 $ 4,500  
April....................................................  12,375 $ 4,125 
May......................................................   20,250 $ 6,750
June......................................................    16,875
Fixed expenses.......................................... 3,000 3,000 3,000 3,000
Purchases:
February.................................................... 6,900   
March........................................................ 21,375 7,125  
April..........................................................  28,500 9,500 
May...........................................................   30,375 10,125
June...........................................................    34,875
Total disbursements............................ $ 48,375 $ 55,500 $ 67,250 $ 71,625
Net cash increase (decrease)......................... $ (4,375) $ (900) $ (14,000) $ 6,875
Budgeting: Capital, Research and Development Expenditures, and Cash; PERT/Cost 232

Additional computations:
February March April May June
Purchases:
Beginning inventory
(60% of 50% of
current month's
sales)................................ $14,400 $18,000 $16,500 $27,000 $22,500
Cost of goods sold
(50% of current
month's sales).................. 24,000 30,000 27,500 45,000 37,500
Ending inventory
(60% of 50% of
next month's sales).......... 18,000 16,500 27,000 22,500 31,500
Cost of goods sold +
ending inventory -
beginning inventory =
purchases......................... $27,600 $28,500 $38,000 $40,500 $46,500

PROBLEM

3.
Four-Month Cash Budget. Bagel Factory Inc. prepared cash estimates for the next four months. The following
estimates were developed for certain items:

Item .................................................................... March April May June


Cash sales.............................................................. $10,000 $6,000 $8,000 $11,000
Credit sales............................................................ 5,000 2,000 6,000 9,000
Payroll................................................................... 2,000 1,500 2,500 3,000
Purchases............................................................... 3,000 2,600 2,800 4,000
Other expenses...................................................... 2,500 2,400 2,600 2,800

In February, credit sales totaled $9,000, and purchases totaled $5,000. January credit sales were $12,000.
Accounts receivable collections amount to 30% in the month after the sale and 60% in the second month after the
sale; 10% of the receivables are never collected. Payroll and other expenses are paid in the month incurred.
Seventy-five percent of the purchases are paid in the month incurred, and the remainder are paid in the following
month. A $15,000 tax payment is due on June 15. The cash balance was $5,000 on March 1. The company
wants a minimum cash balance of $5,000 per month.

Required:

(1) Prepare a cash budget for the four-month period, March through June.
(2) List the amount of funds available for investing or required for borrowing in each month.
Budgeting: Capital, Research and Development Expenditures, and Cash; PERT/Cost 233

SOLUTION

(1) Bagel Factory Inc.


Cash Budget
For March-June, 19--

March April May June


Receipts from:
Cash sales........................................................ $ 10,000 $ 6,000 $ 8,000 $ 11,000
January credit sales (60% x
$12,000).................................................... 7,200   
February credit sales:
30% x $9,000............................................ 2,700   
60% x $9,000............................................  5,400  
March credit sales............................................  1,500 3,000 
April credit sales..............................................   600 1,200
May credit sales...............................................    1,800
Total receipts............................................ $ 19,900 $ 12,900 $ 11,600 $ 14,000

Disbursements for:
Payroll.............................................................. $ 2,000 $ 1,500 $ 2,500 $ 3,000
Other expenses................................................. 2,500 2,400 2,600 2,800
February purchases (25% x
$5,000)...................................................... 1,250   
March purchases.............................................. 2,250 750  
April purchases................................................  1,950 650 
May purchases.................................................   2,100 700
June purchases.................................................    3,000
Tax payment....................................................    15,000
Total disbursements.................................. $ 8,000 $ 6,600 $ 7,850 $ 24,500
Net increase (decrease) in cash.............................. $ 11,900 $ 6,300 $ 3,750 $ (10,500)
Cash balances:
Beginning......................................................... 5,000 5,000 5,000 5,000
Ending.............................................................. $ 16,900 $ 11,300 $ 8,750 $ (5,500)

(2)
Available for investing........................................... $ 11,900 $ 6,300 $ 3,750 
Needed to borrow...................................................    $ 10,5001
1
$5,500 + $5,000 minimum cash balance
Budgeting: Capital, Research and Development Expenditures, and Cash; PERT/Cost 234

PROBLEM

4.
Identifying Critical Path in PERT Network. Jacques Company prepares a bid to salvage a sunken treasure ship.
The PERT network in Figure 16-2 on page 222 is developed to reflect the activities needed to recover the ship.
The numbers in the line segments represent expected completion times in days for activities.

Required:

(1) Identify the critical path and the total time expected for completion.
(2) The company is informed that Steps 5-6 could be shortened by up to two days for a cost of $1,000 per day
or fraction of a day shortened. Each day or fraction of a day shortened would result in a cost savings of
$3,000 before the expenses required to affect the savings. Show the net savings, as well as the new
completion time, as a result of shortening the time required on Steps 5-6.

SOLUTION

(1)
Paths Completion Time (Days)
1-2-3-8 15.8
1-2-4-7-8 20.0
1-5-4-7-8 17.7
1-5-6-7-8 22.0

(2) Steps 5-6 may be shortened by as much as two days before the path 1-2-4-7-8 would become critical.
Savings would equal 2($3,000 - $1,000), or $4,000. The new completion time is 22 - 2 = 20 days.
Budgeting: Capital, Research and Development Expenditures, and Cash; PERT/Cost 235

PROBLEM

5.
Critical Path; Completion Time in PERT Network. F Troop is experiencing difficulties constructing a fort on
the Red River. Col. Storch proposes that a PERT network be used to organize the construction activities for the
fort. Each activity, together with its immediate predecessor activity and the completion time for the activity, is
given below:

Completion
Time
Activity Type of Activity Predecessor Activity (in Weeks)
1 Clear trees 0 12
2 Sort trees by size Clear trees 6
3 Strip trees into logs Clear trees 9
4 Construct walls Strip trees into logs 12
5 Construct gate Strip trees into logs 3
6 Build turret Sort trees by size 6
7 Build captain's house Strip trees into logs 8
8 Build officers' houses Construct walls 9
9 Build enlisted personnel's barracks Build officers' houses 6
10 Hold completion ceremony All others 1

Required: Diagram a PERT network to represent the activities on page 234. Show the completion time above
each path. Identify the critical path and show the completion time in weeks.

SOLUTION
Budgeting: Capital, Research and Development Expenditures, and Cash; PERT/Cost 236

Paths Completion Time (Weeks)


0 - 1 - 2 - 6 - 10 25
0 - 1 - 3 - 7 - 10 30
0 - 1 - 3 - 4 - 8 - 9 - 10 (critical path) 49
0 - 1 - 3 - 5 - 10 25
Budgeting: Capital, Research and Development Expenditures, and Cash; PERT/Cost 237

PROBLEM

6.
PERT Network. A company is faced with the following PERT network situation (time in days):

Required:

(1) Calculate te (expected time) for each activity. For each activity, the estimates are to, tm, tp, in that order.
(2) Calculate the total time for each path and identify the critical path as well as total time for other paths.

SOLUTION:

(1)
Activity (t o + tm(4) + tp) = Total  6 = te
1-2 1 3(4) 5 18 6 3
1-3 2 6(4) 10 36 6 6
1-4 1 4(4) 7 24 6 4
2-6 2 15(4) 16 78 6 13
3-5 4 7(4) 10 42 6 7
4-5 3 4(4) 5 24 6 4
5-6 4 5(4) 6 30 6 5

(2)
Path tos Total te
1-2-6 3 + 13 16
1-4-5-6 4+4+5 13
1-3-5-6 6+7+5 18 critical path
Budgeting: Capital, Research and Development Expenditures, and Cash; PERT/Cost 268

Chapter 19

STANDARD COSTING: INCORPORATING


STANDARDS INTO THE ACCOUNTING RECORDS

MULTIPLE CHOICE

Question Nos. 8-15, 17, and 19-21 are AICPA adapted.


Question Nos. 16, 22, and 23 are ICMA adapted.
Question No. 18 is CIA adapted.

D 1. When the amount for materials inventory in the general ledger represents the standard cost of
materials and the materials ledger cards are kept in quantities only, the materials price variance is:
A. recorded at the time of disposition of the inventory
B. ignored
C. recorded when materials are requisitioned for production
D. recorded when materials are received
E. allocated to cost of sales only

B 2. A company recorded the following journal entry when materials were issued to the factory:

Work in Process....................................................................................... 9,000


Materials Quantity Variance............................................................ 200
Materials........................................................................................... 8,800

Assuming that there was both a price variance and a quantity variance associated with these
materials, this entry indicates that the method used for materials price variances is to:
A. allocate variances to ending inventories and cost of sales
B. record variances at the time materials are received
C. record variances at the time of disposition of work in process
D. allocate variances to cost of sales only
E. record variances at the time materials are used

D 3. Variances resulting from materials price changes that are to be passed on to customers are:
A. charged to cost of goods sold
B. carried as a special credit to inventory accounts
C. recorded as ordinary inflation revenue
D. allocated to inventories and cost of goods sold
E. charged to a special loss account

268
Standard Costing: Incorporating Standards into the Accounting Records 269

E 4. When standard cost variances are significant, Cost Accounting Standards require that the variances
be:
A. charged to cost of goods sold
B. deferred
C. allocated to inventories only if they are allocated solely for financial reporting purposes
D. recorded as extra income in the current period
E. allocated to inventories as well as cost of goods sold

D 5. If new standard costs reflect conditions that affected the actual cost of goods in the ending inventory,
then ending inventories are costed at:
A. the contra amount carried in cost of sales
B. the old standard
C. the amount carried in the variance accounts
D. the new standard
E. actual cost

A 6. A credit balance in the labor efficiency variance indicates that:


A. standard hours exceed actual hours
B. actual hours exceed standard hours
C. standard rate and standard hours exceed actual rate and actual hours
D. actual rate and actual hours exceed standard rate and standard hours
E. none of the above

D 7. A debit balance in a direct labor efficiency variance account indicates that:


A. actual total direct labor costs incurred were less than standard direct labor costs allowed for
the units produced
B. the number of units produced was less than the number of units budgeted for the period
C. the average wage rate paid to direct labor employees was less than the standard rate
D. the standard hours allowed for the units produced were less than actual direct labor hours used
E. all of the above
Standard Costing: Incorporating Standards into the Accounting Records 270

E 8. Josey Manufacturing Corporation uses a standard cost system that records direct materials at actual
cost, records materials price variances at the time that direct materials are issued to work in process,
and prorates all variances at year end. Variances associated with direct materials are prorated based
on the direct materials balances in the appropriate accounts, and variances associated with direct
labor and factory overhead are prorated based on the direct labor balances in the appropriate
accounts.

The following information is available for Josey for the year ended December 31:

Finished goods inventory at December 31:


Direct materials.............................................................................................................. $ 87,000
Direct labor..................................................................................................................... 130,500
Applied factory overhead............................................................................................... 104,400
Direct materials inventory at December 31.......................................................................... 65,000
Cost of goods sold for the year ended December 31:
Direct materials.............................................................................................................. 348,000
Direct labor..................................................................................................................... 739,500
Applied factory overhead............................................................................................... 591,600
Direct materials price variance (unfavorable)...................................................................... 12,500
Direct materials usage variance (favorable)......................................................................... 15,000
Direct labor rate variance (unfavorable)............................................................................... 20,000
Direct labor efficiency variance (favorable)......................................................................... 5,000
Factory overhead incurred..................................................................................................... 690,000

There were no beginning inventories and no ending work in process inventory. Factory overhead is
applied at 80% of standard direct labor cost.

The amount of direct materials price variance to be prorated to finished goods inventory at
December 31 is a:
A. $1,740 debit
B. $2,000 debit
C. $2,610 credit
D. $3,000 credit
E. none of the above

SUPPORTING CALCULATION:

$87,000
_ $12,500 = $2,500
$87,000 + $348,000
Standard Costing: Incorporating Standards into the Accounting Records 271

A 9. Josey Manufacturing Corporation uses a standard cost system that records direct materials at actual
cost, records materials price variances at the time that direct materials are issued to work in process,
and prorates all variances at year end. Variances associated with direct materials are prorated based
on the direct materials balances in the appropriate accounts, and variances associated with direct
labor and factory overhead are prorated based on the direct labor balances in the appropriate
accounts.

The following information is available for Josey for the year ended December 31:

Finished goods inventory at December 31:


Direct materials.............................................................................................................. $ 87,000
Direct labor..................................................................................................................... 130,500
Applied factory overhead............................................................................................... 104,400
Direct materials inventory at December 31.......................................................................... 65,000
Cost of goods sold for the year ended December 31:
Direct materials.............................................................................................................. 348,000
Direct labor..................................................................................................................... 739,500
Applied factory overhead............................................................................................... 591,600
Direct materials price variance (unfavorable)...................................................................... 12,500
Direct materials usage variance (favorable)......................................................................... 15,000
Direct labor rate variance (unfavorable)............................................................................... 20,000
Direct labor efficiency variance (favorable)......................................................................... 5,000
Factory overhead incurred..................................................................................................... 690,000

There were no beginning inventories and no ending work in process inventory. Factory overhead is
applied at 80% of standard direct labor cost.

The total amount of direct materials in finished goods inventory at December 31, after all materials
variances have been prorated, is:
A. $86,500
B. $87,500
C. $88,000
D. $86,000
E. none of the above

$87,000 $87,000
$87,000 + ( $87,000 + $348,000
_ $12,500 ⋅
$87,000 + $348,000 )(
_ $15,000 )
= $86,500
SUPPORTING CALCULATION:
Standard Costing: Incorporating Standards into the Accounting Records 272

C 10. Josey Manufacturing Corporation uses a standard cost system that records direct materials at actual
cost, records materials price variances at the time that direct materials are issued to work in process,
and prorates all variances at year end. Variances associated with direct materials are prorated based
on the direct materials balances in the appropriate accounts, and variances associated with direct
labor and factory overhead are prorated based on the direct labor balances in the appropriate
accounts.

The following information is available for Josey for the year ended December 31:

Finished goods inventory at December 31:


Direct materials.............................................................................................................. $ 87,000
Direct labor..................................................................................................................... 130,500
Applied factory overhead............................................................................................... 104,400
Direct materials inventory at December 31.......................................................................... 65,000
Cost of goods sold for the year ended December 31:
Direct materials.............................................................................................................. 348,000
Direct labor..................................................................................................................... 739,500
Applied factory overhead............................................................................................... 591,600
Direct materials price variance (unfavorable)...................................................................... 12,500
Direct materials usage variance (favorable)......................................................................... 15,000
Direct labor rate variance (unfavorable)............................................................................... 20,000
Direct labor efficiency variance (favorable)......................................................................... 5,000
Factory overhead incurred..................................................................................................... 690,000

There were no beginning inventories and no ending work in process inventory. Factory overhead is
applied at 80% of standard direct labor cost.

The total amount of direct labor in finished goods inventory at December 31, after all variances have
been prorated, is:
A. $126,750
B. $134,250
C. $132,750
D. $133,750
E. none of the above

SUPPORTING CALCULATION:

$130,500 $130,500
$130,500 + ( $130,500 + $739,500
_ $20,000 )( $130,500 + $739,500
_ $5,000 )
= $132,750
Standard Costing: Incorporating Standards into the Accounting Records 273

B 11. Josey Manufacturing Corporation uses a standard cost system that records direct materials at actual
cost, records materials price variances at the time that direct materials are issued to work in process,
and prorates all variances at year end. Variances associated with direct materials are prorated based
on the direct materials balances in the appropriate accounts, and variances associated with direct
labor and factory overhead are prorated based on the direct labor balances in the appropriate
accounts.

The following information is available for Josey for the year ended December 31:

Finished goods inventory at December 31:


Direct materials.............................................................................................................. $ 87,000
Direct labor..................................................................................................................... 130,500
Applied factory overhead............................................................................................... 104,400
Direct materials inventory at December 31.......................................................................... 65,000
Cost of goods sold for the year ended December 31:
Direct materials.............................................................................................................. 348,000
Direct labor..................................................................................................................... 739,500
Applied factory overhead............................................................................................... 591,600
Direct materials price variance (unfavorable)...................................................................... 12,500
Direct materials usage variance (favorable)......................................................................... 15,000
Direct labor rate variance (unfavorable)............................................................................... 20,000
Direct labor efficiency variance (favorable)......................................................................... 5,000
Factory overhead incurred..................................................................................................... 690,000

There were no beginning inventories and no ending work in process inventory. Factory overhead is
applied at 80% of standard direct labor cost.

The total cost of goods sold for the year ended December 31, after all variances have been prorated,
is:
A. $1,693,850
B. $1,684,750
C. $1,675,450
D. $1,683,270
E. none of the above

SUPPORTING CALCULATION:

$348,000 + $739,500 + $591,600 + ($15,000 x .85) - ($2,500 x .80) - .85 ($6,000) = $1,684,750
Standard Costing: Incorporating Standards into the Accounting Records 274

B 12. Kaiser Manufacturing Company uses a standard cost system in accounting for the costs of production
of its only product, Product A. The standards for the production of one unit of Product A are as
follows:

Direct materials: 10 feet of Item 1 at $.78 per foot and 3 feet of Item 2 at $1 per foot
Direct labor: 4 hours at $3.60 per hour
Factory overhead: applied at 150% of standard direct labor costs

There was no inventory on hand at the end of the year. Materials price variances are isolated at
purchase. Following is a summary of costs and related data for the production of Product A during
the year:

100,000 feet of Item 1 were purchased at $.75 per foot.


30,000 feet of Item 2 were purchased at $.90 per foot.
8,000 units of Product A were produced that required 78,000 feet of Item 1, 26,000 feet of Item 2,
and 31,000 hours of direct labor at $3.50 per hour.
6,000 units of Product A were sold.

The total debits to the direct materials account for the purchase of Item 1 should be:
A. $75,000
B. $78,000
C. $58,500
D. $60,000
E. none of the above

SUPPORTING CALCULATION:

100,000 x $.78 = $78,000


Standard Costing: Incorporating Standards into the Accounting Records 275

D 13. Kaiser Manufacturing Company uses a standard cost system in accounting for the costs of production
of its only product, Product A. The standards for the production of one unit of Product A are as
follows:

Direct materials: 10 feet of Item 1 at $.78 per foot and 3 feet of Item 2 at $1 per foot
Direct labor: 4 hours at $3.60 per hour
Factory overhead: applied at 150% of standard direct labor costs

There was no inventory on hand at the end of the year. Materials price variances are isolated at
purchase. Following is a summary of costs and related data for the production of Product A during
the year:

100,000 feet of Item 1 were purchased at $.75 per foot.


30,000 feet of Item 2 were purchased at $.90 per foot.
8,000 units of Product A were produced that required 78,000 feet of Item 1, 26,000 feet of Item 2,
and 31,000 hours of direct labor at $3.50 per hour.
6,000 units of Product A were sold.

The total debits to the work in process account for direct labor should be:
A. $111,600
B. $108,500
C. $112,000
D. $115,200
E. none of the above

SUPPORTING CALCULATION:

8,000 x 4 x $3.60 = $115,200


Standard Costing: Incorporating Standards into the Accounting Records 276

A 14. Kaiser Manufacturing Company uses a standard cost system in accounting for the costs of production
of its only product, Product A. The standards for the production of one unit of Product A are as
follows:

Direct materials: 10 feet of Item 1 at $.78 per foot and 3 feet of Item 2 at $1 per foot
Direct labor: 4 hours at $3.60 per hour
Factory overhead: applied at 150% of standard direct labor costs

There was no inventory on hand at the end of the year. Materials price variances are isolated at
purchase. Following is a summary of costs and related data for the production of Product A during
the year:

100,000 feet of Item 1 were purchased at $.75 per foot.


30,000 feet of Item 2 were purchased at $.90 per foot.
8,000 units of Product A were produced that required 78,000 feet of Item 1, 26,000 feet of Item 2,
and 31,000 hours of direct labor at $3.50 per hour.
6,000 units of Product A were sold.

Before allocation of standard variances, the balance in the materials quantity variance account of
Item 2 was:
A. $2,000 debit
B. $1,000 credit
C. $2,600 debit
D. $600 debit
E. $1,000 debit

SUPPORTING CALCULATION:

26,000 - (8,000 x 3 x $1) = $2,000


Standard Costing: Incorporating Standards into the Accounting Records 277

C 15. Kaiser Manufacturing Company uses a standard cost system in accounting for the costs of production
of its only product, Product A. The standards for the production of one unit of Product A are as
follows:

Direct materials: 10 feet of Item 1 at $.78 per foot and 3 feet of Item 2 at $1 per foot
Direct labor: 4 hours at $3.60 per hour
Factory overhead: applied at 150% of standard direct labor costs

There was no work in process inventory on hand at the end of the year. Materials price variances are
isolated at purchase. Following is a summary of costs and related data for the production of Product
A during the year:

100,000 feet of Item 1 were purchased at $.75 per foot.


30,000 feet of Item 2 were purchased at $.90 per foot.
8,000 units of Product A were produced that required 78,000 feet of Item 1, 26,000 feet of Item 2,
and 31,000 hours of direct labor at $3.50 per hour.
6,000 units of Product A were sold.

If all standard variances are prorated to inventories and cost of goods sold, the amount of materials
quantity variance for Item 2 to be prorated to direct materials inventory would be:
A. $500 debit
B. $500 credit
C. 0
D. $333 credit
E. $333 debit

SUPPORTING CALCULATION:

The variance would be allocated only to finished goods and cost of goods sold.

E 16. The most appropriate time from a control standpoint to record any variance of actual materials
prices from standard is:
A. at the time of materials usage
B. as needed to evaluate the performance of the purchasing manager
C. at the time the materials are issued by the storeroom
D. at year end, when all variances will be known
E. at the time of purchase

C 17. Standard costing will produce the same income before extraordinary items as does actual costing
when standard cost variances are assigned to:
A. work in process and finished goods inventories
B. an income or expense account
C. cost of goods sold and inventories
D. cost of goods sold
E. income summary
Standard Costing: Incorporating Standards into the Accounting Records 278

D 18. When items are transferred from stores to production, an accountant debits Work in Process and
credits Materials. During production, a materials quantity variance may occur. Materials Quantity
Variance is debited for an unfavorable variance and credited for a favorable variance. The intent of
variance entries is to provide:
A. accountability for materials lost during production
B. a means of safeguarding assets in the custody of the system
C. compliance with GAAP
D. information for use in controlling the cost of production
E. all of the above

B 19. At the end of an accounting period, a quantity variance that is significant in amount should be:
A. reported as a deferred charge or credit
B. allocated among work in process inventory, finished goods inventory, and cost of goods sold
C. charged or credited to cost of goods manufactured
D. allocated among cost of goods manufactured, finished goods inventory, and cost of goods sold
E. none of the above

C 20. What is the normal year-end treatment of immaterial variances recognized in a cost accounting
system utilizing standards?
A. reclassified to deferred charges until all related production is sold
B. allocated among cost of goods manufactured and ending work in process inventory
C. closed to Cost of Goods Sold in the period in which they arose
D. capitalized as a cost of ending finished goods inventory
E. none of the above

A 21. An unacceptable treatment of factory overhead variances at an interim reporting date is to:
A. apportion the total only between work in process and finished goods inventories on hand at
the end of the interim reporting period
B. apportion the total only between that part of the current period's production remaining in
inventories at the end of the period and that part sold during the period
C. carry forward the total to be offset by opposite balances in later periods
D. charge or credit the total to Cost of Goods Sold during the period
E. all are acceptable
Standard Costing: Incorporating Standards into the Accounting Records 279

A 22. Sam Company adopted a standard cost system several years ago. The standard costs for the prime
costs of its single product are as follows:

Material (8 kilograms x $5.00/kg.)....................................................................................... $40.00


Labor (6 hours x $8.20/hr.)................................................................................................... $49.20

The operating data in the following column were taken from the records for November:

In-process beginning inventorynone


In-process ending inventory800 units, 75% complete as to labor; material is issued at the beginning
of processing
Units completed5,600 units
Budgeted output6,000 units
Purchases of materials50,000 kilograms
Total actual labor costs$300,760
Actual hours of labor36,500 hours
Material usage variance$1,500 unfavorable
Total material variance$750 unfavorable

The total amount of material and labor cost transferred to the finished goods account for November
is:
A. $499,520
B. $535,200
C. $550,010
D. $561,040
E. none of the above

SUPPORTING CALCULATION:

(5,600 x $40) + (5,600 x $49.20) = $499,520


Standard Costing: Incorporating Standards into the Accounting Records 280

C 23. Sam Company adopted a standard cost system several years ago. The standard costs for the prime
costs of its single product are as follows:

Material (8 kilograms x $5.00/kg.)...................................................................................... $40.00


Labor (6 hours x $8.20/hr.).................................................................................................. $49.20

The operating data in the following column were taken from the records for November:

In-process beginning inventorynone


In-process ending inventory800 units, 75% complete as to labor; material is issued at the beginning
of processing
Units completed5,600 units
Budgeted output6,000 units
Purchases of materials50,000 kilograms
Total actual labor costs$300,760
Actual hours of labor36,500 hours
Material usage variance$1,500 unfavorable
Total material variance$750 unfavorable

The total amount of material and labor cost in the ending balance of work in process inventory at the
end of November is:
A. 0
B. $9,840
C. $61,520
D. $71,360
E. none of the above

SUPPORTING CALCULATION:

(800 x $40) + (800 x .75 x $49.20) = $61,520

C 24. When the amount for materials inventory in the general ledger represents the actual cost of materials
and the materials ledger cards show quantities and dollar values, the materials price variance is:
A. recorded at the time of disposition of the inventory
B. ignored
C. recorded when materials are requisitioned for production
D. recorded when materials are received
E. allocated to cost of sales only

E 25. The treatment of variances depends upon all of the following, except the:
A. type of variance
B. size of the variance
C. cause of the variance
D. timing of the variance
E. it depends upon all of the above
Standard Costing: Incorporating Standards into the Accounting Records 281

The following questions are based on the material in the Appendix to the chapter.

C 26. A company recorded the following journal entry:

Work in Process...................................................................................... 10,310


Factory Overhead Variable Efficiency Variance.................................. 950
Factory Overhead Fixed Efficiency Variance....................................... 425
Factory Overhead Control.............................................................. 11,685

This entry indicates that the:


A. four-variance method is in use and the variance is favorable
B. three-variance method is in use and the variance is favorable
C. four-variance method is in use and the variance is unfavorable
D. two-variance method is in use and the variance is favorable
E. three-variance method is in use and the variance is unfavorable

A 27. In the alternative three-variance method, the amount of over- or underapplied factory overhead is
analyzed as:
A. spending, idle capacity, and efficiency variances
B. volume, variable efficiency, and fixed efficiency variances
C. controllable, spending, and idle capacity variances
D. volume, variable efficiency, and spending variances
E. none of the above

D 28. In the four-variance method, the amount of over- or underapplied factory overhead is analyzed as:
A. spending, idle capacity, efficiency, and volume variances
B. controllable idle capacity, spending, and efficiency variances
C. variable efficiency, fixed efficiency, controllable, and volume variances
D. variable efficiency, fixed efficiency, spending, and idle capacity variances
E. none of the above
Standard Costing: Incorporating Standards into the Accounting Records 282

PROBLEMS

PROBLEM

1.
Journal Entries for Variances. Parrothead Corp. determines that the following variances arose in production
during March:

Variance Amount
Materials purchase price........................................................................................................ $2,400 favorable
Materials quantity.................................................................................................................. 1,000 favorable
Labor efficiency..................................................................................................................... 500 favorable
Labor rate............................................................................................................................... 750 unfavorable
Factory overhead volume...................................................................................................... 1,700 favorable
Factory overhead controllable............................................................................................... 2,950 unfavorable

Materials purchases totaled $90,000 at standard costs, while $77,000 in materials were taken from inventory for
use in production. Labor payroll totaled $144,000, and actual overhead incurred was $256,000.

Required: Prepare the journal entries to record the above variances, including the recording of the actual and
applied factory overhead using a single factory overhead control account.

SOLUTION

Materials....................................................................................................................... 90,000
Materials Purchase Price Variance....................................................................... 2,400
Accounts Payable.................................................................................................. 87,600

Work in Process............................................................................................................ 78,000


Materials................................................................................................................ 77,000
Materials Quantity Variance................................................................................. 1,000

Work in Process............................................................................................................ 143,750


Labor Rate Variance.................................................................................................... 750
Payroll................................................................................................................... 144,000
Labor Efficiency Variance.................................................................................... 500

Factory Overhead Control............................................................................................ 256,000


Various Credits..................................................................................................... 256,000

Work in Process............................................................................................................ 254,750


Factory Overhead Controllable Variance.................................................................... 2,950
Factory Overhead Control.................................................................................... 256,000
Factory Overhead Volume Variance.................................................................... 1,700

or

Work in Process............................................................................................................ 254,750


Factory Overhead Control.................................................................................... 254,750
Standard Costing: Incorporating Standards into the Accounting Records 283

Factory Overhead Controllable Variance.................................................................... 2,950


Factory Overhead Volume Variance.................................................................... 1,700
Factory Overhead Control.................................................................................... 1,250

PROBLEM

2.
Journal Entries, Three-Variance Method. Canelli Products Co. presents the following data related to June
production:

Item 100% Budget 80% Budget Actual


Materials....................................................................................... $ 30,000 $ 24,000 $ 23,600
Labor............................................................................................. 60,000 48,000 52,500
Factory overhead.......................................................................... 280,000 250,000 252,500
$ 370,000 $ 322,000 $ 328,600

Item 100% Budget 80% Budget Actual


Direct labor hours......................................................................... 5,000 4,000 4,200
Labor rate...................................................................................... -- -- $12.50
Materials purchases...................................................................... -- -- --
Production in units....................................................................... 2,500 2,000 2,000

Required: Prepare the journal entries to record the above data, including the recording of the actual and applied
factory overhead using a single factory overhead control account and using the three-variance method. The
company records the materials price variance at the time that materials are purchased. The factory overhead is
based on the budget at 100%. (Hint: To obtain the overhead variances, first solve for the variable overhead rate.)

SOLUTION

Work in Process............................................................................................................ 24,000


Materials Quantity Variance................................................................................. 400
Materials................................................................................................................ 23,600

Work in Process............................................................................................................ 48,000


Labor Efficiency Variance [$12 x (4,200 DLH - 4,000 DLH)].................................. 2,400
Labor Rate Variance [($12.50 - $12) x 4,200 DLH]................................................... 2,100
Payroll ($12.50 x 4,200 DLH).............................................................................. 52,500

Factory Overhead Control............................................................................................ 252,500


Various Credits..................................................................................................... 252,500

Overhead at 100% Overhead at 80% $280,000 $250,000


=
Hours at 100% Hours at 80% 5,000 4,000

= $30 per hour variable overhead


Standard Costing: Incorporating Standards into the Accounting Records 284

Work in Process............................................................................................. 224,000


Factory Overhead Variable Efficiency Variance.......................................... 6,0002
Factory Overhead Volume Variance............................................................. 26,0003
Factory Overhead Control...................................................................... 252,500
Factory Overhead Spending Variance................................................... 3,5001

Actual factory overhead................................................................................. $ 252,500


Budget allowance based on actual hours:
Fixed expense......................................................................................... $ 130,0001
Variable expense (4,200 hours x $30).................................................... 126,000 256,000
Factory overhead spending variance............................................................. $ (3,500) fav.
1
$250,000 - $30 (4,000) = $130,000 fixed overhead
2
Budget allowance based on actual hours..................................................... $ 256,000
Budget allowance based on standard hours allowed:
Variable overhead (4,000 x $30)......................................................... $ 120,000
Fixed overhead ($250,000 - $120,000)................................................ 130,000 250,000
Variable efficiency variance......................................................................... $ 6,000 unfav.
3
Budget allowance based on standard hours allowed.................................... $ 250,000
Standard factory overhead charged to production
($56 x 4,000)........................................................................................ 224,000
Volume variance........................................................................................... $ 26,000 unfav.

PROBLEM

3.
Materials, Labor, and Overhead Variance Analyses. TYPCO Corp. manufactures changeable typeheads for
use on portable typewriters. Each typehead is in a set consisting of the lead alloy typehead itself, a cover for the
key on the typewriter keyboard, and a plastic box to hold the two items. At the beginning and end of June, there
were no materials inventories. The following standards were developed for each unit:

Item Standard per Unit


Materials:
Lead alloy (3 oz. @ $.22)....................................................................................................... $ .66
Cover materials (6 oz. @ $.04).............................................................................................. .24
Container boxes (1 @ $.10).................................................................................................... .10
Direct labor (1/4 hr. @ $12 per hr.)............................................................................................... 3.00
Overhead ($10 per direct labor hour)............................................................................................ 2.50
Total cost................................................................................................................................. $ 6.50

Annual production is estimated at 50,000 units, with fixed overhead of $25,000. During the past year, the
following costs were incurred to produce 40,000 units:

Materials:
Lead alloy: 122,000 oz. @ $.20
Cover materials: 235,000 oz. @ $.04
Container boxes: 40,500 @ $.09
Direct labor: 9,500 hrs. @ $12.50
Overhead: $90,000
Standard Costing: Incorporating Standards into the Accounting Records 285

Required: Compute the variances for each materials and labor item, recording the materials price variance at the
time of usage. Show the overhead variances using the two-variance method. (Indicate whether each variance is
favorable or unfavorable.)

SOLUTION

Materials Variances

Lead alloy:
Actual (122,000 oz. @ $.20)............................................................................................... $ 24,400
Actual usage at standard cost (122,000 oz. @ $.22).......................................................... 26,840
Price variance...................................................................................................................... $ (2,440) fav.

Actual usage at standard cost.............................................................................................. $ 26,840


Standard usage at standard cost (3 oz. per unit x 40,000 units x $.22).............................. 26,400
Quantity variance................................................................................................................. $ 440 unfav.

Cover materials:
Actual (235,000 oz. @ $.04)............................................................................................... $ 9,400
Actual usage at standard cost (same).................................................................................. 9,400
Price variance...................................................................................................................... $ 0

Actual usage at standard cost.............................................................................................. $ 9,400


Standard usage at standard cost (6 oz. per unit x 40,000 units x $.04).............................. 9,600
Quantity variance................................................................................................................. $ (200) fav.

Container boxes:
Actual (40,500 @ $.09)....................................................................................................... $ 3,645
Actual usage at standard cost (40,500 @ $.10).................................................................. 4,050
Price variance...................................................................................................................... $ (405) fav.

Actual usage at standard cost.............................................................................................. $ 4,050


Standard usage at standard cost (40,000 x $.10)................................................................ 4,000
Quantity variance................................................................................................................. $ 50 unfav.

Labor Variances

Actual (9,500 hrs. @ $12.50)..................................................................................................... $ 118,750


Actual hours at standard rate (9,500 hrs. @ $12.00)................................................................. 114,000
Labor rate variance..................................................................................................................... $ 4,750 unfav.

Actual hours at standard rate...................................................................................................... $ 114,000


Standard hours at standard rate (1/4 hr. per unit x 40,000
units x $12.00)..................................................................................................................... 120,000
Labor efficiency variance........................................................................................................... $ (6,000) fav.
Standard Costing: Incorporating Standards into the Accounting Records 286

Overhead Variances

Actual overhead................................................................................................. $ 90,000


Budget allowance based on standard hours allowed:
Fixed overhead........................................................................................... $25,000
Variable overhead [($2.50 per unit x 50,000 units)
- $25,000] x 80%............................................................................... 80,000 105,000
Controllable variance........................................................................................ $ (15,000) fav.

Budget allowance.............................................................................................. $ 105,000


Standard cost charged in (40,000 units x
1/4 hr. per unit x $10)................................................................................ 100,000
Volume variance............................................................................................... $ 5,000 unfav.

PROBLEM

4.
Allocation of Variances to Inventory and Cost of Goods Sold. The management of Paco Products was
presented with the following distribution of materials, labor, and overhead costs in inventories and cost of goods
sold:

Materials Direct Labor Overhead


Costs Costs Costs
Materialsending inventory................................................ $ 100,000 -- --
Work in processending inventory..................................... 150,000 $ 250,000 $ 150,000
Finished goodsending inventory....................................... 50,000 250,000 150,000
Cost of goods sold............................................................... 800,000 2,000,000 1,200,000
$ 1,100,000 $ 2,500,000 $ 1,500,000

During the year, the following variances were noted:

Materials price usage....................................................................................................... $(10,000) favorable


Materials quantity............................................................................................................ 22,280 unfavorable
Labor rate......................................................................................................................... (27,000) favorable
Labor efficiency............................................................................................................... 23,000 unfavorable
Net overhead.................................................................................................................... 31,500 unfavorable

Required:

(1) Allocate the variances to inventories and cost of goods sold.


(2) Determine the cost of goods sold after the allocation of variances.
Standard Costing: Incorporating Standards into the Accounting Records 287

SOLUTION

(1)
Materials price usage variance to:

$150,000
Work in process......................................................... ---------------- x $ (10,000) = (1,500) fav.
$1,000,000

$50,000
Finished goods .......................................................... ---------------- x $ (10,000) = (500) fav.
$1,000,000

$800,000
Cost of goods sold...................................................... ---------------- x $ (10,000) = (8,000) fav.
$1,000,000
Total................................................................... $ (10,000) fav.

Materials quantity variance to:

$150,000
Work in process......................................................... ---------------- x $ 22,280 = $ 3,342 unfav.
$1,000,000

$50,000
Finished goods........................................................... ---------------- x $ 22,280 = 1,114 unfav.
$1,000,000

$800,000
Cost of goods sold...................................................... ---------------- x $ 22,280 = 17,824 unfav.
$1,000,000
Total................................................................... $ 22,280 unfav.

Labor variances to:

$250,000
Work in process......................................................... ---------------- x $ (4,000) = $ (400) fav.
$2,500,000

Finished goods (same as to work in process)............ = (400) fav.

$2,000,000
Cost of goods sold...................................................... ---------------- x $ (4,000) = (3,200) fav.
$2,500,000
Total................................................................... $ (4,000)1 fav.

1
Labor rate variance - Labor efficiency variance = Net labor variance
($27,000) fav. - $23,000 unfav. = ($4,000) fav.
Standard Costing: Incorporating Standards into the Accounting Records 288

Overhead variances to:

$150,000
Work in process......................................................... ---------------- x $ 31,500 = $ 3,150 unfav.
$1,500,000

$150,000
Finished goods........................................................... ---------------- x $ 31,500 = 3,150 unfav.
$1,500,000

$1,200,000
Cost of goods sold...................................................... ---------------- x $ 31,500 = 25,200 unfav.
$1,500,000
Total................................................................... $ 31,500 unfav.

(2)
Standard cost of goods sold:
Materials.................................................................................................................................... $ 800,000
Labor.......................................................................................................................................... 2,000,000
Overhead.................................................................................................................................... 1,200,000
$ 4,000,000
Add unfavorable variances:
Materials quantity...................................................................................................................... 17,824
Overhead.................................................................................................................................... 25,200
Less favorable variances:
Materials price usage................................................................................................................. (8,000)
Labor.......................................................................................................................................... (3,200)
Cost of goods sold after allocation.................................................................................................... $ 4,031,824

The following problems are based on material in the Appendix to the chapter.

PROBLEM

5.
Journal Entries for Factory Overhead; Alternate Three-Variance Method. The practical capacity of Mindy
Manufacturing Company is 10,000 units of product Mork. At the normal capacity level (80% of practical), the
following factory amounts have been budgeted:

Fixed.................................................................................................................................................. $27,000
Variable............................................................................................................................................. $29,000

Standards were set as follows:


Processing time, 2 hours per unit of Mork
Factory overhead, $3.50 per hour of processing

Actual data for November were:


Production, 7,600 units of Mork
Processing time, 15,400
Factory overhead, $55,500
Standard Costing: Incorporating Standards into the Accounting Records 289

Required: Assuming that actual and applied overhead are recorded in separate accounts, give the general journal
entries to record actual overhead, to charge overhead to production, to close the two overhead accounts, and to
record the overhead variances using the alternative three-variance method.

SOLUTION

Factory Overhead Control....................................................................................... 55,500.00


Various Credits................................................................................................ 55,500.00

Work in Process ($3.50 F.O. rate x 7,600 units x 2 SH per unit)........................... 53,200.00
Applied Factory Overhead.............................................................................. 53,200.00

Applied Factory Overhead...................................................................................... 53,200.00


Efficiency Variance [$3.50 F.O. rate x (15,400 AH - 15,200 SH)]....................... 700.00
Idle Capacity Variance [$1.6875 fix. rate x
(16,000 BH - 15,400 AH)]............................................................................... 1,012.50
Spending Variance................................................................................................... 587.50
Factory Overhead Control............................................................................... 55,500.00

PROBLEM

6.
Journal Entries for Factory Overhead; Four-Variance Method. Melvin Corporation charges factory overhead
to production on the basis of the standard processing time allowed for actual production. The following data
relate to the results of operations for December:

Normal capacity in processing hours........................................................................................................ 5,000


Standard processing hours allowed for actual production........................................................................ 4,600
Actual processing hours required during December................................................................................ 5,200

The factory overhead rate per hour of processing based on normal capacity follows:

Variable overhead..................................................................................... $ 45,000  5,000 hours = $ 9


Fixed overhead.......................................................................................... 155,000  5,000 hours = 31
Total factory overhead.............................................................................. $ 200,000  5,000 hours = $ 40

Actual factory overhead incurred during December totaled $199,000.

Required: Give the appropriate general journal entries to record the actual overhead cost, to record the charge to
production for overhead (assuming that actual and applied overhead are recorded in separate accounts), and the
closing of the two overhead accounts along with the appropriate overhead variances using the four-variance
method.
Standard Costing: Incorporating Standards into the Accounting Records 290

SOLUTION

Factory Overhead Control.............................................................................................. 199,000


Various Credits....................................................................................................... 199,000

Work in Process ($40 F.O. rate x 4,600 SH)................................................................. 184,000


Applied Factory Overhead..................................................................................... 184,000

Applied Factory Overhead............................................................................................. 184,000


Variable Efficiency Variance [$9 var. x (5,200 AH - 4,600 SH)]................................ 5,400
Fixed Efficiency Variance [$31 fix. x (5,200 AH - 4,600 SH)]................................... 18,600
Spending Variance................................................................................................. 2,800
Idle Capacity Variance [$31 fix. x (5,000 BH - 5,200 AH)]................................ 6,200
Factory Overhead Control...................................................................................... 199,000
Standard Costing: Incorporating Standards into the Accounting Records 251

Chapter 18

STANDARD COSTING: SETTING


STANDARDS AND ANALYZING VARIANCES

MULTIPLE CHOICE

Question Nos. 11-16, 18, 19, 21, 22, 26-28, 31, 35, and 36 are AICPA adapted.
Question Nos. 23-25 and 30 are ICMA adapted.
Question Nos. 17, 20, 29, 32-34, and 37 are CIA adapted.

D 1. The type of standard that is intended to represent challenging yet attainable results is:
A. theoretical standard
B. flexible budget standard
C. controllable cost standard
D. normal standard
E. expected actual standard

A 2. Standard costs are used for all of the following except:


A. income determination
B. controlling costs
C. measuring efficiencies
D. forming a basis for price setting
E. establishing budgets

C 3. Of the following variances, the one that is most useful in assessing the performance of the
Purchasing Department is the:
A. idle capacity variance
B. overhead price variance
C. materials purchase price variance
D. labor rate variance
E. materials price usage variance

B 4. The labor efficiency variance is computed as:


A. the difference between standard and actual rates, multiplied by standard hours
B. the difference between standard and actual hours, multiplied by standard rate
C. the difference between standard and actual rates, multiplied by actual hours
D. the difference between standard and actual hours, multiplied by the difference between
standard and actual rates
E. a percentage of the labor time variance

251
Standard Costing: Setting Standards and Analyzing Variances 252

B 5. The method used to assure fairness in the rates paid for each operation performed by an employee is:
A. job costing
B. job rating
C. union contracting
D. the agreed-upon wages at the time of employment
E. labor rate variance analysis

D 6. Materials and labor cost standards are generally based on:


A. expected actual conditions, anticipated prices, and desired efficiency levels
B. theoretical conditions, present price levels, and desired efficiency levels
C. capacity conditions, anticipated prices, and desired efficiency levels
D. normal conditions, present price levels, and desired efficiency levels
E. theoretical conditions, anticipated prices, and theoretically attainable efficiency levels

D 7. The most effective standards are set following a careful study of products and operating conditions
by the:
A. Accounting Department, central management, and the Industrial Engineering Department
B. central management and the employees whose performance is being evaluated
C. Accounting Department and engineering staff
D. Industrial Engineering Department and the employees whose performance is being evaluated
E. central management and the Industrial Engineering Department

E 8. In analyzing factory overhead variances, the volume variance is the difference between the:
A. actual amount spent for overhead items during the period and the amount applied during the
period
B. variable efficiency variance and fixed efficiency variance
C. amount shown in the flexible budget and the amount shown in the master budget
D. master budget application rate and the flexible budget application rate, multiplied by actual
hours worked
E. budget allowance based on standard hours allowed for actual production for the period and the
amount of applied factory overhead during the period

D 9. The variance resulting from obtaining an output different from the one expected on the basis of input
is the:
A. mix variance
B. output variance
C. usage variance
D. yield variance
E. efficiency variance

A 10. In its reports to management, a company disclosed the presence of a fixed efficiency variance. The
procedure used to analyze variances was the:
A. four-variance method
B. mix and yield variances method
C. two-variance method
D. alternative three-variance method
E. three-variance method
Standard Costing: Setting Standards and Analyzing Variances 253

D 11. A purpose of standard costing is to:


A. allocate cost with more accuracy
B. eliminate the need for subjective decisions by management
C. determine the "break-even" production level
D. control costs
E. all of the above

A 12. Which one of the following is true concerning standard costs?


A. If properly used, standards can help motivate employees.
B. Unfavorable variances, material in amount, should be investigated, but large favorable
variances need not be investigated.
C. Standard costs are difficult to use with a process costing system.
D. Standard costs are estimates of costs attainable only under the most ideal conditions, but
rarely practicable.
E. All of the above

A 13. When computing variances from standard costs, the difference between actual and standard price
multiplied by actual quantity yields a:
A. price variance
B. volume variance
C. mix variance
D. yield variance
E. combined price-quantity variance

E 14. A company controls its production costs by comparing its actual monthly production costs with the
expected levels. Any significant deviations from expected levels are investigated and evaluated as a
basis for corrective actions. The quantitative technique that is most probably being used is:
A. time-series or trend regression analysis
B. correlation analysis
C. differential calculus
D. risk analysis
E. standard cost variance analysis

C 15. What type of direct material variances for price and usage will arise if the actual number of pounds
of materials used was less than standard pounds allowed but actual cost exceeds standard cost?

Usage Price
A. unfavorable favorable
B. favorable favorable
C. favorable unfavorable
D. unfavorable unfavorable
E. none none

B 16. If a company follows a practice of isolating variances at the earliest time, the appropriate time to
isolate and recognize a direct materials price variance would be when:
A. the purchase order is originated
B. materials are purchased
C. materials are issued
D. the materials requisition is prepared
E. materials are used in production
Standard Costing: Setting Standards and Analyzing Variances 254

A 17. Which of the following would least likely cause an unfavorable materials quantity (usage) variance?
A. labor that possesses skills equal to those required by the standards
B. scheduling of substantial overtime
C. a mix of direct materials that does not conform to plan
D. materials that do not meet specifications
E. machinery that has not been maintained properly

D 18. Information about Sargent Company's direct material costs is as follows:

Standard unit price $3.60


Actual quantity purchased 1,600
Standard quantity allowed for actual production 1,450
Materials purchase price varianceunfavorable $240

What was the actual purchase price per unit, rounded to the nearest penny?
A. $3.06
B. $3.11
C. $3.45
D. $3.75
E. $3.60

SUPPORTING CALCULATION:

$240 = 1,600 (x - $3.60)


1,600 x = $240 + $5,760
x = $3.75

C 19. Using the following symbols, which formula represents the calculation of the labor rate variance?

AH = Actual hours
SH = Standard hours allowed for actual production
AR = Actual rate
SR = Standard rate

A. SR(AH - SH)
B. AR(AH - SH)
C. AH(AR - SR)
D. SH(AR - SR)
E. SH(SR - AR)

D 20. When a change in the manufacturing process reduces the number of direct labor hours and standards
are unchanged, the resulting variance will be:
A. an unfavorable labor usage variance
B. an unfavorable labor rate variance
C. a favorable labor rate variance
D. a favorable labor usage variance
E. both (C) and (D) above
Standard Costing: Setting Standards and Analyzing Variances 255

B 21. The most probable reason a company would experience a favorable labor rate variance and an
unfavorable labor efficiency variance is that:
A. the mix of workers assigned to the particular job was heavily weighted toward the use of
higher paid, experienced individuals
B. the mix of workers assigned to the particular job was heavily weighted toward the use of new,
relatively low-paid, unskilled workers
C. because of the production schedule, workers from other production areas were assigned to
assist in this particular process
D. defective materials caused more labor to be used in order to produce a standard unit
E. the actual price paid for materials that went into production was less than the standard price
that was expected to be paid

C 22. Information on Orman Company's direct labor costs is as follows:

Standard direct labor rate................................................................................................ $3.75


Actual direct labor rate.................................................................................................... $3.50
Standard direct labor hours.............................................................................................. 10,000
Direct labor usage (efficiency) varianceunfavorable.................................................... $ 4,200

What were the actual hours worked, rounded to the nearest hour?
A. 11,914
B. 10,714
C. 11,120
D. 11,200
E. none of the above

SUPPORTING CALCULATION:

$4,200 = $3.75 (x - 10,000)


$3.75 x = $4,200 + $37,500
x = 11,120

D 23. Each unit of Product 8in1 requires two direct labor hours. Employee benefit costs are treated as
direct labor costs. Data on direct labor are as follows:

Number of direct employees........................................................................................... 25


Weekly productive hours per employee.......................................................................... 30
Estimated weekly wages per employee.......................................................................... $240
Employee benefits (related to weekly wages)................................................................ 25%

The standard direct labor cost per unit of Product 8in1 is:
A. $8.00
B. $10.00
C. $12.00
D. $20.00
E. none of the above

SUPPORTING CALCULATION:

$240 + . 25 (240 )
= $20 / unit
30 ÷ 2
Standard Costing: Setting Standards and Analyzing Variances 256

B 24. J. R. Richard Company employs a standard absorption system for product costing. The standard cost
of its product is as follows:

Direct materials................................................................................................................ $14.50


Direct labor (2 direct labor hours x $8).......................................................................... 16.00
Manufacturing overhead (2 direct labor hours x $11).................................................... 22.00
Total standard cost........................................................................................................... $52.50

The manufacturing overhead rate is based upon a normal activity level of 600,000 direct labor hours.
Richard planned to produce 25,000 units each month during the year. The budgeted annual
manufacturing overhead is:

Variable............................................................................................................................ $3,600,000
Fixed................................................................................................................................. 3,000,000
.......................................................................................................................................... $6,600,000

During November, Richard produced 26,000 units. Richard used 53,500 direct labor hours in
November at a cost of $433,350. Actual manufacturing overhead for the month was $250,000 fixed
and $325,000 variable.

The manufacturing overhead controllable variance for November is:


A. $9,000 unfavorable
B. $13,000 unfavorable
C. $9,000 favorable
D. $4,000 favorable
E. none of the above

SUPPORTING CALCULATION:

Actual factory overhead.................................................. $ 575,000


Budget allowance:
Variable factory overhead (52,000 x $6)................ $312,000
Budgeted fixed overhead......................................... 250,000 562,000
Controllable variance...................................................... $ 13,000 unfavorable
Standard Costing: Setting Standards and Analyzing Variances 257

B 25. J. R. Richard Company employs a standard absorption system for product costing. The standard cost
of its product is as follows:

Direct materials............................................................................................................. $14.50


Direct labor (2 direct labor hours x $8)........................................................................ 16.00
Manufacturing overhead (2 direct labor hours x $11).................................................. 22.00
Total standard cost........................................................................................................ $52.50

The manufacturing overhead rate is based upon a normal activity level of 600,000 direct labor hours.
Richard planned to produce 25,000 units each month during the year. The budgeted annual
manufacturing overhead is:

Variable......................................................................................................................... $3,600,000
Fixed ............................................................................................................................ 3,000,000
$6,600,000

During November, Richard produced 26,000 units. Richard used 53,500 direct labor hours in
November at a cost of $433,350. Actual manufacturing overhead for the month was $250,000 fixed
and $325,000 variable.

The manufacturing overhead volume variance for November is:


A. $12,000 unfavorable
B. $10,000 unfavorable
C. $3,000 unfavorable
D. $9,000 unfavorable
E. $1,000 favorable

SUPPORTING CALCULATION:

Budget allowance based on standard hours allowed


[(52,000 x $6) + $250,000].............................................................. $ 562,000
Factory overhead applied at standard...................................................... 572,000
Volume variance...................................................................................... $ (10,000) favorable

C 26. The following information relates to Department 1 of Ruiz Company for the fourth quarter. The
total overhead variance is divided into three variances: spending, variable efficiency, and volume.

Actual total overhead (fixed plus variable)...................................... $178,500


Budget formula................................................................................. $110,000 + $.50 per hour
Total overhead application rate........................................................ $1.50 per hour
Actual hours worked......................................................................... 121,000

What was the spending variance in this department during the quarter?
A. $8,000 favorable
B. $4,500 favorable
C. $8,000 unfavorable
D. $4,500 unfavorable
E. none of the above
Standard Costing: Setting Standards and Analyzing Variances 258

SUPPORTING CALCULATION:

Actual factory overhead.................................................... $ 178,500


Budget allowance:
Variable for actual hours
(121,000 x $.50).................................................. $ 60,500
Fixed.......................................................................... 110,000 170,500
Spending variance............................................................. $ 8,000 unfavorable

A 27. The following information relates to Department 1 of Ruiz Company for the fourth quarter. The
total overhead variance is divided into three variances: spending, variable efficiency, and volume.

Actual total overhead (fixed plus variable)...................................... $178,500


Budget formula................................................................................. $110,000 + $.50 per hour
Total overhead application rate........................................................ $1.50 per hour
Actual hours worked......................................................................... 121,000
Standard hours allowed for production............................................ 130,000

What was the variable efficiency variance in this department during the quarter?
A. $4,500 favorable
B. $8,000 favorable
C. $4,500 unfavorable
D. $8,000 unfavorable
E. none of the above

SUPPORTING CALCULATION:

Budget allowance for actual hours


[(121,000 x $.50) + $110,000].................................. $ 170,500
Budget allowance for standard hours:
Variable (130,000 x $.50)......................................... $ 65,000
Fixed.......................................................................... 110,000 175,000
Variable efficiency variance............................................. $ (4,500) favorable

E 28. Under the two-variance method for analyzing factory overhead, the controllable (budget) variance is
the difference between the:
A. actual fixed factory overhead and the budgeted fixed overhead
B. budget allowance based on standard hours allowed and the factory overhead applied to
production
C. budget allowance based on standard hours allowed and the budget allowance based on actual
hours worked
D. actual factory overhead and the factory overhead applied to production
E. actual factory overhead and the budget allowance based on standard hours allowed

A 29. Materials usage variances are normally chargeable to the:


A. Production Department
B. Purchasing Department
C. Finished Goods Department
D. Materials Storage Department
E. Factory Storeroom Department
Standard Costing: Setting Standards and Analyzing Variances 259

C 30. Todco planned to produce 3,000 units of its single product, Teragram, during November. The
standard specifications for one unit of Teragram include six pounds of material at $.30 per pound.
Actual production in November was 3,100 units of Teragram. The accountant computed a favorable
materials purchase price variance of $380 and an unfavorable materials quantity variance of $120.
Based on these variances, one could conclude that:
A. more materials were purchased than were used
B. more materials were used than were purchased
C. the actual cost of materials was less than the standard cost
D. the actual usage of materials was less than the standard allowed
E. actual cost and usage of materials were both less than standard

D 31. Information on Duke Co.'s direct material costs for May is as follows:

Actual quantity of direct materials purchased and used............................................... 30,000 lbs.


Actual cost of direct materials...................................................................................... $84,000
Unfavorable direct materials usage variance................................................................ 3,000
Standard quantity of direct materials allowed for May production............................. 29,000 lbs.

For the month of May, Duke's direct materials price variance was:
A. $2,800 favorable
B. $2,800 unfavorable
C. $6,000 unfavorable
D. $6,000 favorable
E. none of the above

SUPPORTING CALCULATION:

$3,000 = x (30,000 - 29,000)


1,000 x = $3,000
x = $3
y = $2.80 - $3.00(30,000)
y = ($6,000) favorable

A 32. A company uses a standard cost system to account for its only product. The materials standard per
unit was 4 lbs. at $5.10 per lb. Operating data for April were as follows:

Material used................................................................................................................. 7,800 lbs.


Cost of material used..................................................................................................... $40,950
Number of finished units produced............................................................................... 2,000

The material usage variance for April was:


A. $1,020 favorable
B. $1,050 favorable
C. $1,170 unfavorable
D. $1,200 unfavorable
E. none of the above
Standard Costing: Setting Standards and Analyzing Variances 260

SUPPORTING CALCULATION:

x = $5.10 [7,800 - (2,000 x 4)]


x = ($1,020) favorable

D 33. During the last three months, a manufacturer incurred an unfavorable labor efficiency variance. The
least likely cause of this variance is:
A. substantial materials were purchased at a discount at a previously unused supplier's
liquidation
B. for one week, only half of the workforce, those with the highest seniority, were called in to
work
C. a second production line with all new personnel was started
D. the cost-of-living adjustment for the three-month period was $.10 more per hour than
expected
E. none of the above

D 34. The direct labor standards for producing a unit of a product are two hours at $10 per hour. Budgeted
production was 1,000 units. Actual production was 900 units, and direct labor cost was $19,000 for
2,000 direct labor hours. The direct labor efficiency variance was:
A. $1,000 favorable
B. $1,000 unfavorable
C. $2,000 favorable
D. $2,000 unfavorable
E. none of the above

SUPPORTING CALCULATION:

x = $10 [2,000 - (900 x 2)]


x = $2,000 unfavorable

C 35. Under the two-variance method for analyzing factory overhead, the factory overhead applied to
production is used in the computation of the:

Controllable Volume
(Budget) Variance Variance
A. yes no
B. yes yes
C. no yes
D. no no

D 36. Under the three-variance method for analyzing factory overhead, which of the following is used in
computation of the spending variance?

Actual Factory Budget Allowance


Overhead Based on Actual Hours
A. no yes
B. no no
C. yes no
D. yes yes
Standard Costing: Setting Standards and Analyzing Variances 261

D 37. Compute the variable efficiency variance, using the following data:

Standard labor hours per good unit produced...................................................................... 2


Good units produced............................................................................................................. 1,000
Actual labor hours used........................................................................................................ 2,100
Standard variable overhead per standard labor hour........................................................... $3
Actual variable overhead...................................................................................................... $ 6,500

A. $200 favorable
B. $200 unfavorable
C. $300 favorable
D. $300 unfavorable
E. none of the above

SUPPORTING CALCULATION:

Variable budget allowance for actual hours (2,100 x $3).............................. $ 6,300


Variable budget allowance for standard hours
($3 x 1,000 x 2)....................................................................................... 6,000
................................................................................................................. $ 300
..............................................................................................unfavorable

The following questions are based on materials in the Appendix to the chapter.

A 38. In the alternate three-variance method, the efficiency variance is:


A. Standard factory overhead rate x (Actual units of allocation base - Standard units of allocation
base allowed)
B. Actual factory overhead incurred - Budget allowance based on actual hours
C. Budget allowance based on actual hours - (Actual hours x Factory overhead rate)
D. Budgeted fixed factory overhead - (Actual hours x Fixed overhead rate)
E. none of the above

D 39. The four-variance method reconciles to the two-variance method by combining which of the
following to get the controllable variance?
A. fixed efficiency variance and idle capacity variance
B. spending variance and fixed efficiency variance
C. spending variance and idle capacity variance
D. spending variance and variable efficiency variance
E. none of the above

B 40. The four-variance method reconciles to the two-variance method by combining which of the
following to get the volume variance?
A. spending variance and variable efficiency variance
B. fixed efficiency variance and idle capacity variance
C. variable efficiency variance and fixed efficiency variance
D. spending variance and idle capacity variance
E. none of the above
Standard Costing: Setting Standards and Analyzing Variances 262

PROBLEMS

PROBLEM

1.
Labor Variance Analysis. Last National Bank uses a standard cost accounting system for analyzing its labor
costs in its Proof and Transit Division. The primary task of this division is the encoding of checks with magnetic
ink for reading by the computer. The standard calls for an employee to process 900 checks per hour and to be
paid $10 per hour. During the eight-hour night shift last Wednesday, the production levels attained by the four
employees on that shift, together with their hourly wages, were:

Employee Checks Encoded Hourly Wages


Wilson................................................................................................ 7,020 $11.00
Xavier................................................................................................. 6,480 9.25
Yelding............................................................................................... 7,875 10.50
Ziachin............................................................................................... 7,425 9.75

Required: Compute the labor rate variance and the labor efficiency variance for each employee and for the entire
night shift.

SOLUTION

Wilson Xavier Yelding Ziachin Total


Actual rate................................................... $ 11.00 $ 9.25 $ 10.50 $ 9.75 $ 40.50
Standard rate............................................... 10.00 10.00 10.00 10.00 40.00
Rate difference............................................ $ 1.00 $ (.75) $ .50 $ (.25) $ .50
Multiplied by hours worked....................... x 8 x 8 x 8 x 8 x 8
Labor rate variance..................................... $ 8.00 $ (6.00) $ 4.00 $ (2.00) $ 4.00
unfav. fav. unfav. fav. unfav.

Actual hours worked................................... 8.0 8.0 8.00 8.00 32


Standard hours allowed.............................. 7.8 7.2 8.75 8.25 32
Difference in hours..................................... .2 .8 (.75) (.25) 0
Multiplied by standard rate......................... x $10 x $10 x $10 x $10 x $10
Labor efficiency variance........................... $ 2.00 $ 8.00 $ (7.50) $ (2.50) 0
unfav. unfav. fav. fav.
Standard Costing: Setting Standards and Analyzing Variances 263

PROBLEM

2.
Materials Variance Analyses. Healthy Dinners Inc. packages a frozen fish dinner that consists of 6 ounces of
halibut, 4 ounces of asparagus, 5 ounces of rice, and 3 ounces of yogurt. On October 1, the following price
standards were set for each batch of 1,000 dinners:

Materials
Item Price Standard
Halibut........................................................................................................................................ $.60 per ounce
Asparagus.................................................................................................................................... .25
Rice............................................................................................................................................. .10
Yogurt......................................................................................................................................... .20

The actual cost for 1,000 dinners was: halibut, $.70 per ounce; asparagus $.20 per ounce; rice, $.12 per ounce; and
yogurt, $.22 per ounce.

Quantity variances arise from the cooking process. The materials used for the 1,000 dinners in Batch 1099 were:

Halibut........................................................................................................................................ 5,500 ounces


Asparagus.................................................................................................................................... 3,800
Rice............................................................................................................................................. 4,900
Yogurt......................................................................................................................................... 3,150

Required: Determine the materials price usage variance and the materials quantity (or usage) variance for Batch
1099. (Indicate whether each variance is favorable or unfavorable.)

SOLUTION

(Actual unit price - Standard unit price) x Actual usage = Materials price usage variance

Halibut: ($.70 per oz. - $.60 per oz.) x 5,500 oz....................................................................... $ 550 unfav.
Asparagus: ($.20 per oz. - $.25 per oz.) x 3,800 oz................................................................... (190) fav.
Rice: ($.12 per oz. - $.10 per oz.) x 4,900 oz............................................................................ 98 unfav.
Yogurt: ($.22 per oz. - $.20 per oz.) x 3,150 oz........................................................................ 63 unfav.
Materials price usage variance................................................................................................... $ 521 unfav.

(Actual quantity - Standard quantity allowed) x Standard price = Materials quantity variance

Halibut: (5,500 oz. - 6,000 oz.) x $.60....................................................................................... $(300) fav.


Asparagus: (3,800 oz. - 4,000 oz.) x $.25.................................................................................. (50) fav.
Rice: (4,900 oz. - 5,000 oz.) x $.10........................................................................................... (10) fav.
Yogurt: (3,150 oz. - 3,000 oz.) x $.20........................................................................................ 30 unfav.
Materials quantity variance........................................................................................................ $(330) fav.
Standard Costing: Setting Standards and Analyzing Variances 264

PROBLEM

3.
Materials Mix and Yield Variance Analysis. Kreutzer Candle Co. manufactures candles in various shapes,
sizes, colors, and scents. Depending on the orders received, not all candles require the same amount of color, dye,
or scent materials. Yields also vary, depending upon the usage of beeswax or synthetic wax. Standard ingredients
for 1,000 lbs. of candles are:

Standard Cost
Standard Mix per Pound
Input:
Beeswax................................................................................................... 200 lbs. $1.00
Synthetic wax.......................................................................................... 840 .20
Colors....................................................................................................... 7 2.00
Scents....................................................................................................... 3 6.00
Totals................................................................................................ 1,050 lbs.

Standard output............................................................................................... 1,000 lbs.

Price variances are charged off at the time of purchase. During January, the company was busy manufacturing red
candles for Valentine's Day. Actual production then was:

Input:
Beeswax............................................................................................................................. 4,100
Synthetic wax.................................................................................................................... 13,800
Colors................................................................................................................................. 2,200
Scents................................................................................................................................. 60
Totals.......................................................................................................................... 20,160 lbs.

Actual output............................................................................................................................ 18,500 lbs.

Required: Compute the materials mix variance and the materials yield variance. (Indicate whether each variance
is favorable or unfavorable and round to three decimal places.)

SOLUTION

Actual quantities at individual standard materials cost........................................................... $ 11,6201


Actual input quantity at weighted average of standard
materials cost (20,160 x $.3812)........................................................................................ $ 7,681
Materials mix variance............................................................................................................. $ 3,939 unfav.

Actual input quantity at weighted average of standard


materials cost (20,160 x $.3812)........................................................................................ $ 7,681
Actual output quantity at standard materials cost per
pound of output (18,500 lbs. x $.403)............................................................................... 7,400
Materials yield variance........................................................................................................... $ 281 unfav.
Standard Costing: Setting Standards and Analyzing Variances 265

1
Beeswax............................................................ 4,100 lbs. @ $1 per lb............................................ $ 4,100
Synthetic wax................................................... 13,800 lbs. @ $.20 per lb......................................... 2,760
Colors............................................................... 2,200 lbs. @ $2 per lb............................................ 4,400
Scents............................................................... 60 lbs. @ $6 per lb............................................ 360
20,160 lbs. ............................................................... $ 11,620
2
Weighted average standard materials costs:
Beeswax........................................................... 200 lbs. @ $1....................................................... $ 200
Synthetic wax................................................... 840 lbs. @ $.20.................................................... 168
Colors............................................................... 7 lbs. @ $2....................................................... 14
Scents............................................................... 3 lbs. @ $6....................................................... 18
1,050 lbs. ............................................................... $ 400

$400
Standard materials cost
1,050 lbs .

3
Standard materials costs $400
= = $. 40 per lb . cost per unit of output
Standard output 1,000 lbs.

PROBLEM

4.
Overhead Variance Analysis, Using the Two-Variance Method. Tuxla Products Co. charges factory overhead
into production at the rate of $10 per direct labor hour, based on a standard production of 15,000 direct labor
hours for 15,000 units; 60% of factory overhead costs are variable. Production data for May and June are:

May June
Production................................................................................................... 12,000 hrs. 14,200 hrs.
Units produced............................................................................................ 12,000 15,000
Actual factory overhead.............................................................................. $140,100 $149,300

Required: Prepare a factory overhead variance analysis for May and June, using the two-variance method.
(Indicate whether each variance is favorable or unfavorable.)

SOLUTION

May June
Actual factory overhead.......................................................................... $ 140,100 $ 149,300
Budget allowance based on standard:
Budgeted fixed expense (40% x $10 x
15,000 units).............................................................................. (60,000) (60,000)
Variable expenses:
12,000 hrs. allowed x $10 x .60................................................ (72,000)
15,000 hrs. allowed x $10 x .60................................................ (90,000)
Controllable variance.............................................................................. $ 8,100 unfav. $ (700) fav.
Standard Costing: Setting Standards and Analyzing Variances 266

Budgeted allowance based on standard


hours allowed.................................................................................... $ 132,000 $ 150,000
Standard hours allowed x Standard factory
overhead rate:
12,000 hrs. x $10....................................................................... (120,000)
15,000 hrs. x $10....................................................................... (150,000)
Volume variance...................................................................................... $ 12,000 unfav. 0

PROBLEM

5.
Overhead Variance Analysis, Using the Three-Variance Method. Standard direct labor hours budgeted for
May production were 5,000, with factory overhead at that level budgeted at $25,000, of which $15,000 is variable.
Actual labor hours for the month were 4,800; however, the number of standard labor hours allowed for actual May
production is 5,200. Actual factory overhead incurred during the month was $25,600.

Required: Compute the overall factory overhead variance and analyze it using the three-variance method (i.e., the
spending variance, the variable efficiency variance, and the volume variance). Indicate whether the variances are
favorable or unfavorable.

SOLUTION

Actual factory overhead.......................................................................... $ 25,600


Standard overhead chargeable to production (5,200
standard hours allowed x $5 overhead rate).................................... 26,000
Overall factory overhead variance.......................................................... $ (400) favorable

Actual factory overhead.......................................................................... $ 25,600


Budget allowance based on actual hours:
Variable overhead (4,800 actual hours x $3)................................... $14,400
Fixed overhead................................................................................. 10,000 24,400
Spending variance................................................................................... $ 1,200 unfavorable

Budget allowance based on actual hours (from above).......................... $ 24,400


Budget allowance based on standard hours:
Variable overhead (5,200 standard hours x $3)............................... $15,600
Fixed overhead................................................................................. 10,000 25,600
Variable efficiency variance................................................................... $ (1,200) favorable

Budget allowance based on standard hours (from above)...................... $ 25,600


Standard factory overhead chargeable to production
(from above)..................................................................................... 26,000
Volume variance...................................................................................... $ (400) favorable

Spending variance................................................................................... $ 1,200


Variable efficiency variance................................................................... (1,200)
Volume variance...................................................................................... (400)
Overall factory overhead variance.......................................................... $ (400) favorable
Standard Costing: Setting Standards and Analyzing Variances 267

The following problem is based on the material in the Appendix to the chapter.

PROBLEM

6.
Overhead Variance Analysis, Using the Four-Variance Method. In May, the management of Kentucky Co.
received the following data for its Bluegrass Products Division:

Standard 1 Actual
Units produced............................................................................................................. 5,000 5,100
Direct labor hours......................................................................................................... 10,000 10,300
Fixed factory overhead................................................................................................. $12,000 $13,000
Variable factory overhead............................................................................................ $30,000 $34,500
1
Denotes normal capacity used for predetermined overhead rate computation.

Required: Prepare a factory overhead variance analysis for May, using the four-variance method. (Indicate
whether each variance is favorable or unfavorable.)

SOLUTION

Actual factory overhead..................................................................................... $ 47,500


Budget allowance based on actual hours worked:
Fixed factory overhead............................................................................... $12,000
Variable factory overhead:

$30,000
10,300 actual hrs. x ----------------........................................................ 30,900 42,900
10,000 DLH

Spending variance.............................................................................................. $ 4,600 unfav.

Budget allowance based on actual hours worked............................................. $ 42,900


Actual hours x standard overhead rate:

$30,000 + $12,000
10,300 hrs. x --------------------------..................................................... 43,260
10,000 DLH

Idle capacity variance........................................................................................ $ (360) fav.

Budget allowance based on actual hours worked............................................. $ 42,900


Budget allowance based on standard hours allowed:
Fixed expense.............................................................................................. $12,000
Variable expense (10,200 standard hours
allowed x $3 variable overhead rate).................................................. 30,600 42,600
Variable efficiency variance.............................................................................. $ 300 unfav.

Actual hours (10,300) x fixed overhead rate ($1.20)........................................ $ 12,360


Standard hours allowed (10,200) x fixed overhead
rate ($1.20).................................................................................................. 12,240
Fixed efficiency variance.................................................................................. $ 120 unfav.
Standard Costing: Setting Standards and Analyzing Variances 291

Chapter 20

DIRECT COSTING AND COST-VOLUME-PROFIT ANALYSIS

MULTIPLE CHOICE

Question Nos. 7-10, 11-13, 27, 28, 32, and 33 are AICPA adapted.
Question Nos. 14-16, 25, 26, 29, 30, 31, and 34-35 are CIA adapted.

C 1. The costing procedure that treats fixed manufacturing costs as period costs is:
A. full costing
B. absorption costing
C. direct costing
D. conventional costing
E. none of the above

C 2. The following must be known about a production process in order to institute a direct costing
system:
A. the contribution margin and break-even point for all goods in production
B. the gross profit and margin of safety for all goods in production
C. the variable and fixed components of all costs related to production
D. the controllable and noncontrollable components of all costs related to production
E. standard production rates and times for all elements of production

E 3. A cost that is included as part of product costs under both absorption costing and direct costing is:
A. managerial staff costs
B. insurance
C. variable marketing expenses
D. taxes on factory building
E. variable materials handling labor

B 4. When inventories increase from one period to the next and all other factors remain constant, income
under direct costing:
A. will be irrelevant for decision making
B. will be smaller than under absorption costing
C. cannot be accurately computed
D. leads to smaller federal income tax payments
E. will be greater than under absorption costing

C 5. Of the following, the organization most likely to support direct costing is the:
A. American Institute of Certified Public Accountants
B. Securities and Exchange Commission
C. Institute of Management Accountants
D. Internal Revenue Service
E. Financial Accounting Standards Board

291
Direct Costing and Cost-Volume-Profit Analysis 292

E 6. The following unit costs for the production of laser guns were based on expected capacity in the
coming period:
Direct materials............................................................................................................................ $4
Direct labor................................................................................................................................... 7
Variable overhead........................................................................................................................ 2
Fixed overhead............................................................................................................................. 5
Variable marketing and administrative expenses........................................................................ 6
Fixed marketing and administrative expenses............................................................................ 4

Under the direct costing method, these units are recorded in inventory at a cost of:
A. $11
B. $16
C. $18
D. $19
E. none of the above

SUPPORTING CALCULATION:

$4 + $7 + $2 = $13

B 7. A basic tenet of direct costing is that period costs should be currently expensed. The rationale
behind this procedure is that:
A. allocation of period costs is arbitrary at best and could lead to erroneous decisions by
management
B. since period costs will occur whether or not production occurs, it is improper to allocate these
costs to production and defer a current cost of doing business
C. period costs are uncontrollable and should not be charged to a specific product
D. period costs are generally immaterial in amount and the cost of assigning the amounts to
specific products would outweigh the benefits
E. all of the above

C 8. A term more descriptive of the type of cost accounting often called direct costing is:
A. relevant costing
B. prime costing
C. variable costing
D. out-of-pocket costing
E. full costing

A 9. Costs that are treated as product costs under variable (direct) costing are:
A. only variable production costs
B. all variable costs
C. all variable and fixed manufacturing costs
D. variable manufacturing costs and fixed general and administrative costs
E. only direct costs
Direct Costing and Cost-Volume-Profit Analysis 293

A 10. Direct costing is not in accordance with generally accepted accounting principles because:
A. fixed manufacturing costs are assumed to be period costs
B. direct costing includes variable administrative costs in inventory
C. direct costing procedures are not well known in industry
D. net earnings are always overstated when using direct costing procedures
E. direct costing ignores the concept of lower of cost or market when valuing inventory

D 11. In an income statement prepared as an internal report using the direct costing method, fixed selling
and administrative expenses would:
A. be used in the computation of the contribution margin
B. be inventoried
C. appear in the same section as variable selling and administrative expenses
D. be used in the computation of operating income but not in the computation of the contribution
margin
E. not be used

D 12. A company had income of $50,000 using direct costing for a given period. Beginning and ending
inventories for that period were 13,000 units and 18,000 units, respectively. Ignoring income taxes,
if the fixed overhead application rate were $2.00 per unit, what would the income have been using
absorption costing?
A. $86,000
B. $40,000
C. $50,000
D. $60,000
E. cannot be determined from the information given

SUPPORTING CALCULATION:

$50,000 + $2 (18,000 - 13,000) = $60,000

D 13. In an income statement prepared as an internal report using the direct costing method, which of the
following terms should appear?

Gross Profit
(Margin) Operating Income (Loss)
A. Yes Yes
B. Yes No
C. No No
D. No Yes
E. No Sometimes
Direct Costing and Cost-Volume-Profit Analysis 294

D 14. Using absorption costing, which of the following columns includes only product costs?

.................................................................................................... A B C D
Direct labor........................................................................................... X X X
Direct materials.................................................................................... X X X
Sales materials...................................................................................... X
Advertising costs.................................................................................. X
Indirect factory materials..................................................................... X X X
Indirect labor......................................................................................... X X X
Sales commissions................................................................................ X
Factory utilities..................................................................................... X X X
Administrative supplies expense.......................................................... X
Administrative labor............................................................................. X
Depreciation on administration building............................................. X
Cost of research on customer demographics....................................... X

A. A
B. B
C. C
D. D
E. none of the above

B 15. A company manufactures 50,000 units of a product and sells 40,000 units. Total manufacturing cost
per unit is $50 (variable manufacturing cost, $10; fixed manufacturing cost, $40). Assuming no
beginning inventory, the effect on net income if absorption costing is used instead of variable costing
is that:
A. net income is $400,000 lower
B. net income is $400,000 higher
C. net income is the same
D. net income is $200,000 higher
E. none of the above

SUPPORTING CALCULATION:

$40 (50,000 - 40,000) = $400,000


Direct Costing and Cost-Volume-Profit Analysis 295

B 16. A company has the following cost data:

Fixed manufacturing costs..................................................................................................... $2,000


Fixed selling, general, and administrative costs.................................................................... 1,000
Variable selling costs per unit sold........................................................................................ 1
Variable manufacturing costs per unit................................................................................... 2

Beginning inventory............................................................................. 0 units


Production............................................................................................. 100 units
Sales .................................................................................................... 90 units at $40 per unit

Variable and absorption-cost net incomes are:


A. $320 variable, $520 absorption
B. $330 variable, $530 absorption
C. $520 variable, $320 absorption
D. $530 variable, $330 absorption
E. none of the above

SUPPORTING CALCULATION:

Variable: $3,600 - $180 - $90 - $2,000 - $1,000 = $330


Absorption: $3,600 - $180 - [(90  100) x $2,000] - $90 - $1,000 = $530

C 17. All of the following statements related to the use of break-even analysis are true except:
A. a change in fixed costs changes the break-even point but not the contribution margin figure
B. a combined change in fixed and variable costs in the same direction causes a sharp change in
the break-even point
C. a change in fixed costs changes the contribution margin figure but not the break-even point
D. a change in per-unit variable costs changes the contribution margin ratio
E. a change in sales price changes the break-even point

E 18. The costing method that lends itself most readily to the preparation of break-even analysis is:
A. weighted average costing
B. absorption costing
C. first-in, first-out costing
D. semivariable costing
E. direct costing

E 19. The break-even volume in units is found by dividing fixed expenses by the:
A. unit gross profit
B. total variable expenses
C. unit net profit
D. contribution margin ratio
E. unit contribution margin
Direct Costing and Cost-Volume-Profit Analysis 296

C 20. A major assumption concerning cost and revenue behavior that is important to the development of
break-even charts is that:
A. all costs are variable
B. total costs are quadratic
C. costs and revenues are linear
D. the relevant range is greater than sales volume
E. costs will not exceed revenues

B 21. If the fixed cost attendant to a product increases while the variable cost and sales price remain
constant, the contribution margin and break-even point will:

Contribution Margin Break-Even Point


A. increase increase
B. not change increase
C. not change not change
D. increase decrease
E. decrease increase

E 22. If current sales are $1,000,000 and break-even sales are $600,000, the margin of safety ratio is:
A. 6%
B. 60%
C. 167%
D. 100%
E. 40%

SUPPORTING CALCULATION:

$1,000,000 - $600,000
= 40%
$1,000,000

A 23. Assuming that there is no effect on other products that are manufactured, a company should
discontinue a product line for economic reasons when the:
A. contribution margin from the product line is negative
B. sales of the product are less than the break-even point
C. profit from the product line is less than that for the other products
D. profit from the product line is negative
E. contribution margin from the product line is less than that for other products

E 24. When referring to the "margin of safety," an accountant would be thinking of:
A. the excess of sales revenue over variable costs
B. the excess of budgeted or actual sales over the contribution margin
C. the excess of budgeted or actual sales revenue over fixed costs
D. the excess of actual sales over budgeted sales
E. none of the above
Direct Costing and Cost-Volume-Profit Analysis 297

C 25. Based on the cost-volume-profit chart in Figure 20-1 for a manufacturing company, the correct
statement is:
A. line b graphs total fixed costs
B. point c represents the point at which the marginal contribution per unit increases
C. line d graphs total costs
D. area e (between lines b and d) represents the contribution margin
E. area a represents the area of net loss

B 26. A valid assumption for cost-volume-profit analysis is:


A. an increase in fixed costs will cause the break-even point to rise
B. demand is constant regardless of price
C. a decrease in variable cost per unit will lower the break-even point
D. variable costs per unit are assumed to remain constant within the range of activity analyzed
E. all of the above are invalid assumptions

D 27. The following information pertains to Izzy Co.:

Sales (50,000 units).......................................................................................................... $1,000,000


Direct materials and direct labor...................................................................................... 300,000
Factory overhead:
Variable..................................................................................................................... 40,000
Fixed.......................................................................................................................... 70,000
Selling and general expenses:
Variable..................................................................................................................... 10,000
Fixed.......................................................................................................................... 60,000

How much was Izzy's break-even point in number of units?


Direct Costing and Cost-Volume-Profit Analysis 298

A. 18,571
B. 26,000
C. 9,848
D. 10,000
E. none of the above
Direct Costing and Cost-Volume-Profit Analysis 299

SUPPORTING CALCULATION:

$70,000 + $60,000
= 10,000
($1,000,000 ÷ 50,000 ) - ( $350,000 ÷ 50,000 )

A 28. The following information pertains to Izzy Co.:

Sales (50,000 units).......................................................................................................... $1,000,000


Direct materials and direct labor...................................................................................... 300,000
Factory overhead:
Variable..................................................................................................................... 40,000
Fixed.......................................................................................................................... 70,000
Selling and general expenses:
Variable..................................................................................................................... 10,000
Fixed.......................................................................................................................... 60,000

What was Izzy's contribution margin ratio?


A. 65%
B. 59%
C. 35%
D. 66%
E. none of the above

SUPPORTING CALCULATION:

$300,000 + $40,000 + $10,000


1⋅ = . 65
$1,000,000

A 29. A result from lowering the break-even point is:


A. an increase in the sales price per unit
B. an increase in the semivariable cost per unit
C. an increase in the variable cost per unit
D. a decrease in the contribution margin per unit
E. an increase in income tax rates

C 30. A company manufactures a single product that sells for $30. If the company has fixed costs of
$150,000 and a contribution margin of 40%, the break-even point in sales dollars is:
A. $250,000
B. $275,000
C. $375,000
D. $525,000
E. none of the above

SUPPORTING CALCULATION:

$150,000  .40 = $375,000


Direct Costing and Cost-Volume-Profit Analysis 300

C 31. A company producing widgets expects to incur fixed costs during the next year of $3 million. It also
expects to incur handling costs of $1 per widget, labor costs of $3 per widget, and materials costs of
$2 per widget. The company produces widgets only when ordered and, therefore, does not incur any
carrying costs. It sells widgets for $10 each. The number of widgets that must be sold next year in
order to break even is:
A. 500,000 units
B. 600,000 units
C. 750,000 units
D. 1,000,000 units
E. none of the above

SUPPORTING CALCULATION:

$3,000,000  ($10 - $6) = 750,000

E 32. Clark Co.'s operating percentages were as follows:

Sales................................................................................................................ 100%
Cost of sales:
Variable.................................................................................................... 50%
Fixed........................................................................................................ 10 60
Gross profit..................................................................................................... 40%
Other operating expenses:
Variable.................................................................................................... 20%
Fixed........................................................................................................ 15 35
Operating income............................................................................................ 5%

Clark's sales totaled $2,000,000. At what sales level would Clark break even?
A. $1,900,000
B. $666,667
C. $1,250,000
D. $833,333
E. $1,666,667

SUPPORTING CALCULATION:

[$2,000,000 x 25%]  [1 - (70%  100%)] = $1,666,667

C 33. The following information pertains to Neon Co.'s cost-volume-profit relationships:

Break-even point in units sold......................................................................................... 1,000


Variable costs per unit...................................................................................................... $500
Total fixed costs............................................................................................................... $ 150,000

How much will be contributed to profit when unit 1,001 is sold?


A. $650
B. $500
C. $150
D. 0
E. none of the above
Direct Costing and Cost-Volume-Profit Analysis 301

SUPPORTING CALCULATION:

Break-even point = (1,000 x $500) + $150,000 = $650,000


 Selling price = $650,000  1,000 = $650
Contribution margin = $650 - $500 = $150

C 34. During June, a company expects sales revenue from its only product to be $300,000, fixed costs to
be $90,000, and variable costs to be $120,000. If the company's actual sales revenue during June is
$350,000, its profit would be:
A. $90,000
B. $105,000
C. $120,000
D. $140,000
E. none of the above

SUPPORTING CALCULATION:

Sales .................................................................................................................................. $350,000


Variable costs...................................................................................................................... 140,000
Contribution margin............................................................................................................ $210,000
Fixed costs........................................................................................................................... 90,000
Profit .................................................................................................................................. $120,000

C 35. A company has just completed the final development of its only product, general recombinant
bacteria, that kills most insects before dying. The product has taken three years and $6,000,000 to
develop. The following costs are expected to be incurred on a monthly basis for the production of
1,000,000 pounds of the new product:

1,000,000 Pounds
Direct materials..................................................................................................... $ 300,000
Direct labor........................................................................................................... 1,250,000
Variable overhead................................................................................................. 450,000
Fixed overhead...................................................................................................... 2,000,000
Variable selling, general, and administrative expenses....................................... 900,000
Fixed selling, general, and administrative expenses............................................ 1,500,000
Total............................................................................................................... $ 6,400,000

At a sale price of $5.90 per pound, the sales in pounds necessary to ensure a $3,000,000 profit the
first year would be (to the nearest thousand pounds):
A. 13,017,000 pounds
B. 14,000,000 pounds
C. 15,000,000 pounds
D. 25,600,000 pounds
E. none of the above

SUPPORTING CALCULATION:

[ 12 ($2,000,000 + $1,500,000 )] + $3,000,000


= 15,000,000 pounds
$5 . 90 - $ . 30 - $1 . 25 - $ . 45 - $ . 90
Direct Costing and Cost-Volume-Profit Analysis 302

C 36. A specialized version of direct costing for short-run optimization is :


A. learning theory
B. absorption costing
C. the theory of constraints
D. variable costing
E. none of the above

D 37. The theory of constraints uses which of the following basic measures :
A. throughput
B. operating expense
C. assets
D. all of the above
E. none of the above

B 38. The practice of improving a reported volume or idle capacity variance by producing more than is
currently needed is viewed by the theory of constraints as :
A. a benefit with no cost increase
B. a cost increase with no benefit
C. both a cost increase and a benefit
D. worthwhile from a cost/benefit perspective
E. none of the above

E 39. The theory of constraints is a short-run optimization technique that views which of the following as
relatively constant :
A. resources
B. technology
C. product lines
D. demand
E. all of the above

A 40. The theory of constraints is primarily useful for :


A. short-run decisions
B. medium range decisions
C. long-run decisions
D. both short-run and long-run decisions
E. medium range to long-run decisions
Direct Costing and Cost-Volume-Profit Analysis 303

PROBLEMS

PROBLEM

1.
Income Statement Using Absorption Costing and Direct Costing. Clouseau Corp. developed the following
standard unit costs:

Materials...................................................................................................................................................... $ 6.00
Labor........................................................................................................................................................... 4.25
Variable overhead....................................................................................................................................... 4.80
Fixed overhead............................................................................................................................................ 1.55
Variable marketing expenses...................................................................................................................... 1.50
Fixed administrative expenses.................................................................................................................... 4.50
Total...................................................................................................................................................... $ 22.60

The selling price is estimated at $30, and standard production is 9,000 units. Last year, production amounted to
9,000 units, of which 1,500 units were in inventory at the end of the year. This year, production amounted to
7,700 units; 7,000 units were sold at standard price. There are no work in process or materials inventories.

Required:

(1) Prepare an income statement for the current year, using (a) absorption costing and (b) direct costing.
(Round all computations to the nearest whole dollar and round $.50 up. Any over- or underapplied
factory overhead should be closed to Cost of Goods Sold.)
(2) Compute and reconcile the difference in operating income under the two methods.

SOLUTION

(1)(a) Absorption Costing

Sales (7,000 units @ $30)................................................................... $ 210,000


Cost of goods sold:
Beginning inventory (1,500 units x $16.60 1).............................. $ 24,900
Production costs:
Materials (7,700 units @ $6)................................................ $46,200
Direct labor (7,700 units @ $4.25)...................................... 32,725
Variable overhead (7,700 units @ $4.80)............................ 36,960
Fixed overhead (7,700 units @ $1.55)................................. 11,935 127,820
Cost of goods available for sale................................................... $ 152,720
Ending inventory (2,200 units x $16.60)..................................... 36,520
Cost of goods sold (7,000 units x $16.60)........................... $ 116,200
Volume variance (9,000 - 7,700 x $1.55)............................ 2,015
Cost of goods sold at actual.................................................. 118,215
Gross profit.......................................................................................... $ 91,785

Variable marketing expenses (7,000 units @ $1.50)......................... $ 10,500


Fixed administrative expenses (9,000 units @ $4.50)....................... 40,500
Total marketing and administrative expenses............................. 51,000
Operating income for the current year................................................ $ 40,785
Direct Costing and Cost-Volume-Profit Analysis 304

1
Beginning inventory:
Materials....................................................................................... $ 6.00
Labor............................................................................................ 4.25
Variable overhead........................................................................ 4.80
Fixed overhead............................................................................. 1.55
Total...................................................................................... $ 16.60

(b) Direct Costing

Sales (7,000 units @ $30)................................................................... $ 210,000


Variable cost of goods sold:
Beginning inventory (1,500 units @ $15.05 1)............................. $ 22,575
Variable production cost (7,700 units @ $15.05)....................... 115,885
Variable cost of goods available for sale............................. $ 138,460
Ending inventory (2,200 units @ $15.05)................................... 33,110
Variable cost of goods sold (7,000
units x $15.05)............................................................... 105,350
Gross contribution margin.................................................................. $ 104,650
Variable marketing expenses (7,000 units @ $1.50)......................... 10,500
Contribution margin............................................................................ $ 94,150
Less fixed expenses:
Overhead (9,000 units x $1.55)................................................... $ 13,950
Administrative (9,000 units x $4.50)........................................... 40,500 54,450
Operating income for the current year................................................ $ 39,700
1
Beginning inventory:
Materials....................................................................................... $ 6.00
Labor............................................................................................ 4.25
Variable overhead........................................................................ 4.80
Total...................................................................................... $ 15.05

(2)
Operating income under absorption costing............................................................................................. $ 40,785
Operating income under direct costing..................................................................................................... 39,700
Difference.................................................................................................................................................. $ 1,085

Units produced during year....................................................................................................................... 7,700


Units sold during year............................................................................................................................... 7,000
Increase in finished goods......................................................................................................................... 700
Fixed factory overhead per unit................................................................................................................ x $1.55
Difference.................................................................................................................................................. $ 1,085
Direct Costing and Cost-Volume-Profit Analysis 305

PROBLEM

2.
Distinguishing Between Costing Methods. The president of Symbiotic Systems Inc. asks the controller to
prepare a cost analysis, using both direct costing and absorption costing, as well as an assessment of the impact of
allocating a $25,000 unfavorable labor efficiency variance among inventories. The following income statements
were prepared:

D C B A
Sales ............................................................. $ 1,000,000 $ 1,000,000 $ 1,000,000 $ 1,000,000
Cost of goods sold:
Current cost............................................... $ 480,000 $ 455,000 $ 305,000 $ 330,000
Beginning work in process....................... 39,000 39,000 23,750 23,750
Ending work in process............................ (40,000) (56,667) (41,667) (25,000)
Beginning finished goods......................... 16,000 16,000 10,000 10,000
Ending finished goods.............................. (20,000) (28,333) (20,833) (12,500)
Cost of goods sold..................................... $ 475,000 $ 425,000 $ 276,250 $ 326,250
Gross profit..................................................... $ 525,000 $ 575,000 $ 723,750 $ 673,750
Other costs (not included above).................... 240,000 240,000 390,000 390,000
Net income...................................................... $ 285,000 $ 335,000 $ 333,750 $ 283,750

A few days later, the controller was arrested for embezzlement. The president now asks the assistant controller to:
(1) identify the method that was used to prepare each income statement, (2) compute the total current production
cost at standard under each costing method, and (3) compute the fixed production cost.

Required: Prepare the answers requested by the president.

SOLUTION

Note to instructor: This problem may be made more difficult by eliminating income statement B.

(1)
Income
Statement Costing Method Used
D............................................................................................................... Absorption (no allocation)
C............................................................................................................... Absorption (with allocation)
B............................................................................................................... Direct (with allocation)
A............................................................................................................... Direct (no allocation)

(2)
The total current production cost at standard would equal the current cost (no allocation) less the unfavorable
variance.

Absorption costing................................................................................................... $ 480,000 Current cost in D


- 25,000
$ 455,000

Direct costing........................................................................................................... $ 330,000 Current cost in A


- 25,000
$ 305,000
Direct Costing and Cost-Volume-Profit Analysis 306

(3)
The fixed production cost would be equal to the difference between the current cost under absorption costing and
the current cost under direct costing when both methods use the same allocation method.

$480,000 - $330,000 = $150,000 or

$455,000 - $305,000 = $150,000 or

Use the difference in below-the-line costs:

$390,000 - $240,000 = $150,000

PROBLEM

3.
Direct Costing Income Statements. Pro-Am Products presents the following data from absorption costing
income statements for the last two years:

19A 19B
Sales......................................................................................................................... $2,000,000 $2,500,000
Cost of goods sold (at standard).............................................................................. 800,000 950,000
Over- or underapplied overhead.............................................................................. 25,000 (25,000)
Marketing and general expense............................................................................... 500,000 550,000
Operating income..................................................................................................... 675,000 1,050,000

Required: Prepare the direct costing income statements for each year, assuming that there were no changes in
capacity between years and that the unit variable costs are constant. (Hint: Use the high- and low-points method
to determine the fixed and variable portions of each cost element.)

SOLUTION
19A 19B
Sales ...................................................................................................................... $ 2,000,000 $ 2,500,000
Variable cost of goods sold..................................................................................... $ 400,000 $ 500,000
Variable marketing and general expenses............................................................... 200,000 250,000
Gross contribution margin....................................................................................... $ 1,400,000 $ 1,750,000
Fixed expenses:
Manufacturing expenses.................................................................................... $ 425,000 $ 425,000
Marketing and general expenses....................................................................... 300,000 300,000
Total fixed expenses.................................................................................... $ 725,000 $ 725,000
Operating income..................................................................................................... $ 675,000 $ 1,025,000

Additional computations:
Actual overhead:
19A ($800,000 + $25,000)................................................................................. $ 825,000
19B ($950,000 - $25,000).................................................................................. 925,000
Difference..................................................................................................... $ 100,000
Direct Costing and Cost-Volume-Profit Analysis 307

$925,000 - $825,000
Variable production cost: = 20% of sales
$2,500,000 $2,000,000

Fixed production cost = Total cost - Variable cost


= $825,000 - ($2,000,000 x 20%)
= $425,000
or

$925,000 - ($2,500,000 x 20%) = $425,000

$550,000 $500,000
Variable marketing and general expenses: = 10% of sales
$2,500,000 $2,000,000

Fixed marketing and general expenses: $500,000 - ($2,000,000 x 10%) = $300,000

PROBLEM

4.
Absorption Costing Income Statement. Fong Products Co. manufactures restaurant equipment. The direct
costing income statement for last year is given below:

Sales......................................................................................................................................................... $ 370,000
Less:
Variable manufacturing cost............................................................................................................ 98,000
Variable marketing and general expenses....................................................................................... 64,000
Contribution margin................................................................................................................................. $ 208,000
Less:
Fixed manufacturing cost................................................................................................................. 50,000
Fixed marketing and general expenses............................................................................................ 70,000
Operating income..................................................................................................................................... $ 88,000

The variable and fixed costs in inventories for last year were:

Beginning Ending
Inventory Inventory
Work in process:
Variable cost........................................................................................................... $ 6,000 $ 9,000
Fixed cost................................................................................................................ 8,000 10,000
Total.................................................................................................................. $ 14,000 $ 19,000

Finished goods:
Variable cost........................................................................................................... $ 26,000 $ 20,000
Fixed cost................................................................................................................ 16,000 8,000
Total.................................................................................................................. $ 42,000 $ 28,000

There were no cost variances.

Required: Prepare an absorption costing income statement for last year, including inventory details.
Direct Costing and Cost-Volume-Profit Analysis 308

SOLUTION

Sales.............................................................................................................................. $ 370,000
Cost of goods sold:
Current manufacturing cost.................................................................................. $ 148,000
Add work in processbeginning inventory.......................................................... 14,000
$ 162,000
Less work in processending inventory...................................................................... 19,000
Cost of goods manufactured........................................................................................ $ 143,000
Add finished goodsbeginning inventory.................................................................... 42,000
$ 185,000
Less finished goodsending inventory........................................................................ 28,000
Cost of goods sold........................................................................................................ 157,000
Gross profit................................................................................................................... $ 213,000
Marketing and general expenses.................................................................................. 134,000
Operating income......................................................................................................... $ 79,000

PROBLEM

5.
Terminology on Break-Even Chart. A traditional break-even chart is illustrated in Figure 20-2.

Required: Identify each letter on the chart, using the proper terminology.
Direct Costing and Cost-Volume-Profit Analysis 309
Direct Costing and Cost-Volume-Profit Analysis 310

SOLUTION

Lettered Item in
Break -Even Chart Terminology
A Fixed cost area
B Variable cost area
C Profit area
D Break-even point
E Loss area
F Total cost line
G Sales line
H Fixed cost line
I y-axis
J x-axis

PROBLEM

6.
Contribution Margin; Break-Even Sales in Dollars. The management of Ivory Coast Products Co. is presented
with the following data:

Sales............................................................................................................................... $ 500,000
Direct materials............................................................................................................. $ 60,000
Direct labor.................................................................................................................... 90,000
Factory overhead........................................................................................................... 100,000 250,000
Gross profit.................................................................................................................... $ 250,000
Marketing expenses....................................................................................................... $ 70,000
General expenses........................................................................................................... 100,000 170,000
Net income..................................................................................................................... $ 80,000

Fifty percent of factory overhead is fixed, while 40% of marketing expenses and all general expenses are fixed.

Required:

(1) Compute the contribution margin ratio.


(2) Compute the break-even point in sales dollars.
(3) New factory equipment may be purchased that will not affect total costs at this sales level but will
increase fixed factory overhead costs to 75% of factory overhead. Assuming that this purchase is made,
show its effect by recomputing the answer to (1).
(4) Assuming that the new factory equipment is purchased, show its effect by recomputing the answer to (2).
(Round all percentages to the nearest tenth of a percent and all dollar amounts to the nearest whole dollar.)
Direct Costing and Cost-Volume-Profit Analysis 311

SOLUTION

(1 )
Sales Variable costs $500,000 $60,000 $90,000 $50,000 $42,000
=
Sales $500,000

$258,000
= = 51.6%
$500,000

(2)
Fixed costs $50,000 + $28,000 + $100,000 $178,000
= = = $344,961
C/M ratio . 516 .516

(3 )
Sales Variable costs $500,000 $60,000 $90,000 $25,000 $42,000
=
Sales $500,000

$283,000
= = 56.6%
$500,000

(4 )
Fixed costs $75,000 + $28,000 + $100,000 $203,000
= = = $358,657
C/M ratio . 566 . 566

PROBLEM

7.
Expected Profits; Break-Even Point in Units; Margin of Safety; Effect of an Increase in Sales. Panko's
Pickles Inc. estimates sales of 500,000 units at $5 per unit. Variable costs generally equal $1 per unit. Fixed
expenses for this planned sales level would equal $2 per unit.
Direct Costing and Cost-Volume-Profit Analysis 312

Required: Compute the following (round all answers to the nearest whole number):

(1) Estimated profit for the planned level of sales


(2) Break-even point in units and dollars
(3) Margin of safety ratio (M/S)
(4) Increase in profit that would result from a 10% increase in sales
(5) Profit as a percentage of the planned level of sales
Direct Costing and Cost-Volume-Profit Analysis 313

SOLUTION

(1)
500,000 units x Unit profit = 500,000 x ($5 - $2 - $1) = $1,000,000 Estimated profit

Total fixed expenses 500,000 units _ $2 $1,000,000


= =
Contribution margin per unit $5 $1 $4

= 250,000 Breakeven point in units

250,000 _ $5 = $1,250,000 Break-even point in dollars


(2)

Planned sales Breakeven sales $2,500,000 $1,250,000


=
Planned sales $2,500,000

= 50% Margin of safety (M/S) ratio


(3)
(4)
Contribution margin per unit x Unit increase = $4 x (500,000 x 10%) = $200,000

(5)
Profit = C/M ratio x M/S ratio = 80% x 50% = 40%

PROBLEM

8.
Break-Even Point in Dollars; Direct Costing Statement; Net Income as a Percentage of Last Year's Net
Income. Mordeci Manufacturing Co. shows the following comparative income statement data for the last two
years:

19A 19B
Sales (in units)......................................................................................................... 15,000 20,000
Sales......................................................................................................................... $ 300,000 $ 400,000
Cost of goods sold:
Materials........................................................................................................... $ 150,000 $ 200,000
Labor................................................................................................................. 75,000 100,000
Overhead........................................................................................................... 30,000 35,000
Total........................................................................................................... $ 255,000 $ 335,000
Gross profit.............................................................................................................. $ 45,000 $ 65,000
Other expenses......................................................................................................... 30,000 40,000
Net income............................................................................................................... $ 15,000 $ 25,000

Required:
Direct Costing and Cost-Volume-Profit Analysis 314

(1) Compute the 19B net income as a percentage of 19A net income.
(2) Prepare a direct costing income statement for 19A and 19B. (Hint: Use the high- and low-points method
to determine the fixed and variable portions of each cost element.)
(3) Compute the break-even point in dollars as determined from the above data.
(Round all answers to the nearest whole number.)
Direct Costing and Cost-Volume-Profit Analysis 315

SOLUTION

$25,000
= 167%
$15,000
(1)
(2)
19A 19B
Sales......................................................................................................................... $ 300,000 $ 400,000
Less variable expenses:
Materials........................................................................................................... $ 150,000 $ 200,000
Labor................................................................................................................. 75,000 100,000
Overhead (5% of sales)1................................................................................... 15,000 20,000
Other variable (10% of sales) 2......................................................................... 30,000 40,000
Total........................................................................................................... $ 270,000 $ 360,000
Contribution margin................................................................................................ $ 30,000 $ 40,000
Less fixed expenses:
Overhead 3.......................................................................................................... 15,000 15,000
Net income............................................................................................................... $ 15,000 $ 25,000

Additional computations:

1 Change in overhead
Variable overhead =
Change in sales

$35,000 $30,000 $5,000


= = = 5%
$400,000 $300,000 $100,000

2 Change in other expenses


Other variable expenses =
Change in sales

$40,000 $30,000 $10,000


= = = 10%
$400,000 $300,000 $100,000
3
$30,000 - $15,000 or $35,000 - $20,000

(3)
Direct Costing and Cost-Volume-Profit Analysis 316

Fixed expenses $15,000


=
Contribution margin (C/M) ratio $40,000 ÷ $400,000

$15,000
or
$30,000 ÷ $300,000

$15,000
= = $150,000 Breakeven point
. 10
Direct Costing and Cost-Volume-Profit Analysis 317

PROBLEM

9.
Break-Even Point in Units and Dollars. Professional Products Inc. manufactures two productsType A and
Type B. Relevant budgeted sales and cost data for the coming year are:

Variable Expenses
Product Unit Sales Unit Price per Unit
Type A.................................................................... 100,000 $15 $6
Type B.................................................................... 150,000 10 7

The fixed costs for the company amounted to $1,000,000.

Required: Compute the break-even point in units and in dollars for Type A and Type B.

SOLUTION

Type B 150,000
= 1 .5 or 3:2
Type A 100,000

Contribution margin per hypothetical package = [2 x ($15 - $6)] + [3 x ($10 - $7)]


= $18 + $9
= $27

Total fixed costs $1,000,000


=
Contribution margin per hypothetical package $27
¿

¿ = 37,037 breakeven point in


hypothetical package

Product Break -Even Point


Type A..................................................... 37,037 x 2 = 74,074 units; 74,074 @ $15 = $1,111,110
Type B..................................................... 37,037 x 3 = 111,111 units; 111,111 @ $10 = $1,111,110
Direct Costing and Cost-Volume-Profit Analysis 313

Chapter 21

DIFFERENTIAL COST ANALYSIS

MULTIPLE CHOICE

Question Nos. 9, 11-13, 15-20, and 29 are AICPA adapted.


Question Nos. 21, 22, 28, and 32-34 are ICMA adapted.
Question Nos. 10, 14, 30, and 31 are CIA adapted.

B 1. Additional output that results in a positive difference between differential revenues and differential
costs is beneficial to a company if and only if:
A. other sales are affected
B. other sales are unaffected and other unit costs are unaffected
C. other unit costs are increased and idle capacity is decreased
D. other sales are unaffected but other unit costs are increased
E. there is no idle capacity

C 2. The effect of discontinuing a department with a contribution to overhead of $30,000 and allocated
overhead of $48,000, of which $26,000 cannot be eliminated, would be to:
A. increase profit by $8,000
B. decrease profit by $26,000
C. decrease profit by $ 8,000
D. decrease profit by $22,000
E. increase profit by $ 4,000

SUPPORTING CALCULATION:

($48,000 - $26,000) - $30,000 = ($8,000)

D 3. Gizmo Manufacturing is considering dropping a product line. It currently produces a multipurpose


woodworking clamp in a simple manufacturing process that uses special equipment. Variable costs
amount to $6.00 per unit. Fixed factory overhead costs, exclusive of depreciation, have been
allocated to this product at a rate of $3.50 a unit and will continue whether or not production ceases.
Depreciation on the special equipment amounts to $20,000 a year. If production of the clamp is
stopped, the special equipment can be sold for $18,000; if production continues, however, the
equipment will be useless for further production at the end of one year and will have no salvage
value. The clamp has a unit sales price of $10. Ignoring income tax effects, the minimum number
of units that would have to be sold in the current year to make it worthwhile to keep the equipment
(on a cash-flow basis) is:
A. 20,000
B. 5,000
C. 3,000
D. 4,500
E. 36,000

313
Direct Costing and Cost-Volume-Profit Analysis 314

SUPPORTING CALCULATION:
x ($10 - $6) = $18,000
$4x = $18,000
x = 4,500

B 4. The costing method used to determine the lowest price that could be quoted for a special order that
would use idle capacity within a production area is:
A. process
B. direct
C. standard
D. absorption
E. job order

E 5. In deciding whether to manufacture a part or buy it from an outside vendor, a cost that is irrelevant
to the short-run decision is:
A. indirect materials
B. direct labor
C. variable factory overhead
D. fixed factory overhead that will be avoided if the part is bought from an outside vendor
E. fixed factory overhead that will continue even if the part is bought from an outside vendor

C 6. Faced with a long-run make-or-buy decision, the manager should do all of the following except:
A. compare the making of the parts with alternative uses that could be made of the firm's own
facilities if the parts are purchased
B. compare the cost of making the parts with the cost of buying them
C. use a cost study with only the differential costs and with no allocation of existing fixed
overhead or profit
D. consider differences in the required capital investment and the timing of cash flows
E. consider the quantity and quality of the parts as well as the technical know-how required

E 7. An opportunity cost is:


A. a cost that may be saved by not adopting an alternative
B. a cost that may be shifted to the future with little or no effect on current operations
C. a cost that cannot be avoided because it has already been incurred
D. the difference in total costs that results from selecting one alternative instead of another
E. the profit foregone by selecting one alternative instead of another

E 8. The term "differential cost" refers to:


A. the profit foregone by selecting one alternative instead of another
B. a cost that does not entail any dollar outlay but that is relevant to the decision-making process
C. a cost that continues to be incurred even though there is no activity
D. a cost common to all alternatives in question and not clearly or practically allocable to any of
the alternatives
E. the difference in total costs that results from selecting one alternative instead of another

314
Differential Cost Analysis 315

A 9. In a make-or-buy decision:
A. fixed costs that can be avoided in the future are relevant
B. only variable costs are relevant
C. only prime costs are relevant
D. fixed costs that will continue regardless of the decision are relevant
E. only conversion costs are relevant

D 10. For the past 12 years, the Jolt Company has produced the small electric motors that fit into its main
product line of dental drilling equipment. As materials costs have steadily increased, the controller
of the Jolt Company is reviewing the decision to continue to make the small motors and has
identified the following facts:
1. The equipment used to manufacture the electric motors has a book value of $150,000.
2. The space now occupied by the Electric Motor Manufacturing Department could be used to
eliminate the need for storage space now being rented.
3. Comparable units can be purchased from an outside supplier for $59.75.
4. Four of the people who work in the Electric Motor Manufacturing Department would be
terminated and given eight weeks of severance pay.
5. A $10,000 unsecured note is still outstanding on the equipment used in the manufacturing
process.

Which of the items above are relevant to the decision that the controller has to make?
A. 1, 2, 4, and 5
B. 1, 3, 4, and 5
C. 1, 3, and 4
D. 2, 3, and 4
E. 2, 3, 4, and 5
Differential Cost Analysis 316

D 11. Ely Electronics has the following standard costs and other data:

Part A4 Part B5
Direct materials............................................................................................ $ .40 $ 8.00
Direct labor................................................................................................... 1.00 4.70
Factory overhead.......................................................................................... 4.00 2.00
Unit standard cost......................................................................................... $ 5.40 $ 14.70

Units needed per year................................................................................... 6,000 8,000


Machine hours per unit................................................................................. 4 2
Unit cost if purchased................................................................................... $ 5.00 $ 15.00

In past years, Ely has manufactured all of its required components; however, this year only 30,000
hours of otherwise idle machine time can be devoted to the production of components. Accordingly,
some of the parts must be purchased from outside suppliers. In producing parts, factory overhead is
applied at $1.00 per standard machine hour. Fixed capacity costs that will not be affected by any
make-or-buy decision represent 60% of the applied overhead.

The 30,000 hours of available machine time are to be scheduled so that Ely realizes maximum
potential cost savings. The relevant unit production costs that should be considered in the decision
to schedule machine time are:
A. $5.40 for A4 and $14.70 for B5
B. $5.00 for A4 and $15.00 for B5
C. $1.40 for A4 and $12.70 for B5
D. $3.00 for A4 and $13.50 for B5
E. none of the above

SUPPORTING CALCULATION:

A4 = $.40 + $1.00 + .40($4.00) = $3.00


B5 = $8.00 + $4.70 + .40($2.00) = $13.50

E 12. Production of a special order will increase the contribution margin when the additional revenue from
the special order is greater than:
A. the nonvariable costs incurred in producing the order
B. the direct materials and labor costs in producing the order
C. the fixed costs incurred in producing the order
D. the indirect costs of producing the order
E. the marginal cost in producing the order

C 13. In considering a special order that will enable a company to make use of present idle capacity, which
of the following costs would be irrelevant?
A. fixed factory overhead that can be avoided
B. materials
C. depreciation of the factory building
D. direct labor
E. variable overhead
Differential Cost Analysis 317

E 14. In deciding whether to manufacture a part or buy it from an outside vendor, a cost that is relevant to
the short-run decision is:
A. direct labor
B. variable overhead
C. fixed overhead that will be avoided if the part is bought from an outside vendor
D. direct materials
E. all of the above

D 15. A company owns equipment that is used to manufacture important parts for its production process.
The company plans to sell the equipment for $10,000 and to select one of the following two
alternatives: (1) acquire new equipment for $80,000, or (2) purchase the important parts from an
outside company at $4 per part. To select the best alternative, the company should compare the cost
of manufacturing the parts:
A. plus $80,000 to the cost of buying the parts less $10,000
B. to the cost of buying the parts less $10,000
C. less $10,000 to the cost of buying the parts
D. plus $80,000 to the cost of buying the parts
E. none of the above

C 16. The following standard costs pertain to a component part manufactured by Rob Co.:

Direct materials............................................................................................................................ $ 4
Direct labor................................................................................................................................... 10
Factory overhead.......................................................................................................................... 40
Standard cost per unit................................................................................................................... $ 54

Factory overhead is applied at $1 per standard machine hour. Fixed capacity cost is 60% of applied
factory overhead and is not affected by any make-or-buy decision. It would cost $49 per unit to buy
the part from an outside supplier. In the decision to make or buy, what is the total relevant unit
manufacturing cost?
A. $54
B. $38
C. $30
D. $5
E. none of the above

SUPPORTING CALCULATION:

$4 + $10 + .40($40) = $30


Differential Cost Analysis 318

E 17. The Reno Company manufactures Part No. 498 for use in its production cycle. The cost per unit for
20,000 units of Part No. 498 are as follows:

Direct materials............................................................................................................................ $ 6
Direct labor................................................................................................................................... 30
Variable overhead......................................................................................................................... 12
Fixed overhead applied................................................................................................................ 16
........................................................................................................................................... $ 64

The Tray Company has offered to sell 20,000 units of Part No. 498 to Reno for $60 per unit. Reno
will make the decision to buy the part from Tray if there is a savings of $25,000 for Reno. If Reno
accepts Tray's offer, $9 per unit of the fixed overhead applied would be totally eliminated.
Furthermore, Reno has determined that the released facilities could be used to save relevant costs in
the manufacture of Part No. 575. In order to have a savings of $25,000, the amount of relevant costs
that would be saved by using the released facilities in the manufacture of Part No. 575 would have to
be:
A. $80,000
B. $60,000
C. $125,000
D. $140,000
E. $85,000

SUPPORTING CALCULATION:

$60 - ($6 + $30 + $12 + $9) = $3


$3(20,000) + 25,000 = $85,000

C 18. At December 31, Zar Co. had a machine with an original cost of $84,000, accumulated depreciation
of $60,000, and an estimated salvage value of zero. On December 31, Zar was considering the
purchase of a new machine having a five-year life, costing $120,000, and having an estimated
salvage value of $20,000 at the end of the five years. In its decision concerning the possible
purchase of the new machine, how much should Zar consider to be a sunk cost at December 31?
A. $120,000
B. $100,000
C. $24,000
D. $4,000
E. none of the above

SUPPORTING CALCULATION:

$84,000 - $60,000 = $24,000


Differential Cost Analysis 319

B 19. Stewart Industries has been producing two bearings, components B12 and B18, for use in
production. Data regarding these two components are:

.................................................................................................... B12 B18


Machine hours required per unit................................................................ 2.5 3.0

Standard cost per unit


Direct material.................................................................................... $ 2.25 $ 3.75
Direct labor......................................................................................... 4.00 4.50
Manufacturing overhead
Variable 1..................................................................................... 2.00 2.25
Fixed2.......................................................................................... 3.75 4.50
.................................................................................................... $ 12.00 $ 15.00
1
Variable manufacturing overhead is applied on the basis of direct labor hours.
2
Fixed manufacturing overhead is applied on the basis of machine hours.

Stewart's annual requirement for these components is 8,000 units of B12 and 11,000 units of B18.
Recently, Stewart's management decided to devote additional machine time to other product lines
resulting in only 41,000 machine hours per year that can be dedicated to the production of the
bearings. An outside company has offered to sell Stewart the annual supply of the bearings at prices
of $11.25 for B12 and $13.50 for B18. Stewart wants to schedule the otherwise idle 41,000 machine
hours to produce bearings so that the company can minimize its costs (maximize its net benefits).

The net benefit (loss) per machine hour that would result if Stewart Industries accepts the supplier's
offer of $13.50 per unit for component B18 is:
A. $.50
B. $(1.00)
C. $1.50
D. $(1.75)
E. some amount other than those given above

SUPPORTING CALCULATION:

($3.75 + $4.50 + $2.25) - $13.50 = ($3.00)  3 = ($1.00)

The following questions are based on the material in the Appendix to the chapter.

D 20. If plant capacity for cutting time and shaping time is 80 hours and 100 hours, respectively, and it
takes four hours to cut and two hours to shape a standard model and two hours to cut and five hours
to shape a deluxe model, the maximum number of standard and deluxe models that can be produced
are:
A. 50 standard and 40 deluxe
B. 20 standard and 40 deluxe
C. 40 standard and 20 deluxe
D. 20 standard and 20 deluxe
E. 20 standard and 50 deluxe
Differential Cost Analysis 320

SUPPORTING CALCULATION:

Cutting = 4x + 2y  80
Shaping = 2x + 5y  100
Standard = 80  4 = 20
Deluxe = 100  5 = 20

A 21. Plant capacity for cutting time and shaping time is 80 hours and 100 hours, respectively, and it takes
four hours to cut and two hours to shape a standard model and two hours to cut and five hours to
shape a deluxe model. If the standard models were represented on the horizontal axis and the deluxe
models were represented on the vertical axis of a graph, the shaping constant would be expressed as:
A. 2x + 5y  100
B. 2x + 5y  100
C. 4x + 2y  100
D. 4x + 2y  100
E. 2x + 2y  100

D 22. If there is one unique optimal solution to a linear programming problem, that solution would be
found at:
A. the highest point on the y-axis
B. the objective function
C. the origin
D. a corner point
E. the highest point on the x-axis

D 23. An iterative, stepwise procedure that is used to solve linear programming problems is the:
A. graphical technique
B. identity matrix
C. cost minimization problem
D. simplex method
E. matrix algebra approach

A 24. Linear programming is an operations research technique that allocates resources. Mathematical
expressions are used to describe the problem. The measure of effectiveness that is to be maximized
or minimized is called the:
A. objective function
B. derivative of the function
C. nonlinear function
D. constraints
E. decision variables

A 25. The term "constraints" in a linear programming model generally describes:


A. scarce resources
B. dependent variables
C. inefficiencies
D. the objective function
E. costs
Differential Cost Analysis 321

B 26. A 200-bed hospital serves 500 meals per day. An analytic tool that would help management plan
meals to meet nutrition goals at minimum cost is:
A. Monte Carlo simulation
B. linear programming
C. material requirements planning
D. Markov analysis
E. exponential smoothing

E 27. Linear programming is a mathematical technique designed to help an organization allocate its
resources. A linear programming problem takes the form presented below:

f = A1X1 + A2X2 + ... + AnXn subject to B1X1 + B2X2 + ... + BnXn  C1

What is the name for the following function?

f = A1X1 + A2X2 + ... + AnXn


A. revenue function
B. cost function
C. constraint function
D. linear function
E. objective function

B 28. Pleasant Valley Company makes two ceramic products, vases (V) and bowls (B). Each vase requires
two pounds of material and three hours of labor. Each bowl requires two pounds of material and one
hour of labor. During the next production week, there will be 100 pounds of material and 60 hours
of labor available to make vases and bowls. Each pound of material costs $4 and each hour of labor
costs $10. All factory overhead is fixed; it is estimated to be $200 for this production process for a
week. Pleasant Valley sells vases for $50 each and bowls for $35 each. The objective function for
Pleasant Valley would be:
A. maximize Z = $50V + $35B
B. maximize Z = $12V + $17B
C. minimize Z = $38V + $18B
D. maximize Z = $12V + $17B = $200
E. some function other than those given above

SUPPORTING CALCULATION:

V = $50 - (2 x $4) - (3 x $10) = $12


B = $35 - (2 x $4) - (1 x $10) = $17
Differential Cost Analysis 322

PROBLEMS

PROBLEM

1.
Effect of Special Order on Profits. Markham Modems, Inc. recently received a special order to manufacture
10,000 units for a Brazilian company. This order specified that the selling price per unit should not exceed $50.
Because the order was received without the effort of the Sales Department, no commission would be paid.
However, an export-handling charge of $2 per unit would be incurred. Management anticipates that acceptance of
the order will have no effect on other sales.

The company is operating at 80% of capacity, or 80,000 units, and expects to continue at this level for the coming
year, without the Brazilian order. Unit selling price and costs, based on estimated actual capacity for the coming
year, are:

Selling price............................................................................................................................................... $65.00

Expenses
Direct materials................................................................................................................................... $15.00
Direct labor.......................................................................................................................................... 20.00
Variable factory overhead................................................................................................................... 7.50
Fixed factory overhead........................................................................................................................ 3.00
Sales commissions............................................................................................................................... 5.00
Other marketing expenses (75% variable).......................................................................................... 2.00
General expenses (25% fixed)............................................................................................................ 4.00
Total............................................................................................................................................ $56.50

Required: Prepare an analysis showing the effect on profits if the company accepts the special order.

SOLUTION

Markham Modems, Inc.


Effect of Special Order on Profits
Per Unit
Differential costs....................................................................................................................................
Direct materials................................................................................................................................... $ 15.00
Direct labor.......................................................................................................................................... 20.00
Variable factory overhead................................................................................................................... 7.50
Other marketing expenses................................................................................................................... 1.50
Export-handling charge....................................................................................................................... 2.00
General expenses (75% variable)........................................................................................................ 3.00
Total............................................................................................................................................ $ 49.00
Differential selling price............................................................................................................................ 50.00
Profit per unit............................................................................................................................................. $ 1.00

Profit per unit x Units sold = $1 x 10,000 = $10,000 increase in profit


Differential Cost Analysis 323

PROBLEM

2.
Effect of New Order on Profits. Island Cyclery, Inc. manufactures trail bikes. Management is considering the
expansion of its sales to the tourist market. All sales for this market would be made to a rental agency in St. John,
and the rental agent would receive a discount. An investigation of the trail-bike market reveals the following
data:

Trail bikes sold year-to-date...................................................................................................................... 1,000


Production capacity of trail bikes.............................................................................................................. 2,000
Costs for this year (at standard):
Materials.............................................................................................................................................. $200,000
Direct labor.......................................................................................................................................... 275,000
Factory overhead
Variable....................................................................................................................................... 100,000
Fixed............................................................................................................................................ 175,000
Sales commissions............................................................................................................................... 45,000
Packing and shipping.......................................................................................................................... 25,000
Advertising.......................................................................................................................................... 85,000
Administrative expenses..................................................................................................................... 190,000

A trail bike regularly wholesales for $800, but the St. John transaction calls for a selling price of $700. There
would be no sales commission. Because the St. John rental agency will pay cash, the allowance for doubtful
accounts (which usually amounts to 2% of sales and is included in administrative expenses) will not be required.
However, packing and shipping costs would increase by $25 per unit to compensate for transpacific freight. All
other selling and administrative expenses are fixed. The company estimates that 500 trail bikes will satisfy the St.
John tourist market.

Required: Prepare an analysis showing the effect on profits if the new order is accepted by the company. (Show
both per-unit selling price and costs and total sales and costs for the new order.)

SOLUTION

Per Unit Total


Sales............................................................................................................................... $ 700 $ 350,000
Differential costs
Materials ($200,000  1,000 units)........................................................................ $ 200 $ 100,000
Direct labor ($275,000  1,000 units).................................................................... 275 137,500
Variable factory overhead ($100,000  1,000 units)............................................. 100 50,000
Packing and shipping [($25,000  1,000) + $25].................................................. 50 25,000
Total.................................................................................................................. $ 625 $ 312,500
Increase in profit............................................................................................................ $ 75 $ 37,500
Differential Cost Analysis 324

PROBLEM

3.
Decision to Add New Product Line. The management of Banter Inc. is considering the entry of its new Jokes
product line in the market. Because its existing product line, Riddles, has similar characteristics to the new
product line, management expects the Jokes sales to require a minimum of additional expense. It is also
anticipated that Riddles sales will increase if both product lines are offered in a package deal. The Jokes product
line would be manufactured in a company-owned facility that is now being rented to another firm for $600,000
per year. Depreciation on this facility and all other building expenses are presently $100,000 per year. In
addition, the company will need to rent equipment to manufacture the new product line at an additional cost of
$150,000 per year. The contribution margin for the Jokes product line would be $50 per unit, and annual sales are
estimated at 10,000 units. Last year, sales for the Riddles product line amounted to 40,000 units. Other relevant
per unit sales and cost data were:

Selling price.................................................................................................................................................... $100


Variable cost of goods sold............................................................................................................................. 25
Fixed cost of goods sold.................................................................................................................................. 20
Variable marketing and administrative expenses........................................................................................... 10
Fixed marketing and administrative expenses............................................................................................... 15

If the new Jokes product line is undertaken, the company expects a 10% increase in Riddles sales. Otherwise,
Riddles sales will remain unchanged. Additional facilities will not be needed to manufacture the additional
Riddles units.

Required: Prepare an analysis showing the effect on profit if this new product line is accepted by the company.

SOLUTION

Contribution from:
Jokes contribution margin (10,000 units @ $50)............................................ $ 500,000
Riddles sales (40,000 units x 10%):
Sales (4,000 units @ $100)..................................................................... $ 400,000
Variable cost of goods sold (4,000 units @ $25).................................... (100,000 )
Variable marketing and administrative expenses
(4,000 units @ $10)............................................................................ (40,000 ) 260,000
Total......................................................................................................... $ 760,000
Less:
Rental foregone on company-owned facility.................................................. $ 600,000
Depreciation of new equipment....................................................................... 150,000 750,000
Increase in profits................................................................................................... $ 10,000
Differential Cost Analysis 325

PROBLEM

4.
Decision to Eliminate a Division. Major League Company has two operating divisions American and National.
The January income statements for each division and the company as a whole are:

American National
Division Division Total
Sales................................................................................................. $ 112,500 $ 60,000 $ 172,500
Cost of goods sold:
Prime cost................................................................................. $ 20,000 $ 15,000 $ 35,000
Variable factory overhead........................................................ 15,000 12,000 27,000
Fixed factory overhead............................................................. 22,500 18,000 40,500
Total.................................................................................. $ 57,500 $ 45,000 $ 102,500
Gross profit...................................................................................... $ 55,000 $ 15,000 $ 70,000
Other expenses:
Sales commissions.................................................................... $ 10,000 $ 5,000 $ 15,000
Packing and shipping................................................................ 9,000 7,000 16,000
Advertising................................................................................ 12,000 8,000 20,000
Administrative.......................................................................... 16,000 8,000 24,000
Total.................................................................................. $ 47,000 $ 28,000 $ 75,000
Operating income (loss).................................................................. $ 8,000 $ (13,000 ) $ (5,000 )

Company creditors recently informed management that the company must attain more profitable operations before
further credit will be extended. One possible move that would aid this situation would be to sell the National
Division. One prospective buyer would buy this division for $200,000; the money from the sale could be invested
at 8% interest.

One effect of the sale of the National Division would be that all of its variable manufacturing costs could be
eliminated; however, none of the fixed factory overhead expenses would be avoided. The sales commissions and
packing and shipping expenses are completely variable. The advertising expenses for the company as a whole
would be $15,000 after the elimination of the National Division. Finally, half of the administrative expenses
charged to the National Division would be eliminated if the division were sold.

Required:

(1) Prepare a revised income statement for the company as a whole for the month of January if the National
Division is eliminated to improve the credit rating.
(2) Should the National Division be eliminated to improve profits?
Differential Cost Analysis 326

SOLUTION

(1)
Major League Company
Income Statement
For Month Ended January 31, 19--

Sales........................................................................................................................ $ 112,500
Cost of goods sold:
Prime cost......................................................................................................... $20,000
Variable factory overhead................................................................................ 15,000
Fixed factory overhead.................................................................................... 40,500
Total......................................................................................................... 75,500
Gross profit............................................................................................................. $ 37,000
Other expenses:
Sales commissions........................................................................................... $10,000
Packing and shipping....................................................................................... 9,000
Advertising....................................................................................................... 15,000
Administrative.................................................................................................. 20,000
Total......................................................................................................... $ 54,000
Operating loss......................................................................................................... $ (17,000 )
Nonoperating income:
Interest income................................................................................................. 1,333 1,333*
Loss before taxes.................................................................................................... $ (15,667 )
*(200,000 x .08)  12

(2)
No, the National Division should not be eliminated. The company will incur $10,667 less loss if the division is
continued because the fixed overhead cannot be eliminated.
Differential Cost Analysis 327

PROBLEM

5.
Make-or-Buy Decision. TGIF Inc. manufactures party games. Most games are played on boards that are
purchased from an outside supplier at the cost of $1 each. The company uses 50,000 boards a year. Management
requests that an analysis be made to determine the profitability of producing the boards internally.

The materials required to manufacture each board cost $.15 per board. To print the game pattern and to glue the
pattern to the board includes a direct labor cost of $.20 per board. The company would also have to lease a board
press costing $20,000 for a four-year lease. Presently, there is adequate space in the Producing Department for the
manufacture of 20,000 boards per year.

If the company were to produce all of its boards internally, it would be necessary to cease its manufacture of
checkers and to purchase these pieces from the outside, resulting in an additional $25,000 cost. Also, a checker
caster costing $8,000 with a $4,000 book value would have to be scrapped without a salvage value.

Required: Prepare a recommendation to management to aid in the make-or-buy decision for the game boards; use
an analysis of the differential costs required for the manufacture of 20,000 and then 50,000 boards vs. the cost of
purchasing each quantity from an outside supplier.

SOLUTION

20,000 50,000
Boards Boards
Cost to purchase from outside................................................................................ $ 20,000 $ 50,000
Manufacturing costs:
Direct materials ($.15 per board)..................................................................... $ 3,000 $ 7,500
Direct labor ($.20 per board)........................................................................... 4,000 10,000
Leasenew board press ($20,000/4 yrs.)......................................................... 5,000 5,000
Cost to purchase checkers from outside.......................................................... -- 25,000
Total......................................................................................................... $ 12,000 $ 47,500
Differential profit (loss) from manufacturing........................................................ $ 8,000 $ 2,500

The recommendation to management would be to make 20,000 game boards and purchase 30,000 from the outside
because it results in more differential profit than producing 50,000 boards. The cost of the checker caster is
irrelevant to the make-or-buy decision because it is a sunk cost.
Differential Cost Analysis 328

PROBLEM

6.
Sell or Process Further. From a particular joint process, Gusher Company produces three productsKerosene,
Gasoline, and Fuel Oil. Each product may be sold at the point of split-off or processed further. Additional
processing requires no special facilities, and the production costs of further processing are entirely variable and
traceable to the products involved. In 19--, these products were processed beyond split-off. Joint production
costs for the year were $60,000. Sales values and costs needed to evaluate Gusher's 19-- production policy follow:

Additional Costs
and Sales Values
if Processed Further
Gallons Sales Values Sales Added
Product Produced at Split-Off Values Costs
Kerosene........................................................ 6,000 $25,000 $42,000 $12,000
Gasoline......................................................... 4,000 41,000 45,000 6,000
Fuel Oil.......................................................... 2,000 24,000 32,000 8,000

Joint costs are allocated to the products based on the percentage of the individual product's sales value to the total
sales value of all products.

Required:

(1) For gallons of Kerosene, compute the unit production cost most relevant to a "sell" or "process further"
decision.
(2) Determine which products the company should subject to additional processing in order to maximize
profits.

SOLUTION

(1) Added costs/Gallons produced = $12,000/6,000 = $2.00. Two dollars is the most relevant Kerosene unit
production cost for a "sell" or "process further" decision.
(2) Gusher Company should subject only Kerosene to additional processing in order to maximize profits
because the added sales value exceeds the added costs. For Gasoline, the added costs exceed the added
sales value; for Fuel Oil, the added sales value equals the added cost, so other factors would be dominant.

Kerosene Gasoline Fuel Oil


Sales value if processed further........................................................ $ 42,000 $ 45,000 $ 32,000
Sales value at split-off...................................................................... 25,000 41,000 24,000
Added sales value............................................................................. $ 17,000 $ 4,000 $ 8,000
Added costs....................................................................................... $ 12,000 $ 6,000 $ 8,000
Differential Cost Analysis 329

The following problems are based on material appearing in the Appendix to the chapter.

PROBLEM

7.
Problem Formulation. Barby-Q Inc. manufactures gas grills and charcoal grills. Each gas grill sells for $160,
and each charcoal grill sells for $60. The variable costs of a gas grill are $100 and the fixed costs allocated to the
normal production level equal $25. Variable costs of a charcoal grill are $30, and the fixed costs allocated to
charcoal grills are $15 per unit. There is a limitation of 10,000 pounds of steel used to manufacture both products.
The charcoal grills require 1 pound of steel per unit; each gas grill requires 2 pounds. The demand for gas grills is
limited to 5,000 units. Both products must be completed in the Finishing Department, where the equipment is
limited to 500 hours of use per period. Ten charcoal grills can be finished in one hour, but only three gas grills
can be finished per hour.

Required: Formulate the objective function and the constraints. Use x to represent gas grills and y to represent
charcoal grills. (Do not solve this problem.)

SOLUTION

Maximize: $60x + $30y


Constraints: 2x + y  10,000 (steel constraint)
x  5,000 (demand constraint)
.33x + .1y  500 (finishing constraint)

PROBLEM

8.
Profit Maximization: Graphic Method. Consider the following linear programming problem:

Maximize: $2x + $3y


Constraints: x + y < 5,000
x + 3y < 8,000

Required: Solve this problem using the graphic method.


Differential Cost Analysis 330

SOLUTION

Letter Total
on Graph Corner Point Profit Profit
A 0x ; 2,667 (0 x $2) + (2,667 x $3) $ 8,000
B 3,500x ; 1,500 y (3,500 x $2) + (1,500 x $3) 11,500 1
C 5,000x ; 0y (5,000 x $2) + (0 x $3) 10,000
1
Optimal solution
Solution of the simultaneous equations: x + y = 5,000
x + 3y = 8,000

Rearranging second equation and solving for x: x = 8,000 - 3y

Substituting into the first equation: (8,000 - 3y) + y = 5,000


-2y = -3,000
y = 1,500
Differential Cost Analysis 331

Substituting into the second equation: x + 3(1,500) = 8,000


x = 3,500
Differential Cost Analysis 331

Chapter 22

PLANNING FOR CAPITAL EXPENDITURES

MULTIPLE CHOICE

Question No. 4 is AICPA adapted.


Question No. 3 is ICMA adapted.
Question No. 2 is CIA adapted.

D 1. The type of costs presented to management for a decision to replace equipment should be limited to:
A. controllable costs
B. conversion costs
C. historical costs
D. relevant costs
E. standard costs

B 2. A company can replace the machinery currently used to manufacture its product with more efficient
machinery. The new machinery will reduce labor and also will reduce the percentage of spoiled
units. It is expected to have a useful life of 5 years. The most appropriate technique for determining
whether or not the company should replace its machinery with the new, more efficient machinery is:
A. cost-volume-profit analysis
B. capital-budgeting analysis
C. regression analysis
D. linear programming
E. none of the above

D 3. Depreciation is incorporated explicitly in the cash flow analysis of an investment proposal because
it:
A. is a cost of operations that cannot be avoided
B. results in an annual cash outflow
C. is a cash inflow
D. reduces the cash outlay for income taxes
E. represents the initial cash outflow spread over the life of the investment

E 4. Common problems related to ethical considerations in the capital budgeting include all of the
following, except:
A. superiors and associates sometimes apply pressure to circumvent the approval process
B. pressure may exist to write-off or devalue assets below their true value to justify replacement
C. the economic benefit of capital projects may be exaggerated to increase the likelihood of
approval
D. the accountant may mistakenly go to the individuals involved in the ethical conflict first,
rather than first discussing it with the accounting supervisor
E. all of the above are ethical problems related to capital budgeting

331
Planning for Capital Expenditures 332

D 5. Maxwell Company has an opportunity to acquire a new machine to replace one of its present
machines. The new machine would cost $90,000, have a 5-year life, and no estimated salvage value.
Variable operating costs would be $100,000 per year. The present machine has a book value of
$50,000 and a remaining life of 5 years. Its disposal value now is $5,000, but it would be zero after
5 years. Variable operating costs would be $125,000 per year. Ignore income taxes. Considering
the 5 years in total, what would be the difference in profit before income taxes by acquiring the new
machine as opposed to retaining the present one?
A. $10,000 decrease
B. $15,000 decrease
C. $35,000 increase
D. $40,000 increase
E. none of the above

SUPPORTING CALCULATION:

Additional depreciation on the new machine................................................................ $ (40,000)


Loss on sale of old machine........................................................................................... (45,000)
Operating cost savings.................................................................................................... 125,000
Increase in income.......................................................................................................... $ 40,000

D 6. Effective planning and control is important for the effective administration of a capital expenditure
program because:
A. the long-term commitment increases financial risk
B. the magnitude of expenditures is substantial and the economic penalties for unwise decisions
are usually severe
C. decisions made in this area provide the structure for operation of the firm
D. all of the above
E. none of the above

D 7. A company manual used for detailing policies and procedures required for administering the capital
expenditure program should:
A. encourage people to work on and submit new ideas
B. focus attention on useful analytical tasks
C. facilitate rapid project development and expeditious review
D. all of the above
E. none of the above

D 8. A number of evaluations of a single capital expenditure proposal may be necessary because of:
A. circumstances that change during the time span from the origin of the project idea to its
completion
B. alternative solutions of the problem for which the project is designed
C. assumptions that vary as to the amount and timing of cash flows
D. all of the above
E. none of the above
Planning for Capital Expenditures 333

A 9. The following capital expenditures that compare the future costs of the old assets with the future
costs of the new assets as a basis for making a decision are:
A. replacement expenditures
B. expansion expenditures
C. improvement expenditures
D. allowance expenditures
E. none of the above

B 10. In which of the following types of capital expenditure decisions does the basis for a decision most
markedly shift from cost savings to increased profits and cash flow?
A. replacement expenditures
B. expansion expenditures
C. improvement expenditures
D. allowance expenditures
E. none of the above

C 11. The capital expenditures in which the benefits are most difficult to quantify are:
A. replacement expenditures
B. expansion expenditures
C. improvement expenditures
D. allowance expenditures
E. none of the above

C 12. Primary motivations for computer integrated manufacturing, robotics, and flexible manufacturing
systems include all of the following, except:
A. the need to improve product quality in the face of increasing competition
B. the desire to be able to adjust production output quantity quickly to satisfy changing consumer
demand
C. cost savings
D. the desire to be able to adjust production output variety quickly to satisfy changing consumer
demand
E. all of the above are primary motivations

B 13. All of the following are common cash inflows related to capital expenditure proposals, except:
A. additional revenues from increased sales
B. increased working capital requirements
C. reduction in inventory carrying costs
D. salvage value at the end of the project
E. all of the above are cash inflows

E 14. All of the following are common cash outflows from capital expenditure programs, except:
A. equipment installation
B. employee training
C. computer programming and fine tuning
D. increased working capital requirements
E. salvage value at the end of the project
Planning for Capital Expenditures 334

C 15. The system for recovering the cost of capital expenditures through federal income tax deductions
that was required for tangible, depreciable property placed in service after 1980 is known as:
A. MACRS
B. 200% declining balance
C. ACRS
D. 150% declining balance
E. none of the above

A 16. Under the Tax Reform Act of 1986, the system that increased the number of property classes and
lengthened the recovery periods of most kinds of depreciable property is known as:
A. MACRS
B. 200% declining balance
C. ACRS
D. 150% declining balance
E. none of the above

D 17. An example of 5-year property under MACRS is:


A. most manufacturing machinery
B. railroad cars
C. commercial aircraft
D. light trucks
E. none of the above

B 18. An example of 7-year property under MACRS is:


A. automobiles
B. most manufacturing machinery
C. light trucks
D. small aircraft
E. none of the above

A 19. An example of 27.5-year property under MACRS is:


A. residential rental property
B. commercial aircraft
C. nonresidential buildings
D. railroad cars
E. none of the above

C 20. Under MACRS, the depreciation on tangible personal property is computed as if the property were
placed into service at the:
A. beginning of the year
B. end of the year
C. midpoint of the year
D. midpoint of the month
E. none of the above

D 21. Under MACRS, the depreciation on real property is computed as if the property were placed into
service at the:
A. beginning of the year
B. end of the year
C. midpoint of the year
D. midpoint of the month
E. none of the above
Planning for Capital Expenditures 335

D 22. A machine that cost $50,000 and is fully depreciated is sold for $10,000. The $10,000 is then used
as a down payment on the purchase of a new machine costing $75,000. Assuming a 40% tax rate,
the out-of-pocket cost of the new machine is:
A. $75,000
B. $71,000
C. $65,000
D. $69,000
E. none of the above

SUPPORTING CALCULATION:

Cost of new machine.......................................................................................................... $75,000


Less: After-tax inflow from old machine ($10,000 x .60)............................................... 6,000
.................................................................................................................................. $69,000

C 23. A machine that cost $50,000 and is fully depreciated is allowed as a $10,000 trade-in on a machine
costing $75,000. Assuming a 40% tax rate, the out-of-pocket cost of the new machine is:
A. $75,000
B. $71,000
C. $65,000
D. $69,000
E. none of the above

SUPPORTING CALCULATION:

Cost of new machine.......................................................................................................... $75,000


Less: Trade-in allowance................................................................................................... 10,000
.................................................................................................................................. $65,000
Planning for Capital Expenditures 336

PROBLEMS

PROBLEM

1.
Estimating Pretax Cash Inflows. Skyway Corporation is considering purchasing a new machine to be used to
manufacture a new product, called Jax, which will sell for $15 a unit. Variable manufacturing cost is expected to
be $5 for each unit of Jax manufactured, and variable marketing cost, $2 for each unit sold. The machine being
considered could produce 10,000 units a year, all of which the Marketing Department believes could be sold for
$15 a unit. The proposed machine would cost $250,000. Although the machine would probably last 8 years,
management believes that the product's life cycle would be only 5 years. The salvage value of the new machine at
the end of the product's 5-year life cycle is expected to be $50,000. Management does not believe the machine
could be used to manufacture any of the company's other products.

Required: Compute the pretax net cash inflows expected from the capital expenditure proposal for each year, and
ignoring the effect of income taxes, determine the excess of cash inflows from all sources over the cost of the
machine.

SOLUTION

Estimated Unit Unit Net Pretax


Demand Unit Sales Variable Contribution Cash Inflows
Year in Units Price Cost Margin From Sales
1 10,000 $15 $7 $8 $ 80,000
2 10,000 15 7 8 80,000
3 10,000 15 7 8 80,000
4 10,000 15 7 8 80,000
5 10,000 15 7 8 80,000

Total net pretax cash inflows from sales....................................................... $ 400,000

Initial cash outflow (cost of asset)................................................................. $ 250,000

Less pretax estimated salvage value.............................................................. (50,000) 200,000

Excess of net pretax cash inflows over cost.................................................. $ 200,000


Planning for Capital Expenditures 337

PROBLEM

2.
Estimating Pretax Cash Inflows With Inflation. Speedi Corporation is considering a capital expenditure
proposal which will require an initial cash outlay of $50,000. The project life is expected to be 6 years. The
estimated salvage value for the equipment (based on today's market price for similar used 6-year old equipment) is
$2,500. Estimated annual net cash inflows from operations during the life of the project follow:

Estimated Annual
Year Cash Inflow
1 $10,000
2 15,000
3 15,000
4 15,000
5 10,000
6 5,000

Required: Compute the excess of cash inflows over cash outflows assuming management expects a constant 4%
rate of inflation during the 6-year period. (Round your price level index to three decimal places.)

SOLUTION

Estimated Net 4% Annual Price-level


Pretax Cash Price-level Adjusted Net
Year Inflows Adjustment Cash Inflows
1 $10,000 (1 + .04)1 = 1.040 $ 10,400
2 15,000 (1 + .04)2 = 1.082 16,230
3 15,000 (1 + .04)3 = 1.125 16,875
4 15,000 (1 + .04)4 = 1.170 17,550
5 10,000 (1 + .04)5 = 1.217 12,170
6 5,000 (1 + .04)6 = 1.265 6,325

Total price-level adjusted net pretax cash inflows


from operations..................................................................................................... $ 79,550

Plus cash inflow from salvage...................................................................................... $2,500

Price-level adjustment................................................................................................... 1.265 3,163

Total price-level adjusted net pretax cash inflows


over initial cash outflow....................................................................................... $ 82,713

Less initial cash outflow............................................................................................... 50,000

Excess of net pretax cash inflows over initial cash outflow........................................ $ 32,713
Planning for Capital Expenditures 338

PROBLEM

3.
Estimating After-tax Cash Flows for CIM Project. Athens Corporation is considering the various benefits that
may result from the shortening of its production cycle by changing from the company's present manufacturing
system to a computer integrated manufacturing (CIM) system. The proposed system can provide productive time
equivalency close to the 25,000 hours available annually with the company's present system. The present system
costs $50 per hour more to operate than the proposed CIM system. The company expects to operate the system at
full capacity. The annual out-of-pocket costs of maintaining the proposed CIM system is $500,000 more than the
company's present system. The proposed CIM system will require an initial investment of $1,000,000. The
system is expected to have a useful life of 6 years with no expected salvage value. The company is in a 40% tax-
rate bracket.

Required: Compute the relevant annual after-tax cash flows expected from the CIM project. (Assume the
equipment is 5-year class MACRS property and use the rates provided below.)
(AICPA adapted)

MACRS 5-year
Year Recovery Rate
1 0.200
2 0.320
3 0.192
4 0.115
5 0.115
6 0.058
1.000
Planning for Capital Expenditures 339

SOLUTION

(1) (2) (3) (4) )


Annual )
Operating Additional Taxable )
Savings With Maintenance Tax Income (Loss) )
Year CIM* Cost With CIM Depreciation** (1)-(2)-(3) )
1 $1,250,000 $500,000 $200,000 $550,000 )
2 1,250,000 500,000 320,000 430,000 )
3 1,250,000 500,000 192,000 558,000 )
4 1,250,000 500,000 115,000 635,000 )
5 1,250,000 500,000 115,000 635,000 )
6 1,250,000 500,000 58,000 692,000 )

( (5) (6)
( Tax Liability
( With 40% Net After-tax
( Tax Rate Cash Inflows
( 40% x (4) (1)-(2)-(5)
( $220,000 $530,000
( 172,000 578,000
( 223,200 526,800
( 254,000 496,000
( 254,000 496,000
( 276,800 473,200

Total net after-tax cash inflows....................................................................................................... $ 3,100,000

Less initial cash outflow to purchase system.................................................................................. 1,000,000

Excess of net after-tax cash inflows over initial cash outflow....................................................... $ 2,100,000

*Annual hours of operating capacity.............................................................................................. 25,000

Savings per hour with CIM............................................................................................................. x $50


$ 1,250,000

** MACRS 5-year Tax


Year Recovery Rate Depreciable Basis Depreciation
1 0.200 $1,000,000 $ 200,000
2 0.320 1,000,000 320,000
3 0.192 1,000,000 192,000
4 0.115 1,000,000 115,000
5 0.115 1,000,000 115,000
6 0.058 1,000,000 58,000
1.000 $ 1,000,000
Planning for Capital Expenditures 340

PROBLEM

4.
Computing After-tax Cash Inflows. Stevie Company is considering a capital expenditure with the following
estimated net cash inflows:

Estimated Pretax Inflation


Year Adjusted Net Cash Inflow
1 $ 70,000
2 80,000
3 90,000
4 110,000
5 100,000
6 80,000

The equipment required for the project would have an initial cost of $500,000, and it is not expected to have any
salvage value at the end of the life of the project. The equipment will be depreciated using the straight-line
method over its economic life of 6 years for book purposes; however, it qualifies as 5-year property for tax
purposes. The company's effective tax rate is 40%.

Required: Determine the estimated after-tax net cash inflows for each of the project's 6 years, and the total excess
of cash inflows over the life of the project over cash outflows. (Use the MACRS rates provided below to compute
tax depreciation.)

MACRS 5-year
Year Recovery Rate
1 0.200
2 0.320
3 0.192
4 0.115
5 0.115
6 0.058
1.000
Planning for Capital Expenditures 341

SOLUTION

(1) (2) (3) (4) (5)


Net
Estimated Taxable Tax Liability After-tax
Inflation Income With 40% Cash
Adjusted Net Tax (Loss) Tax Rate Inflows
Year Cash Inflows Depreciation* (1) - (2) 40% x (3) (1) - (4)
1 $ 70,000 $100,000 $(30,000 ) $(12,000 ) $ 82,000
2 80,000 160,000 (80,000 ) (32,000 ) 112,000
3 90,000 96,000 (6,000) (2,400) 92,400
4 110,000 57,500 52,500 21,000 89,000
5 100,000 57,500 42,500 17,000 83,000
6 80,000 29,000 51,000 20,400 59,600

Total net after-tax cash inflows....................................................................................................... $ 518,000

Less initial cash outflow to purchase system.................................................................................. 500,000

Excess of net after-tax cash inflows over initial cash outflow....................................................... $ 18,000

* MACRS
5-year
Recovery Depreciable Tax
Year Rate Basis Depreciation
1 0.200 $500,000 $100,000
2 0.320 500,000 160,000
3 0.192 500,000 96,000
4 0.115 500,000 57,500
5 0.115 500,000 57,500
6 0.058 500,000 29,000
1.000 $500,000

PROBLEM

5.
Effect of Inflation and Taxes on Investment Decision. Weighout Company is evaluating a capital expenditure
proposal that will require an initial cash investment of $100,000. The project will have a 6-year life; however,
the property will qualify as 5-year property for income-tax depreciation purposes. The income tax rate is 40%.
The annual cash inflows from the project, before any adjustment for the effects of inflation or income taxes, are
expected to be as follows:

Unadjusted
Estimate of
Year Cash Inflows
1 $25,000
2 27,000
3 29,000
4 23,000
5 20,000
6 15,000
Planning for Capital Expenditures 342

The expected salvage value of the property is zero. Cash inflows are expected to increase at the anticipated
inflation rate of 4% each year.

Required: Compute the inflation adjusted after-tax cash inflow from the proposal for each year, and the excess of
total net cash inflows over the initial cash outlay. (Use the MACRS depreciation rates provided below to compute
tax depreciation, and round the price-level index to three decimal places.)

MACRS 5-year
Year Recovery Rate
1 0.200
2 0.320
3 0.192
4 0.115
5 0.115
6 0.058
1.000

SOLUTION

(1) (2) (3)


Inflation
Adjusted
Estimate of
Periodic Cash
Cash Inflows
Year Inflows 4% Price-level Adjustment (1) x (2)
1 $ 25,000 (1 + .04)1 = 1.040 $ 26,000
2 27,000 (1 + .04)2 = 1.082 29,214
3 29,000 (1 + .04)3 = 1.125 32,625
4 23,000 (1 + .04)4 = 1.170 26,910
5 20,000 (1 + .04)5 = 1.217 24,340
6 15,000 (1 + .04)6 = 1.265 18,975
$ 139,000 $ 158,064

(1) (2) (3)


5-Year
Depreciable Property Tax
Basis of Recovery Depreciation
Year Property Percentage (1) x (2)
1 $100,000 0.200 $ 20,000
2 100,000 0.320 32,000
3 100,000 0.192 19,200
4 100,000 0.115 11,500
5 100,000 0.115 11,500
6 100,000 0.058 5,800
$ 100,000
Planning for Capital Expenditures 343

(1) (2) (3) (4) )


Adjusted Taxable Federal and )
Estimate of Income State )
Net Cash Tax (Loss) Income Tax )
Year Inflows Depreciation (1) - (2) Rate )
1 $26,000 $20,000 $ 6,000 40% )
2 29,214 32,000 (2,786 ) 40% )
3 32,625 19,200 13,425 40% )
4 26,910 11,500 15,410 40% )
5 24,340 11,500 12,840 40% )
6 18,975 5,800 13,175 40% )

( (5) (6)
( Net
( After-tax
( Cash
( Income Tax Inflows
( (3) x (4) (1) - (5)
( $ 2,400 $23,600
( (1,114 ) 30,328
( 5,370 27,255
( 6,164 20,746
( 5,136 19,204
( 5,270 13,705

Total estimated net after-tax cash inflows from project.................................................................. $134,838

Less initial cash outflow for machinery........................................................................................... 100,000

Excess of after-tax cash inflows from project over initial cash outflow......................................... $ 34,838
Planning for Capital Expenditures 344

Chapter 23

ECONOMIC EVALUATION OF CAPITAL EXPENDITURES

MULTIPLE CHOICE

Question Nos. 8-11, 15-19, 21, 28, 30, and 31 are AICPA adapted.
Question Nos. 12, 13, 20, and 27 are ICMA adapted.
Question Nos. 14, 22-26, and 29 are CIA adapted.

E 1. In order to calculate the payback period for a project, it is necessary to know the:
A. salvage value
B. useful life
C. minimum desired rate of return
D. net present value
E. annual cash flow

C 2. The component of the capital investment decision that would most likely concern an accountant is
the:
A. social responsibility factors
B. competition
C. time value of money
D. imponderables
E. legal restrictions

C 3. The net present value of a proposed project represents the:


A. cash flows less the original investment
B. present value of the cash flows plus the present value of the original investment less the
original investment
C. present value of the cash flows less the original investment
D. present value of the cash flows less the cost of the old machine being replaced
E. cash flows less the present value of the cash flows

B 4. If an initial investment outlay is $60,000 and the cash flows projected are $15,000, $20,000,
$25,000, and $10,000 in each of the first four years, respectively, the payback period in years would
be:
A. 3.3
B. 3.0
C. 2.5
D. 4.0
E. 5.0

SUPPORTING CALCULATION:
$15,000 + $20,000 + $25,000 = $60,000 or 3 years

344
Economic Evaluation of Capital Expenditures 345

C 5. R. D. Inc. purchased a machine for $240,000. The machine has a useful life of six years, no salvage
value, and straight-line depreciation is to be used. The machine is expected to generate cash flows
from operations, net of income tax, of $70,000 in each of the six years. R. D. Inc's cost of capital is
12%. The net present value is:
A. $180,000
B. $35,490
C. $47,770
D. $121,680
E. $123,330

SUPPORTING CALCULATION:

($70,000 x 4.111) - $240,000 = $47,770

E 6. Conte Inc. invested in a machine with a useful life of six years and no salvage value. The machine
is expected to produce annual cash flows from operations, net of income tax, of $2,000. If the
estimated internal rate of return is 10%, the amount of the original investment was:
A. $9,000
B. $11,280
C. $12,000
D. $5,640
E. $8,710

SUPPORTING CALCULATION:

4.355 x $2,000 = $8,710

D 7. If a company's desired long-run proportion of funds to be provided consists of 60% common stock
and retained earnings with an after-tax cost of 15%, and 40% bonds with an after-tax cost of 8%, its
weighted-average cost of capital will be:
A. 13.7%
B. 15%
C. 10.8%
D. 12.2%
E. 8%

SUPPORTING CALCULATION:

Proportion of Weighted-
Source Total Funds After-tax Cost Average Cost
Common Stock..................... .60 .15 .090
Bonds..................................... .40 .08 .032
.122
Economic Evaluation of Capital Expenditures 346

B 8. The method of project selection that considers the time value of money in a capital budgeting
decision computes the:
A. accounting rate of return on average investment
B. internal rate of return
C. payback period
D. return on investment
E. accounting rate of return on initial investment

A 9. The net present value and the internal rate of return methods of decision making in capital budgeting
are superior to the payback method in that they:
A. consider the time value of money
B. are easier to implement
C. consider accrual-based accounting income
D. require less input
E. reflect the effects of depreciation and income taxes

A 10. Which of the following capital expenditure planning and control techniques has been criticized
because it might mistakenly imply that earnings are reinvested at the rate of return earned by the
investment?
A. internal rate of return method
B. accounting rate of return on initial investment method
C. payback method
D. average return on investment method
E. present value method

A 11. A company is considering the purchase of a new conveyor belt system for carrying parts and
subassemblies from building to building within its plant complex. It is expected that the system will
have a useful life of at least ten years and that it will substantially reduce labor and waiting-time
costs. If the company's average cost of capital is about 15% and if some evaluation must be made of
cost/benefit relationships (including the effects of interest) to determine the desirability of the
purchase, the most relevant quantitative technique for evaluating the investment is:
A. present value (or internal rate of return) analysis
B. Program Evaluation and Review Technique (PERT)
C. accounting rate of return analysis
D. cost-volume-profit analysis
E. payback analysis

D 12. Cinzano Inc. wants to use discounted cash flow techniques when analyzing its capital investment
projects. The company is aware of the uncertainty involved in estimating future cash flows. A
simple method some companies employ to adjust for the uncertainty inherent in their estimates is to:
A. ignore salvage values
B. average the expectations of several different managers
C. use accelerated depreciation
D. adjust the minimum desired rate of return
E. increase the estimates of the cash flows
Economic Evaluation of Capital Expenditures 347

A 13. Depreciation is incorporated explicitly in the discounted cash flow analysis of an investment
proposal because it:
A. reduces the cash outlay for income taxes
B. represents the initial cash outflow spread over the life of the investment
C. is a cost of operations that cannot be avoided
D. results in an annual cash outflow
E. is a cash inflow

C 14. Deficiencies associated with using the payback method to evaluate investment alternatives include
all of the following, except that:
A. the present value of cash inflows is ignored
B. inflows of different time periods are treated equally
C. it may be used to select those investments yielding a quick return of cash
D. cash flows after the payback period are ignored
E. all of the above

B 15. For $45,000, Harmon Company purchased a new machine with an estimated useful life of five years
with no salvage value. The machine is expected to produce cash flow from operations, net of
income taxes, as follows:

1st year................................................................................................................................. $ 9,000


2d year.................................................................................................................................. 12,000
3d year.................................................................................................................................. 15,000
4th year................................................................................................................................. 9,000
5th year................................................................................................................................. 8,000

Harmon will use the sum-of-the-years-digits' method to depreciate the new machine as follows:

1st year................................................................................................................................. $15,000


2d year.................................................................................................................................. 12,000
3d year.................................................................................................................................. 9,000
4th year................................................................................................................................. 6,000
5th year................................................................................................................................. 3,000

What is the payback period?


A. 3 years
B. 4 years
C. 5 years
D. 2 years
E. none of the above

SUPPORTING CALCULATION:

$9,000 + $12,000 + $15,000 + $9,000 = $45,000 or 4 years

B 16. The capital budgeting method that assumes that funds are reinvested at the company's cost of capital
is:
A. accounting rate of return
B. net present value
C. internal rate of return
D. return on investment
E. payback
Economic Evaluation of Capital Expenditures 348

A 17. The basis for measuring the cost of capital derived from bonds and preferred stock, respectively, is
the:
A. after-tax rate of interest for bonds and stated annual dividend rate for preferred stock
B. pretax rate of interest for bonds and stated annual dividend rate less the expected earnings per
share for preferred stock
C. pretax rate of interest for bonds and stated annual dividend rate for preferred stock
D. after-tax rate of interest for bonds and stated annual dividend rate less the expected earnings
per share for preferred stock
E. none of the above

D 18. The weighted-average cost of capital approach to decision making is not directly affected by the:
A. proposed mix of debt, equity, and existing funds used to implement the project
B. stated annual dividend rate for preferred stock
C. value of the common stock
D. current budget for expansion
E. cost of debt outstanding

D 19. If income tax considerations are ignored, how is depreciation expense used in the following capital
budgeting techniques?
Internal Rate of Return Payback
A. Excluded Included
B. Included Excluded
C. Included Included
D. Excluded Excluded

A 20. Perot Inc. is in the enviable situation of having unlimited capital funds. The best decision rule, in an
economic sense, for it to follow would be to invest in all projects in which:
A. the net present value is greater than zero
B. the payback is less than four years
C. the accounting rate of return is greater than the earnings as a percentage of sales
D. the payback reciprocal is greater than the internal rate of return
E. the internal rate of return is greater than zero

A 21. Orab Co. has the chance to invest in a 2-year project expected to produce cash flows from
operations, net of income taxes, of $100,000 in the first year and $200,000 in the second year. Orab
requires an internal rate of return of 20%. The present value of $1 for one period at 20% is 0.833;
for two periods at 20% is 0.694. For this project, Orab should be willing to invest immediately a
maximum of:
A. $222,100
B. $208,200
C. $283,300
D. $249,900
E. none of the above

SUPPORTING CALCULATION:

($100,000 x .833) + ($200,000 x .694) = $222,100


Economic Evaluation of Capital Expenditures 349

A 22. The three frequently used methods for ranking investment proposals are payback, net present value,
and internal rate of return. One of the three is defined as the interest rate that equates the present
value of expected cash flows and the cost of the project. A second method finds the present value of
expected cash flows and subtracts the initial cost of the project. The following terms that match
these respective definitions are:
A. internal rate of return and net present value
B. internal rate of return and payback
C. net present value and internal rate of return
D. net present value and payback
E. none of the above

E 23. Your company is comparing internal rate of return to net present value computations as alternative
criteria for evaluating potential capital investments. Which of the following best describes these
computations?
A. The internal rate of return method ignores the initial cost of the investment in its
computations.
B. The net present value method ignores the company's cost of capital.
C. The net present value method is more appropriate to use during periods of inflation.
D. The two methods will give the same rankings because they both consider the time value of
money.
E. The internal rate of return method assumes that the positive cash flows generated each year
are reinvested at the computed rate of return for the investment being evaluated.

C 24. A project has a cost of $5,000 and is expected to produce a cash flow of $1,220 a year for five years.
Using the table given, what is the internal rate of return? (Note: Annuity factors are rounded to two
places.)

Future Value of an Present Value of an


Annuity of $1 per Period Annuity of $1 per Period
for 5 Periods for 5 Periods
7% 5.75 4.10
8% 5.87 3.99
9% 5.98 3.89
10% 6.11 3.79

A. 9%
B. 10%
C. 7%
D. 8%
E. none of the above

SUPPORTING CALCULATION:

4.10 x $1,220 = $5,002  7% IRR


Economic Evaluation of Capital Expenditures 350

E 25. At a company's cost of capital of 15%, a prospective investment has a negative net present value.
Based on this information, it can be concluded that:
A. the internal rate of return is greater than 15%
B. the payback period is shorter than the life of the asset
C. the accounting rate of return is less than 15%
D. the accounting rate of return is greater than 15%
E. the internal rate of return is less than 15%

D 26. Risk can be controlled in capital budgeting situations by assuming a:


A. high accounting rate of return
B. large net present value
C. high net income
D. short payback period
E. all of the above

D 27. The net present value (NPV) method and the internal rate of return (IRR) method are used to analyze
capital expenditures. The IRR method, as contrasted with the NPV method:
A. is considered inferior because it fails to calculate compounded interest rate
B. is an internal rate of return method, while the NPV method is not
C. almost always gives a different decision as to the acceptability of a given proposal
D. assumes that the rate of return on the reinvestment of the cash proceeds is at the internal rate
of return of the project analyzed rather than at the cost of capital used
E. is preferred in practice because it is able to handle multiple desired hurdle rates, which is
impossible with the NPV method

D 28. The net present value capital budgeting technique can be used when cash flows from period to
period are:
Uniform Uneven
A. no yes
B. no no
C. yes no
D. yes yes

C 29. Which of the following is always true of the net present value (NPV) approach?
A. If a project is found to be acceptable under the NPV approach, it would also be acceptable
under the internal rate of return (IRR) approach.
B. The NPV and the IRR approaches will always rank projects in the same order.
C. If a project is found to be acceptable under the NPV approach, it would also be acceptable
under the payback approach.
D. The NPV and the payback approaches will always rank projects in the same order.
E. None of the above

A 30. If income tax considerations are ignored, how is depreciation expense used in the following capital
budgeting techniques?
Internal Rate of Return Net Present Value
A. Excluded Excluded
B. Excluded Included
C. Included Excluded
D. Included Included
Economic Evaluation of Capital Expenditures 351

D 31. A proposed project has an expected economic life of eight years. In the calculation of the net present
value (NPV) of the project, salvage value would be:
A. excluded from the calculation of the NPV
B. included as a cash inflow at the estimated salvage value
C. included as a cash inflow at the future amount of the estimated salvage value
D. included as a cash inflow at the present value of the estimated salvage value
E. none of the above
Economic Evaluation of Capital Expenditures 352

PROBLEMS

PROBLEM

1.
Payback Period; Rate of Return; Net Present Value. The management of Elite Cookies Inc. is considering the
purchase of a new shaping machine. The machine will cost $100,000 and will have a useful life of 10 years with a
salvage value of $10,000 at the end of ten years. The investment will result in cost savings of $16,000 per year
for each year of the machine's life. The tax rate is zero, and the appropriate discount rate for the company is 10%.
(The present value factor for $1 received at the end of 10 years is .386, and the factor for $1 received annually for
10 years is 6.145.)

Required:

(1) Compute the payback period.


(2) Compute the accounting rate of return on the average investment.
(3) Compute the net present value.
(Round answers to two decimal places.)

SOLUTION

(1) $100,000/$16,000 = 6.25 years payback period


(2) [($160,000 - $90,000)/10 yrs.] / [($100,000 + $10,000)/2] = 12.73% accounting rate of return on average
investment
(3)
Net Present value Present value of Original
present = of ten annual cash + inflow from salvage - investment
value inflows of $16,000 in tenth year

$2,180 = ($16,000 x 6.145) + ($10,000 x .386) - ($100,000)

PROBLEM

2.
Effect of Inflation on Investment Decision. Ranchero Company is evaluating a capital budgeting proposal that
will require an initial cash investment of $100,000. The project will have a 3-year life. The net after-tax cash
inflows from the project, before any adjustment for the effects of inflation, are expected to be as follows:

Unadjusted
Estimate of
Year Cash Inflows
1 $50,000
2 40,000
3 30,000

No salvage is expected at the end of the project. The anticipated inflation rate is 10% each year. The company's
cost of capital rate is 16%.
Economic Evaluation of Capital Expenditures 353

Required:

(1) Compute the estimated cash inflow for each year, adjusted for the anticipated effect of inflation.
(2) Determine the net present value of the cash flows before and after the adjustment for the anticipated
effects of inflation.
(The present values of $1 @ 16% at the end of years 1, 2, and 3 respectively are .862, .743, and .641. The present
value of an annuity of $1 @ 16% for 3 years is 2.246.)

SOLUTION

(1)
Inflation Adjusted
Unadjusted Estimate Inflation Estimate
Year of Cash Inflows Adjustment of Cash Inflows
1 $ 50,000 1.100 $ 55,000
2 40,000 1.210 48,400
3 30,000 1.331 39,930
$ 120,000 $ 143,330

(2)
PV of PV of
Unadjusted Adjusted PV of $1 Unadjusted Adjusted
Year Cash Flows Cash Flows @ 16% Cash Flows Cash Flows
0 $(100,000 ) $(100,000 ) 1.000 $ (100,000 ) $ (100,000 )
1 50,000 55,000 .862 43,100 47,410
2 40,000 48,400 .743 29,720 35,961
3 30,000 39,930 .641 19,230 25,595
Net present value of investment........................................................................ $ (7,950 ) $ 8,966

PROBLEM

3.
Payback Period; Net Present Value; Internal Rate of Return. The president of Eradicator Corp. is considering
the purchase of new demolition equipment costing $100,000, with a useful life of five years and no salvage value.
The new equipment would yield an annual after-tax cash flow of $29,129. An appropriate discount rate for this
type of equipment is 12%. (The present value of an annuity of a dollar @ 12% for five years is 3.605. The
present value of a dollar @ 12% received at the end of the fifth year is .567.)

Required:

(1) Compute the payback period to the nearest tenth of a year.


(2) Compute the net present value to the nearest whole dollar.
(3) Compute the internal rate of return on the purchase.
(4) Should the equipment be purchased?
Economic Evaluation of Capital Expenditures 354

SOLUTION

(1) $100,000/$29,129 = 3.4 years payback period

(2) ($29,129 x 3.605) - $100,000 = $5,010 net present value

(3) $100,000/$29,129 = 3.433 discount factor


Present value of $1 received annually for 5 years at 14% interest has a factor equal to 3.433. The internal
rate of return on the purchase is 14%.

(4) Yes. The net present value is positive and the internal rate of return (14%) is more than the discount rate
(12%).

PROBLEM

4.
Cost of Capital. Molloy Company wishes to compute a weighted-average cost of capital for use in evaluating
capital expenditure proposals. Earnings, capital structure, and current market prices of the company's securities
are:

Earnings:
Earnings before interest and tax................................................................................................. $ 400,000
Interest expense on bonds........................................................................................................... 100,000
Pretax earnings............................................................................................................................ $ 300,000
Income tax (40%)........................................................................................................................ 120,000
After-tax earnings....................................................................................................................... $ 180,000
Preferred stock dividends............................................................................................................ 75,000
Earnings available to common stockholders.............................................................................. $ 105,000
Common stock dividends............................................................................................................ 50,000
Retained earnings........................................................................................................................ $ 55,000

Capital structure:
Mortgage bonds, 12%, 20 years................................................................................................. $ 500,000
Preferred stock, 15%, $100 par.................................................................................................. 500,000
Common stock, no par, 25,000 shares........................................................................................ 300,000
Retained earnings (equity of common stockholders)................................................................. 700,000
$ 2,000,000

Market price of the company's securities:


Preferred stock............................................................................................................................ $100
Common stock............................................................................................................................ 30

Required: Determine the company's cost of capital to the nearest hundredth of a percent.
Economic Evaluation of Capital Expenditures 355

SOLUTION

Proportion After-tax Weighted


FundsSource of Funds Cost Cost
Bonds........................................................................... .25 .071 .018

Preferred stock............................................................. .25 .152 .0375

Common stock and retained earnings......................... .50 .143 .070


1.00 .1255 or 12.55%

Computations:
1
.12 - (.12 x .4)
2
$15/$100 = .15
3
$105,000/25,000 = $4.20; $4.20/$30 = .14

PROBLEM

5.
Investment Analysis; Uniform Cash Flow. Denali Company is evaluating a capital budgeting proposal,
requiring an initial investment of $45,000. The project will have a five-year life. The after-tax annual cash
inflow from this investment is $12,000. The cost of capital is 10%. (The present value of $1 @ 10% received at
the end of five years is .621. (The present value of $1 @ 10% received each year for five years is 3.791.)

Required:

(1) What is the payback period?


(2) Compute the net present value of the project.
(3) What amount should Denali have invested five years ago, at 10% compounded annually, to have $45,000
now?

SOLUTION

(1) $45,000 initial outlay/$12,000 annual outflow = 3.75 payback years

(2) Cash inflow ($12,000 x 3.791).................................................................................................... $ 45,492


Investment.................................................................................................................................... 45,000
Net present value......................................................................................................................... $ 492

(3) $45,000 x .621 = $27,945


Economic Evaluation of Capital Expenditures 356

PROBLEM

6.
Use of Net Present Value to Evaluate Asset Acquisition. Jurassic Park is considering the acquisition of a new
ride at a cost of $250,000. The ride has an estimated useful life of 7 years, but it qualifies as 5-year property for
tax purposes under MACRS. The annual pretax cash inflows from the new ride, net of annual operating expenses,
is expected to be $50,000 in each of the 7 years the ride will be used. At the end of the 7-year period, company
executives believe that the ride can be sold for $60,000. The company is in a 40% income tax bracket, and its
weighted-average cost of capital is 15%.

Required:

(1) Determine the net present value of the investment in the new ride. (Use the MACRS rates and net present
value factors provided below.)

(2) Should the investment be made?

MACRS
5-year Property Recovery Present Value of Present Value of an
Percentage $1 @ 15% Annuity of $1 @ 15%
Year Percentage 1 .870 1 .870
1 .200 2 .756 2 1.626
2 .320 3 .658 3 2.283
3 .192 4 .572 4 2.855
4 .115 5 .497 5 3.352
5 .115 6 .432 6 3.784
6 .058 7 .376 7 4.160

SOLUTION

(1) (1) (2) (3)


Tax Depreciation
Tax Basis of 5-year Property on New Ride
Year New Ride Recovery Rate (1) x (2)
1 $250,000 0.200 $ 50,000
2 250,000 0.320 80,000
3 250,000 0.192 48,000
4 250,000 0.115 28,750
5 250,000 0.115 28,750
6 250,000 0.058 14,500
$ 250,000
Economic Evaluation of Capital Expenditures 357

(1) (2) (3) (4) (5) )


Increase Increase )
Annual (Decrease) (Decrease) )
Pretax Tax in Taxable Income in Income )
Cash Depreciation Income Tax Tax )
Year Inflows Deduction (1) - (2) Rate (3) x (4)
)
1 $50,000 $50,000 0 40% 0 )
2 50,000 80,000 (30,000) 40% (12,000) )
3 50,000 48,000 2,000 40% 800 )
4 50,000 28,750 21,250 40% 8,500 )
5 50,000 28,750 21,250 40% 8,500 )
6 50,000 14,500 35,500 40% 14,200 )
7 50,000 0 50,000 40% 20,000 )

( (6) (7) (8)


( Present
( Value of
( After-tax After-tax
( Cash Present Cash
( Inflows Value of Inflows
( (1)-(5) $1 @ 15% (6) x (7)
( 50,000 0.870 43,500
( 62,000 0.756 46.872
( 49,200 0.658 32,374
( 41,500 0.572 23,738
( 41,500 0.497 20,626
( 35,800 0.432 15,466
( 30,000 0.376 11,280

Present value of periodic after-tax cash inflows................................................................................. $ 193,856


Plus present value of after-tax salvage [$60,000 x (1-40%) x .376].................................................. 13,536
Present value of cash inflows over useful life of new ride................................................................. $ 207,392
Less initial cash outflow (cost of ride)................................................................................................ 250,000
Net present value of investment.......................................................................................................... $ (42,608 )

(2) No. The negative net present value indicates that the rate of return on this proposed ride will be less than
the company's weighted-average cost of capital.
Economic Evaluation of Capital Expenditures 358

Chapter 24

DECISION MAKING UNDER UNCERTAINTY

MULTIPLE CHOICE

Question Nos. 1, 2, and 19 are AICPA adapted.


Question Nos. 4-6, 8-11, and 14-17 are ICMA adapted.
Question Nos. 7, 12, 13, and 18 are CIA adapted.

A. 1. Which of the following best identifies the reason for using probabilities in capital budgeting
decisions?
A. uncertainty
B. cost of capital
C. time value of money
D. projects with unequal lives
E. all of the above

D 2. In probability analysis, the square root of the mean of the squared differences between the
conditional values and the expected value is the:
A. objective function
B. optimum corner point
C. EOQ
D. standard deviation
E. none of the above

E 3. Which of the following utilizes statistical sampling techniques in capital budgeting in order to obtain
a probabilistic approximation of the profitability of a capital expenditure proposal?
A. sensitivity analysis
B. decision tree
C. linear programming
D. probabilistic budgeting
E. Monte Carlo simulation

B 4. The Social Club plans to apply the expected value decision rule (criterion) to determine the number
of cups of hot cider to stock. The expected value is the:
A. sum of the conditional profit (loss) for each event
B. sum of the conditional profit (loss) of each event times the probability of each event occurring
C. conditional profit (loss) for the best event times the probability of each event occurring
D. sum of the conditional opportunity loss of each event times the probability of each event
occurring
E. revenue less the costs

358
Decision Making Under Uncertainty 359

D 5. The Social Club plans to use a payoff table to apply the expected value decision rule (criterion) to
determine the number of cups of hot cider to stock. The Social Club would select the demand level
that:
A. is closest to the expected demand
B. has the greatest probability of occurring
C. has the greatest expected opportunity loss
D. has the greatest expected monetary value
E. includes the event with the greatest conditional profit

E 6. The Social Club plans to apply the expected value decision rule (criterion) to determine the number
of cups of hot cider to stock. The maximum expected value of additional information is the:
A. same as the expected profit under certainty
B. sum of the conditional profit (loss) for the best event of each act times the probability of each
event occurring
C. difference between the expected profit under certainty and the expected opportunity loss
D. difference between the expected profit under certainty and conditional profit for the best act
under certainty
E. difference between the expected profit under certainty and the expected monetary value of the
best act under uncertainty

C 7. Solutions provided by quantitative techniques based on probabilities should be considered to be:


A. numerically precise and correct
B. approximations based solely on past experiences
C. the best estimate of expected results
D. unaffected by environmental changes
E. none of the above

C 8. Decisions are frequently classified as those made under certainty and those made under uncertainty.
Certainty exists when:
A. the probabilities for each outcome of an event can be assigned with a high degree of
confidence
B. the probability of the event is less than 1
C. there is absolutely no doubt that an event will occur
D. there is more than one outcome for each possible action
E. the standard deviation of an event is greater than 0

C 9. Barkley & Co. has been sued by a client for breach of warranty. Barkley's controller has
accumulated data from the outcomes of similar cases. Barkley & Co. can best quantify its exposure
to a loss in this situation by using:
A. regression analysis
B. Markov analysis
C. expected value analysis
D. queuing theory
E. Matrix algebra
Decision Making Under Uncertainty 360

B 10. Arlington Inc. is attempting to predict the profitability of a new product line. The Marketing
Department has developed three different forecasts of annual demand and their related probabilities
of occurrence for the coming yearlow (.2), medium (.5), and high (.3). To develop an estimate of
the annual profit figure for the new product line, Arlington Inc. should employ:
A. queuing theory
B. expected value analysis
C. correlation and regression analysis
D. discounted cash flow techniques
E. PERT/CPM analysis

B 11. Expected value in decision analysis is:


A. a standard deviation using the probabilities as weights
B. an arithmetic mean using the probabilities as weights
C. the square root of the squared deviations
D. the standard deviation divided by the coefficient of variation
E. a measure of the difference between the best possible outcome and the outcome of the
original decision

D 12. A proprietor who just inherited a building is considering using it in a new business venture.
Projections for the business are: revenue of $100,000, fixed cost of $30,000, and variable cost of
$50,000. If the business is not started, the owner will work for a company for a wage of $23,000.
Also, there have been two offers to rent the building, one for $1,000 per month and one for $1,200
per month. What are the expected annual net economic profits (losses) to the owner if the new
business is started?
E. $20,000
F. $(3,000)
G. $(15,000)
H. $(17,400)
E. none of the above

SUPPORTING CALCULATION:

$100,000 - $30,000 - $50,000 - $23,000 - (12 x $1,200) = $(17,400)

C 13. A firm obtained the following data based on the results shown below for 100 runs simulating the
introduction of a new product.
Net Profit Before Tax: ($5,000 ) $0 $5,000 $10,000 $15,000
Frequency: .30 .30 .20 .15 .05

The firm should:


A. expect to break even if the product is introduced
B. not introduce the product
C. expect to make a profit if the product is introduced
D. expect to lose money if the product is introduced
E. none of the above
Decision Making Under Uncertainty 361

SUPPORTING CALCULATION:

Profit Probability Expected Value


$(5,000 ) .30 $(1,500 )
0 .30 0
5,000 .20 1,000
10,000 .15 1,500
15,000 .15 2,250
$3,250

B 14. The Prep Club sells fresh hot cider at Ivy University's home football games. The frequency
distribution of the demand for cups of hot cider per game is presented below.

Unit Sales Volume Probability


10,000 cups .10
20,000 cups .15
30,000 cups .20
40,000 cups .35
50,000 cups .20
1.00

The hot cider is sold for $1.00 a cup, and the cost per cup is $.40. Any unsold hot cider is discarded
because it will spoil before the next home game.

The estimated demand for hot cider at the next Ivy University home football game using an expected
value approach is:
A. 30,000 cups
B. 34,000 cups
C. 40,000 cups
D. 50,000 cups
E. some amount other than those given above

SUPPORTING CALCULATION:

10,000 x .10 = 1,000


20,000 x .15 = 3,000
30,000 x .20 = 6,000
40,000 x .35 = 14,000
50,000 x .20 = 10,000
34,000
Decision Making Under Uncertainty 362

A 15. The Prep Club sells fresh hot cider at Ivy University's home football games. The frequency
distribution of the demand for cups of hot cider per game is presented below.

Unit Sales Volume Probability


10,000 cups .10
20,000 cups .15
30,000 cups .20
40,000 cups .35
50,000 cups .20
1.00

The hot cider is sold for $1.00 a cup, and the cost per cup is $.40. Any unsold hot cider is discarded
because it will spoil before the next home game.

The conditional profit (loss) per game of having 30,000 cups of hot cider available but only selling
20,000 cups of cider is:
I. $8,000
J. $12,000
K. $18,000
L. $3,000
E. some amount other than those given above

SUPPORTING CALCULATION:

$1(20,000) - $.40($30,000) = $8,000

C 16. The Prep Club sells fresh hot cider at Ivy University's home football games. The frequency
distribution of the demand for cups of hot cider per game is presented below.

Unit Sales Volume Probability


10,000 cups .10
20,000 cups .15
30,000 cups .20
40,000 cups .35
50,000 cups .20
1.00

The hot cider is sold for $1.00 a cup, and the cost per cup is $.40. Any unsold hot cider is discarded
because it will spoil before the next home game.

The conditional profit (loss) per game of having 30,000 cups of hot cider available but being able to
sell 40,000 cups of hot cider if it were available is:
A. $14,000
B. $12,000
C. $18,000
D. $24,000
E. some amount other than those given above

SUPPORTING CALCULATION:
30,000 ($1 - $.40) = $18,000
Decision Making Under Uncertainty 363

E 17. Boyer Company is considering designing an educational computer software package. Boyer's
management is aware that this project may not be feasible, that demand for the software may be low,
and that competitors may offer a similar package before Boyer does. Boyer can best evaluate the
possible payoffs of the computer software project by using:
A. differential calculus
B. critical path analysis
C. linear programming
D. regression analysis
E. decision tree analysis

C 18. A decision tree has been formulated for the possible outcomes of introducing a new product line.

.7
/------------- $100,000
/
#1----------
\
\------------- $70,000
.3

.8
/------------- $170,000
/
#2----------
\
\------------- $80,000
.2

Branches related to Alternative #1 reflect the possible payoffs from introducing the product without
an advertising campaign. The branches for Alternative #2 reflect the possible payoffs with an
advertising campaign costing $40,000. The expected values of Alternatives #1 and #2, respectively,
are:
A. #1: (.7 x $100,000) + (.3 x $70,000)
#2: (.8 x $170,000) + (.2 x $80,000)
B. #1: (.7 x $100,000) + (.3 x $70,000)
#2: (.8 x $130,000) + (.2 x $40,000)
C. #1: (.7 x $100,000) + (.3 x $70,000)
#2: (.8 x $170,000) + (.2 x $80,000) - $40,000
D. #1: (.7 x $100,000) + (.3 x $70,000) - $40,000
#2: (.8 x $170,000) + (.2 x $80,000) - $40,000
E. none of the above

B 19. A firm wishes to compare the effects of using a new labor-saving machine with present direct labor
methods. These comparisons will be made over a wide variety of operations on several typical days.
The demands placed upon each operation as well as the sequence of individual operations can be
described by probability distributions. The most relevant quantitative technique is:
A. cost-volume-profit analysis
B. Monte Carlo simulation
C. Program Evaluation and Review Technique (PERT)
D. statistical sampling
E. time-series or trend-regression analysis
Decision Making Under Uncertainty 364

C 20. When several unit sales volumes are multiplied by the probability of their occurrence and those
products are summed, the result is the:
A. median
B. standard deviation
C. expected value
D. best estimated sales level
E. average sales level

C 21. The quantitative technique that would be most useful in projecting revenues is:
A. linear programming
B. PERT/cost analysis
C. probability theory
D. learning-curve analysis
E. queuing theory

B 22. Probabilistic estimates are most frequently used with which of the following methods of capital
expenditure evaluation?
A. payback
B. present value
C. internal rate of return
D. accounting rate of return
E. none of the above

D 23. The measure of the variability of expected outcomes in a probability distribution is known as the:
A. coefficient of variation
B. standard deviation
C. expected value
D. variance
E. none of the above

A 24. Which of the following can be computed and compared for each alternative to determine the relative
riskiness of investments that have different levels of expected return?
A. coefficient of variation
B. variance
C. standard deviation
D. expected value
E. none of the above

C 25. Which of these could occur in practice where the capital expenditure relates to the production of an
established product or service, the demand for which is expected to vary in response to temporary
changes in consumer taste?
A. perfectly correlated cash flows
B. negative cash flows
C. independent cash flows
D. mixed cash flows
E. none of the above
Decision Making Under Uncertainty 365

E 26. In capital expenditure analysis, which of the following can be constructed to evaluate alternative
levels of investment?
A. normal distribution
B. bar graph
C. nonnormal distribution
D. pie chart
E. payoff table

A 27. Which of these is useful in that it gives the manager a visual map of the expected levels of each
alternative action?
A. decision tree
B. Monte Carlo simulation
C. Markov chain
D. sensitivity analysis
E. none of the above

E 28. The standard deviation of the expected net present value is determined by summing the discounted
standard deviations for each period over the life of the project when the cash flows in each of the
periods are:
A. independent
B. positive
C. mixed
D. negative
E. perfectly correlated

E 29. If events are related, computational procedures must be modified by substituting:


A. random variables
B. slack variables
C. dependent variables
D. independent probabilities
E. conditional probabilities

A 30. An expenditure evaluation tool that explicitly incorporates both quantitative and nonquantitative
factors into the decision analysis is known by the acronym:
A. MADM
B. FMS
C. CIM
D. JIT
E. none of the above
Decision Making Under Uncertainty 366

PROBLEMS

PROBLEM

1.
Probability Analysis. The operator of an office building concession stand wishes to know how many doughnuts
to stock each day. The doughnuts cost $.25 each and are sold for $.35 each. Those unsold at the end of the day
have no value. From past experience, the following probability distribution has been calculated:

Number of
Doughnuts Sold
per Day Probability
40 .25
50 .60
60 .15

Assume that only the three quantities listed are ever sold and that the occurrences are random events.

Required:

(1) What is the average number of doughnuts sold per day? If the operator stocked this average number of
doughnuts each day, what would the expected daily contribution margin be? (Round to two decimal
places.)
(2) Compute the variance, the standard deviation, and the coefficient of variation of the expected value.
(Round intermediate calculations to 4 decimal places and round the standard deviation and the coefficient
of variation to the nearest whole cent.)

SOLUTION

(1)
Number of Average Number
Doughnuts Sold Probability of Doughnuts Sold
40 .25 10
50 .60 30
60 .15 9
Average number of doughnuts sold per day..................................................... 49

Expected daily contribution margin if 49 doughnuts stocked:

Number of Expected Daily


Doughnuts Sold Contribution Margin Contribution
per Day (Conditional Value) Prob. Margin
40 (40 x $.10) - (9 x $.25) = $1.75 .25 $.44
50 49 x $.10 = 4.90 .60 2.94
60 49 x $.10 = 4.90 .15 .74
Expected daily contribution margin (expected value)......................................................... $ 4.12
Decision Making Under Uncertainty 367

(2)
(1) (2) (3) (4) (5)
Contribution Difference
Margin from
(Conditional Expected Value Variance
Value) ($4.12) (2) Squared Probability (3) x (4)
$1.75 $(2.37) $5.6169 .25 $1.4042
4.90 .78 .6084 .60 .3650
4.90 .78 .6084 .15 .0913
$1.8605

Standard deviation = square root(Column 5 total) = square root($1.8605)


= $1.3640

$1.36
Coefficient of variation = ---------- = .33
$4.12

PROBLEM

2.
Decision Trees. The management of Seoul Industries is trying to decide whether to build a large, medium, or
small plant at a new location. Demand for the company's product in the new area is uncertain, but the marketing
manager has assigned probabilities to three levels of demand. These probabilities, as well as the contribution
margins (conditional values, in millions of dollars) for each plant size and demand level, are as follows:

Demand Level
Plant Size High Moderate Low
Large.......................................................................................................... $7 $2 $ -1
Medium...................................................................................................... $6 $4 $ 0
Small.......................................................................................................... $5 $3 $ 1
Probability................................................................................................. .3 .5 .2

Required:

(1) Construct a decision tree for this situation.


(2) Choose the most profitable of the expected alternatives.
Decision Making Under Uncertainty 368

SOLUTION

(1)

Expected
Contribution
Demand Margin

/ ---------- HIGH (.3) $2.1


/ $7
/
T /---------------- MODERATE (.5) 1.0
N/ $2
A/
L /-------------------- LOW (.2) -.2
P/ $-1 $2.9 expected value
/
E/
G/
R/
A/ /--------------- HIGH (.3) $1.8
L/ / $6
/
DECISION / MEDIUM PLANT------------ MODERATE (.5) 2.0
POINT \ $4
S\ \
M\ \-------------- LOW (.2) 0
A\ $0 $3.8 expected value
L\
L\
\
P\
L \-------------------- HIGH (.3) $1.5
A\ $5
N\
T \ -------------- MODERATE (.5) 1.5
\ $3
\
\ -------- LOW (.2) .2
$1 $3.2 expected value

(2) Based on expected contribution margins, management should build the medium plant, which has the
highest expected value.
Decision Making Under Uncertainty 369

PROBLEM

3.
Standard Deviation for Perfectly Correlated Cash Flows. Gayle Company is considering a capital expenditure
for which the periodic cash inflows are expected to be normally distributed and perfectly correlated. The
expected net present value of the proposal is $10,000, and the standard deviation of the cash inflows is $2,500 in
each period. The initial cash outflow has a zero standard deviation. The company's weighted-average cost of
capital is 12%, and the project is expected to have a life of 4 years.

Required: Compute the standard deviation, rounded to the nearest dollar, of the expected net present value for the
Gayle Company investment. The present value of $1 @ 12% at the end of four periods is .636 and the present
value of an annuity of $1 for four periods is 3.037.

SOLUTION

Periodic Standard Present Value of Present Value of


Year Deviation $1 at 12% Standard Deviation
0 0 1.000 0
1-4 $2,500 3.037 $7,593
Standard deviation of net present value................................................................................ $7,593

PROBLEM

4.
Standard Deviation and Coefficient of Variation for Perfectly Correlated Cash Flows. Laurens
Manufacturing Co. is considering the purchase of a machine that will cost $100,000 and produce a new product.
The machine is expected to have a useful life of 5 years and no salvage value. The after-tax cash inflows for each
year are expected to be $30,000. The cash flows are expected to be normally distributed with a standard deviation
of $3,000. The periodic cash flows are expected to be perfectly correlated. The weighted-average cost of capital
is 12%. The present value of $1 @ 12% at the end of five periods is .567 and the present value of an annuity of
$1 for five periods is 3.605.

Required:

(1) Compute the expected net present value of the capital expenditure proposal.
(2) Determine the standard deviation of the expected net present value.
(3) Compute the coefficient of variation. (Round to two decimal places.)
Decision Making Under Uncertainty 370

SOLUTION

(1)
Expected Value of Present Value of
After-tax Present Value of After-tax
Year Net Cash Flows $1 @ 12% Net Cash Flows
0 $(100,000) 1.000 $ (100,000)
1-5 30,000 3.605 108,150
Expected net present value............................................................................................... $ 8,150

(2)
Standard Deviation Present Value of Present Value of
Year of Cash Flows $1 @ 12% Standard Deviation
0 0 1.000 0
1-5 $3,000 3.605 $10,815
Standard deviation of expected net present value............................................................ $10,815

(3) Coefficient of variation = 10,815/8,150 = 1.33

PROBLEM

5.
Revising Probabilities. Health Foods Manufacturing Company plans to introduce a new product known as oat
bran chips. The vice-president of marketing believes that the demand for oat brand chips will be between 50,000
and 80,000 bags. The following probabilities have been assigned to each possible level of demand:

Demand Probability
50,000 .20
60,000 .20
70,000 .50
80,000 .10

The president of the company requested that the market demand be analyzed by an expert system computer
program that resulted in the following output:

Demand Probability
50,000 .10
60,000 .10
70,000 .50
80,000 .30

Required: Using Bayes' theorem, compute the posterior probabilities for the various levels of demand for oat bran
chips, assuming that the demand probabilities generated by the expert's system provide new information (i.e.,
assume the expert system probabilities are conditional probabilities). (Round to four decimal places.)
Decision Making Under Uncertainty 371

SOLUTION

(1) (2) (3) (4) (5)


Prior
Probability x Posterior
Conditional Probability
Prior Conditional Probability (4) Line Item
Demand Probability Probability (2) x (3)  (4) Total
50,000 .20 .10 .02 .06250
60,000 .20 .10 .02 .06250
70,000 .50 .50 .25 .78125
80,000 .10 .30 .03 .09375
1.00 1.00 .32 1.00000

PROBLEM

6.
Payoff Table. Sara Company buys and resells a perishable product. A large purchase at the beginning of each
month provides a lower per unit cost and assures that Sara can purchase all the items it wishes. However, unsold
units at the end of each month are worthless and must be discarded. If an inadequate quantity is purchased,
additional units of acceptable quality are not available.

The units, which Sara sells for $3 each, are purchased at a fixed fee of $100,000 per month plus $1 each, if at
least 100,000 units are ordered and if they are ordered at the beginning of the month.

The needs of Sara's customers limit the possible sales volumes to only four quantities per month  100,000,
125,000, 150,000, or 175,000 units. However, the total quantity needed for a given month cannot be determined
prior to the date Sara must make its purchases. The sales managers are willing to place a probability estimate on
each of the four possible sales volumes each month. They noted that the probabilities for the four sales volumes
change from month to month because of the seasonal nature of the customers' businesses. Their probability
estimates for December, 19A, sales quantities are 25% for 100,000, 35% for 125,000, 30% for 150,000, and 10%
for 175,000.

Required: Prepare a payoff table showing the expected value of each of the four possible strategies of ordering
units, assuming that only the four quantities specified are ever sold and that occurrences are random events.
Identify the best strategy. (ICMA adapted)
Decision Making Under Uncertainty 372

SOLUTION

Table of expected values of possible strategies (000s omitted):

Purchases/Sales 100 125 150 175 Expected Value


100 $100 $100 $100 $100 $100
125 75 150 150 150 131.25
150 50 1251 200 200 136.252
175 25 100 175 250 118.75
Probability .25 .35 .30 .10
1
Contribution margin for ordering 150,000 units and selling 125,000 units:

Sales (125,000 x $3)............................................................................................................................... $375,000


Cost of units [$100,000 + (150,000 x $1)]............................................................................................. 250,000
$125,000
2
Expected value for purchasing 150,000 units:

$50 x .25.................................................................................................................................................. $ 12.50


125 x .35.................................................................................................................................................. 43.75
200 x .30.................................................................................................................................................. 60.00
200 x .10.................................................................................................................................................. 20.00
$ 136.25

Sara Company should purchase 150,000 units for December, according to the expected value decision model
because this number of units produces the largest expected value, $136,250.
Decision Making Under Uncertainty 373

Chapter 25

PROFIT PERFORMANCE MEASUREMENTS AND


INTRACOMPANY TRANSFER PRICING

MULTIPLE CHOICE

Question Nos. 12, 17, and 18 are AICPA adapted.


Question No. 10 is ICMA adapted.
Question Nos. 9, 11, and 14-16 are CIA adapted.

E 1. If profits are $20,000, sales are $100,000, and capital employed is $50,000, the capital-
employed turnover rate would be:
A. 4
B. 5
C. 0.40
D. 0.20
E. 2

SUPPORTING CALCULATION:

$100,000  $50,000 = 2

C 2. If profits are $100,000, sales are $500,000, and capital employed is $400,000, the rate of
return on capital employed would be:
A. 400%
B. 125%
C. 25%
D. 20%
E. 500%

373
Decision Making Under Uncertainty 374

SUPPORTING CALCULATION:

$100,000  $400,000 = 25%

D 3. The profit figure that is preferred in connection with the analysis of a division or department is:
A. income before income tax
B. taxable profit
C. net income
D. operating income
E. net income exclusive of bond interest

374
Profit Performance Measurements/Intracompany Transfer Pricing 375

A 4. All of the following are arguments that favor the use of the original cost basis for valuing plant
assets for determining the investment base except:
A. depreciated values reflect the objective that the capital base should be maintained by
replacing assets used up (depreciated) during the current period
B. nonuniformity of depreciation methods and differing ages of assets impedes comparison
among plants
C. assets of manufacturing companies should be considered to be used on a continuing
basis
D. accumulated depreciation is not deducted from the gross asset value of property
because it represents retention of the funds required to keep the stockholders' original
investment intact
E. plant assets are used to produce income over their entire life; therefore, the full cost is
considered an investment until the assets are retired from use

C 5. A limitation to using the rate of return on capital employed for internal profit measurement
would be that:
A. managers are influenced to make decisions that are good for the company only in the
long run; thus, they often miss current opportunities
B. none of the data required for allocating assets to segments are available in the
accounting records
C. lack of agreement on the optimum rate of return might discourage managers who
believe the rate is set at an unfair level
D. weaknesses with respect to the use or nonuse of individual assets, particularly
inventories, would not be detected
E. the ratio cannot be used for measuring efficiency in managing the company or the
division

A 6. Reporting income by divisions, where there are frequent purchases and sales among divisions,
has been criticized because of the arbitrary nature of the:
A. transfer prices
B. gross revenues assigned to products sold
C. return-on-capital-employed computations
D. depreciation methods used
E. product pricing methods
Profit Performance Measurements/Intracompany Transfer Pricing 376

C 7. The transfer pricing method that is the best objective profitability and performance
measurement is based on:
A. cost
B. negotiated pricing
C. market pricing
D. return on capital employed
E. arbitrary methods

E 8. The transfer pricing method that allows managers the greatest degree of authority and control
over the profit of their units is:
A. market pricing
B. return on capital employed
C. arbitrary methods
D. cost
E. negotiated pricing
Profit Performance Measurements/Intracompany Transfer Pricing 377

A 9. The return on investment (ROI) ratio measures:


A. both asset turnover and earnings as a percentage of sales
B. asset turnover and earnings as a percentage of sales, correcting for the effects of
differing depreciation methods
C. only asset turnover
D. only earnings as a percentage of sales
E. none of the above

E 10. Return on investment (ROI) is a term often used to express income earned on capital invested
in a business unit. A company's ROI would be increased if:
A. sales decreased by the same dollar amount that expenses increased
B. sales and expenses increased by the same percentage that total assets increased
C. net profit margin on sales increased by the same percentage that total assets increased
D. sales increased by the same dollar amount that expenses and total assets increased
E. sales remained the same and expenses were reduced by the same dollar amount that
total assets decreased

C 11. Which of the following is the most valid reason for not using a cost plus transfer price between
decentralized units of a company? A cost plus transfer price:
A. does not reflect the excess capacity of the supplying unit
B. is typically more costly to implement
C. does not ensure the control of costs of a supplying unit
D. is not available unless market-based prices are available
E. all of the above

B 12. In a decentralized company in which divisions may buy goods from one another, the transfer
pricing system should be designed primarily to:
A. minimize the degree of autonomy of division managers
B. aid in the appraisal and motivation of managerial performance
C. increase the consolidated value of inventory
D. discourage division managers from buying from outsiders
E. all of the above

E 13. To avoid waste and maximize efficiency when transferring products among divisions in a
competitive economy, a large diversified corporation should base transfer prices on:
Profit Performance Measurements/Intracompany Transfer Pricing 378

A. full cost
B. replacement cost
C. product cost
D. variable cost
E. market price
Profit Performance Measurements/Intracompany Transfer Pricing 379

A 14. A company has two divisions, A and B, each operated as a profit center. A charges B $35
per unit for each unit transferred to B. Other data follow:

A's variable cost per unit............................................................................ $ 30


A's fixed costs............................................................................................. $ 10,000
A's annual sales to B.................................................................................. 5,000 units
A's sales to outsiders.................................................................................. 50,000 units

A is planning to raise its transfer price to $50 per unit. Division B can purchase units at $40
each from outsiders, but doing so would idle A's facilities now committed to producing units for
B. Division A cannot increase its sales to outsiders. From the perspective of the company as
a whole, from whom should Division B acquire the units, assuming B's market is unaffected?
A. Division A, in spite of the increased transfer price
B. outside vendors
C. Division A, but only at the variable cost per unit
D. Division A, but only until fixed costs are covered; then should purchase from outside
vendors
E. none of the above

SUPPORTING CALCULATION:

Cost of buying outside $40/unit


Incremental cost of making inside $30/unit
Savings from buying inside $10/unit

C 15. Given a competitive outside market for identical intermediate goods, what is the best transfer
price, assuming all relevant information is readily available?
A. average cost of production
B. average cost of production plus average production department's allocated profit
C. market price of the intermediate goods
D. market price of the intermediate goods less average production department's allocated
profit
E. none of the above

A 16. What is the most appropriate base to use in computing a return on investment for a business
Profit Performance Measurements/Intracompany Transfer Pricing 380

segment?
A. total segment assets employed
B. total segment assets employed less allocated liabilities of the company
C. current assets of the segment
D. noncurrent assets of the segment
E. none of the above

A 17. The calculation of a company's return on investment is affected by a change in:

Capital Turnover Profit Margin on Sales


A. yes yes
B. no yes
C. no no
D. yes no
Profit Performance Measurements/Intracompany Transfer Pricing 381

B 18. The price that one division of a company charges another division for goods or services
provided is called the:
A. market price
B. transfer price
C. outlay price
D. distress price
E. none of the above

D 19. The following data relate to the Happy Division of Euphoria, Inc.:

Sales ............................................................................................................. $10,000,000


Variable costs................................................................................................. 3,000,000
Direct fixed costs............................................................................................ 5,000,000
Invested capital............................................................................................... 2,000,000
Capital charge................................................................................................ 12 %

The divisional residual income is:


A. $7,000,000
B. $240,000
C. $2,000,000
D. $1,760,000
E. none of the above

SUPPORTING CALCULATION:

($10,000,000 - $3,000,000 - $5,000,000) - ($2,000,000 x 12%) = $1,760,000

B 20. The following data relate to the Happy Division of Euphoria, Inc.:

Sales ............................................................................................................. $10,000,000


Variable costs................................................................................................. 3,000,000
Direct fixed costs............................................................................................ 5,000,000
Invested capital............................................................................................... 8,000,000
Capital charge................................................................................................ 12 %

The divisional return on investment is:


Profit Performance Measurements/Intracompany Transfer Pricing 382

A. 50%
B. 25%
C. 20%
D. 12%
E. none of the above

SUPPORTING CALCULATION:

($10,000,000 - $3,000,000 - $5,000,000)  8,000,000 = 25%


Profit Performance Measurements/Intracompany Transfer Pricing 383

E 21. Common forms of management incentive compensation include all of the following, except:
A. deferred compensation
B. stock options
C. stock appreciation rights
D. performance shares
E. all of the above are forms of management incentive compensation

D 22. Generally, performance measurements and related incentive compensation plans should do all
of the following, except:
A. reward long-term performance
B. tie incentive compensation to achieving strategic goals
C. evaluate operating profits before gains from financial transactions
D. evaluate operating profits after deductions for the incremental amount of accelerated
depreciation
E. all of the above should be done
Profit Performance Measurements/Intracompany Transfer Pricing 384

PROBLEMS

PROBLEM

1.
Rate of Return on Capital Employed, Using Depreciated Cost Method. Quik Energy Corp. has
$1,500,000 in total assets. Plant and equipment have a book value of $600,000 (original cost,
$800,000). There is a cash balance of $200,000, and accounts receivable total $250,000. The
remainder of the assets is in the form of materials inventories. The company produces two
productsJuicers and Blenders. Sales and production data are:

Juicers Blenders
Units sold......................................................................................................... 30,000 50,000
Sales price....................................................................................................... $20 $33
Materials cost................................................................................................... 8 16
Labor and overhead......................................................................................... 6 8
Marketing and administrative expenses........................................................... 4 6

All sales are on account. All overhead and marketing and administrative costs are variable and in the
same proportion between products as labor costs. Plant and equipment are allocated on the basis of labor
and overhead costs. Cash is allocated to products on the basis of the anticipated cost of goods sold.
Inventory is allocated on the basis of materials cost.

Required: Compute the rate of return on capital employed for each product and for the company as a
whole, using the depreciated cost method. (Round allocation percentages and answers to the nearest
whole percent.)
Profit Performance Measurements/Intracompany Transfer Pricing 385

SOLUTION

Rate of Return on
Capital Employed

Juicers..................................................................... $ 60,000  $ 409,000 = 15%


Blenders.................................................................. $ 150,000  $ 1,091,000 = 14%
Company................................................................. $ 210,000  $ 1,500,000 = 14%

Additional computations: Juicers Blenders


Sales price........................................................................................................... $ 20 $ 33
Less unit cost...................................................................................................... 18
30
Net income per unit............................................................................................. $ 2 $ 3
Multiplied by unit sales........................................................................................ x 30,000 x 50,000
Net income.......................................................................................................... $ 60,000 $ 150,000

Net income for the company............................................................................... $210,000

Allocation of Capital
Allocation Total to Total
Item Basis Allocate Basis
Cash Cost of goods sold $200,000 $1,620,000
100 %
Accounts receivable Sales 250,000 2,250,000
100 %
Inventories Materials cost 450,000 1,040,000
100 %
Plant and equipment Labor and overhead cost 600,000 580,000
100 %

Basis Used By Cost Allocated To


Item Juicers Blenders Juicers Blenders
Cash $420,000 $1,200,000 $ 52,000 $ 148,000
26 % 74 %
Profit Performance Measurements/Intracompany Transfer Pricing 386

Accounts receivable 600,000 1,650,000 67,500 182,500


27 % 73 %
Inventories 240,000 800,000 103,500 346,500
23 % 77 %
Plant and equipment 180,000 400,000 186,000 414,000
31 % 69 %
Total.............................................................................................. $ 409,000 $ 1,091,000

Total for company......................................................................... $1,500,000


Profit Performance Measurements/Intracompany Transfer Pricing 387

PROBLEM

2.
Percentage of Profit to Sales; Capital-Employed Turnover Rate; Rate of Return on Capital Employed.
The president of Black Hills Mining Company compared the Copper Mining Division, the Zinc Mining
Division, and the Nickel Mining Division, using the relevant data below:

Copper Zinc Nickel


Mining Mining Mining
Division Division Division
Sales..................................................................................... $ 5,000,000 $5,000,000 $5,000,000
Division expenses................................................................. 4,000,000 4,000,000 4,900,000
Capital employed.................................................................. 20,000,000 2,000,000 2,000,000

Required:

(1) Compute the percentage of profit to sales, the capital-employed turnover rate, and the rate of
return on capital employed for the three divisions.
(2) Do the Copper Mining Division and the Nickel Mining Division have the same low rate of return on
capital employed for the same reasons? Offer any suggestions for improving the various divisions'
rates of return on capital employed.

SOLUTION

(1)
Percentage
of Profit
to Sales
Copper Mining Division.......................................... $ 1,000,000  $5,000,000 20%
Zinc Mining Division............................................... $ 1,000,000  $5,000,000 20%
Nickel Mining Division............................................ $ 100,000  $5,000,000 2%

Capital-Employed
Turnover Rate
Copper Mining Division.......................................... $ 5,000,000  $20,000,000 .25
Zinc Mining Division............................................... $ 5,000,000  $ 2,000,000 2.5
Profit Performance Measurements/Intracompany Transfer Pricing 388

Nickel Mining Division............................................ $ 5,000,000  $ 2,000,000 2.5

Rate of Return
on Capital
Employed
Copper Mining Division......................................... 20% x .25 5%
Zinc Mining Division.............................................. 20% x 2.5 50%
Nickel Mining Division........................................... 2% x 2.5 5%
Profit Performance Measurements/Intracompany Transfer Pricing 389

(2)
No; although both Copper and Nickel have the same 5% rate of return on capital employed, it is for
different reasons. Using the Zinc Division as a benchmark, Copper has an acceptable percentage of profit
to sales ratio and an unacceptable capital-employed turnover rate. Nickel has an unacceptable percentage
of profit to sales ratio and an acceptable capital-employed turnover rate.

Copper will best be able to improve its return on investment by reducing its assets employed. Nickel will
best be able to improve its return on investment by cutting costs to increase its percentage of profit to
sales.

PROBLEM

3.
Market-Based Transfer Pricing System vs. Standard Cost System. Corbin Cement Products sells 100,000
bags of cement each year at $10 per bag. Its plant has a capacity to produce 150,000 bags of cement
per year; fixed costs related to the plant amount to $400,000 per year. Variable costs per bag are $5.

Cohoes Concrete Products, a subsidiary located in another city, uses cement, sand, and gravel to produce
bags of concrete. One-half bag of cement is needed for each bag of concrete. At present, the bags of
concrete sell for $9 per bag and cost $6 per bag (all variable costs, including the cost of cement). The
subsidiary sells 100,000 bags per year and, at present, purchases its cement from an outside supplier at
$9 per bag. Corbin Cement Company asks its subsidiary to buy 50,000 bags of cement at the $10 market
pricean offer that is refused by Cohoes Concrete Products.

Required: Compare gross profits under the present market-based transfer pricing system for Corbin
Cement Products, its subsidiary, and the corporation as a whole with the gross profits if the transfer pricing
system were based on standard costs for a production level of 150,000 bags of cement.
Profit Performance Measurements/Intracompany Transfer Pricing 390

SOLUTION

Corbin Cohoes Corporation


Cement Concrete as a
System Products Products Whole
Market-based transfer pricing:
Sales to outsiders...................................... $ 1,000,000 $900,000 $ 1,900,000
1
Cost of goods sold.................................... 900,000 600,000 1,500,000
Gross profit................................................ $ 100,000 $300,000 $
400,000

Standard cost transfer pricing


(using 150,000 bags of
cement as basis for
allocating fixed costs):
Sales to outsiders...................................... $ 1,000,000 $ 900,000 $ 1,900,000
2
Intracompany sales (costs)....................... 383,333 (383,333) --
3 4
Cost of goods sold.................................... (1,150,000) (150,000) 1,300,000
Gross profit................................................ $ 233,333 $ 366,667 $ 600,000

1
($5 x 100,000 bags) + $400,000 = $900,000
2
[$5 + ($400,000/150,000)] x 50,000 bags = $383,333
3
($5 x 150,000 bags) + $400,000 = $1,150,000
4
[$6 - ($9*/2)] x 100,000 bags = $150,000

*The $9.00 is the cost per bag of cement purchased on the outside that would not be needed if the
purchase were made within the company. Since one bag of cement is used to produce two bags of
concrete, the per-unit cost of cement for one bag of concrete is equal to $4.50 (or half the cost of a bag
of cement).

With the standard cost system, the concrete subsidiary will profit, because $1.33 will be saved per bag of
cement purchased [($9.00 - ($5 + $2.67)]. Corbin Cement Products will also profit because its fixed
costs can be spread over a larger number of units. Most important, the corporation's overall gross profit
will be increased by $200,000.
Profit Performance Measurements/Intracompany Transfer Pricing 391

PROBLEM

4.
Listed below are relevant Company Z data for component Smurf that is produced by both Division X and
outsiders and that is an integral part of product Widget that is produced by Division Y:

Y's annual purchase of Smurf.......................................................................................................... 50,000


X's variable cost per unit of Smurf................................................................................................... $10
X's fixed cost per unit of Smurf........................................................................................................ $2

Required: Assume that both divisions are profit centers and have the right to buy and sell outside if their
sister divisions don't meet the external market price.

(1) If Division X currently has some idle capacity, will Company Z, as a whole, be better off if Division
Y buys Smurfs outside for $14 each rather than internally for the $15 per-unit selling price that
allows Division X its normal markup?
(2) If Division X could sell all 50,000 units to outside buyers at $15 per unit, will Company Z be better
off if Division Y buys Smurfs outside for $14 each rather than internally for the $15 per-unit selling
price?
(3) If Division X has some idle capacity and the outside market price drops to $11 per unit, which is
below the full cost of $12 per unit in Division X, will Company Z be better off if Division Y buys
Smurfs externally?

SOLUTION

(1) No, the company will be worse off by $200,000 (the difference between the $14 per-unit outside
price and the $10 per-unit variable cost, multiplied by 50,000 units).
(2) Yes, the company will be better off by the difference between the $250,000 contribution margin on
the external sales [$50,000 x ($15 - $10)] and the $200,000 difference in part (1) above, or
$50,000.
(3) No, the company will be worse off by $50,000, which is the difference between the outside price
of $11 per unit and the $10 per-unit variable cost, multiplied by 50,000 units.

PROBLEM
Profit Performance Measurements/Intracompany Transfer Pricing 392

5.
Transfer Pricing. The Chemical Division of Bill Company produces lawn-care chemicals. One-third of
Chemical's output is sold to the Lawn Services Division of Bill; the remainder is sold to outside customers.
The Chemical Division's estimated sales and standard cost data for the year follow:

Lawn Services Outsiders


Sales.................................................................................................... $ 15,000 $40,000
Variable cost........................................................................................ (10,000) (20,000)
Fixed cost............................................................................................. (3,000) (6,000)
Gross profit........................................................................................... $ 2,000
................................................................................................$14,000

Gallons sold......................................................................................... 5,000 10,000


Profit Performance Measurements/Intracompany Transfer Pricing 393

The Lawn Services Division has an opportunity to purchase 5,000 gallons of identical quality from an
outside supplier at a cost of $1.75 per gallon on a continuing basis. Assume that the Chemical Division
cannot sell any additional products to outside customers, that the fixed costs cannot be reduced, and that
no alternative use of facilities is available.

Required: Should Bill allow its Lawn Services Division to purchase the chemicals from the outside
supplier? Support your answer by computing the increase or decrease in Bill Company operating costs.
(AICPA adapted)

SOLUTION

Yes, because buying the chemicals would save Bill Company $1,250 determined as follows:
Variable cost to manufacture by Chemical Division............................................................... $ 10,000
Outside supplier cost ($1.75 x 5,000).................................................................................. 8,750
Savings to Bill if the Lawn Services Division purchases from the
outside supplier............................................................................................................... $ 1,250

You might also like